You are on page 1of 418
Solucién: Este probléma puede resolverse de dos formas: con el métode de campo magnético (pro- cedimiento directo) 0 mediante analogia con un cireuito eléctrico (procedimicnte ine directo), Método 1. Puesto que el valor de p, €s elevado en comparacién con el de a,a partir del ejemplo 7.6, NE botteNE “ia € 2p. oy 2p. _ 210 x 1077)(0.5 x 10-9) Método 2, El nucleo toroidal de Ia figura 8.26(a) es andlogo al circuito eléetrico de la figura 8.26(). Con base en el cireuito y Ia tabla 8.4, € 2m. SF aN aw HR = we y2T ho HS pogpiytre 2p9¥ eee OA “PokerNa — 3-978 como se obtuvo anteriormente. En el circuito magnético que aparece en Ja figura 8.27, caleule Ia corriente que una densidad.de flujo magnético de 1.5 Wb/m?en el entrehierro producira en la bobina, su. poniendo que 42 = 50, ¥ que todas las ramas comparten una misma area tle seccion fruneveread de 10 mot. 7. =o Sotuctén: El circuito magnético de Is figura $27 es andlogo al circuito eléetrico de Ia En In figura 8.27, 9, Jt, By y Oh, son tas reluctancias en las trayectorias 143, 1 y 50 (entrehierro), respe: poate Hoses 3x10 7206 se 9107 0.9 x 108 9 — Ge 10-7)(50)40 TOY 20% o 4x 10-7 S108 — Gae 10VA)GO IO) ~~ 20 Al combinar 2, y &; como resistores en paralelo, Mh, _ My _S x 10% PMN a= Ge ge = B= SS 7.4 X10" Dey = Peg + My + Ml Ty — PARA e £ za,sa, . a, => Fale, : © o Figura 8.28. Circuito eléctrico anislogo al cireuito magnético de 1a figura 8.27. La fuerza magnetomotriz. es = N= ¥,Rr S. Por consiguiente po BeS8ir _ 1S x 10 x 10 N 400 > 207 4416.0, Rjercicto 8.15 ; i By torcide de 1a figura 5.26(a) tiene tna bobini de 1000 vueltas en torno a su_nd- ee coc y a~ lem, qué corriente se requiere para establecer un flujo, magnetico de 0.5 mWb i @) sicl nucleo es ne Magnético? b) si el nticleo es de 4, = 500? Respucstas: a) 795.84 y b) 1592.4, Un electroimén en forma de U como el que se muestra en la figura 8.29 esta disenado Pate ievamtar una masa de 400 kg Gincluida la del contacto). La horquilla de hierro (Gu, B00) tiene una seccion transversal de 40 em?y longitud media de 50 em, ¥ los entrehierros ore eae largo cada uno. Ignore la reluctancia del contacto y caicule cl ntimero de vuel- cea tla bobina cuando la corriente de excitacin es de 1A. Solucién: Ta fuerza de tracciGn en uno y otro de los dos entrehierros debe equilibrar el peso. Por tanto, BE £2 SD ome Figura 829. Blectroimén en forma de U; para el ejemplo 8.16. Resumen By = Bite _ 400 x 9.8 x Aer X< 1077 ag 4010 By = 1.41 Wom? Pero FANT = HR, + HD ae 2 0.1 x 10 6 x 108 Puesto que 6x ar x 107%1 ) Su velocidad y energfa cinética en esa ubieacisn, Una carga de -2 mC xe pone en movimiento en el punto (0,1, 2) a una velocidad de fen un campo magnética B= 6a, Wh/m®, Determine la posicion y velocidad de la 10 § despues suponiendo que la asa de la carga en de I geamo, Describa el movi imeage Be “18. Promenas a ase) Figura 8.31. Para el problema 85. ‘Tras inyectar un haz de clectrones en forma normal al contorno del plano de un campo uni forme Ba, 08 posible dispersarion segn su velocidad, come se observa en la figura 831. 4) Demuestre que los electrones serian expulsados del campo en trayectorias paralel haz de entrada, como se muestra en la figura. b) Deduzca una expresion para Ia distancia de salida d sobre el punto de entrada. al Dado que B = Gra, — 9ya, + 32a, Wo/m?, halle Ia fuerza total experimentada por Ia espira rectangular (en el plano 2 ~ 0) que aparece en la figura 8.32. Un elemento de corriente de 2 em de longitud se localiza en el origen en el vacto y porta una corriente de 12 mA a lo largo de n,,Una corriente filamentosa de 15a, A se localiza a su vez 1o largo de x — 3, ~ 4. Halle la fuerza sobre el filamento de corriente. “Tres Kinens infinitas Ly, Ls ¥ Ly definidas por x — 0,y — 0:x = 0,y = 4.y x = 3.9 = 4,respec- tivamente, portan corrientes filamentosas de 100 A, 200A y 360 A a'lo largo de a,. Halle Ia fuctza por unidad de longitud sobre 4) Ly debida a Ly by 4, detwan a £5 ©) Ly dobida a Ly A) Ly debida a Ly y Ly Indique en cada caso si se trata de una fuerza de repulsion © Figura 8.32. Para el problema 8.6. Figura 8:33. Para el problema 8.9. + 89. Un conductor de 2 m de largo que porta'3 A s¢ coloed en paralelo al eje za una di P, = 10-cm, como se muestra en Ia figura #33, Si el campo en la region es (pia) (eudnto trabajo se requicre para que ci conduetor dé una vuelta alrededor del ope Galeule a) Ia fuerza sobre el lado I de Ia espira triangular y b) 1a fuerza total sobre la “BIL. Una linea de transmisién triffsicn se compone de trex conductores sostenidod en I ‘A.B Y € para formar un tidngulo equildtero como el que aparece en la figura 85. stante, tanto el conductor A como el conductor 4 portal una corriente de 75 A, que el conductor © porta una corr Slconductor Cen ese instante. 8.12. Un tubo de longitud infinita de radio interno a y radio externo 6 es de un material magnético, porta una corriente total /y esta colocada # lo largo del eje =. Si se halla toa un campo magnético constante ga, determine la fuerza por unidad de longi Figura 8.34, Para el problema 6.10. ; Figura 8.35. Para cl problema 8.11... 8.13. Un conductor de longitud infinita esté incrustado en una masa de hierro (j= 2000/4,),de la auc esta nisiado, como te muestra on in Hgura S90. Con base en la teora Ge las imagenes, Sstime Ia densidad de flujo magnético en el punto P- 4.14. Un galvanometro dispone dle una bobina rectangular de 10 por 30 mim por lado que gira en tomo al centro del iado mas corto, Montada en un campo magnetico radial, en su plano: neta un campo magnético constante de 0.4 Wh/m?. Si cuenta con 1000 Vueltas y ports una Corriente de 2 mA, halle el torque que experiment, + 8:15. Un pequefto imén ubicado en el origen produce B = —0.5a, mWb/m? en (10, 0,0). Halle @) ©.3.0) ») G40) GAA) RAG. Un blaque de hierro (J4 = 5000}4,) s6 coloca en tn campo magnético uniforme con 1.5 Whim. Stel hicrro se compone de 8.5 > 10™ dtomosim, calcula: a) la magnetizacion Mb) la corriente mapnética promedio. y Figura £36. Para el problema 8.13. vaa7. sas, 820, +821. naa. 823, En cierto material con pt = 6:Sptor = 108, + 25a, — 40a, Am halle 4) La susceptibilidad magnética x, del material 2) La densidad de flujo magnético B. ©) La magnetizacién M. 4d) La densidad de energia magnetics Bn un material ferromagnético (44 = 4.54,). B ~ 4ya, mWb/m? eateute: 4) Xiq, 8) He) M.A) Sy Gierto material registra una intensidad de campo magnético Hf = 1200 A/m cuando # Si Ff se reduce 2 400 A/m, 8 = 1.4 Wh/m?, Caleule el cambio en la magnetizacion M. Un conductor cilindrico de longitud infinita de radio « y permesbilidad 44, se sitda, g0 del eje =. Si porta una corionte J uniformemente distribuida & lo large de a, respecto de 0 = p= a 4 Sim —42(-ya, + 3m) em un cube de tamano a, halle Jy. Supongn que ke 4) Con relacion a a frontera entre dos medios magnéticos que spaiece en la figural Imuestre que las condicionct en In irontera sobre el vecior de magnetieacian son May _ Mas Mn 4 1) Sila frontera no est libre de corriente, demuestre que en lugar de Ia ecuacién, obtiens tat Slt ae) Sipts = 2stg en Ia region 1 (0 < dh < 7), a = Sptg en la region 2 (ar a. 831. Cuando dos alambres paralelos idénticos estan separados 3 m, Tongitud es de 2.5 j2H/m, Calcule el didmetro de cada alambre. inductancia por unidad de x Figues 8.37. Para el problema 8:27. 8.32, Un solenoide de 10 em de longitud y 1 em de radio cuenta con 450 vueltas, Cal tanela, 18.33. El auicleo de un toroide ex de 12 em? y el material con el que est hecho tiene ys, =! radio medio ex de S0-em,caleule el homere de vucltas necesarias para obtenct ag Gade 2512 ‘Demuestre que Ia inductancia mutua entre la espira rectangular y la corricnte lineal se presentaron on Ia figura #4 os vin= Boo] Caloule My cuando a = b = p, = 1m lomprucbe que Ia meuctanei muta entre los solenoides coaxialés devahddon Tongitud €, y €, (€, >> €,), vueltas Ny y Nay radios "yy rsCOn =r es BNNs Me a Un anillo de cobatto (14, ~ 600) tiene un radio medio de 30 em. Si una bobina el anillo porta 12.A, calcule el numero de vucltas resqucricias para establecer una d flujo magnetico promedio de 1.3 We/m an el anil 8.37. Remitase a la figura 827, Si In corriente en Ia bobina es de 0.5 A, halle la fuerza tiz y la intensidad de campo magnético en el entrehierro. Suponga que Ha = S00 ji, ‘das las ramas comparten un aren de seecidn transversal de 10 er" 838. La bobina de 2000 vueltas del circulto magnético que aparece en Ia figura 8.38 Corriente de 10 A. Suponga que todns Ins ramus tienen una secciGn transversal de 2 elmaterial del nucleo es bietro con #4, = 1500. Calesle RF y Wrespecto de. Bt maces, 2) Elentrehierro. zm Figura 8.38, Para el problenia 8.38. eRe aTA| = iE q 2 be 839. 8.40, aan Figura 8.39. Para el problema 8.39. Considere el circuito magnético que se presenta en Ia figura §.39-Suponiendo que cl nucleo G2 "1600,2,) poses una scecin tranvefsal uniforme de 4 cm#, determine la deasidad de fu joen elentrenterro. Considere el relevador clectromagnético que se muestra en la figura 8.40. ,Qué fuerza actin Sobre su armadura (parte movil) si el flujo en el entrehierro es de 2mWb? El area de éste es ‘de0.3 em? y su longitud de 1.5 mm. ‘Un toroide con entrehierro come el que aparece en In figura 8.41 poxee una seceiGn transver- Sal cuasirada, Un conductor largo portador de corriente 7, esta insertada en l entrehierro, Siz, — 200 mA, N= 750,p, = 10.em,a— 5mm y ¢,— 1mm, caleule 4) La fuerza sobre el entrehierro ciiande J, ~ Oy la permeabilidad relativa del toroid es de 300. 1b) Ta fuerza sobre el conductor cuando f ~ 2 mA y la permeabilidad del toroide os infinita, Ignore en ambos casos el efecto de borde en el entrehierro. En Ia figura 8.42 se muestra una secci6n de tn electroimén bajo el cual se halla una placa que soporta una carga, El electroiman poses un area de contacto de 200 em? por polo, en tanto Gue cl pole intermedia cuenta con una bobina con 1000 vueltas e = 3 A. Caleule In masa Mécind que ct cicetroman poxeia tevantan, Suponga que Ia relucrancia ciel leckrotmar 9 ht Placa es despreciable, En Ja figura 8.43 se presenta la seccién transversal de un sistema electromecénico cuyo Embolo se mucve lbremente entre dos casquillos no magneticos Suponiendo que todos lox {ramos comparten la misma rea de seecion transversal 5, demuestre que NPS wn asa Figura 6.40. Para el problema 8.40, Figura 8.42, Para el problema 8.42. Figuea 8.43. Para el problema 843, IV. ONDAsS yY APLICACIONES ;cuaciones de Maxwell 2Quieres ser un héroe? No te contentes con ver hacer proezas a los demas 0 ignorar lo que ocurre a tu alrededor. Actia. Quienes actaan desean ardientemente cumpltr sits Propésitos, avanzar, servir a sus semejantes, ser lox mejores y cambiar su mundo, [GLENN VAN ExtREN Introduccion En la parte IT (capmtulos 4 a 6) de este libro nos ocupamos de los campos electrostéticos, denotados con E(x, y, 2), mientras que en la parte III (eapitulos 7 ¥°8) estudiamos los campos magnetostaticos, representados con H(x, y, z). Esto significa que hasta aquf he. mos restringido nuestro andlisis a los campos electromagnéticos estaticos, 0 invariables en el tiempo. En lo sucesivo examinaremos situaciones con campos eléctricos ¥ magnet. cos dindimicos, 0 var ies en el tiempdBSenalemos en primer término que los campos eléctrico y magnético estaticos son independiente: ontre sf, of tanto que los dinamivos ‘son interdependientes. En otras palabras, un campo elecirico variable en el tiempayimpli- ‘a necesariamente un campo magnético correspondiente variable en el tiempectin we. Bundo término, los campos clectromagnéticos variables cn el Hlempe, fepresemeados con Ee, y, 2, Dy H(z, 9. £. ), possen mayor valor practice que los estuiisos No obstante at Zonoeimicnito de los campay estaticos proporcion: ‘Dara comprender los dinamicos. Pn tercer lugatecucrdese que los campos clectrowtaticos auclen ser prod, Gidos por cargas eléctricas éstitieas y que Tos Campos magnetostiicos se deben al ‘magnéticas estaticas (polos magnéticos); en cambio, los campos variables en el tiempo U ondas suclen deberse a cargas accleradas o corrierite: variables en sl tiempo como i Gue se muestran en Ia figura 9-1 Una confidante pulestoria producird radi Variables en el tiempo). El tipo de caricnte is cue a Ja causa de la emision radiada en tarjetas Iogicas digitales, En suma: i Sea meta El proposito de este capitulo es sentar las bases para el estudio subsecuente: Esto supone la presentacién de dos importantes conceptos{ la fuerza clectromotriz, basada en experimentos de Faraday, (la corriente de desplasamiento, producto de hipstesis de Maxwell. Como resultado de estos conceptos, las ecuaciones de Maxwell —tal como se les, doysmaiid.on in beccides 9.6» los comndicdomes on ia tromiars henna cummnawloatacnagadeicen © Figura 9.1. Divertos tipos de corriente variable on el tiempo: Gi tinuaiaen, Go) nestnngulnr () eiangular 86 estaticos se modificardn para dar cuenta de la variacin temporal de los camp. he destacar que las ccusciones de Maxwell resumen las leyes del electrom: Sorvirdn de fundamento a nuestros andlisis en lo que Festa del libro. Por tal mat Sion 9.5 debe considerarse cl nucleo de este texto. 9.2. Ley de Faraday ras ¢l descubrimiento experimental de Oersted (en el que Biot, Savart y Any ron sus leyes) de que una corriente estacionaria produce un campo magné Igico indagar si el mapnetismo producfa electricidad. Once aftos después del hi ‘Gersted, en 1831, Michacl Faraday en Londres y Joseph Henry en Nueva York: ron que un campo magnético variable on el tiempo producta una corriente elé& De acuerdo con los experimentos de Faraday, un campo magnético estitico ns Ay (bo variable en el tiempo produce un voltaje induci tin eireuito cerrado, el cual provoea un flujo de: eae ae Haut aii enoiab!-en7Obu siber obieiedts al 9b sche Esta es 1a (aMeNRaRaday, In cual puede expresarse como an ay a Nae veo donde N es el ndmero de vueltas en el circuito y Vel flujo a través de cada una. "Para detalles sobre los experimentos de Michael Raraday (1791-1867) y Jéseph Henry (1 vense W.F: Magic, A Source Book in Physics, Flarvard University Press, Cambridge, MA, ale Figura 9.2. Circuito en cl que se muestra lun campo generador de fuer clectromotriz E, Yun campo electrostatics E, es a su vez laJBBMBIIER? sepiin In cual la direccién del flujo de corriente en el circuito es tal gue el campo magnetico inducido resultante de la corriente inducida se opondra al campo magnético original. Recuérdese que un campo eléctrico se describis como aquel en el que cargas eléctricas experimentan fuerza. Los campos eléctricos considerados hasta este punto son causados Por cargas eléctricas; en ellos, las linens de flujo comicnzan y terminan en las cargas. No Ststaute, caisten Uuus Lipos de campos clécrrleos no directamente causados porcargas cléctricas Estos son Tos campos producido: BOF fucrza electromatsiz, Los pensrado: res eléctricos, las baterias, pilas termoeléctricas, pilas de Grove y pilas fotovoltaicas son fuentes de fuerza clectromotriz; todos ellos convierten energin no cléctrica en cléctrica, ~™*" Considérese el circuito electrico que aparece en la figura 9.2, en el que una bateria es fuente de fuerza electromotriz. La accion electroquimiea de Ia baterfa da como resul- tado un campo producido por fuerza electromotriz E, La acumulacin de carga en las terminates de la baterfa causa asimismo un campo clectrostatico E, (~ VV). El campo eléetrico total en cualquier punto es E-E,+8, 2) (Cabs hacer notar que Eyes de cero fuera de la baterfa, Ey E, siguen direcciones opues- tas dentro de ésta y la direccion de E, en la baterfa cs ia contraria ala que sigue fucra de ella. Sise integra la ecuacidn (9.2) sobre el circuito cerrado, fering ej-ac0= [ea (atevéedetanmers) 030) donde $ E.- dl = 0, porque E. es conservativo. La fuerza electromatriz de In haterta es Ia integral de linea del campo producido por esa fuerza: es decir, ven [oman [neta 8 te oa @ Un campo ctectrostistico F, no puede mantener una cofriente estacionaria en un Gircuito cerrado, ya que $, , =a — 0 — 1 Un campo producido por tucrza clectromotriz, E, no es conservativo, Excepto en electrostatica, voltaje y diferencia dé potencial por lo general no son equivalentes *Asf llamada én honor a Heinrich Friedrich Emil Lenz (1804-1865), profesor de fisiea de nacio- ilo 9.3. Fuerza electromotriz estatica y cinética Una vez analizada Ia relacién entre fuerza clectromotriz y campo eléctrica, Telacion entre los campos eléetrico y magnético en el marco de la ley day. Fin el caso de un cireuito con una vuelta (N= 1), la ecuacién (9-1) se convi ae Veo aT de En (érminos de Fy R, 1a ecuacién (9.4) puede expresarse como = ven feat [aoa ee ee once ma pa reaspnec ree er eae aren Se te ee ee ee sas a eee ee te arasee eee Una espira estaci en un campo B variable en el tiempo. ‘Una espira de drea variable en el tiempo en un campo B estatico. Una espira de drea varinble en el tiempo en un campo B variable en el ti Consideremos por separado cada una de estas posibilidades. A. Espira estacionaria en un campo B variable en el tiem| Guerza electromotriz estatica) oc Este caso se representa en Ja figura 9-5, en Ja que una espira conductora estacte Ubica en un campo magnético B variable en el tempo. Fn estas condiciones la. (95) se convierte en 1B ( creciente Figura 9.3. Fuerza clectromotriz inducida debida Cetacionaria en un campo Bt variable en el tiempo. ’ saree ches Fe ea eee an. J, oe Para igualar estas dos integrales, sus integrandos deben ser iguales; es decir, [ce x) -as = - as on B vxE--2 8) Esta es una de las ecuaciones de Maxwell para campos Variables en el tiempo. Indica que él campo E variable en el tiempo no es conservative (V * E. #0). Esto no quiere decir que se infrinjan los principios de 1a conservacion de la energia. Fl trabajo realizado para incorporar una carga alrededor de una trayectoria cerrada en un campo eléctriea va~ Hable cn cl tempo, por cjemplo, ae debe a la energia procedente del campo mapnetica Variable en el tiempo. Obsérvese que la figura 9.3 obedece a ley de Lenz: el flujo de la Corriente inducida 7 produce un campo magnético que se opone @ BC). B. Espira mévil en un campo B estatico (fuerza electromotriz cinética) (Cuando una espira conductora se halla en movimiento en un campo B estatico, en ella se induce una fuerza electromotriz. Recuérdese que, de acuerdo con la ecuacion (8.2), 18 fuerza sobre una carga en movimiento a una velocidad uniforme wen un campo magne tico Bes oe oy A, i BSTEESRROTSMRAEEP. se detine como =, =F2-uxn os) oe se parte del supuesto de que una espira conductora en movimiento a una velocidad uniforme u se compone de gran ndmero de electrones libres, In fuerza clectromotriz in- ducida en ella es ven fea fxm 10) Se cos 2 onc ecm mate ee, ee ae eee eee Sa ee ee Figura 9.4, Maquina de corriente de fuerza clectromotriz cinética, consistente esta vez en una varilla que se muey 'y # son perpendiculares, de modo que, binacién con la ecuacion (8.2), la couacion (3.9) se convicrte on FE, = 16x B Fy LOR y la ecuacion (9.10) en Veg = ute ‘ras aplicar el teorema de Stokes a la ecuacion (9-10), [ow xy as = [ex cox m-as VaE, ve @=B) Nétese que, a diferencia de la ecuacién (9.6), en Ia ecuacién (9.10) no hay ne un signo negativo, puasto que ya se ha tenide en cuenta la ley de Lenz, x Figura 9.5. Fuerza clectromotriz indy 1B Gnducide) Gebide s una expire movil en un campe os cestatico. No siempre es facil aplicar la ccuncisn (9.10). En su uso deben tomarse las precau ciones siguientes: 4. La integral de esta ecuacién es igual a cero a lo largo de la porciGn de 1a espira en Ia que t= 0. Ast-dldebe considerarse a lo largo de ia porcidn de la espira que cru- EAT Campo (ia varilla en el caso de la figura 9.5), donde el valor de w es diferen- te de cero. . Ta direesion de la corriente inducida es la misma que la de E,, ow * B. Los limi- tor de la integral de esta ecuacién se seleccionan en Ia direcciGn opuesta a ta de la Corfiente inducida, lo que satisface In ley de Lenz. En la ecuaciGn (9.13), por ejem- fio, Ia Integracion sobre Les a lo largo de —a,, mientras que la corriente induci- da fluye en Ia varilla a lo largo de a,. a mévil en un campo B variable en el tiempo Esta situacién general corresponde a la de una espira conductora en movi bn un campo magnético variable en el tiempo. En este caso estan presentes tanto la fuer- 2a clectomotris estatica como la einética, La combinacion de las ecuaciones (9.6) ¥ (9.10) Ga como resultado la fuerza electromotriz total, de esta manera _[, eB Ve ‘aL Een 0,8 partir de las ecuaciones (9.8) y (9.14), as fxm eas) + Vx (x B) (16) “Adviértase que la ecuaci6n (9.15) es equivalente a la ccuacién (9.4), de manera que Vs puede hallarse mediante cualquiera de ambas. Mas aun, Ia ecuacion (9.4) puede sup! Siempre a las ecuaciones (9.6), (9-10) y (9-15). ° ‘Una barra conductora puede deslizarse libremente sobre dos ricles conductores, como se muestra en la figura 9.6, Calcule el voltaje inducido en ella a) Si se estaciona en y = Sem y B= 4cos 10% a, mWb/m? 'b) Si se desliza a una velocidad de u = 20a, m/s y B = 4a, mWb/m? ) Sise desliza a una velocidad de u = 20a, m/s y B = 4 cos (10% — y) a, mWb/m? Figura 9.6. Para cl ejemplo 9. Solucién: a) Este caso corresponde al de fuerza clectromotriz estatica, la cual ost dada Ao uae (eae poo bine sear 4(10°)(0.08)(0.06) sen 10% 19.2 sen 10% V De acuerdo con Ia ley de Lenz, la polaridad del voltaje inducido es tal que el pot del punto P en la barra es menor que el de Q cuando B se incrementa, 1b) Este caso corresponde al de fuerza clectromotriz cinética: j Vie = | Cox y= | um, Fa) dem, ube = —20(4.10-(0.06) = A4'smv ©) En este caso estan presentes tanto la fuerza electromotriz estatica como la cing te problema puede resolverse de dos maneras. Método 1. Mediante la ecuacisn (9.15) va — f $ = [0% [410° coy encote — ya" ae as + [cox mya : + [f,208, > 4.107 cos(0"t — ya.) - dx ay = 240 cos(10%r = y*)|” — 80¢10-)(0.06) eos(10% — yy = 240 cos(10%r — y) — 240 cos 10% — 4.8(10-) costo" — 9) = 240 cos(10"r — y) — 240 cos 10% ‘ya que la fuerza electromotriz cinética es despreciable en comparacién con la est Se emplea Ia identidad trigonométrica A+B cos A — cos B= ~2 sen 4+ re — 2) son Va = 480000 (20% — 2) senv Método 2. Opcionalmente, es posible aplicar la ecuacisn (9.4), uae Vee = (9.1.4) donde we [reas = i [Ps coutces— ya dy —4(0.06) sen(10%r — y) 0.24 sen(10% — y) + 0.24 sen 10% mwb Pero. ay sis 8 Yo ey = ut = 20 Por tanto, W = 0.24 sen(10% — 202) + 0.24 sen 10% mWb Vie = —2% = 0,24(10° — 20) cos(10% — 20f) — 0.24(10*) cos 10% mv = 240 cos (10% — y) — 240 cos 10% V 4.5) igual resultado que en la ecuaci6n (9.1.2). En la ecuaci6n (9.1.1), Ia dependencia de y Fespecto del tiempo se considera en J (u * TB) - dl, lo que en cambio ya no preocupa en aB/or. ;Por qué? Porque al calcular la Tuerza electromotriz estitica se da por supuesto que Im eopira os cotacionaria. Por ol contrario, cata sutilorn debe tenerc om vucnita al aplica he ecuacion (9.1-1). Esto explica que el segundo método sea mas sencilla. Bjercicio 9.4 Considérese la espira que se presents en la figura 9.5. Si B = 0.5a, Whim’, = 20.0, €= 10 om y la varilla se mueve a una Velocidad constante de Sia, m/s, halle 4) La fuerza electromotriz inducida en la varilla, B) La corriente a través del resistor ©) La fuerza cinética sobre Ia varilla, 4) La potencia disipada por el resistor. Respuestus: @) 0.4 V, b) 20mA, 6) ~ LmN yd) 8 mi La espira que aparece en la figura 9.7 se halla dentro de un campo magnética. 1B — 30a, mWb/nr Si cl lado DC de la espira corta las I{neas de flujo en la free SO Hz y ia espira se encuentra en el plano yz en cl instante r= 0, calcule a) La fuerza electromotriz indueida en ¢ = 1 ms, b) La corriente inducida en 3 ms. Solucién: Sede sere eee cance Va= [taxman os av pao ae a Oh Oe arf = 100% Como wy di estén en coordenadas cilindricas, IS se tanstorma en coordenadas cas mediante la ecuacion (2.9): a Aloe dem, pr AD=4em, w B = Baa, = B, (cos da, — senda) = 0.05. Por tanto, axe lo” pe, con Bac Figura 9.7. Para el ejemplo 9.2: In'ps Indica la fuerza electromotriz creck GEED)- alm “pei coe $= abtCNnas (obs um pag arene We ee ieee eeneny de \ o- Bogart donde G, es una constante de integracion. En ¢ = 0, 6 = 7/2, ya que la espira se situa en, el plano 2 en ese instante, C, — 7/2, Bn consecuencia, bros e Enr=1ms, Vie = 67 Sen(O.177) = 5.825 mV b) La corriente inducida es = 607 sen (100m) mA Ené=3ms, 1 = 60m sen(0.32) mA = 0.1525 A Bjercicio 92 Repita el ejemplo 9.2 con los mismos datos, excepto que el campo B car 4) B = 50a, mWb/m’; esto es, cl campo magnético se orienta a lo largo de la direc cin de 3: b) B= 0.024 1, Wb/m?; esto es, cl campo magnético es variable en el tiempo. Respuestas: a) 17.93 mV, 0.1108 A y b) 20.5 #V, —41.92 mA. Fl circuito magnético que se presenta en la figura 9.8 pose una seccion transversal uni- forme de 10 m2. Siesta energizado por una corriente de iy(2) = 3 sen 100m! A en la bo- bina de N, — 200 vueltas, halle la fuerza electromotriz inducida en la bobina de N, = 100 vueltas. Suponga que 2 = 500 j2,. Figura 9.8. Circuito mag elejemplo 93. Sotuctén: El flujo en el cireuito es F_ Nah _ Nits. YH” GS 2m, De acuerdo con Ia ley de Faraday, la fuerza electromotriz induelda en la segunda boi N28 2 NiNauS di = CoE 2mp, dt 100 = (200) « (500) = (427 % 10"7) « (10-*) + 30007 cos 10077 ‘2m + (10 x 10 *) = —6m cos 100m7V rriente de desplazamiento En Ia seccién anterior reconsideramos en esencia Ia ecuacién del rotacional de Maxwell para campos electrostaticos y la modificamos para situaciones de variacion en el tiempo fa fin de satisfacer la ley de Faraday. Reconsideremos ahora la ecuacidn del rotacional de Maxwell para campos magnéticos (ley de los circuitos de Ampere) en funcidn de la va- Fiacion en el tempo. Recuérdese que en el caso de campos clectromagnéticos estaticos venus ean Sin embargo, la divergencia del rotacional de un campo vectorial es idéntica a cero (véa- se el ejemplo 3.10). Por consiguiente, viqvxm-0-ves eas) No obstante, la continuidad de corriente en la ecuaci6n (5.43) exige que 2p via = =28: so a9) Es evidente, asf, que las ecuaciones (9.18) y (9.19) son incompatibles respecto de condi- clones de variacidn en el tiempo. Debe modificarse entonces la ecuacion (9.17), a fin de que sea acorde con la couacidn (9.19). Se afiade para ello un término a la ecuacién (9.17), Ta que se convierte en. vxMastay (9.20) donde J, esté por determinarse y definirse. De nueva cuenta, Ia divergencia del rotacio- nal de un vector es igual a cero. Por tanto: V-(V XH) =0=V-3+0-3, 21) Para que la ecuacidn (9.21) sea acorde con la ecuacién (9.19), Vids = ve = 28 2 0y-p) ~ vB (220) oD = 2 (e220) {La sustitucién de ta ecuactén (9.226) en la ecuacién (9.20) resulta en a vxnas+ 2D 023) Esta es la ccuacién de Maxwell (basada en Ia ley de los circuitos de Ampere) para un campo variable en el tiempo. Fl término J, = aD/or se conoce como, ‘en tanto que J es la densidad de corriente de conducciom Figura 9.10. Dos superficies ds 1 ie demestran In mecesics Circuits de Ampere f “fe o © Gi = cB). La insercidn de J, en la ccuacién (9.17) fue una de las mayores contsi de Maxwell, Sin ese término, la propagacion de ondas clectromagncticas (ondas d televisiGn, por ejemplo) seria imposible A hajns frecnancine 4 2 comparacion con J, pero en radiofrecuencias son comparables, no existian atin fuentes de alta frecuencia, de modo que la comprobacion ex} ll de la ecuacién (9.23) era irrealizable. Tuvieron que pasar varios anos para que consiguiera, tras generar y detectar ondas de radio, Este es uno de los contad due Ia argumentacién matemstica se ha adelantado a la investigacion experi Con base en la densidad de corriente de desplazamiento, la RIERA se define como Ma ‘Téngase presente que la corriente de desplazamiento es resultado de campos variables en el tiempo. Un ejemplo comin de tal corriente es la que pasa por tor (@ condensador) cuando se aplica a sus placas una fuente de voltaje alt ejemplo, representado en la figura 9.10, ilustra la necesidad de Ia corriente de miento, La aplicacion de la version estricta de la ley de los circuits de Ampi yectoria cerrada Z que aparece en la figura 9.10(a) resulta en fomeanfs-as= nent i donde / es la corriente a través del conductor y S; Ia superficie plana delimitay | En el caso, en cambio, de la superfice 5, en forma de globo que pasa entre las ph | capacitor cn la figura 9.10(), fovaa [sas porque por S, no fluye corriente de conducci6n (J = 0). Esto es contradictorio, ¥ Ambos casos se utiliza Ia misma trayectoria cerrada L. Para resolver esta conti ° te 3 Reousniese que J = p,0'es-e'eu ves la Gensidad de bor:teite US CGA es preciso incluir la corriente de desplazamiento en la ley de los circuitos de Ampere. La densidad de corriente total es J+ J, Fn In ecuacion (9.23) Jy = 0, de manera que la ecua- cin no pierde validez. Fn la ecuacién (9.26),J— 0, de modo que fuca- [acd 2[ peas 42—7 oan En ambas superficies se obtiene entonces la misma corriente, aunque en S, se trata de co- riente de conduccién y en S de corriente de desplazamiento, Un voltaje de 50 sen 10° FV se aplica a las placas paralelas de un capacitor, con area, de 5 em? y 3mm de separacidn. Caleule la corriente de desplazamiento suponiendo que «= 22. Soluctén: v Daecene¥ aD _ sav Ma a a ae esav_ gay Ty Je+ 8 = SM lo que equivale a la corriente de conduccién, dada por 4a -2-s 147.4 cos 10% nA. Bjercicio 9.4 Bn el vacto, E = 20 cox (wt ~ 50x)a, Vin. Calcule ade De ow Respuestas: a) —20we, sen(wt — S0x)a, Alm’, b) 0.4 we, cos(ot — SOx)a, Alm y ©) 15 = 10% rad/s, |] 9.5. Versién definitiva de las ecuaciones de Maxwell James Clerk Maxwell (1831-1879) es considerado el padre de la teoria elect mo, en la que, ademas de reunir todos los resultados experimentales y tedricos 6 hasta entonces en esas materias, introdujo la corriente de desplazamiento y p existencia de ondas electromagnéticas. Sus ecuaciones no fueron plenament das por muchos cientificos hasta ser confirmadas mas tarde por Heinrich Ri das de radio. En la tabla 7.2, inserta en Ia seccién 7.6, se presentaron las leyes del ele mo pra condiciones extiticns que Maxwell condensé en cimtre eciaciones Sim ‘a aplicacion es la referente a CO) variables en el tiempo, Ia cual se presenta en la tabla 9.1. Como puede observ: table Cional exhiben ciertas modificaciones. La forma integral de las ecuaciones de describe las leyes fisicas subyacentes, en tanto que la forma diferencial es la de. en la resolucién de problemas. Para que un campo pueda “calificarse" como el tico debe satisfacer las cuatro ecuaciones de Maxwell. Es imposible exagetar lai cia de estas ecuaciones, puesto que resumen todas las leyes electromagnéticas hasta la fecha. En lo que resta de este libro se har4 frecuente referencia a ellas ‘Puesto que el proposito de esta seccidn es compendiar el contenido del lib ne citar ecuaciones que van de la mano de las de Maxwell. Se asocian con éstas ecuacidn de la fuerza, de Lorentz, 4 F=O@ +uxB) Inexistencia de cargns magnéticas ais i Tinmbien conocide somo ey le Gaus para cans rami ‘James Clerk Maxwell (1831-1879), fisico excocés, recogié sus estudios en el libro A ii Eleciricity and Magnetism, Dover, Nueva York, 2 vols. 1954. como In ecuacisn de In continuidad (29) Los conceptos de linealidad, isotropia y homogencidad de-un medio material también ‘ables a los campos Variables en el tiempo; en el caso de un medio lineal, homo- son ap! géneo e isotrdpico, caracterizade por ,¢ y 44, las relaciones constitutivas Deb e+e (9.302) B- yH = 1,04 + M) (9.308) 7 (9.306) mantienen validez cn campos variables en el tiempo, lo mismo que, en consecuencia, las condiciones en la frontera Ey= Ey 0 (By —E) Xan =0 (31a) Hy = Hy Ko 0 (HH) ayy - K 1b) Din Da =P, 0 (Dy DD) Maz = Pr (@si6) By By =0 0 (Br — By) - ay, = 0 31d) Sin embargo, respecto de un conductor perfecto! (@ ~ 6) en un campo variable en el tiempo, — s=0 (32) Y por tanto, B,-0, BE, =0 ss) La ccuacién (9.31) también es aplicable a uf dieléctriéo perfects (7 ~ 0), salvo que en este caso K — 0. Aunque las ecuaciones (9.28) a (9.33) no son ecuaciones de Maxwell, ox tn vinculadas con ellas. Para completar esta seccin de resumen, en la figura 9.11 se ofrece una asociacién es- tructurada de los potenciales y campos vectoriales de los campos eléctrico y magnético, Este diagrama de flujo electromagnetico permite visualizar las relaciones basicas entre cantidades de esos campos. Indica asimismo Ia posibilidad de hallar facilmente formu- laciones opcionales para un problema dado. En las figuras 9.11(b) y (c) se introdujo p" co- mo densidad magnética libre (similar a p,) —cuyo valor es de cero, desde luego y A, como densidad de corriente magnética (andloga a 3). Empleando términos del andlisis de esfuerzos, la simbolizacién de las principales relaciones es la siguiente: 4) ecuaciones de compatibilidad ate me. 93a). @ o Figura 9.11. Diagrama de flujo electromagnético que muestra la relacin entre p: J campos vectoriales: (a) sistema electrostatieo, (b) sistema magnetostético, Ce) sist Ursctromapnetico, (Adaptacion autorizada por el IEEE Publishing Department) y vxKE b) ecuaciones constitutivas = ant y DE ©) eeuaciones de equilibrio vD=p y aD veH=s+2> Al referirnos a los campos electromagnéticos estiticos, obtuyimos el potencial eléetrico cscalar como pes v= [ue 40) gy ot potatétat zaagasticp vector al corn _ fuse a- [a eat ‘Examinemos ahora qué ocurre con estos potenciales cuando los campos varfan en el tiempo. Recuérdese que A se definid a partir del hecho de que V - B — 0, lo que también Hige cael casu de los vampos variables on el Gempo. De ah que la relacién B Ze 0.42) sea vélida en situaciones de variacion en el tiempo. La combinacién de la ley de Faraday expresada en la ecuacidn (9.8) con la ecuacién (9.42) da como resultado, vxm e207 x A) 430) vx(n+2) 0 ease) En vista de que el rotacional del gradiente de un campo escalar ¢s idéntico a cero (véase cf ejercicio 3.10), la solucion de la ecuacién (9.43) es nvtho_w as) Bow 2 cas) Con fundamente en las ecuaciones (9.42) y (9.45) es posible determinar los campos vec toriales By E, siempre que se conozean los potenciales A y V.Sin embargo, hemos de ha- lar expresiones de A y V similares a las formuladas en las ecuaciones (9.40) y (9.41) que se adecuen n campos variables en el tiempo. ‘La tabla 9.1 y la ecuacién (9.38) revelan que V - D = p, ex vélida para condiciones de variacién en el tiempo. De la adopcion de la divergencia de ia ecuacién (9.45) y el uso de las ecuaciones (9.37) y (9.38) se obtiene =P a vy 29s vee == -vy—S-ay ° ot vv +2v-ay--& La adopeién del rotacional de la ecuacién (9.42) y Ia incorporacién de las 6 (8.23) ¥ (0.45) resulta en Pe ean neem (a same (t donde se han supuesto D = eR y B = HI. Ta apticacién de la identidad vee VM UMA YA) — 2A, a la ecuacidn (9.47) produce 2a Wan vee ay = nt + ne (22) + ne Un campo vectorial se define inequivocamente cuando se especifican su rot divergencia. En la ecuacién (9.42) ya se ha especificado el rotacional de A: por que se aclararén mas adelante, es posible expresar la divergencia de A como Via RH -ne % Esta expresin, en la que se relacionan A y V, se llama condicion de Lorentz, ciales. Esta condicién se tuvo en mente al optar por V > A — 0 para campos mi ticos en la ecuacién (7.59). Al imponer la condicién de Lorentz de la ecuacién (2 ‘ecuaciones (9.46) y (9.49) se convierten respectivamente en ee roa y PA VA pel = -us e que son écwaciones de dndas que Se explicariin en el capitulo siguiente! La r ber optado por la condicion de Lorentz salta a la vista al examinar las ccuacior ¥ (9:52). Tal condicién disocia las ecuaciones (9.46) y (9.49) y produce una simetrh las ecuaciones (9.51) y (9.52). Es posible demostrar que la condicion de Lorent obtenerse de Ia ecuacién de continuidad: ast, la eleceidn de la ecuacién (9.50) no: traria. Cabe destacar que las ecuaciones (6.4) y (7.60) son casos estaticos especi ccuaciones (9.51) y (9.52), respectivamente. En otras palabras, los potenciales tisfacen las ecuaciones de Poisson para condiciones de variacién en el tiempo. Asi como las ecuaciones (9.40) y (9.41) son las soluciones o formas integrales de las ecuaciones (6-4) y (7.60), puede demostrarse que las soluciones® de las ecuaciones (9.51) y (9.52) son, (play |, 4reR ve 53) (say Et término [p] (© [BD significa que el instante £ en pyCe, 9. 2.0 [0 4x, y-,0] 8 reemplan Ferrer ty A dein panera ps ve po & 233) r’] es Ia distancia entre el punto de origen ® y el punto de observacion ry doride R = |r (9.56) es In velocidad de propagacisn de Ia onda. En el vacto, u = ¢ ~ 3X 108 mys es Ia veloci dad de la luz. Los potenciales Vy A de las ecuaciones (9.53) y (9.54) se llaman potencial eléctrico escalar retardado y potencial magnético vectorial retardado, respectivamente, Dadas py 3, Vy A pueden determinarse mediante las ecuaciones (9-53) y (9.54); a partir de Vy A. Ey B pueden determinarse por medio de las ecuaciones (9.45) y (9.42), respec pe . Campos arménicos en el tiempo Hasta aqus, la dependencia temporal de los campos electromagnéticos ha sido arbitraria Para ser espeeificos, supondremos que los campos son arménicos en el tempo. _ Un campo nemonico en el tempo es ol que varia perie ance ie Seni 3 a Sad nde poe 2 Ademis de ser de vilor priictico, el Jnusoidal puede prolongarse a la mayoria de {as formas de ondas por medio de técnicas de transformacién de Fourier. Los sinusoides son de facil expresion en fasores,con los cuales es muy sencillo trabajar. Sin embargo, an- tes de aplicar fasores a campos clectromagnéticos precisemos el concepto de fasor. ‘Un Jasor z es un numero complejo que puede expresarse como eo biyorZs sr 3 Véase, por ejemplo, D. K. Cheng, Field and Wave Electromagnetics, Addison-Wesley, Reading, MA. 198h pe apt een. re ren h flan 6) z ! donde j = V/A, x es la parte real de c:y es Ia parte imaginaria de zir es de z, dada por Vere r= tel y #es la fase de z, dada por os tant No deben confundirse x, y, z, ry ¢ con las variables de coordenadas, pese a (habrian podido usarse otras letras, pero es diffeil hallar mejores). Fl fasor z my senturse en forma rectangular como z — x + jy y en forma polar como 2 Estas dos formas de representacién de z se refieren en las ecuaciones (9.57) ilustran en Ia figura 9.12. Es recomendable efectuar la adicién y sustraccion, forma rectangular, y la multiplicacion y division en forma polar. ‘Dados los mtimeros complejos zoxtiyor i n=mtimanlh yo ao mth ae cabe poner de relieve las propiedades basicas siguientes. Adicion: ata Gt) +101 +9) ‘Sustraccion: 2 — ta Gi) FIO Multipticacion: } j seca ra Aba | de Division: i B= Dy de Figura 0.12. Representacidn de un fasor == + Rafe cuadrada: Va Vir 02 oie) Conjugade complejo: waxy esp, En el apéndice A.2 se detallan otras propiedades de los nimeros complejos. ‘Para introducir él elemento tiempo, sea enarte (9.62) donde 9 puede ser una funcién de coordenadas temporales o espaciales o una constante, [as partes real (Re) © imaginaria (Im) de relt = reltrehe 63) estan dadas respectivamente por Re (re!) = r cos (wt + 8) (64a) Im (rei#) = sen (wt + 8) (646) ‘Ast, una corriente sinusoidal 4) = , cas(oot + 0), por ejemplo, equivale a la parte real de itne ia corrieate 1'@) = J, sen(ot + 8), la cual es Ia parte imaginaria de Jee", tam= {ge Suede fepresentarse como in parte real de Z,e"e™e >. ya que sen a = cos(a — 90°). eine icar operaciones matematicas, sin embargo, se debe ser congruente en el empleo AU ISSUE SET'G Gmmaginaria de una cantidad: no es posible usar ambas al mismo tiempo: ‘di tecmino compigjo Ze, el cual resulta de la eliminacién del factor de tiempo en 1(), se lama corriente de fasor y es denotado con 1,; es decir, 1, = Ke" = 1.20 es) donde el subindice + denota la forma de fasor de I(1). Ast, 1() = 1s cos(ot + 0), 1a forma Instantanea, puede Expresarse Como 1) = Reem (0.66) {En general, un fasor puede ser un escalar o un vector. Si.un vector A(x,y. 2,1) es un cam= Pa damonico en el tempo, la forma de fasor de Aes A,(x,9.£): la relacion entre estas dos Eantidades esta dada por A= Re (Asem) (9.67) = A, cos(wt — Bx) a,, A puede expresarse como A Re(Age Maem) Si, por ejemplo, A oss) ‘La comparacion de esta expresi6n cén|la ecudciGn (9.67) indica quella forma de fasor de rn ALA Ae ay 69) Nétese que a partir de Ia ecuacién (9.67) BA 8 ncn aie 2A ocaemy ReGaaem) lo que demuestra que tomar Ia derivada temporal de la cantidad instanténe; multiplicar su, forma de fasor por ji, Esto es, 2a nT ik De igual modo, frase a Ba In ecuncion (9.67) ne opi por parts modo del anti te napria pode optntse por in Bate vinghtatin Cons senescent Beaton Entre tn trina natantdnen Aes) y su fora de ses Stee depot taceps yee macenoncne teks sce ces sntnan complies lateconclloaberons 2 soir A a aestr ae a Roh ote ante 8 Seaton tbl Silos alstvarhanttcon ¢] sompo! cer cutdaaae dot Suan 9 SCS ake en a Selig liar g ageor toy wifactintel pata epee ieee a Ins abuacignes O80} yay bons tale 53 ce preteen to Ge are te pee art etic aetna a autos oe erties a a ioreopice y homopence, n'cia' els sade sfscior as ues oe, ca rosie tli aed Tarcantauay por tee easton sored el tector de tote pucde axcutee Sel antici cobporarenontci at fcimpo.cm Habra sido iguntonenue pie poner el acto se ene Sa BSED Scat Magia ea Pestle Tabla 9.2. "Ecuacioties de Maxwell para campos artnicos Srl tempo suponiende el factor de terme = vB o fora [ on vemno fous ~0 Ft hes (soncf Bert mh Ba vam nate, fia fa, + iD) a8 Eyalde los nimeros complejos iG = 18)" oie 1+ 72 +P Les pe » an [a] Solucién: @) Fste problema puede resolverse de dos ma gular o en forma polar, Método 1 (forma rectangular). Sea \eras: trabajando con z en forma rectan. ae donde i (3 — 74)" = el conjugado complejo de (3 — 4) a34j4 (Para hallar el conjugado complejo de un ntimero complejo, sen cada j por ~j.) z= -1+76 wee Qtsyas-14 nats fats = 1G+ 14) = 4 + sete = (1 4 JOA 4 ja) = 3 — ja + fie — 24 “a7 tina y ae a Dare fia De la multiplicacion y division de z por y entre 27 — jd (ractonalizaci6n) se obticne = (54 + 18-27 = ay __ 150 ~ pas =~ Car + flay—at— fia) ~ BP 1a = 0.1622 ~ 0.027 = 0.1644 79.46" Método 2 (forma polar): a= f= 1790" 26-74)" (L=53.13%)* = 5/53.13° 25 = (-1 + j6) = V37 799.46" 6 = (2 +H)? = (VS 226.56")? = 5 £53.13° Por tanto, _ 200°) 53.13" (V7 199.46°)(5 /53.13°) ene : is 1 piadé = Vag 0 = 99:46" = 0.1644 /—9.469 = 0.1622 — j0.027 como se obtuvo anteriormente. ieee ee [z] wa lt+j= V2 448° y a= 4 — 8 = 4V5 Z-03.4° pei a V2 745° 8- We Sbe- Kaos Soveaioe : 2p = V01581 /108.4°/2 = 03976 /54.2° Puesto que A — 10 cos (10% — 10x + 60°) a, ¥ B, = 20//) a, + 10e"* a, exprese A en forma de fasor y B, en forma instantanea. Solucton: A= Re [10100040 a, donde @ = 10%, Por tanto, A= Re [100K60-100 a, oi] = Re (Ae) A, = 106160309 a, si B= 220, 2 tae, = pay + 10cm, Soe ra, + 106, B= Rewer Be Sis San.en ot = ann + 10 008 (at + a, + Wee tnadg.} ane 3 2 fn wt a+-10 oon (+ 25 El campo eléetrico y el campo magnético en el vacio estén dados por = 32 oe (10 © 82) a Vim 14 = 42 cos (10% + Bz) a, Alm Exprese estos enunciados en forma de fasor y determine las constantes #7, y 6 de mane- ra que los campos satisfagan las ecuaciones de Maxwell. Soluciém: ‘ ‘Las formas instantaneas de K y Hise expresan como. EB Re@e), HW = Re Ge") donde w = 10*y los fasores E, y H, estéin dudos por 50 cans = He £,- Demy, En el vaio, py = 0,0, = 0,8 = &¥ 4 = Ht, de modo que las ecuaciones de Mi Ve De2V-B=0 +V-E,=0 VB =4,V-H=0+0-H,=0 Vx Ho +E VKH, = Jone, on vxEs—y Vx B= jon, 5 au E, = — joe, Al sustituir la ecuacién (9.7.2) en las ecuaciones (9.7.3) y (9.7.4) se comprucba mente que se satisfacen dos ecuaciones de Maxwell, es decir, 1a vin, 35 ee) = 0 1a vem, 12 (H,.) =0 Bip Hod) A, ven, =v x (Ze oma,) = LEE cra, De lu suytltucion de tas ecuaciones (9.7.2) y (9:/-/) en Ia ecuacion (9.7.5) se obti JHB 50 IP city = jurn, 22 ten, Hf ~ 50 ae, De forma similar, Ia sustitucién de la ecuacién (9.7.2) en Ia ecuacién (9.7.6) result 50. A, Ee 8 ela, = — footy “i Ha! 2 $0: Bey La multiplicacién de la ecuaci6n (9.7.8) por In ecuacién (9.7.9) produce #3 = (50)? 2 50. = * Rom £50VeJnn 0.1326 [De In division de la ecuacién (9.7.8) entre Ia ecuacién (9.7.9) se obtiene ones Ba t0Vae =e te 2 223.33 «10-7 En vista de Ia ecuacién (9.7.8), 1, = 0.1326, 6 = 3.33 x 10-0 H, = —0.1326, 8 ~ — X10"; s6lo estos valores satisiacen lus cuniro ecuaciones de Maxwell. Ejereicio 9.7 En claire, E = 94 con (6 x 107 — prjag Vim. Halle By H. 3 "= 0.2r)u, = 354 sen 0 x Respuesta: 02 rad/m, 1,1 cos 0sen (6 X 10% = 02h)a, — ° 08 (6 < 107 —0.2r)ay Alm. En un medio caracterizado por o = 0,40 = ft £5 ¥ HE =20sen (10% — fz)u, vin caleule By A. Solucién: Este problema puede resolverse directamente en el dominio temporal o con el empleo de fasores. Como en el ejemplo anterior, 8 y Hse determinan mediante el recurso de «ie Fy satistagan las cuatro ecusciones de Maxwell. Método 1 (dominio temporal). Resolvamos este problema del modo mas dificil: en el do- minio temporal. Ps evidente que se satisface la ley de Gatiss para campos eléctricos; esto ¢s, Ey v-n--0 Conforme a la ley de Faraday on 1 vxme nw 4 wet ferxmer Pero nem |= 2 o| PE, VIET lo. ce /0 az * ox = 208 cos (10% — Bz)a, + 0 Por tanto, w= 228 feos 10% ~ 2) dae 208 Fibs sen (10% ~ iz) a, ‘Se comprueba facilmente que at ven = th to aie daminanton einen utes in ny ds Chinen pare cummpon magnétions mn ain eas vxnecese® 4 B-2 lexi porque o = 0. Pero. donde se ha sustituido HI de la ecuacién (9.8.1). Ast, la ecuacién (9.8.2) se convi 2 = 228, [oon aot — as) dt, _ 2067 ~ (2B, ten 10% — A) a, La comparacién de esta expresién con el E dado resulta en 9.7. CAMPOS ARMONICDS EN EL TIMP 399 10%) 02) SNES ea Sei A partir de la ecuacién (9.8.1), 20 (2/3) Hm getaor ry een (10% = Fm sree si sen (10 Método 2 (empleo de fasores). E=Im(e) > 83) donde » = 10% De nueva cuenta, vee, ==0 vxeE, VxE, =~ = Sy = Jone, a m= ead Notese que se satisface V +H, = 0. Vx Hy = jock, > 85) La sustitucién de H, de la ecuncion (9.8.4) en la ecuacién (9.8.5) da como resultado ge, = 1b tHe, 208 jon 02 ~~ wie De la comparaci6n de esta expresién con el E, dado en la ecuacién (9.8.3) se obtiene oe muy BR tw = como se obtuvo anteriormente. Con base en la ecuacién (9.8.4), el a > TORGar x 1077) 8 Sa = Im Ce") ee Eden tO a Be) a, arm como se obtuvo anteriormente. Como puede advertirse, trabajar con fasores mie concillo que hacerlo dircetamente on el dominic (euspenal: Noteve wsiznismo. Pleamos ~ A=Im (Ace) Porque el E dado esta en forma de seno, no de coseno. Habriamos podido usar A= Re (Ae™), en cuyo caso el seno se expresa en términos de coseno y Ia ecuacicin (9: IE = 20 cos (10% — 2 — 90°) a, = Re (Ee™) 3) serfa B, = 20€-=-P a, = —200-/a, tras de lo cual se sigue el mismo procedimiento. (UIRERFREAYY 1. En este capstuto se presentaron dos conceptos fundamentales: ta fuerza ele: Enis Us), Deaada on chpectimentos de Faraday y Id corhonta as debpiomnenta Sultante de hipotesis de Maxwell. Estos conceptos tmponen modificacion ecuiaciones del rotacional de Maxwell obtcnidas para campos electron estaticos, com objeto de incluir en cllas Ia dependencia de lee campos renee Sempo, 2. La ley de Faraday establece que la fuerza electromotriz inducida esté dada por ( aw as i hd i AGE tei dcttcn, Ve, = [SBSH Yc cole ats ruecs essemoertastion Va= [een at La corriente de desplazamiento tim faeces Joy a ke tirctins Se, Arnphee costa lua Maral pedo ar Coan olectroeaayt carlos afios ante do que furran contimuidan erperizentatmente por Hartz amiento), es una modificaci6n de la En su forma diferencial, las ecuaciones de Maxwell para campos dindmicos son: v-D=p, v-B-o oB yxeu 2 2D yxu-3+2 Estas ecunciones diferenciales tienen sus correspondicntes formas integrales (véanse las tablas 9-1 ¥ 9.2), las cuales se deducen de aquétlas aplicando el teorema de Stokes © de la divergencia, Todo campo electromagnético debe satisfacer las cuatro ecuacio- hes de Maxwell Fl potencial eléctrico éscalar variable en el tiempo V(x, y. z, 0) y el potencial magnéti- co Vectorial variable en el tiempo A(x y, z, 1) satisfacen comprobadamente las ecua clones de ondas si se cumple la condicién de Lorentz. Loe campor armaniras on al Gempa son lot que varfan sinusaidalmente an al tampa Se les expresa facilmente en fasores, con los que es muy sencillo trabajar. Empleando Ia referencia del coseno, la cantidad vectorial instanténea A(x, y. 2,4) se relaciona con su forma de fasor A,(x, y, 2) de acuerdo con AG.9. 2,0 = Re[A.Ge, y, 2) ole El flujo a través de cada vuelta de una bobina de 100 vueltas es de (e° ~ 21) mWb, donde tse mide on togundos [a fuerza eldetromotriz inducida en #2 sex aiv b) ~1v ©) Amv a) 04 ©) -04V J Bereciente Bdecreciente Figura 9.13. Para la pregunta de rr fae Cl 92, O38. 1s0zva, Wh/m®, como se ilustra en la figura 9.15. V; no es igual a Vs. @) Ciento b) Falso eo Bigorn 9.14, Para in pregunta de repaso 93. Tr @ © Leg [7 © @ Suponiendo que las espiras son estacionarias y que el campo magnético B varia induce corrionte J, cuales de las configuraciones que aparecen ef la figura 9.13 son Dos bobinas conductoras 1 y 2 (Idénticas salvo por el hecho de que In bobina 2 esté Ga) se sittan en un campo magnético uniforme que decrece un indice constante, muestra on Ia figura 9.14, Stel plano en el que se encuentran las bobinas es perper las Iineas del campo, (cual de los siguicntes enunciados es cierto? 44) En ambas bobinas se induce fuerza clectromotriz. 1) Se induce fuerza electromotriz en la bobina 2, 1a bobina fracturada. ‘¢) En ambas bobinas ocurre igual calentamiento en joules. 4) Bn ninguna de las bobinas ocurre calentamiento en joules. Una espira rota alrededor del eje y en un campo magnético B= B, sen or a, voltaje inducide en ella se debe a 4) Fuerza electromotriz cinética, 2) Pucenn cloctromoteis eatation ©) La combinacién de fuerzas electromotriz cinética y estatica. 4) Ninguna de las eausas anteriores, ‘Una espira rectangular se localiza en el campo magnético variable en el empo B © © Figura 9.18. Para ln pregunta de repaso 9.5 y el problema 9.10. Ter 2 © oY © ° El concepto de corriente de desplazamiento fue unit importante contribucién de a) Faraday, b) Lenz ©) Maxwell 4) Lorentz. 2) El profesor de este curso, Identifique entre las expresiones siguientes las que no son ecuaciones de Maxwell para cam- eb cabled an howtiocs @ v-5+% a0 vp p re fiat f (8a Sat) can 9 facasso Se dice que un campo clectromagnético no existe o no es maxwelliano si no satisface las ecua- clones de Maxwell y las ecuaciones de ondas deducidas de cllax: Cuales de los siguientes ‘campos en el vaclo no son maxwellianos? 2) = conn con 10%, 2) = 100008 wen, ©) D= «acm (0! — 109) a, @ = 04 son 10%, coe (20% — 5a, $28 co (we — ree Vinee) So .: ee on Den - o. | Problemas_y 9.9. ,Cusl de lox enunciados siguientes no es cierto con relacién aun fasor? 1a) Puede ser un etealar o un vector ) Bs una cantidad dependiente del tiempo. ©) Un fasor V, puede representarse como Vg /00 Vee, donde Ve — IV4| 4) Fs una cantidad compleja. 9:40. Sik, = 10 2 a,, zcudl de Ins siguientes no es una representacion correcta de E? @) Re (em) b) Re (Beh) ©) Im (ee) 4) 10 cos (ws + J4x) a, ©) 10sen (wr + 4x) a, Respuestas: 91b,9.2b, d,9.3a, 9.Ac, 95a, 9.66, 97a, by das, 94D, 9.90, €,9:10d. 9.1. Una espira conductora circular de 20 cm de radio se sitdia en el plano z = O.en wn nético B — 10 cos 377¢ a, mWb/m?, Caleule el voltaje inducide én ella. 9.2 Una varilla de longitud € gira on torno al eje £ con una velocidad angular 81 calcule cl voltaje inducido en el conductor. 9.3, Una espira rectangular de 30 por 40 em gira a 130 rad/s en un campo may Whim normal al eje de rotacién. Si tiene 50 vueltas, determine el voltaje induel 9.4, Ein Ia figura 9.16 aparece una espira conductora de 20 em? de dre y resistenel B40 con 10%, mWb/m’, halle la corriente indueida em ella e indique su di 9.5. Determine la fuerza electromotriz inducida en Ia espira en forma de V que se figura 0.17, a) Adopte B= 0.1, Whim" y «™ 2a, may suponga que Ia varilla pone en movimiento en el origen cuando t ~ 0.) Repita el inciso 2) si B~ 0. Figura 9.16. Para el proble +97. 99, Figura 9.17. Para el problema 9.5. ‘Una espira cusdrada de lado a retrocede n una velocidad uniforme ua, desde un filamento de longitud infinita portador de corriente [lo largo de a, como se mucstra en Ia figura 9.18, Suponiende que pp, en el momento ¢— 0, demuestre que la fuerza clectromotriz induci- da‘en la espira ens > Oe atl tol Vg meet rae + a) Una varilla conductora se mueve a una velocidad constante de 3a,.m/s en paralelo a.un cable recto y largo que porta una corriente de 13 A, como se ilustra on Ia figura 0.19. Caleute La fuerza clectromotriz inducida en la varilia ¢ indique en cual de sus extremos ocurre mayor potencial ‘Una barra conductora esta conectada con conductores flexibles a un par de rieles en un campo iagnético B= 6 cos 10r 9, mWb/m:, como se muestra en la figura 9.20. Siel eje © es 1a posicion de equilibrio de Ia barra y la velocidad de ésta es de 2 cos 10r a, mis, halle el voltaje Inducido en ells Un automévil vinja a 120 km/h. Si el campo magnético terrestre es de 4.3 10- Wo/m*, ha- lic el voltaje inducido en la defensa del auto, de 1.6 m de longitud, Suponga que el ngulo en- tre el campo magnétice terrestre y Ia normal al auto es de 65" 9.10. ‘Si el Grea de ta expira quc se presents en In figura 9.15 ex de 10 em? caleule V, y Vs = Figura 9.18 Para el problema 9.6. Figura 9.19. Para el problema 9.7. asa, ; .11, Tal como se observa en Ia figura 9.21, una barra imantada es lanzada hacia el bobina de 10 vueltes y resistencia de 15 (1, Siel flujo magnatice a traves de la de 0.45 Wb a 0.54 Wh on 0.02 s, geudles sn Ia magnitud ¥ direcelon (consideradas Jado cercano a la barra) de la cortiente inducida? 9:12. En Ia figura 9.22 aparece Ia secci6n transversal de un disco gencrador homopolar terno p; — 2.em y radio externo p, ~ 10 cm que gira en un campo magnetica Uni 15 mW/m? a una velocidad de 60 rad/s. Caleule el voltaje inducido. 9.13. Las placas paralelas, de 2.8 em? de area y distancia de separacién de 0.2 mm, de un con aire como dicléctrico estin conectadas 4 un generador con voltaje de 30 V a ‘alle cl valor maximo de la densidad de corriente de desplazamiento y de la corr plazamiento. _ 9-14, La razon JJ, (densidad de corriente de conduccién a densidad de corricnte de to) es muy importante a altas frecuencias. Caleule el valor de esa razon 8.1 GHz en 4) agua destilada (J4 ~ phy, ~ Sle, o7 ~ 2 < 10-* Sim) by agua de mar (jt = [ty, © = 8184; 07 = 25 Sim) 3 ©) piedra caliza Gu = py = 584, 2X 10° Simm) 9.15. Suponiendo que el agua de mar tiene 42. fly, & ~ S16," = 20 Sim, determine la cla cn la que la magnitud de la densidad de corriente de conduccion cs 10 veces, de Ia densidad de corriente de desplazamiento, Figura 9.20. Para el problema 9.8, 9.16. oar. sas. 9.9, 9.20. ‘una corriente de con ‘Un conductor chya sccciGn transversal pose un fren de 10 em’ POM CIcuie Ia magnitud de Suosion de Oz sen 10" mA. Puce que 23 1g Sie y He = In densidad de corriente de desplazamicnto, .co en términos de E, ‘@) Facriba las eounciones de Maxwell pila’ tin iieidio lineal y bomoke™ YH, suponiondo sélo el factor de tiempo ¢™. E 1 a for ) Esctiba en coordenadas cartesianas y ocho ecuaciones seals! ccunciones de Maxwell referidas en la tabla 9.2. sma puntual de tas jones de Maxwell pueden Demuestre que, on una regién sin origen (J — 0, p, = 0). las ecu reducirse a dos: Identifique esas dos ccuaciones globales omogéneo € isotropico Demuestre que Ia densidad de carga p, de un conductor lineah a) Suponga una repisn sin ofiiten y deduzea la ecuacién de difusio” we = no - jerna 9.12. mane rH CO 9.21. fn cierta rexisn, 4 = Qya, + x2, + Payson 10% Alm Malle p, si px. 9.0.0) = 0. 9.22. Tin una region sin carga en In que o = 0,e = 5 cos(10ie — ay), Alm eee ye Halle: a) 349 D,b) 6, 9.23, En cierta region con o = 0.1 = ty ¥ © = 6.25e,,¢l campo magnético de una magnetica er HL = 0.6 cos Bx cos 10% a, Alm Determine fy el E correspondiente mediante Ins ecuaciones de Maxwell *9.24. En un medio no magnético, B= 50 cos(10" — avya, + 40 sen(10% ~ Bx)a, Vim Halle la constante dieléctrica e, y el H correspondiente. 925. Compruche si lox campos siguientes son campos electromagnéticos gentiinos; Satisfacen las couaciones do Maxwell. Suponga que existen on Tesiones sin cnr 2) A= do sonlor + 108), 0) w= Peowat ~ 20a cost 2), 6m (sotcottey + S820, son 0 Dd oble *°9.26. Dads ia eneraia electromagnetics total wa 3 forge me 1 demuestre con base en las ecuaciones de Maxwell que we ae ~ fe myeas — [esa 9.27. En el vacio, H= p(sen dha, + 2.608 dom) com 4 10% Alm es 928. 933. os. ‘Una antena radia en el vacio y = Bone ‘cos(2er < 10% — Br)apmA/m Determine 61 E correspondiente en términos de B. Et campo eléétrico en el aire esta dado por E = pte~*~‘ag Vim: halle By J. En el vacio (p, = 0.3 = 0), demuestre que A= EE (cosoa, — sen 0 meh satisface la ecuncién de onda dada en la ecuacién (9.52). Halle el V correspondiente. Con. Sidere ¢ como Ia velocidad de la luz en el vacto. Evalde los siguientes nimeros complejos y exprese sus respuest a) (4 230" — 10 290°) 1372 ”) ora 105: + jay O Ta 71 + Ce + floy* (ie 2-200) Ga ass + Escriba como fasores los siguientes campos arménicos en el tiempo: cen forma polar: 5 a a) B= A .contwt — 3x — 10°), — sen (or + 3x + 20")my by = PF casleor = Site . €) B= 6e-% senor — 2x), + 10e—* cos(we — Sx)m, Exprese los fasores siguientes en su forma instanténea: A= = 3 Se ee © Gn 12. + pe son te Dados A = 4sen ota, +3 cos wma, y B, forma instantanes. nOze/a,,exprese A en forma de fasor y B, on ‘Denucsize que en una fegién lineal, homogénea, isotropica y sin fuente, tanto E, como Hi, ‘ben satisfacer Ia eeuacion de ondas VA, + PA, — 0 a ee 10 Propagaci6n de ondas electromagnéticas Llegarés muy lejos en Ia vida si eres amable con los jévenes, compasive can los anc os, solidario con los esforzados ¥ tolerante con los debiles y los fuertes. Porcue alg dia te contards entre todos ellos, Grosce W. 10.1. Introduccién. Aplicaremos inicialmente las ecuaciones de Maxwell # Ia propagacién de ondas magnéticas, Predicha por tales ecuaciones la existencia de esas ondas fue omy por Heinrich Hertz. Luego de varios céleulos y experimentos, Hertz logré gene tectar ondas de radio, lamadas en su honor oridas hertzianas. Bjemplos comunes de ondas electromagnéticas son as ondas de radio, las sen: levision, Ios haces de radar y los rayos luminosos. Todas las formas de energia eles nética comparten tres caracteristicas fundamentales: se desplazan a gran ve Adoptan al hacerlo propiedades de ondns ¢ irradian hacia fuera desde una fuent ayuda de ningun vehiculo fisico discernible. Fl problema de la radiacion se abi el capitulo 13. El principal objetivo de este capitulo es resolver las ecuaciones de Maxwell y cir el movimiento de las Ondas electromagnéticas en los siguientes medios 1. El vacio ( = 0,¢ = ey, 4 = Ha) 2! Dieléctricos sin pérdidas (@ "0, ¢ = b,25. 44 = pyttg 0 7 SS 008 2. Dicléctricos disipativos (2 # 0,« — ,es 1 = pone) 4. Buenos conductores (a = 00,0 = 85.4 = 1,40, 00 > ane) donde @ es la frecuencia angular de la onda. Por ser el caso mas general, nos ocup: en primer término de los dieléctricos disipativos, de los que sencillamente de mos los casos especiales 1,2 y 4 modificando los valores de 7, # y J. Sin embargo, examinar el movimiento de ondas en esos diferentes medios, analizaremos las carad ticas generales de las ondas, esenciales para comprender las ondas clectromagné! lector versado en el concepio de ondas puede omitir el estudio de Ia seein 10.2. seeciones finales de este capitulo se expondran consideraciones de potencia, reflexion y transmision entre dos medios distintos. 2. Estudio general de las ondas El detallado conocimiento de In propagacién de ondas electromagnéticas implica el de Jas ondas en general, peeeueemcus puny mz conus “Tina onda es ana funcidn tanto, Qeurre movimiento de‘ondas cuando una perturbaci6n en el punto A en el instante ¢, se Felaciona con lo que sucede en el punto B en el instante / >"f,, Una ecuacion de onda, como las ejempliticadas por las ecuaciones (9.51) y (9.52), es una ecuacién diferencial parcial de segundo orden. En una dimension, una ecuaciGn escalar de onda adopt la forma de HE. PE an act donde wes la velocidad de onda. La ecuacin (10.1) es un caso especial de Ia ecuaci6n (51), en la que el medio earece de fuente (p, = 0,5 — 0). Se le resuclve siguiendo un, procedimiento similar al que se describio en cl ejemplo 6.5. Sus soluciones son de Ia o aoa) forma B= fle us (10.20) EO ga + un) (10.26) BAS SHO tle HO 025 donde fy g denotan cualquier funciGn de z — uty 2 + ut, respectivamente. Son ejemplos de tales funciones z= ur sen k(z = un),cos k(z = uf) y wits), donde k es una constante. Podria demostrarse facilmente que todas estas funciones satisfacen la ecuacion (10.1). Si se adopta en particular la dependencia de tiempo arménico (© sinusoidal) ela ecuacién (10.1) se convierte en @E, az + PE, =0 aos) donde f = alu y E, es la forma de fasor de B. La resoluci6n de I ecuaci6n (10.3) es se- micjante al caso C del ejemplo 6.5 [véase la ecuaci6n (6.5-12)} Habiendo insertado el fac- tor de tiempo, las posibles soluciones de la ecuacién (10.3) son Et & Acie 90 aoa) E- = Belk 00) (10.40) y B= Ache 20 + pelo donde A y B son constantes reales. ‘Consideremos por ¢l momento la solucign formulada en la ecuaciSn (1044) ma la parte imaginaria de esta ecuacidn se obtiene E = Asen (wr — 82) Se ha optado por esta onda sinusoidal en razén de sti simplicidad; de la cle. arte real de la ecuaciGn (10.44) habrfa resultado una onda cosinusoidal. Repai iguientes caracterfsticas de In onda expresada en In ecuacién (10.5): 1. Es arménica en el tiempo, ya que para arribar a tal ecuaci6n se adopte dencia del tiempo 2 Acs la amplitud de la onda, de unidades iguales a las de F. (or ~ Bs) es la fase (en radianes) de In onda; depende del tiempo fy de Ble espacial z. 4. wes Ia frecuencia angular (en radianes/segundo) y B la constante de fave ro de onda (en radianes/metro) 2 3 Dada su variaci6n tanto con el tiempo 1 como con la variable espacial 2, scntarse grificamente como una funcion de manteniendo constante = ¥ viceve figuras 10.1(a) y 10.1(b) aparecen los diagramas de £(z,¢— constante) y Et. 2 tante), respectivamente. Fn la primera de ellas se observa que la onda tarda cn una distancia A, 1a que por este motivo recibe el nombre de longittad de onda Ce En la segunda, la onda tarda en repetirse el tiempo T, el periodo (en segundi que para que la onda recorra la distancia 4a la velocidad t transcurre el ti Suponer que Awur Pero T = Uf, donde fes la frecuencia (el namero de ciclos por segundo) de Ia hertz (Hz). Asi, uf Por efecto de esta relaci6n fija entre longitud de onda y frecuencia, la posicién de tacion de radio en su banda puede identificarse con una u otra, aunque sucle pi Ia frecuencia, Asimismo, a causa de que o = 20f Bed y rela? © Figura 101, Diagrama de E(e,0) = A sen(wt ~ 62): (a) con # constante, (b) con = constante, de las ecuaciones (10.6) y (10.7) es de esperar que ‘ae a- 2m aoe) La ecuacion (10,8) indica que cualquiera que sea la distancia comprendida por su longi tad, un onda sufre un cambio de fase de 27 radianes. ‘Demostremos ahora que la onda representada por la ecuaci6n (10.5) se desplaza a una velocidad u en la direccién +z. Para hacerlo se considera un punto fijo P en la onda y se traza la ecuaciOn (10.5) en los instantes ¢~ 0, 7/4 y 7/2, como en la figura 10.2. En Esta es cvidente que el punto P se mucve a lo largo de Ia direccion “+z a medida que la ‘Onda avanza en el tiempo. El punto P es un punto de fase constante, de manera que (ot — fz = constante dz Gag a0.) p Figura 10.2. Disgrama dh EG.) =A sen(eot — fie) instantes (a) #— 0, ro we a ccuacién equivalente a la ccuacién (10.75). La ecuacién (10.9) indica que la 0} plaza a-una velocidad wen la direccion +z. De la misma forma podria demostr Ia onda B sen (wr + 2) representada por la eeuacion (10.45) se desplaza a dad w en la direecién ~2. En suma, cabe sefalar to siguiente: 1. Una onda es una funcién tanto del tiempo como del espacio. 2 Nets fin el iustannie ¢ = Ove elige arbitrariamente como, referencia. 3. Cuando el signo de (wt + 62) es negativo, la propagacién de la onda ocut direceién +2 (onda de avance © de marcha positiva); cuando ¢ positive, gacicn ocurre en la direccion —z (onda de retraceso o de marcha negativa) 4, Puesto que sen (4) = —sen = sen (y, * 7), mientras que cos(—W). sen (Wh © /2) = = cos w sen (ih =m) = sen 0s (i + /2) = + sen y cos (= 2) = — cosy donde yy = wr + Bz. Mediante la ecuacisn (10.10), toda onda arménica en el tie de representarse en forma de sefio © coseno. TFenomenon eloctromannsticos Bjemplos de sos Tnervato de frecudncn aproimedo Rae genns” {Terepia contra et edncer toe tome Rayos X. 1o'—10" Gre via foie anie Rotwgratia iol Gre Radares relevidorts de 3300 one ‘microondas, comunieaciga, Radioondas selevisign URE 470-806 Matte La clasificacién de mdltiples frecuencias en orden numérico constituye un espectro. En Ia tabla 10.1 se detallan las frecuencias en que ocurren diversos tipos de energfa en el Cspectro clectromagnético, Las frecuencias utiles para la comunicacidn por radio ocurren, Cerca del extremo inferior del espectro. Conforme la frecuencia aumenta, la manifesta- cidn de energia clectromagnética comporta riesgos para los seres humanos.’ Los hornos de microondas, por ejemplo, pueden ser peligrosos sino se les blinda adecuadamente. Las dificultades practicas para el empleo de energia electromagnética con fines de comunica- Gion también aumentan al incrementar Ia frecuencia, al grado de volver imposible el uso de tal cnergia. No obstante, el limite de la frecuencia utilizable se ha elevado gracias a micjores métodos de comunicacién. Hoy los satélites de comunicacién operan con fre~ Cuencias proximas a los 14 GHz, Esta frecuencia esta aun muy por debajo de la de la luz, Ia que sin embargo ya se emplea para la radiocomunicacién en el restringido ambit de la fibra optica” El campo eléctrico en el vacfo esté dado por E = 50 cos (10% + Ax) @) Halle la direecion de la propagacién de la onda. }) Caleule By el tiempo que tarda en recorrer una distancia de 2/2. ¢) Trace la onda ent = 0, T/4 y T72. Soluciém: @) Del signo positivo en (ot + Ax) se infiere que la onda se propaga a lo largo de — Esto se confirmara on ol inciso 6) de este ejemplo. 1 Véase a edicién especial de marzo de 1987 de la JERE Engineering in Medicine and Biology Magazine sobre "I:fectos de la radiacion electromagnética” 2 Vase la edicion de octubre de 1980 de [EEE Proceedings sobre “Cominicaciones mediante fibr éptica”. 2) Bn el vacto, u 108 pote. ad 73x10 "3 (6 = 0.3333 rad/m Si Tes el periodo de la onda, ésta tarda 7-segundos en recorrer una distancia 4. locidad c. De ahf que en recorrer una distancia de A/2 tarde T liz ye Da 52m oF = 14a ns ‘Opcionalmente, y a causa de que la onda viaja a la velocidad de la luz c, eee gb» dee 9 ned Pero 2m an 2 = on Por tanto, on 46 5g Sap — 3142 as como se obtuvo anteriormente, En p= 0, B= 50cos ae Ba 1 = 114, By = 500s («+22 + px) = 5060s (Bx + 0/2) = —50 sen ax En 15 72,8, 7 50608 (w 37 + px) ~ 500s (ox +m) = —50 cos Bx Ey en t = 0, 74, 72 se thaza contra x, como se observa en Ia figura 10.3. Ni pinto P en la onda (seleccionado arbitrariamente) se mueve a lo largo de mentarse 1,10 que demuestra que la onda se desplaza a lo largo de —a, Figura 10.3. Para el ejemplo 10.1; In onda 26 despinza a lo largo de —a, “En el vacto. Ht = 0.1 cos (2 10% eed, Alm ob a) Caleule Bay TI) AT 2) Determine el tiempo f, que 1a onda tarda en recorrer A/8. Eyromeaath ots clsmacapecoacet sien ti ‘vouboig (EOD) 1) 0.667 rad/m, 9.425 m, 31.42 ns, b) 3.927 ns ye) véase la figura 10.4. 10.3. Propagacién de ondas en dieléctricos disipativos ce ic ‘Como se mencioné en la secci6n 10.1, la propagacién de ondas en dieléctricos disipativos es un caso general del que pueden dedicirse los casos especiales de la propagacion de ‘ondas en otros tipos de medios. Por tanto, est seeciOn servira de fundamento a las tres ler soocones posteriones, } { i ince imperfecto o conductor imperfecto) en el que a #0, s Uierencia de uh sin pérdidas (dieléctrico perfecto © buen dieléctrico), en el que « — 0. ‘Considérese un medio dieléctrico disipativo lineal, isotrépico y homoge (, = 0). Tras adoptar y suprimir el factor de tiempo elas ecuaciones de Mi Ia tabla 0.2) se convierten en. VrE,=0 v-H,-0 VX EB, = “joe, Vx H,= (o + jose, Al tomar el rotacional de ambos miembros de Ia ecuacién (10.13) se obtiene V KV XE, = jon ¥ OH, La aplicaci6n de la identidad vectorial VX VX A=V(V-A) WA al miembro izquierde de la ecu: (0.14) produce ae AVE, = jane + jon), VE, — YE, =0 1m (10.15) y Ia invocacion de las ecunciones donde P= jane + play Loy donde ¥ #8 In constante de propagacién (por metro) del medio. Siguicndo un procedi Iniento semejante, es posible demostrar que en cuanto al campo HM, WH, — YH, = 0 0.19) “Lag ecuaciones (10.17) y (10.19) son las ecuaciones vectoriales homogéneas de Helmholtz, 6 ecuaciones vectoriales de onda, La ecuacion (10.17), por ejemplo, equivale en coorde” is cartesianas a tres ecuaciones escalares de onda, una por cada componente de Ea Jo largo de a.m, ¥ a ‘Puesto queen las ecuaciones (10.17) a (10.19) y es una cantidad compleja, conceda- mos que =+ ie (40.20) ey P se obticnen de las ecuaciones (10.18) y (10.20), en el entendido de que Re = 9 — a? = one ao21) IPA) s+ ot = om Vos oe (40.22) Dé las ecuaciones (10.21) y (10:22)'se obtiene suponemos que Ia onda se propaga a lo largo) Sin menoscabo de la generalizacion, de fay que B, solo cucuta von ha cos r, 40.25) La sustitucion de esta expresion en Ia ecuacién (10.17) resulta en (= PEA) (10.26) PR Az) , Pfalz) , OBZ) $e + ee PEe®) — Yes) 3 3 ié Por consiguiente, (10.27) ¥ lew i La solucion de esta ecuacién escalar de onda, una ecuacién diferencial lines ces (véase el caso B del ejemplo 6.5) ji E,,fz) = Bye" + Exe" | donde £, y £% son constantes. FI hecho de que el campo deba ser finito en el i pone que £{~ 0. Opeionalmente, ya causa de que e”* denota una onda que flo largo de —a, mientras que suponemos que la propagacién de Ia onda go de a,, Z5— 0. Desde cualquier punto de vista, asf, A= 0. De la insercisn tiempo een la ecuacién (10.28) y el empleo de la ccuacién (10.20) se Obti BG, 9 = Re [E.(2)e™a] Re (Eye “el 2a, En la figura 10.5 se presenta el diagrama de [| en los instantes ¢ = 0 y ¢ = Af Vista que E slo cuenta con la componente x y se desplaza a lo largo de la dir Hlabiendo obtenido E(z, #), H(z, 0) se obtiene con pasos similares mediante In (410.19) 0 aplicando Ia ecuscién (10.29) en combinacién con las ecuaciones de como se hizo en el ejemplo 9.8. De un modo u otro se llega finalmente a Wz.) = Re (te elt B2n,) donde E Hy ” y donde 7 ¢s una cantidad compleja conacida como impedancia intrinseca (en Inedio. Siguienda los mismos pasos que en el cjemplo 9.8, es posible demostrar: jou eh jos ” InlZen= Inte Slo largo de In direecion +2 Tos instantes ¢= Oy t= At as Indican valores instantaneos ia tam 20q = 10.33) ral eta donde 0 = 6, = 45°. La sustitucién de las ecuaciones (10.31) y (10.32) en Ia ecuacisn (40.30) da como resultado Vale ey emt Pay] Bs eat cos(wt — Pz — On) @y 0.34) De las ecuaciones (10.29) y (10.34) se desprende que conforme Ia onda se propaga a lo largo dea, su amplitud decrece o se atenta en un factor e~=", motivo por el cual a re be el nombre de constante de atenuacion 0 factor de alenuacton del medio. Esta constan- te mide el indice espacial de Ia declinacion de la onda en el medio y se enuncia en nepers por metro (Np/m) o en decibeles por metro (dii/m). Una stenuacion de 1 neper equiva Tea una reduccion dee! del valor original, mientras que un incremento de 1 neper eq vale a un aumento en un factor de ¢. En el caso del voltaje, asi, Np = 20 logio ¢ ~ 8.686 dB. 03s) En cuanto a la ecuacién (10.23), vale hacer notar que si = 0, como es el caso tanto de un medio sin pérdidas como del vacio, a = 0, de modo que la Onda no se atentia al pro- pagarse. La cantidad ff es una medida del corrimiento de fase por longitud y se Hama Constante de fase o numero de onda. En términos de B, la velocidad de onda wy Ia lon: tud de onda A estan dadas respectivamente por [véanse las ecuaciones (10.76) y (10.8)] Beas ZZ (0.36) Respecto de las ecuaciones (10.29) y (10.34) es posible observar asimisme que Ey H. estan fuera de fase por 0, en cualquier instante, a causa de la impedancia intrinseca com- pleja det medio, n cualduier momento, asf, E se adelanta a Hi (o Hse rezaga de E) por 0,, Sefidlese por ultimo que, en un medio dad de corriente de conduccion J # la magnitud de la densidad de corriente de desplaza- miento J, es IL low Vaal ~ Tet] tno oar tan @ donde tan 0 es la tangente de pérdida y 0 el dngulo de pérdida del medio, Ia figura 10.6. Aunque no es facil trazar una linea de demarcacin entre buen res y dieléctricos disipativos, tan 0 0 0 pueden usarse para determinar cusn dis medio. Un medio es un buen dieléctrico (sin pérdidas o perfecto) si, en su de tan @ es muy reducido (a ~* we), y un buen conductor si tan é es muy all Desde el punto de vista de la propagacién de ondas, el comportamiento ca un medio depende-no s6lo de sus pardmetros constitutivos o,¢ y 1, sino frecuencia de operacion. Un buen conductor en bajas frecuencias podria ser Iéctrico en altas frecuencias. Fn lo que se refiere a las ecuaciones (10.33) y (I 0=20n Con base en la ecuaci6n (10.14) eel ie — fore jock, donde ees et je" ye! = 2,0" = afm, mientras que'e, es la permitividad compleja del medio, Advi Ta raz6n de «a #! os In tangente de pérdidn del medio: es decir, tan En las secciones siguientes se examinard la propagacién de ondas en otros. medios, casos especiales del considerado en esta secciGn. Por tanto, de las formal nidas para este caso general deduciremos simplemente las que rigen en aquellos comienda al estudiante no limitarse a memorizarlas, sino observar ademds ¢ obtencisn a partir de las formulas relativas al caso general, . igura 10.6. Angulo de pérdida de un me oe disipative, léctricos sin pérdidas En un dieléctrico sin pérdidas, << we, Este ex un caso especial del referide en la seceién, 10.3, salve que = Hatt (0.42) Al sustituir estos valores en las ecuaciones (10.23) y (10.24) se obtiene @=0, B= aVue 0.43a) He Bee an 0.436) a-JfEae cao.aay de manera que Ey Ht comparten la misma fase temporal. Asimismo, ;. Ondas planas en el vacio Este es también un caso especial del considerado en Ia secciGn 10.3. Fsta vez, C0 Meet BoB 0.45) sta situacion puede interpretarse asimismo como unt case especial del descrito ex I see cidn 10.4. En consecuencia, basta reemplazar © por €, ¥ 4 Por 4a en la ecuacin (10.43) 0 Sustituir directamente a ecuacin (10.45) en las ecuaciones (10.23) y (10.24). De una u otra forma se obtiene (10.46a) Vem Ane 0.460) donde ¢ ~ 3 = 10% m/s, la velocidad de Ia luz en el vaefo. EI hecho de que las ondas elec tromagnéticas viajen en el vacio a la velocidad de la luz es importante, pues indica que la luz es manifestacion de una onda electromagnética, En otras palabras, la luz es propia- mente clectromagnética. Al sustituir los parémetros constitutives de! Ia ecuacién (10.45) eit (2033), 0, OY 7 ~ mu donde 7, es la impedancia intrinseca del vacto y esi na = [2 = 1200 = 3770 E = £, cos(wr ~ Bz) 0 E. HE = 11,008 (wt — Bz) a, = 22 costwt — Bz)ay fo ©08 (wt. — Bz) ay = 5 costut — Bz) En la gus 10.7(a) aparcee cl dingrama de Ey HM. Bn general; si ag: ity te £0n| Unitarioe a lo large del campo E, el campo Hy la direccion de propagacion de la posible demostrar que (véase el problema 10.14) ‘Figura 10.7. (a) Diagrama de E Some funcionus de zen 1 0: Ge'Ey Hen 2 ~ 0. Las flech: © a0.49) Los campos (u ondas clectromagnéticas) tanto E como Hi son normales en cualquier pun- toa la direceion de propagacion de onda a, Esto significa que se sitian en un plano transversal ortogonal a esa direcci6n. Asi, forman una onda clectromagnética sin com- ponentes de campo eléctrico y magnétice a lo largo de la direccién de propagacion, Famaca onda electromagnética transversal (ET). Ey Hi son a s¥ vez, ¥ por separado, una onda plana uniforme, puesto que F. (o FX) manticne igual magnitud a todo lo largo de un Plano transversal, definido por 7 — constante. La direccién en la que apunta el campo Eleetrico es la polarizacion de una onda ET. La onda de la ecuacién (10.29), por ejemplo, Seta polarizada en la dlireceién de x, Esto se advierte en la figura 10.7(b), ilustrativa de ‘Gndas planas uniformes, La existencia fisica de una onda plana uniforme es imposible, ya Que cefentenderta al infinite y representaria nna energia infinita. Pese a su simplicidad. So carece de importancia pues sirve como aproximacion de ondas practicas —las proceden- tes de una antena de radio, por ejemplo— alejadas de fuentes de radiacion. Aunque estas precisiones se refieren al vacio, también se aplican a cualquier otro medio isotrépico. .6. Ondas planas en buenos conductores Este es otro caso especial del expuesto en la secci6n 10.3. Un conductor perfecto, buen conductor, es aquel en el que « 52 ae, de modo que o/ae— oo; es decir, ote 0.50) Asi, las ecuaciones (10.23) y (10.24) se convierten en a=p- (= Vafer aosiay o_ [Pw 22 un g- 2 1-3 ao.sib) Asimismo, na ft fs 0.52) de modo que E se adelanta a Hen 45°. Si rE ye" cos(wt — 52) a0.s3a) S]En algunos textos la polarizacién se define de otra manera, ** costwt — pz — 45°) a, De esta forma, a medida que la onda E (0 H) se desplaza en un medio cond plitud es atenuada por el factor e-**. Indicada en la figura 10.8, la distancia, de Ia cual la amplitud de onda decrece en un factor e-? (alrededor de 37%), didad pelicular © profundidad de penetracion del medio; esto es, Eye = Bye! La ccuacién (10.54a) sucle ser aplicable a cualquier medio material. En el buenos conductores, las ecuaciones (10.512) y (10.54a) producen . ees Value a Con referencia a un buen conductor, la imagen contenida en la figura 10.8 rest rada, pero la profundidad pelicular de un medio parciaimente conductor puede: considerable. En cuanto a las ecuaciones (10.514), (10.52) y (10.540), alusivas aun buen conductor, 1 fon 147 ee os Figura 10.8. tlustracion de Ia profundidad “abla 10.2. Profundlidad polleular del Cobre.” Frecuencia (2) 306010050010 10" 108 Profundidad pelicular (iam) 26299065 6H IO GH 10 Ven cobra. 58 10" who, a = 65A/ VF fon em Con relacion asimismo a los buenos conductores, la ecuacién (10.532) puede expresarse lo que indica que 4 mide Ia disminucién exponencial de la onda durante su recorrido por Cl Gunductor. En la tabla 10.2 26 presenta la profundidad pelieular del cobre a varias freeuencias, la cual decrece al aumentar la frecuencia. Asf, Ey Hi dificilmente pueden propagarse a través de buenos conductores. El fendmeno por el que la intensidad de campo decrece répidamente en un conduc tor se conoce como efecto pelicular. Los campos ¥ corrientes asociadas son confinados a una capa muy delgada (la “piel”) de la superficie del conductor. Respecto de un cable de radio , por ejemplo, es valido suponer quc, a altas frecuencias, toda la corriente fluye en. Clanillocircular de grosor 8 que se muestra en Ia figura 10.9. El efecto pelicular —el cus adopta distintas apariencias en problemas tales como la atenuacion en gufas de ondas, la resistencia efectiva o en corriente alterna de neas de transmision y el blindaje electro- magnetico— es util en numerosas aplicaciones, Puesto que, por ejemplo, Ia plata presenta Una profundidad pelicular muy reducida y es insignificante la diferencia de rendimiento entre un componente de plata pura y uno de cobre con revestimiento de plata suele recurrirse a Cste para abatir el costo de materiales de componentes de gufas de ondas. Esto explica asimismo que en las antenas exteriores de television se empleen conducto- res tubulares huecos en lugar de conductores solidos. De igual manera, los aparatos eléc- tricos pueden ser protegidos eficazmente contra ondas clectromagnéticas mediante Pubes pemeuctewse do opepestanas costes mectinrtidacioeinclivsilarse |e aroeqe ‘La profundidad pelicular sirve para caleular la resistencia en corriente alterna debida al efecto pelicular. La resistencia de la ecuacién (5.16) se llama resistencia en corriente di- recta; es decir, - Ra & G18) Figur 10.9. Profundidad pelicular a altas frecuencia, 6 = a, La resistencia superficial o pelicular R, (en Q/m?) es la parte real de la 9 de un ductor. Asi, a partir de la eeuacion (16.55) Esta es Ia resistencia de ura yuna unidad de longitud del tor. Equivale a la resistencia en corriente directa de una unidad de longitud ductor con drea de seccidn transversal 1 3. Con referencia, asf, a una anchura’ Tongitud ¢ dadas, Ia resistencia en corriente alterna se calcula recurriendo a la ya da relaciGn de resistencia en corriente directa de la ecuacion (5.16) y suponienda, uniforme de corriente en el conductor de prosor 3; esto es, donde S ~ 41. Respecto de un cable conductor de radio a (fig. 10.9), w = 27a, de € Ry Brad a ey Tie) va Puesto que 6-4 a altas frecuencias, esto indica que R., es mucho mayor que Ra: neral, la razon de la resistencia en corriente alterna a la resistencia en corriente: comienza en 1.0 en corriente directa y muy bajas frecuencias y crece conforme al Ia frecuencia. Asimismo, aun si la mayor parte de la corriente no esta distribuida mente en un conductor de 58 de grosor, la pérdida de potencia es la misma que si estuviera distribuida uniformemente en un grosor de 6 y cero. Esta es una razén que a5 se le denomine profundidad pelicular. Un dielécirice disiputive Gene win impeduncia ints fiseca de 200 2307 @ cu une bu cia particular. Si, en esa frecuencia, la onda plana que se propaga por el dieléctri como componente de campo magnético = 102+ co0( ar 32) 2, Alm halle Ey @. Determine la profundidad pelicular y la polarizacién de la onda, Solucis La onda dada se desplaza a lo largo de a,,de modo que a ~ a5 a7 ~ p= & X ay — aKa, a, Asimismo, 17, = 10, de manera que Eo He Salvo por su diferencia de amplitud y fase, Ey H tienen siempre la misma forma. Por tanto, = = 200 £307 = 200 ef" +, = 20006" E = Re (20000/e~rei/a,) B= ~26-“ cos( or — $+ Z)ackv/em En conocimiento de que /# = 1/2, es preciso determinar a. Puesto que Sin embargo, = tan 20, ~ tan 60- = V3. Bn comecuencin, 2V3 = 3.464m La onda tiene una componente E,; asf, esté polarizada a lo largo de la direcci6n de z. mE RAE am . ae ‘Una onda plana que se propaga por un medio con e, = 8,4, = 2 tiene | sen G0" ps) a.Vim. Determine a i ieee 5) La tangente de pérdida, @)-Laimpedancia de lacnda. @) La velocidad de la onda, @) El campo Ht. : Respuestas: g) 1.374 rad/m, B (0.5154, €) 177.72 £13.63° Q, d) 7.278 x © ¥ 6) 2.817¢"9 sen(10¥¢ ~ Bz ~ 13.63°)a, mA/m, iy fj En un medio sin pérdidas en el que 7 = 607, 4, = Ly H = —0.1 cos (wt — 2) a, 4 i Hempte 103 | Cae — z)a, A/m, calcule 6, 0 ¥ E- | Solucién: i, En este caso, ¢ = 0, — Oy # = 1,de modo que | n= Vala = [Bs fe — 1208 Ve, i St Sow A pai sarrolladas en este capitulo (basadas en las ecuaciones de Maxwell) 0 usando dir mente las ecuaciones de Maxwell, como en el capitulo anterior. Método 1. Para emplear las (éenicas desarrolladas en este capitulo, sea 1 E-H,+H, donde Hi, = —0.1 cos (wt ~ 2) a, y Hy — 0.5 sen (wr — 2), y el campo eléetrico corres- pondiente E-E,+E, donde E, = £,, 08 (ot — 2) iy, ¥ Bz = Fay sen (at — 2) ay,. Nétese que aunque H tiene Componentes a'lo largo de a, y'a,,no tiene ninguno a lo largo de la direccién de propa- gacidn:; se trata en consecuencia de una onda ET. En el caso de Ky, 8, = Ca Xan) Byg = 9 Hyg = 607 (0.1) = Gr Por tanto, F, = 67 cos (wt — 2) a, En el caso de Ex, Be, = — Ge X ay) = C0, X a) = Bag — 1) Hag = 07 (0.5) = 300 Por tanto, E, = 307 sen (or ~ z)a, La adicion de F, y E, da como resultado E; es decir, E = 94.25 sen (1.5 < 10% — 2) a, + 18.85 cos (1.5 < 10% — z) Método 2. Es posible aplicar directamente las ecuaciones de Maxwell. ae 1 eye, E = entf[yxna porque @ = 0. Pero oe 6 sete |= Dl ort VAM Vinge) Hye) 0 co 2 = Hy, 008 (wt = 2) a, + Hy. sen (ot — 2a, donde H,, = -0.1 y Hz, = 0.5. Por consiguiente, fox mat = Been oa — Bit con ot = 24, 94.25 sen (wr ~ 2) a, + 18.85 cos (wr — 2) a, Vim. snanecagsiin aedinae. perisemeitiienomnoatse {Une onda plana en oa «tains ion sein oe de DD Bric | Fees oo) M. seam a te ecten inks aust at = Bb nie neieoeare we of Btt ea % ee cel Ejempio 10.4 | Una onda plana uniforme que se propaga on cierto medio tiene E = 2e-* sen (10% — Bz) a, Vim. Si el medio se caracteriza por ¢, = 1, #1, — 20 @ = 3 mhos/m, halle a, 6 y H. Solucién: Para saber si el medio es un dieléctrico disipative © un buen conductor debe se Ia tangente de pérdida. 3393 >= 1 108 x 1 x 3 lo que indica que el medio es un buen conductor en la frecuencia de operation. i. p= fee — [sere cease @=614Npim, 8 = 61.4 rad/m Asimismo, iw _ [4m x 10°? x 20(104) ]! Pie = Se [So 3 tan 20, == 3393 —> m4 donde to! “Woineeet ain (109 — ottae — 2) apmArm Rueroicta an aeaiceal ve panies igh ans ee sb cos dom? w/aba, Vien yO. Determine ee eae saa 1) Lia dbtuncta recorridn por in onda para presentar un sorrimlento de fase de 10" 6) 1a dtancia recorridn por i onda pars que s amplivud se redizcn 40%, Ditleayeanes ane ‘Respuestas: a) 2.787a, Vim, b) 8.325 mm, ¢) $42mm y d) —4.71a, mA/m. Una onda plana E = E, cos (wt — Bz) a, incide en un buen conductor en z = 0. Halle la densidad de corriente en el conductor. Sotucion: Puesto que la densidad de corriente J = oF, es de esperar que J satisfaga la ecuacion de ‘onda formulada en la ecuacion (10.17); es decir, vs, — 73, = 0 Asimismo, la E incidente cuenta sélo con componente x y varfa con 2, Por tanto, FHL NY Goins Phen 0 1a cual es una ecuacién diferencial ordinaria cuya solucién es (véase el caso B del ejemplo 6.5) Ue = AeT + Bete i Ejemplo 10.6 La constante B debe ser de cero porque J,, es finita cuando z —> 00! Pers, ductor, 7 2= we de manera que a — 8 — 1/5. Por tanto, 22H yrat pm a+) - SEP Jog = Ams 100 Jpg = FO) eH +18 donde J,, (0) e8 Ia densidad de corriente en Ia superficie del conductor, jercicio 10.5 7 Buils atin be ald Vite eid Con relacién a la densidad de corriente del ejemplo 10.5, halle Corriente total a través de tna franja del conductor de prot largo de zy anchurs w alo argo de y. 7 ae ea Les eee che ‘Con referencia al cable coaxial de cobre mostrado en la figura 7.12, sea a = 2mm y£— 1 mm. Calcule la resistencia de 2 m de longitud del cable en corriente 100 MHz. Sotucion: Sea R=R+R, donde R, y R, son las resisténcias de los conductores externa e inter: En corri¢nte directa, eee 2 Ri SS Gad? ~ SSRI a[s MIO TE 7A ¢ e ‘ oe oS ~ anile + oP — 8) ~ ome + Be) ———E Sax AGRI + Ia] x 10 = 0.8429 mo 4 748% 0.8429 Serene penif = 100 Mrz, Re ve ee Re Ws = rb2ma ~ Daa ees [=p IO" X Ae X10 Iw x2 x 10% 5B x 107 0.410 Pucsto que 6 = 6.6 pm 5.8 x 10” = 0.13840 En consecuencia, R= 0.41 + 0.1384 = 0.5484 0 valor superior en aproximadamente 150 veces al de Rig. Con relaci6n, asf, a la misma co- friente efectiva f, la pérdida Ghmica (2R) del cable a 100 MHz es mucho mayor que la en un factor de 150. pérdida de potencia en corriente direct SfRRERM BIBER ‘Respecto de un alambre de aluminio con didmetro de 2.6 mm, calcule la raz6n de resistencia en corriente alterna a resistencia en corriente directa a a) 10 MHz D2GHz © Respuestast 4) 24.16 eas 10:7. Potencia y el vector de Poynting Be Como ya se menciond, por medio de ondas clectromagnéticas es posible transportar Gherpta de un punto (sede de un transmisor) a otro (con un feceptor). La rapidez de tal transmision de energia puede obtenerse de las ecuaciones de Maxwell: (a0.s8a) (40.588) 436m _ PROMGACION DE ONDAE HLEETROMAONETINS 1 mtipican de amon miembnos acne 058) Po Boo) ok oe Pero en todo campo vectorial Ay H (véaue el spéndice 6.10) Vi(AXB) = B-(V XA) AL(V xB). | La aplicacién de esta identidad vectorial a It écuacién (10.59) (concediendo, i yB— B) dacomo resultado. $09 XE) + V(X E) sok? +e ‘Con base en Ia ecuacién (10.580), He(v xe) = (HSE) = SF ca) de modo que Ia ecuacién (10.60) se convierte en. ae 241, ant BOP WV KH) ~ okt + 30S Si se reordenan los términos y se obtiene Ia integral de volumen de ambos [vexm a = 8 f [Leer Bare ]er fede ‘Ataplicar al teorema dela civergenca nl miomtee inrverdgieg cha formas 8 | [Lemred uae] arf ont ae ‘ ‘ ‘ Potencia Rapier de decrement Potcadia mica oat eas — ania aioe ebsieie en aii del volumen — los campos eléetrico y magnético, La ecuacién (10.63) es el teorema de Poynting. cuyos términos se identifican aqut} mentos de conservacién de energfa aplicados a campos electromagnéticos. El pri no del micmbro derecho de esta ccuncion es In rapides de decremento de Imacenada en los campos elécirico y magnético, y el segundo la potencia disip: je que el medio es un conductor (¢. # 0). La cantidad FE x Hen el miembro izqui vector de Poynting ® (el cual se mice on watts por metro cuadrado [W/m=); es d } “On the transfer of energy it \ Esto representa el vector instantaneo de densidad de potencia asociado con el campo Clectromagnético en un punto dado. La integracién del vector de Poynting sobre cual- Guicr superficie cerrada da como resultado la potencia neta que sale de esa superficie. Este teorema se ilustra ch Ia figura 10.10. tanto a E como a FL y ocurre, por tanto, a Io largo de mak X ay 0.49) El hecho de que @ apunte a lo large de mp provoos que ol nombre de este vector degene Tara en vector de apuntamiento (pointing, en inglés). ‘Sisuponemos de nuevo que EG, = Bye" cos (wt ~ Bz), entonees az, 2) = 22 e-«© cos (wt — Bz — 0,)a, Potencia de salide Figura 10.10. MlustraciGn del equilibrio de potencia en campos clectromagnéticos, Ejemplo 10.7 M0) = 67" cos (wt ~ Bz) COS (wt ~ Bz ~ 0p)m, al om ial 2%" [608 0, + cos (2a — 282 — Oy) me puesto que cos A con = 2 [con (A — B),+ cou (A. B)]. Fara determinar Poynting promedio temporal Fyrom (2) (en W/m), de mayor valor practico que. de Poynting instantaneo 2(z, 1a ecuacisn (10.66) se integra sobre el periods es decir, Pram(e) = [ME de Ess posible demostrar (véase el problema 10.28) que esto equivale a Porom() = E Re (E, XH) Al sustituir la ecuaci6n (10.66) en la ecuacisn (10.67) se obtiene Poroml2) = Zpap © COS Oy Me 2Iml Ta patancia promedia temporal total que atravices una euperficie © determin: dada por ol oan Reparese en Ia diferencia entre P, Pyar Y Pycom- PCC, Ys Z- 1) 3 el vector de watts/metro y varfa en el tiempo. PyraqiCz, y, 2) tambien se mide en watts/metro y es medio temporal del vector de Poynting %; es un vector, pero invariable en el f Pree 05 1a potencia promedio temporal total a través de tina superficie, en Wal En un medio no magnético E = 4 sen Qn x 10% — 0.82) a, Vim 7. POTENCIAY EL VECTOR OF POYNTING M439 Hralle a enn b) La potencia promedio temporal que porta la onda. ¢) La potencia total que atraviesa 100 em? del plano 2x + y = Solucion: a) Puesto que a = Oy 8 # wie, el medio de referencia no es el vacfo, sino un medio sin pérdidas. B=08, w=24x10, = 1, (no magnético), Por tanto, fot B= OV pe = WV ne, = 2S Be _ 0.8(3 x 10") _ 12 Qe x 107 7 Be = 1202 = 1200+ = 1097 ye Bx Hm Been\on — Bx), Peas 16 Porm = & [ae = .-5h5" = Sia, mWim ©) Enel plano 2x + y = 5 (véanse los ejemplos 3.5 u 8.5), Asf, la potencia total es en fotos Fyrom” Sy = @1 < 10-%a,) - (100 x 10-4 [7] = 16210 4s ww Bjercicio 10.7 z in ol vacio, 1 Sid Soeilens se) aia /easesMlieiiy porerniaterealy @) Una placa cuadrada de 10 cin por lado en'ét plano.x #21 b) Un disco circular de Sem de radio enelplanox=1.0 ee ee ‘sebibaae. 10.8. Reflex nde una onda plana en incidencia normal Hasta aquf hemos considerado ondas planas uniformes que se desplazan en m mogéneos ilimitados. Pero cuando una onda plana procedente de cierto medio se 6 tra con un medio diferente, es parcialmente reflejada y parcialmente transmit proporcin de Ia onda incidente por ser reflejada y por ser transmitida depende pardmetros constitutivos (e, 4, ¢) de los dos medios implicados. Aqut supondrem El plano de la onda incidente ¢s normal a la frontera entre los medios: la incidenci cus de ondas planas xe tratard en In seccién siguiente, una ver comprendido el simple, de la ineidencia normal, ‘Supongamos que una onda plana que se propaga en forma normal en la {rontera 2 ~ O entre el medio 1 (z = 0), caracterizado por. iy el medio 2 (z > 0), caracterizado por o, #, ¥ fz, como se muestra en En ésta, los subindices ry ¢ denotan las ondas incidente, reflejada y tran: tivamente. Las ondas incidente, reflejada y transmitida que aparecen en la figura obtienen de Ia siguiente manera: (Onda incidente: Jone E,fz) = Ewe ay entonces He) = Hae ra, = Be a, Onda relejada: (&,, H) se desplaza a lo largo de —! os a Medio 2 65st) ithe OL Figura 10.11. Onda plana de incidencia normal on una interfan Entre dos medios distintos, Hy) = Hoe (a0.74) donde E,, se halla presumiblemente a Io largo de a,; supondremos en forma sistemstica que, en incidencia normal, E,, E, y E, tienen la misma polarizacién. Onda transmitida: (&,, H) se desplaza a lo largo de +a, en el medio 2. Si Eu (2) = Exe ™ ay 40.75) entonces E, (2) = Hye a, = Bera, 10.76) HAG) = Hue’ a 0.76) En las ecuaciones (10.71) a (10.76), Eon Exe ¥ Em 80 las magnitudes en z = 0 de los cam- pos [éctricos incidente, reflejado y transmitido, respectivamente. ‘Adviertase en la figura 10-11 que el campo total en el medio 1 comprende los cam- pos tanto incidente como reflejado, mientras que el medio 2 s6lo contiene al campo trans- mitido: es decir, E, = 8, +E, +H, E, = E, m=, En la interfaz z = 0, las condiciones en Ia frontera exigen que'las componentes tangen- dales\de Tos caiupol lS > 1 start eontiniias: Pucete GOs las chides ect! traneversales, los campos E y H son enteramente tangenciales a Ia interfaz.En z= 0, asf, Ejay Han — His iinplican que E,(0) + F,(0)= BO) > Eq + Exe Ho) +0) = 0) Ea A partir de las ecuaciones (10.77) y (10.78) se obtiene se a A mam y ze. 2a Con base en las ecuaciones (10.79) y (10.80), el coeficiente de reflexion Ty el de transmision 7 se definen como ‘i 2 = Be 2m En mt Cabe hacer notar que Litres 2) Tanto T' como 7 son adimensionales y pueden ser complejos 2o=sir=1 El caso hasta aquf expuesto es el general. Consideremos ahora un caso especial, medio Les un dieléctrico perfecto (sin pérdidas, 7, — 0) y el medio,2 un condi fecto (@, ~ 00). En estas circunstancias, 1, — 0; asf, = —1 y 7 = 0, lo que indi ‘onda es totalmente reflejada. Esto no es de sorprender, puesto que en un condu fecto los campos tienden a cero, de modo que es imposible que exista una onda tida (E, ~ 0). La onda totalmente reflejada se une con la onda incidente para onda estacionaria. Como su nombre lo indica, una onda estacionaria se “estaci ‘vViaja, ya que se compone, de dos ondas en movimiento (E, y E,) de igual amplitt campos E y H son enteramente tangenciales a Ia interfaz.En z= 0, asf, Ejay Han — His iinplican que E,(0) + F,(0)= BO) > Eq + Exe Ho) +0) = 0) Ea A partir de las ecuaciones (10.77) y (10.78) se obtiene se a A mam y ze. 2a Con base en las ecuaciones (10.79) y (10.80), el coeficiente de reflexion Ty el de transmision 7 se definen como ‘i 2 = Be 2m En mt Cabe hacer notar que Litres 2) Tanto T' como 7 son adimensionales y pueden ser complejos 2o=sir=1 El caso hasta aquf expuesto es el general. Consideremos ahora un caso especial, medio Les un dieléctrico perfecto (sin pérdidas, 7, — 0) y el medio,2 un condi fecto (@, ~ 00). En estas circunstancias, 1, — 0; asf, = —1 y 7 = 0, lo que indi ‘onda es totalmente reflejada. Esto no es de sorprender, puesto que en un condu fecto los campos tienden a cero, de modo que es imposible que exista una onda tida (E, ~ 0). La onda totalmente reflejada se une con la onda incidente para onda estacionaria. Como su nombre lo indica, una onda estacionaria se “estaci ‘vViaja, ya que se compone, de dos ondas en movimiento (E, y E,) de igual amplitt 444 BE PROPAGAGION DF ONOAS ELECTROMAGNETICAS Figura 10.12. Ondas estacionarias E — 2£,, sen fz s0n wt las curvas 0, 1,2,3,4,.. -, corresponden alos momentos #20, 178, Ti, 3375, Ti2) "|" "Tespoctivamente; A= 27/B. ¥ que los valores minimos de [E,| ocurren en Bitmm = Gn +1) caso B Sim, < m,T < 0. neste caso, la ubicacién de los valores maximos de |E;| esta Ia ecuacion (10.89), y la de sus valores minimos por la ecuacién (10.88), como #e | en la figura 10.13. Conviene senalar que 1. El valor minimo de || ocurre en asociacién con el valor maximo de |B 2. La onda transmitida (no representada en la figura 10.13) en el medio 2 onda puramente mévil, de manera que en esa region no hay valores minimos. La raz6n de [Ejlmix 8 [Eilmin CO 4¢ [Hislmix & [Hylmin) 8¢ Hama razén de onda estaci es decir, oe lBilewe Mile 2 IPI TEjlem THilam — = ITI Riipto 10.5 10.8. REFLEKION DE UNA ONDA LANA EN INCADENIA NORMAL M445, ‘Figura 10.13. Ondas estacionarias debida a reflexi6n en una interfaz Gntre dos medion sin perdidas; A = 27/B,- ir) = 254 ao.91) Puesto que |T| = 1 de ello se desprende que 1 = s = 00, La raz6n de onda estacionaria es Adimensional y suele expresarse en decibeles (dB), de esta forma: sen dB = 20 loses 0.92) En al vaote (e = 0), una onda plana con Hi = 10 cos (10% — Bz) a, mA/m incide normalmente en un medio sin pérdidas (© = 269. 4 Sy1,) en la region z = 0.De- termine la onda reflejada H,, EF, y la onda transmitida Hi, E, Solucién: Este problema puede fesolverse de dos maneras. Método 1. Considérese que la figura 10.14 ilustra este problema. Con referencia, @_ 1 Am eax Ie 3 m= 1, = 1207 el ejemplo 10.8. @ Con referencia al medio dieléctrico sin pérdidas, pra wV/ je = V/V, = 28) = 483 o-VE-VEVE Pueste que H, = 10 cos (10% ~ 8,2) a, es de esperar que ole 2m = Big cos (10% = Riz) ae, donde y Eg = MHig = 10-76 Por tanto, E, = —10n, cos (10% — #2) a, mV/m Ahora bien, np 4 Ero = $B Ast de donde es facil obtener H,, en esta forma: De modo similar, Ast, FE, = Fig cos(0% — fz) ae, donde ap, 2, — —a, Hn consecuencia, de lo que resulta 4, = 22cos (10% — $<), mavm Método 2. Opcionalmente, es posible obtener H, y H, directamente de H, mediante Foo eee Ass, a 10 Hyg = 3 Hy = 32 Davi Ae eo Ho = 322m, = 2H = y 4, — —12 000 10" + pie) ama len 20 14, = 22 cos (lo — fas) a, mA/m como se obtuvo antes, Notese que de esta forma quedan satisfechas en z = 0 las condiciones en la frontera, Bat) 418(0)-— 180) — —"2 ns cos t10%) =, y 0) + 440) ~ #400) ~ 22cos Ore, arse invariablemente para la comprobacion cruzada de E Jas cuales pueden ut ie pA |A.5 G¥1z, una onda plana uniforme E,, = 10e—!#* w,Vim en ol vacio inci mente en na I lieléetrica sin pérdidas plana y de gran tamano ( Dorin ak ae ae aston ‘Respuesta: — 3.354 expUf2) a, V/m, 6.667 eXp(—/p.z) a, Vim, donde — By = 28, = 2000/3. { Ejercido 108 00 [Dada una onda plana uniforme en el aire como 1B, = 40 cos (wt — 2) a, + 30 sen (wr — Bz) a, Vim a) Halle H, b) Siesta onda se enclentra con una placa perfectariienté conductora normal al 2 = 0, halle Ia onda reflejada E, y H,. ©) {Cudles son los campos totales E y Hien z = 0? @ €aleule los vectores de Poynting promedio temporal en z= Oy < = 0. Solucion: a) Este problema se asemeja al formulade en el ejemplo 10.5. La onda puede a nerse en dos ondas FE, y E,,, donde Ey — 40 cos (ot — fz) @, Eq — 30 sen (wt — Bz) a, A la presion atmosférica, el aire tiene ©, ~ 1.0006 ~ 1. Asf, es posible consideran Vacio. Sea Hy = Hy + Hy Wy = Hayy cos (wt — Bz) an, donde Por tanto, fi Fay 008 (wt — Bz) a, De igual forma, Hy = Haesen (wt — 62) an, donde Ens 301 Hee "ng 200 ae an, oe arm a, = om ct Ha — —Z,, sen (a — Bz) a y <4, +H, if 1 = — giz sen (or — Bz) a, + 3h 908 (ot — Az) a, mAs Este problema también puede resolverse siguiendo el método 2 del ejemplo 10: ) Puesto que el medio 2 es un conductor perfecto, 21 = nen esto es, rey 70 Jo que indica que los campos incidentes E y Hf son totalmente reflejados, Ewe =T Ew = Ew Por tanto, K, = —40 cos (wt + Bz) — 30 sen (wr + Bz) a, Vim 1H, puede hallarse a partir de E, justo como se hizo en el inciso w) de este mismo ejemplo ©.a partir de Hy siguiend el méiodo 2 del ejemplo anterior. Cualquiera que sea cl proce- dimiento, se obtiene i 1 HH, = bcos (wt + Az) a, — Gersen (wr + Bz)a, Alm ©) Es posible demostrar que, en el aire, los campos totales BSB +E, y Bm +H, pueden formar una onda estacionaria. En el conductor, los campos totales B,-E,=0, H,=H,-0. Enz 30, 4 2, Pigrom a Ua — Flea = 40? + 3070.1 =o Enz=0, Ie. Pee arg st See porque la potencia incidente es reflejada en su totalidad, Ejeraiets 10.9 . | ybacnda plana ¥ — 40 ecn (at — 55) 0, Vim on un medio sin. pees 2 m,) sh ancnentrs can un media disipative Ga pnt Ane © normal al eje x en x — 0. Halle = | bajfes giemiato: aoe Hy 3 estnabiani coc oH pot f DEY, <2 SS ORY EH, Se 4) Los vectores de Poynting promedio temporal en ambas regiones_ | Rempoertne ) OSIRIA, 02905 £33,562 AP 02546) 4023 nencfar + ph 171.9") a, Vien, 54.3 sen (we + Se+ 171.9") a, mam, ©) 11.47 €- sen (wt ~ 7.8268 + 33.56") a, Vim, 120.2 0-60 (or ~ 7.826% ~ 4.01") a, mAlm y d) 0.5469 a, Wim, 0. 28 12.04) a, When, M | Reflexi6n de una onda plana Consideremos ahora una situaciGn mds general que a descrita en la seccién 10.8. Para simplificar el andlisis, supondremos que tratamos con medios sin pérdidas. (Bastaria reemplazar © por e, para prolongar el andlisis a medios disipativos.) Es posible demostrar (weanse los problemas 10.14 y 10.15) que una onda plana uniforme adopta la forma ge- neral de BE, = By cost - F — 0) 20.93) = Re [E,e 4] donde r = xa, + ya, + za, ¢s el radio 0 vector de posicion yk = kaa, + kya, + K.a, el vee~ tor de numero dé onda 0 vector de propagacion: k sigue siempre la direceion de propaga- ¢lon de Ja onda. La magnitud de & se Felaciona con @ de acuerdo con ja relacion de dispersion = + +E = one 0.94) En medios sin pérdidas, asf, & es en esencia lo que fi en las secciones anteriores. Dada la forma general de E de la ecuacién (10.93), las ecuaciones de Maxwell se reducen a kx E = ont 0.954) kX Hs wer 0.955) kom (0.95e) k-E=0 0.952) Jo que indica que 1. E, Hy k son mutuamente ortogonales y 2. F y Hse sittan en el plano ker = kx + ky + kz = constante Con base en la ectiacién (10.95), 61 campo H correspondiente al campo E de la ecuacién 10.93) es x = ae axe ” we Dexe- 096) Habiendo expresado E y H en forma general, consideremos ahora la incidenc oblicua de una onda plana uniforme en una frontera plana, como se ilustra en la figu 10.15(a). EI plano definido por el vector de propagacién K y un vector unitario #, nor, mal ala frontera se llama plano de incidencta, El angulo 0, entre ky a, 08 el dngulo de Incidencia. ‘También en este caso las ondas incidente y reflejada se encuentran en el medio 1, y Ia onda transmitida (0 refractada) en el medio 2. Sea E, = F.008 (kx + kegy + gz — 1) a0.97@) B= Ep 008 (kya + ky + kak — 0,1) 10.97) E, = Bye 008 (ku + egy + Kuz — 0) 0.976) Mediot («\.44) | Medio2 (en @ lessen, Fo ome, An Boos © Figura 10.15. Incidencia oblicua de una onda plana: (a) ilustracién de 6.8, ¥ 8,:(b) llustracion de las componentes normal ¥ tangencial de K. donde k, k, y k, con sus componentes normal y tangencial se muestran en 10.15(B). Puesto que la componente tangencial de E debe ser continua en la front E(z = 0) + Bz = 0) = EXz = 0) Para que las ondas representadas por la ecuacion (10.97) cumplan esta condi frontera respecto de todas las x y'y es indispensable que Ses cpeetbe eres o Be Sig Renae Feast 3 ym key ky = ky La condicisn 1 implica que la frecuencia no cambie. Las condiciones 2 y 3 (lam: diciones de acoplamiento de fase), requieren que las componentes tangenciales de tores de propagacién sean continuas. Esto significa que los vectores de prop: yk, deben situarse en el plano de incidencia. Ast, por efecto de las condicion« k,sen 0, flexion y 0, €) dngulo de transmision. Sin embargo, en el caso ky ke = By = OV ne ao.o1a) ky = Ba = Vp 40.1015) 40.99) y (10.1014) se desprende claramente que (0.102) Angulo de reflexién 9, es igual al dngulo de incidencia 0, como en Spti- ‘nto asimismo en las ecuaciones (10.100) ¥ (10.101), pa 0.103) ny sen 0, = 7 sen 0.104 eV me, = clay yma = eV pat bn base en estas generalidades preliminares sobre Ia incidencia oblicua, considere- fos casos especiales: uno en el que el eampo E es perpendicular al plano de inciden- ‘iro en el que el campo E_es paralelo a ese plano, Cualquier otra polarizacion fesenta una combinacién lineal de estos dos casos. elt: son los indices de refraccién de los . Polarizacién paralela fesste caso se Hustra cu Ia fyuia 10.16, en la que cl campo Ee ubiea on ol plano x2, ol pla- no de incidencia, Situados en el medio I, los campos incidente y reflejado estan dades por E,, — Ee(©0s 0, — sen Om.) e7HPitven sore a) (40.1054) engl Aparomnteier 0.1056) B,, = E,{C08 0, a, + sen 6, m,)eHhrtente-remay (10.1064) a, — Est emma se), (40.1060) ™ donde f, = Vays Reparese detenidamente en Ia forma como se lleg6 a cada Componente de estos campos. La clave en Ia deduccién de sus componentes es obte- her en primer término el vector de polarizacion k de las ondas incidente, reflejada y tanemitida. como se muestra en la figura 10.15(b). Una vez conocido k, se define E, | Figura 10.16. Incidencia obticua en la, ai plane de inciden ere ie) de manera que V+ E, = 0.0 k + E, = 0, tras de lo cual se obtiene HY, a ‘Situados a su vez en el medio 2, los campos transmitidos estén dados por Ey, = E,(cos 0, ay — sen 0, a.) e~MAesenss sou) donde 6; = 0 V/gnes. Si nuestro supuesto sobre las direcciones relativas rel ectiaciones (10.105) a (10.107) es erréneo, el resultado final nos lo indicaré por su signo. ‘De la imposicion de la condiciones de que 0, = 0, y de que las component ciales de Ey H sean continuas en la frontera = = 0 se obtiene Big + Bn) €08 0; = By £08 8, 1 a 9 Blo — Ere) = 5 Eve La expresion de E,, y E,, en términos de E,, produce Bre _ ma Soa 8, — 7 008, Bi nes ores te cone, io 72 €08 8, +m €08 8 Fn= 15. ao.1108) Las ecuaciones (10.109) y (10.110) son las ecwaciones de Fresnel. Cabe referir que estas ‘ecuaciones se reducen a las ecuaciones (10.81) y (10.82) cuando 8, = 0; = 0,.como es de. esperar, Puesto que 0, y 8, se relacionan conforme a la ley de Snell, formulada en la ecu: ion (10.103), las ecuaciones (10-109) y (10.110) pueden expresarse en términos de 0, sus tituyendo Goa {con base on las ccuaciones (10.109) y (10.110) 68 fll demontrar ve cout it ryt a (SEB) aoa La ecuaci6n (10.109) evidencia que es posible que I} = 0, porque el numerador es Ja diferencia de dos términos En estas condiciones no hay feflexin (,, — 0), y el angulo incidente en el que esto ocurre se llama dngulo de Brewster 8,0 dngulo de polarizacion, puesto que una onda incidente arbitrariamente polarizada se reflcjara con solo Ia com- Ponente de E perpendicular al plano de incidencia. El efecto de Brewster se aprovecha €n tubos laser para controlar la polarizacion de la luz emitida mediante la colocacion de cristales de cuarzo en el éngulo de Brewster. Este angulo se obtiene disponiendo que 6, ~ 0», cuando Ti,~ 0 en la ecuacién (10-109); es decir, 2 605 8, = m COS Op, ah (1 ~ sen? 0,) = nf (1 — sen* dy.) La introduccién de la ecuacién (10-103) 6 (10.104) resulta en sen? 0», 0.113) Por su valor practico, conviene considerar el caso en que, ademas de carecer de pérdidas, Jos medios dieléctricos son no magnéticos: esto es, 4, — 44 — My, En esta situaci6n, Ia ecuacién (10-113) se convierte on 6m, = Sh -v0en 6h = of cently ache r e+ tite, = fl aos) a nn Ee Cn ee ny 456m PRomcACiON DF ONDAT LECMOMAGNENCAS B. Polarizaci6n perpendicular od En este caso, el campo E es perpendicular al plano de incidencia (el plano xz), ilustra en la Hgura 10.17, aunque también podria decirse que en esta situacion Hes paralelo al plano de incidencia. Los campos incidente y reflejado en el trutsceipes hate SB aii cla decasg, ib Be (eos 905+ son ja) eter necent) 4 Sl etirukrors d 14, £22 (cos 0, my + som dm) «IMO end ty r mientras que los campos transmitidos en el medio 2 estan dados por Ky = Fe lke ant dios gs ¢ 4, = 2 (—cos 0, a, + sen 0, a,) eW/elenenty toms) Me sae a € c 0.117) Satisface las ccunciones (10.95) de Maxwell. Al imponer ests vez las de que las componentes tangenciales de E y H sean continuas en z — 0 y de qi Notese que Ia definicion de las componentes de campos en las ecuaciones ( igual a 6,20 obtiena, Eg + Ew = En An — F.) cos 0, = 1 F,,c08 0, a ay Eve ~ Ene) 008 01 = 7 Bi. 608 0, La expresion de E,, ¥ Ey en términos de £,, produce 7. Incidencia oblicua en Ia 4 perpendicular al plano de ineidencia. Ew =T Em 0.1198) y Ee 2m 60s 6, 74 Bye” a 608 0 +m COS O, Geta} Fe = 71 Ew 0.1208) las ecuaciones de Fresnel para polarizacién perpendicular. Con fundamento en Ias ecua- ciones (10-119) y (10.120) es facil demostrar que deren aoaz1y ‘ecuacién semejante a la ecuacién (10.83) para incidencia normal. Asimismo, cuando 0, = 6, = 0, las ecuaciones (10.119) y (10.120) se convierten en las ccuaciones (10.81) 0.82), como debe ser. En el caso en que no hay reflexion, I, = 0 ( £, = 0), Io que equivale al caso de transmisién total (+, = 1). Al reemplazar 8, por el correspondiente angulo de Brewster Oy, Se obtiene 12 608 Op, = 7; £08 9, nH (1 — sen® 0p.) = nf (1 — sen® 0.) La incorporaciéa dé In.ectacién (10.104) resulta en: a tag (0.122) En medios no magnéticos (14) — 4 — 144), Sen” 0p, > 00 en Ia ecuacisn (10-122), de modo que no existe 8», ,ya que el seno de un dngulo nunca es mayor que la unidad. Asi Si sy # Ha Ym ~ tla ecuacion (10.122) se reduce a 0.123) Auman: panies atedilaiitie, en tate entn gaiciion, ‘Una onda electromagnética se desplaza en el vacfo con la componente de campo, EE, = 100 62691059 a, Vim, Fiemplo 10.10 Determine aoyr : }) La componente de campo magnético. ©) La potencia promedio temporal en la onda, Sotuctén: @) Al comparar el E dado con EB, = Beh = Byron tla, resulta claro que k, = 0.866, Ast, k= VETTE ~ VORGE) + OS = 1 Pero en el vacto, Por tanto, 3X 10" rad/s Qn a = 7E = an = 6.283m b) A partir de la ecuaci6n (10.96), el campo magnético correspondiente esté di nm, - sox x 8, (0.8668, + 0.58.) Ge x 10-7 3 x1 100 a,el*"r HL, = (1.33 a, — 2.3.0.) f+ 050 mAs ©) La potencia promedio temporal es rR Prom = Re (B, t) = Fay an (1007 2C120%) = 11.499, + 6.631 a, Wim? (0.866, + 05.8,) Ejercicio 10.10 ste Repita el ejemplo 10.10 E = G0 a, + Sa.) cos (wr + 2y — 42) Vim en el vacto. ‘Respuestas: @) 1.342 < 10° rad/s, 1.405 m, b) ~29.66 cos (1.342% 10% + 2y — 42) A. mA/m yc) —0.07415 a, © 0.1489 a, Wim? ‘Una onda plana uniforme en el aire con We = 8.605 (wr — ax — 32) a, vim incide en una lamina dieléctrica (z = 0) con 4, = 1.0, #, = 2.5, = 0. Halle 4) La polarizacion de la onda. 6) El Angulo de incidencia ¢) El campo E reflejado. @) El campo Hi transmitide, Solucién: a) Del campo F incidente se desprende claramente que el vector de propagacién es Ky = day + 30, ky = 5 = OV a Por tanto, (= Se = 15 < 10% rads Un vector unitario normal a Ia interfaz (z — 0) es a,. Fl plano que contiene a k y a, es » = constante, el cual es el plano xz, cl plano de incidencia. Puesto que E, es normal a @ste plano, la polarizacién es perpendicular (como Ia representada en la figura 10.17) 4b) Los vectores de propagacién se ilustran en la figura 10.18, donde es evidente que ku 4 fe-Gre tan 6, 53.13° Opcionalmente, y prescindiendo de Ia figura 10.18, 6, puede obtenerse del hecho de que es.el dngulo entre ky a,; es decir, ©) Para determinar E, bastarfa recurrir a la ecuaciGn (10.116a), ya que este ‘Similar al planteado en el apartado B de la secci6n 10.9. Pero si suponemos 41 mos al tanto de ello, sea B, = F,. 608 (wt — k, + Da, el cual es de forma semejante el E, dado. Se ha clegido aqui el vector unitatia Sideracién del hecho de que Ia componente tangencial de E debe ser contin terfaz. Con base en la figura 10.18, KS pe Me — Bre donde i Keg = Kp SCN On Keng = Hep 008 8, a No obstante, 0, = 6, y k, = k; = 5, puesto que tanto k, como k, se encuentran mo medio. De ahi que k, = 4a, — 3a, i Para hallar £,,, se precisa de 6, De acuerdo con la ley de Snell sen 9, = 7 sen 9, = < 10% BRIE VI X25 ~ 7.906 Con base en la figura 10.18, Keg = k,8en 0, = 4 Kj. = K, 608 0, = 6.819 i, = 4a, + 6.819 @, Notese que ky = ky = Kun como era de esperar. ‘Bi 2 mcos 0, 7 E, ~ M608 0, +m CORO, 2 X 238.4 cos 53.13" 238.4 cos 53.13° + 37 cos 30.39" = 0611 jsmo resultado podrfa obtenerse de la relacién 7, = 1 + TL Asf, Eg = 1 Fig = 0.611 X 8 = 4.888 1, = 4.888 cos (15 < 10% — 4x — 6.8192) a, 462 PROPAGACGON DE ONOAS FLECTROMAGNETICAS H, se obtiene facilmente de F, en esta forma a ke ee Wy = Zag XB, = EE 4a, + 68191 ~ 7806 @38.4) H, = (~17.69 a, + 10.37 a,) cos (15 * 10% — 4x ~ 6.8192) mA/ 2 4.BHE a, cos (wt — Ken) Wjercicto 10.11 Sila onda plana descrita en el ejercicio 10.10 incide en un medio dic} © = Oe = Fey. 1 = My ¥ OCUPA z= 0, calcule 4) Los angulos de incidencia, reflexién y transmisisn. ee b) Los coeficientes de reflexidn y transmision. ©) El campo F total en el vacto. 4) El campo F total en el dieléetrico. ob @) El angulo de Brewster. Respuestas: a) 26.56", 26.56", 12.92", b) 0.295, 0.647, c) (10 a, + 5a) oot + 2y'= dz) + (—2.946a, + 1.473 0.) cos (wot + 2y + a @) (7.055a, + 1.618a,) cos (wt + 2y ~ 8.7182) Vim, e) 63. Resumen 1. La ecuaciGn de onda es de la forma con la solucién, @ = A sen (wt — Bz) donde # = velocidad de onda, A = amplitud de onda, YB = constante de fase. Asimismo, B ~ wie = 27/A.0 u tud de onda y 7 = periodo. 2. En un medio disipativo sin carga, la ecuacion de onda basada en las 6&4: Maxwell es de la forma frecuencia angular ( PA= NT, donde & VA, — YA, =0 donde A, es E, 0 H, yy = « + jf es la constante de propagacién. Si suponen FE, ~ £2) a,, se obtienen Onda electromagnéticas de la forma, EG. 0)'= Ee cos (wt — Bz) a, H(z.) = He7** cos (wt ~ Bz — 0,) a, donde « = constante de atenuacién, 8 = constante de fase, 1 — || 0, = impedan- cia intrinseca del medio. El recfproco de « es la profundidad pelicular (8 — Ve). La re- lacién entre B, @ y A formulada anteriormente también es valida para ondas clectromagnéticas. 3. De la propagacion de ondas ent medios disipativos puede deducirse Ia que ocurre en otros tipos de medios, casos especiales de aquél. En el vaclo,7 = 0,8 = ey.) — fast on medios dieléctricos sin pérdidas, «= 0,2 — a6, ¥ # ~ Molt, ¥ en Buenos conducts fea @ = 00,8 — on. pe = peoolae—> 0. 4. Un medio puede ser dicléctrico disipativo, dieléctrice sin pérdidas o buen conductor dependiendo de su tangente de pérdida, dada por Ibe et tan@ = = La, 7 oe 7 6? donde ¢, = e’ — je” es la permitividad compleja del medio. En dicléctricos sin pérdi- das, tan 8 << 1; en buenos conductores, tan @ > 1, y en dicléctricos disipatives tan 0 es def orden de la unidad, 5. En un buen conductor, los campos tienden a concentrarse en la distancia inicial § con- siderada desde la superficie del conductor. Este fendmeno se lama efecto pelicular. En el caso de un conductor de anchura w y longitud ¢, la resistencia efectiva 0 en corrien- te alterna es € Re ZG donde 8 es la profundidad peticular. 6. El vector de Poynting, ®, es el vector de flujo de potencia, de direcci6n igual a Ia de I Propagacién de la onda y de magnitud igual a In de In potencia que fluye por una uni dad de area normal a su direccion. P= EXH, Fyrom = 1/2 Re CE, x Hz) 7. St una onda plana procedente del medio 1 incide en forma normal en el medio 2, l coeficiente de reflexi6n I” y el coeficiente de transmision r estan dados por Ee ™@—-1 45 Fa m+ Eve <14r La razén de onda estacionaria, s, se define como epee 1=1rI 8. En el caso de incidencia obticua de un medio 2)10s éoeficientes de Fresnel son, in pérdidas 1 a un medio sin pérdidas _ m.c0s 8, — 1m 608 _ _2mcos 6, cn 60s 8, +7, c08 8,7 ~ 2 6088, + 1 COS 8, Yr, donde « = constante de atenuacién, 8 = constante de fase, 1 — || 0, = impedan- cia intrinseca del medio. El recfproco de « es la profundidad pelicular (8 — Ve). La re- lacién entre B, @ y A formulada anteriormente también es valida para ondas clectromagnéticas. 3. De la propagacion de ondas ent medios disipativos puede deducirse Ia que ocurre en otros tipos de medios, casos especiales de aquél. En el vaclo,7 = 0,8 = ey.) — fast on medios dieléctricos sin pérdidas, «= 0,2 — a6, ¥ # ~ Molt, ¥ en Buenos conducts fea @ = 00,8 — on. pe = peoolae—> 0. 4. Un medio puede ser dicléctrico disipativo, dieléctrice sin pérdidas o buen conductor dependiendo de su tangente de pérdida, dada por Ibe et tan@ = = La, 7 oe 7 6? donde ¢, = e’ — je” es la permitividad compleja del medio. En dicléctricos sin pérdi- das, tan 8 << 1; en buenos conductores, tan @ > 1, y en dicléctricos disipatives tan 0 es def orden de la unidad, 5. En un buen conductor, los campos tienden a concentrarse en la distancia inicial § con- siderada desde la superficie del conductor. Este fendmeno se lama efecto pelicular. En el caso de un conductor de anchura w y longitud ¢, la resistencia efectiva 0 en corrien- te alterna es € Re ZG donde 8 es la profundidad peticular. 6. El vector de Poynting, ®, es el vector de flujo de potencia, de direcci6n igual a Ia de I Propagacién de la onda y de magnitud igual a In de In potencia que fluye por una uni dad de area normal a su direccion. P= EXH, Fyrom = 1/2 Re CE, x Hz) 7. St una onda plana procedente del medio 1 incide en forma normal en el medio 2, l coeficiente de reflexi6n I” y el coeficiente de transmision r estan dados por Ee ™@—-1 45 Fa m+ Eve <14r La razén de onda estacionaria, s, se define como epee 1=1rI 8. En el caso de incidencia obticua de un medio 2)10s éoeficientes de Fresnel son, in pérdidas 1 a un medio sin pérdidas _ m.c0s 8, — 1m 608 _ _2mcos 6, cn 60s 8, +7, c08 8,7 ~ 2 6088, + 1 COS 8, Yr, 10.6. 10s. 109. 10.10. ©) La amplitud de onda es de 10 Vin. 4) El namero de onda k= 0.33 rad/m, . ©) La onda se atenda al desplazarse. Puesto que HL incorrectos? 0.5 e-8% sen (10% — 22) a, A/m, zeusles de los enunciades siguientes son a) @ = 0.1 Npim. 2) B= ~2 radim, ©) w= 10° rads. 4) La onda se desplaza a lo largo de a, ©) La onda esta polarizada en la direccion de 2. PD El periodo de ta anda es de 1 js. {Cudl es el principal factor para detorminar si un medio es vacfo, dieléctrico sin pérdidas, dieléctrico disipative © buen conductor? 4) Constante de at 1b) Parametros comstitutivos (7, ©. 8). ©) Tangente de pérdida. d) Coeficiente de reflexion, En cierto medio, F = 10 cos (10% — 3y) a, V/m. {Qué tipo de medio es? a) Vacto. b) Dieléctrico perfecto. ©) Dieléctrico sin pérdidas, 4) Conductor perfecto. Las ondas electromagnéticas se desplazan con mayor rapidez en conductores que en dicléc- a) Cierto. 1) False. En un buen conductor, Ey Hi comparten misma fasie temporal @) Cierto. 2) also, El vector de Poynting denota fisicamente la densidad de patencia que sale o entra aun vo- lumen dado en un campo variable en el tempo. @) Cierto, 2) False. Respuestas: 10.1, 10.24, f, 10:34, 10.4b, €, 10.5b, f, 10.6, 10.76, 10.88, 10.98, 10.108. Pfobigyiaiy 101. Una onda etectromagnética que se progaga én cierto medio esta descrita por —— = 25 s0n Qa X 10% — 6x) a, Vim 4) Determine la direceién de propagacin de la onda. b) Caloule el periodo 7, 1a longitud de onda Ay la velocidad 1 ©) Trace la onda en 1 = 0, 718, 774, 772 10.2.) Deduzea Ins ecuaciones (10.23) y (10.24) de las ecuaciones (10.18) y (10.20), 2) Emplee la ecuscién (10.29) en combinaci6n con las ecuaciones de Maxwell war que _ fom “a ©) Deduzea del inciso b) Ins eetiaclones (10.32) y (10.33). 10.3. A 50 Miz, un material dielécirico disipative se earacteriza por & = 3.64, & = 008 Sim. SHE, — Ge? a, Vm, ealeule: a) Yb) Ave) td) 2) FY, 10.4. Un material disipative tiene pL = Stig, & 10 raul/m, ealeule 20, Si a5 MHz In constante a) La tangente de pérdida, 2) La conduetividad del material ©) La permitiviciad compleja. 4) La constante de atenuacién. fe) La impedancia intrinseca, 485, Unmedio no megoéticn t 4) Tangente de pérdida. 1) Constante dieléetrica. ©) Permitividad compleja 4) Constante de atenuacisn a 1 MHz. yne una impedancia intriaseca de /30° 0. Halle su 410.6. La amplitud de una onda que se desplaza a través de un medio disipativo no. reduce 18% cada metro, Sila onda opera a 10 MIEz y el campo eléctrico we a campo magnéticn, caleule: a) Ia constants de propagacion, ») fa longitud de ond fundidad pelicular,d) la conduetividad del medio. 10.7. El agua de mar desempena una funcién vital en el estudio de las comuniencio Finas Suponga que respecto del agua de mar o = 4'Sim,e, = 80, = 19 f= calcule: 4) la velocidad de fase, b) la longitud de onda, c) la profundidad 3) Ia impedancia intrinsesa. 40.8. En cierto medio con 4 = Hort = dey, HL = 126° gen (ar 10% — fy) a, Alm Determine: @) el periodo T de 1a onda, b) la longitud de onda A, ¢) el campo @) la diferencia de fave entre E y Hi. 109. 10.10. soa. 10.14, toas. Fa cierto medio, 166845 sen (2 X 10% — 2x) a, Vim Halle: a) Ia constante de propagacién, b) la longitud de onda, ¢) la velocidad de la onda, ‘d) la profundidad pelicula ‘Una onda uniforme en el aire tiene = 10 co (2m X 10% — Bz) a, a) Caloule By A. 5) Trace Ia onda en z= 0, A/4, ©) Malle H. La componente del campo magnético de una onda electromagnética que se propaga 4 tr ‘és de un medio no magnético (42 ™ #4.) es HE = 25 sen (2 10% + 6x) a, may Determine: 4) La direccin de propagacién de ta onda, b) La permitividad del medio. ©) La intensidad de campo eléctrico. SiH = 10 sen (wr ~ 42)a, mA/m en un materi AY Ie en el cual o = 0,41 — Hy, # = 46, calcule Un fabricante produce un ferrito con 41 = 750Htg, € = Se, ¥ a = 10°" Sim a 10 Ez. 4) {Clasificarta usted este material como un médio sin pérdidas, dis 2) Cateule By 2. ©) Determine Ia diferencia de fase entre dos puntos separadas por 2 mi 4) Halle 1a impedancia inteinseca, ipativo © conductor? “Tras suponer los campos dependientes del tiempo Fi = Bye 1°) y Hi = Hye 1°, don- dek = k,n, + km, + fea, ox el vector de nomero de onda yr = xa, + ya, + 2a, cl vector de radio, demuestre que V XE = — oB/ar puede expresarse como kX E'— ww y deduzea aX ae ae ‘Suponga los mismos campos descritos en el problema 10.14 y demuestre que en una region sin fuente Ins ceuaciones de Maxwell pueden exprosarse como k-E=0 Kx ES out XH = —oek: 10.16. 10.17. a8. 10.20. 10.22, 10.23. Deduzea de estas ecunciones Xap ay ym X ay = Op La componente del campo magnético de una onda plana en un dieléetrico sin HI = 30 sen (2m X 10% — Sx) a, mA/m. a) Sip, = 1, halle b) Caleule Ia longitud de onda y la velocidad de onda, ©) Determine la impedancia de la onda. 4) Determine la polarizacién de la onda. ©) Halle la correspondiente componente del campo eléetrico. P) Malle 1a densidad de corriente de desplazamiento. En un medio no magnético, E = 50 cos (10% — Bx) a, + 40 sen (10% — Bx) a, View Halle In constante dieléctrica #, y el Hi correspondiente. En cierto meclio E = 10 cos (2% % 10% — Bx\(a, + a) Vim Sip. = 50p1y, 8 = 20, ¥ = 0, halle By Ht. 4) Tierra pantanosa hmeda (6 = 1564, 40 = Hay © = 10"? Sim). 2) Germanio inteinseco (© = 166,44 = Hy, & ~ 0.025 Sim), ©) Agua de mar (6 = 81e,. 44 = fay, & = 25 Sim), Calcule la profundidad pelicular y la velocidad de propagacién de una dada pl que se desplaza en eloruro polivinilice (i, ~ 1,6, 4, tan 8 — 7 10-2) au dee Mie, Una onda plana uniforme en un medio disipative tiene una constante de fase 4 10” Fiz, en tanto que su magnitud se reduce 60% por cada 2m recorridos, didad pelicula y Ia velocidad de tn onda, 4) Determine la resistencia en corrieiite directa de un cable ‘Hedon« (@ =5.8 © 107 Sim, 4, = 1,8, = 1) de 1.2 mm de radio y 600 m de longit +b) Hille In resistencia on corriente alterna 100 MHz. a ©) Caloule Ia frecuencia aproximada en corriente alterna son iguales. “ Un tubo de aluminio (a = 3.5 107 Sim, 4, — 1,£, — 1) de 40 m de largo cont no'y externo de 9'y 12 mm porta una corriente total de 6 sen 10°77 A. Hall didad pelicular y Ia resistencia efectiva del tubo. Demuestre que en un buen conductor la profundidad pelicular 8 es siempre: que la longitud de onda. 1025. Las gufes de ondas de cobre austen recubrirse de plata para reducir piétdidas. 8 ol grosor minimo de la plata (44 = #1, € = ey. = 6.1 10" Sm) debe ser de 53, determine 6! pro Sor mfnimo requerido para una guia de ondas que opere a 12 GHz. 10.26. ‘Una onda plana uniforme en un medio disipative no mngnétice tiene: Ga, + 12a,e-%,y = 02 + B.A/m. 4) Caloule la magnitud de In onda én =~ 4m. 44) Halle 1a pérdida en dB sufrida por la onda en el intervalo 0< 2 <3 m, ¢) Caloule el vector de Poynting en z =4,¢= 778. Adopte w = 10" rads. 10.27. En un material no magnético, B= 30 cos (Zar 10% — 6x) a, may Halle: 4) 1a impedancia intrinscea, 6) el veetor de Poynting, c) la potencia promedio tempo- ral que atravieta Ia superficie x= 1,0- 1'm,2 mm s 2. Los conductores de cada linea se caracterizan por oq fe ¥ & = #4: €N tanto que el dicléctrico homogéneo que los separa se caracteriza por 0, 4 ¥ ©. 3. G+ 1/R: Res la resistencia en corriente alterna por unidad de longitud de los con- 4, El valor de Z referido en la tabla 11.1 es la inductancia externa por unidad de lon- gitud, os decir L = Iq. Los efectos de la inductancia interna Ly_ (— Ri) son insignificantes a altas frecuencias, en las que opera Ia mayor parte de los sistemas de comunicacion. 5. En cada linea, Lempe y Gat ay En prevision de la siguiente seccidn, considérese In propagacién de una onda electro magnética a través de una linea de transmisién de dos conductores como la linea coaxial que ‘conecta a un generador 0 fuente con una carga en la figura 11.4(a). Cuando el interrup- tor Sse cierra, el conductor interno se vuelve positive respecto del externo, de modo que @ © © Figura 11.2. Lincas de transmision comunes: (a) linea coaxial, (b) linea (Grdanchadivan 40 Wack pide. Figue 11.3. Parémn rom distribuidos de un: linea de transmision de dos conductores, el campo F irradia hacia fuera, como se ilustra en la figura 11.4(b). En ésta muestra que, de acuerdo con Ia ley de Ampere, el campo H circunda al conduc tador de corriente. Fl vector de Poynting (E x H) apunta a lo largo de la linea mision. Ast, cl cierre del interruptor causa sencillamente una perturbacion qu Ia forma de onda clectromagnética transversal (ET), la cual se propaga a lol Mea. Esta onda es una onda plana no uniforme por medio de la cual se transi cia a través de Ia Iie © Figura 11.4. (@) Linea coaxial que conecta al geniétador con la cary Fea a5" gr ing Ecuaciones de linea de transmisién Como se mencion6 en la seccién anterior, una Iinea de transmisién de dos conductores soporta una onda ET; es decir, los campos eléctrico y magnético en la line son transver~ sales ala direccién de propagacién de la onda. Una propiedad importante de las ondas ETT es que los campos Ey H se relacionan en forma especifica con el voltaje Vy la co- rriente 7, respectivamente: -fe-a ro fara az) Asi pues, en la resolucién de problemas de Iineas de transmision emplearemos las canti- dades de circuitos Ve Jen lugar de las cantidades de campos Ey Hi (es decir, en ver dle Tas echncionss de Maraiil y Inn qrmiiciones am In frotitern), Fil modelo de cieuseat ad sn este caso mds simple y practico. Examinemos una poreién incremental de longitud Az de una linea de transmisién de dos conductores. El propésito es hallar un circuito equivalente a esta inca y deducir las ‘feuaciones de Iimea de transmisiOn. De la figura 113 se desprende que el cireuito que aparece en la figura 11.5 es el cireuito equivalente a una poreidn de la Iinea. Este mode- Io hace suyos los pardmetros R, L. Gy © de las linens de transmision y puede represen- tar # cualquiera de las Iineas de ‘dos conductores de la figura 113. Liamado cireuiro equivalente tipo L,este modelo no es el Gnico posible; hay otros (véase el problema 11-1), En ¢1 se supone que Ia onda se propaga a lo largo de la direceién +z, del generador a i carga. ‘De la aplicacién de la ley del voltaje de Kirchhoff a la espira externa del cireuito de Ja figura 11.5 se obtiene Vie) = RaeKen + eet + vee + ae) ¥@ + 2.00 — Vz. t) = Rie) +z UG D) a1) a ht tet Mee + Aen DASE = a Figura 11.8. Modelo de circuito tipo L de longitud diferencial Az ‘equivalente a una linea de transmision de dos conductores. La adopeicn del limite de la ecuacién (11.3) cuando Az —> 0 produce r av(z.t) ana.) MED pie.) + LUE De igual manera, la aplicaci6n de la ley de la corriente de Kirehhoff al nodo cireuito dela figura 11.5 da como resultado, Hz. Ia + Azo + Ar ovis + Age), = Me + Ant) + GAVE + Az + C AZ = Met On) 189 ayes and + ME TSE Cuando Az — 0,1a ecuaci6n (11.5) se convierte en VEO ale. #) GO LGva.n +e Si suponemos dependencia de tiempo arménico de tal forma que Ve.) = Re[V,@) em} Mz, 8) = Re[L(2) &) donde V,C2) y 1,(2) son las formas de fasor de V(z.1) € I(z, ), respectivament clones (IT.4)'y (11.6) se convierten en V, y J, estsin acoplados en estas ecuaciones diferenciales, Para separarlos se ol da derivacia de V, de la ecuacion (11.8) y se emplea la ecuacion (11.9),de lo que) ey, az = (K+ jal VG + jac) Ve donde ana ‘Al obtener a su vez ln segunda derivada de J, Alobeenst de In ecuacién (11.9) y emplear la ecuacion 11.8) resulta, @.a2y Cabe hacer notar que las ecuaciones (11.10) y (11.12) las ecuaciones de onda para vol obs Morslente, respectivamente-- son de forma similar a Ia de las ecuaciones de onde we Zonas planas (10.17) 9 (10.19), En nuestra noracién usual, axf,on le ceuacign (11-11) ear ee Condtante de propagscin (por metro), « Ia constante de avenuuscion (en nepers se eee decibeles: por metro) ¥ 6 la constante de fase (en radianes por metro). La Rou ginid ae onda Ay la velocidad de onda 1 estan dadas respectivainente por an q1asy ua Sap aiaay La solucin de Las ecuaciones diferen« semejante al caso B del ejemplo 6.5. ies lineales homogéneas (11.10) y (11.12) es Via = Veer + Veer a1asy 12) = Ie + Tze cis) donde Vi, Vz, If ¢ Ig son amplitudes de onda y los signos + y —denotan que Ia onda se gene Med XG laigo de In direceion -+2 ¥ ~z, respectivamente, como lo indican asimismo {ke Rochas, De este modo, la expresi6n instanténea del voltaje es V(z.0 = Re (Vs) eo] we et cos (we — Bz) + V5 ev cos (we + 87) quay ,, de una linea de transmision es la razon de 1a onda Foslive's la’ oncla de corviente en cualquier punto en ‘FRecuérdeve que, de acuerdo con Ia ecuacisn (10.35), 1 Np = 8.686 dB. Z, 6s andloga a 7, la impedancia intrinseca del medio de propagaci6n de la or {ituir las ecuaciones (11.15) y (11.16) en las ecuaciones (11.8) y (11.9) © ig cientes de los términos e” ye” se obtienc. an A y Poet Fist ¥ G+ jac REL : Ze Gnd Re te donde R, y Xq ron las partes reals imaginaria de Zn. Ry no debe confuiliras Se mide en ohms por metro, y R, en ohms. La constants de propagacion ¥ yl cia caracteristica Z, son propiedades importantes de la Ifnea, porque ambas di los parametros R, £, Gy C'y la frecuencia de operacién. El reetproco de Z,, tancia caracteristica ¥,,es decir ¥, = V/Zq. ‘La Ifnea de transmision considerada hasta aquf es del tipo disiparive, ya ductores que Ia componen son impertectos (7, 4 00) y el dicléetrico en el qi mersos es disipativo (o » 0). Habiendo deserito este caso general, examinemos casos especiales: los de Ins Iineas de transmisiGn sin pérdidas y sin distorsion, A. Linea sin pérdidas (R = 0 = G) Una linen dé transinision sin pérdiday Consta de conductores perk ses LSI Sie 5 TEES Como se desprende claramente de la tabla 11.1, cuando &. = coy & = 0. R=0=G condicién necesaria de una Iinea sin pérdidas. Respecto de una linea de este n (11.20) convierte las ecuaciones (11.11), (11.14) y (11.19) en @=0, y= 1 = jo VEE 117.3. FeUACIONES OF LINEA OE THANSMNSION, 481 B. Linea sin distorsién (R/L = G/C) Una sefal consiste normalmente en una banda de frecuencias; en una linea disipativa, la amplitud de onda de componentes a distinta frecuencia se atenuara de diferente manera, Puesto que @ depende de la frecuencia. Esto resulta en distorsion. De acuerdo con Ia expresin general de @ y [referida en la ecuacién (11.11)} una linea sin distorsion es consecuencia de que los parametros adopten la forma siguiente Bog 1.22) En una Ifnea sin distorsién, ast, vo Vee ECE) ~ VRE (14/2) =a sip . a=VRG, po oVIE «a1.234) lo que indica que mientras que @ no depende de la frecuencia, f es una'fuincion lineal de Ia frecuencia, De igual modo Z n-fE-JE x. <0 1230) y an Ze—Lep (11.236) a" Vee Aaviértase que 1. La velocidad de fase es independiente de la frecuencia, a causa de que la constan- tede fase f depende jente de la frecuenci, A menos «ue q-y «sean inde pendientes de la frecuencia, la forma de las senales sufrira distorsion, 2. wy. Zeon iguales que en. las lineas sin pérdidas. Bjemplo 11-1 “abla 11.2. Caracteristicas de las linens de tansmisién, cast qa treet natinme neh General Vin eth fot) RIE eee rear ee pas er eve VE+w 3. Una linea sin pérdidas carece también de distorsi6n, pero una linea sin dis ma nnensariamentn cxrece re pardiclan Law ineas sin pérdidas son deseahl (ransmisiGn de polencia, en tanto que las Ifneas telefonicas deben ser I distorsion. Bn Ia tabla 11.2 se presenta un resumen del contenido de esta seccién, Nuestro se restringird casi exclusivamente a lineas de transmisin sin pérdidas. Una Iinea en el aire tiene impedancia caracteristica de 70 © y constante de f¢ radim a 100 MHz. Calcule su inductancia por metro y capacitancia por metro. Solucis: Una Iimea en el aire puede considerarse una Ifnea sin pérdidas, ya que « ~ 0. Por La division de ta écuacién (11.1.1) entre Ia ecuacién (11.1.2) prodyee Re _ 1 Bas a 3 © aR, ~ Bar % 100 = 10°70) = 68.2 prim A partir de la couacion (11.1.1), LL = RIC = (70)(68.2 * 107!) = 334.2 n/m eee ee dte Ian (aeade (komution tis geese iu ceee 2, = 15 rad/m. Fale sus pardmetros R. 1. Gy C- Respuesta: 3.2 0/m, 38.2 nH/m, 5 x 10-* S/m, 5.97 pF/m. ‘Una Hinea sin distorsién tiene Z, = 60.0, « ~ 20 mNpim, u = 0.6¢, donde es Ia veloci- dad de la luz en el vacio. Halle &, LG, Cy Aa 100 MHz, Solucién: ep una linea sin dictorsisn, RC RO=GL.0 o=*E Por tanto 124) 1.2.24) R=aZ, 1.220) Pero wed ue o he 12%) A partir de la ecuacion (1.2.26), R= 0 Z,= QO x 10-)(60) = 1.2.0/m La division de Ia ecuaci6n (11.2.1) entre la ecuaci6n (11.2.3) resulta en Fe Oe sag acim um” 0.6 10%) ~ 399 BF Con base en Ia ecuacién (11.2.2), a _ 400 x 10-6 ouR 12 33 wS/m, 11.4. Impedancia de entrada, razén de onda estacionaria y potenc Al multiplicar las ecuaciones (11.2.1) y (11.2.3) se obtiene a uz Lee - UZ, ~ 06 G * 10) 66 ‘Respuestas: ) 70.7: ZELB6T? O, b) 2.121 10-4 + jB.8BB 107 ‘os mis. : : Considérese una linea de transmisi6n de longitud € caracterizada por-7¥ Z. ¥ a una carga Z,, como se muestra en la figura 11,6, Si se exai , Carga represcata para el generador Uns impedancia de entrada Zyq. Nuts Gn esta scecion es determinar la impedancia de entrada, la razén de ond (ROE) y el flujo de potencia en la linea. 1 ‘Concedamos que la Iinea de transmisin se extiende de z — 0 én el generat en la carga, Antes que nada, precisamos de las ondas de voltaje y corrientte. ciones (11.15) y (11-16), es decir, de Vq) = Viren + Vers | oe ee donde se ha incorporado la ecuacién (11.18). Para hallar Vj y V5. debe disp Condiciones en la terminal. Si, por ejemplo, estan dadas las’ condiciones en Ia gamos Ve=Ve=0. t=Ke=0) © © términos de Z,,€n la entrada Ja sustitucion de estos valores en las ecuaciones (11.24) y (11.25) resulta en VE BU Zale) 1278) We shes 2 127) Nie whe Si la impedancia de entrada en las terminales de entrada es Z,,., el voltaje de entrada V, va corriente de entrada Z, se obtienen [acilmente de la figura 11.6(b) como i Vs Me gre ei te So ee ae Si, por otra parte, estan dadas las condiciones en la carga, digamos Vp =VE-0. hale 129) La sustitucin de estos valores en las ecuaciones (11.24) y (11.25) da como resultado vs a ve EM, + Zhe? 1.302) jet (41.308) 1 -im - Después se determina la impedancia de entrada Zou ~ V,(Z\/I(z) en, to en la linen. En el generador, por ejemplo, las ecuaciones (11.24) y (11.28) | Vile) _ Zl +VE) | Zon = Ta) Ve = Ve Al sustituir la ecuacisn (11.30) en Ta ecuacién (11.31) y utilizar el hecho deg etre coon ye, = =] x senh ye eo tanh ¥6 = Cosh 6 et ee se obtiene [EE] cations a Zq + Z, tanh ye Aunque deducida con referencia a la impedancia de entrada Zoy. en el € Gon. la ecuacién (11.33) es una expresién general para determina Zags punto de la linea, Para hallar Z,,, a una distancia €” desde la carga, como Ti6ra), se reemplaza € por ¢”. En el apéndice A.3 se proporciona una formula lar la tangente hiperbéliea de un numero complejo, necesaria en la ecua En cl caso de una linea sin pérdidas, y — jf, tanh jB€ = j tan BE y Z, que la ecuacién (11.33) se convierte en an of con penny Zi (Zain se) Z,, + jZ tan Be to que indica que la impedancia de entrada varta periddicamente con Ia dis | dela carga, La cantidad é de la ecuacin (11.34) usualmente es la /ongitud i la linea y puede expresarse en grados 0 en radianes. Definamos ahora I’, como el coeficiente de reflexion por voltaje (en la | la razon de la onda de reflexion por Voltaje a la onda incidente en la carga, ¢ ver Vee ‘La sustitucién de V,~ y Va* de la ecuaci6n (11.30) en la ecuacion (11.35) ¥ | cién de V, = Zp, resultan en nu maz I et > mI | El cocticiente de reftexién por voltaje on cualquier punto de ia linea es ia raza de | lwemagnitud de la onda reflejada por voltaje a la de la onda incidente, uae oe Sin embargo, 2 = € — Al sustituir y combinar con Ia ecuacién (11.35) se obtiene re) - fF ae = Tyenset @137) TEAC REE Heniius seuss ee ee ccal ne maral Coaracna Gamaplanan igyneee te-ondere wetonaria (HOR) «0 define como Tas 1+ IPL Tin 1 — IEet te} Es facil demostrar que Imi = VnudlZe € Inn = Vinw/Zq- Los valores maximo y minimos de la impedancia de entrada Z, de la ccuscién (1134) ocurren en los valores maximos Ininimos, respectivamente, de la onda estacionaria de voltaje y corriente, También es posible demostrar que Vande 1Zeetl a 82 (11.394) Tie 1.395) Para comprobar estos conceptos, considérese una linea sin pérdidas con impedaneia caracteristica de Z, = 50 Q. Para efectos de simplificacion, supongamos que esta linea termina en una carga resistiva pura Z, — 100 ©-y que el voltaje en la carga es de 100 V (ems). Las condiciones en la nea se presentan en Ia figura 11.7, en la que se advierte que tales condiciones se repiten cada semilongitud de ond: 0 Be radianes afew Figura 11.7. Patrones de ondas de voltaje ¥ corriente en una linen sin pérdicas terminada en una carga resistive, Como se mencion6 al principio de este capitulo, una Iinea de transmision transferir potencia de una fuente a una carga. La potencia de entrada promedi tancia € desde la carga esta dada por tuna ccuacién semejante a la ecuacién (10.68); 1 os Foam = 3 KE (VET) | donde el factor 1/2 es nécesario, puesto que tratamos con los valores pico en I valores rma. Suponiendo una linea sin pérdidas, se sustituyen las ceuaciones (11.25) para obtener a 1 Ppa =F pa fprertss rere BE Geme— rhes SareflE a ire ere reer] Puesto que los dos diltimos términos son puramente imaginarios, se tiene El primer térniino es la potencia incidente P; y el segundo Ia potencia reflejada P,. Ast, Ia ecuacién (11.40) puede expresarse como P.=P,-P, donde P, es la potencia de entrada 0 transmitida y el signo negative se debe a la onda de direccion negativa, puesto que la direccién de referencia es la del voltaje/eorriente que se desplaza hacia la derecha.. Cabe sefialar con relacién a la ecuacion (1.40) que la poten ia es Constante y no depende de €, ya que tratamos con una linea sin pérdidas. Advicriase fasimismo en que la carga recibe la potencia maxima cuando I’ = 0, como es de esperar. Examinemos ahora los casos especiales representados por la conexién de la linea & una carga Z, ~ 0,2, — Soy Z, — Z,. Estos casos pueden deducirse fécilmente del caso general. A. Linea en cortocircuito (Z; — 0) En este caso, Ia ecuaci6n (11.34) se convierte en Zee = Zom|_ = iZa tan Be asia) Asimismo, Eo -i, ssc aay Debe sefalarse respecto de Ia ecuacién (11.412) que Z.., es una reactancia pura, la cual puede ser capacitiva o inductiva segun el valor de €. La variacin de Z.,, con € se mues- tra en la figura 11.8(@). B. Linea en circuito abierto (Z, = ©) Esta vez Ia ecuacion (11.34) se convierte en z Jim Zen = FZ 25g = —J%Z, cot BE 11.820 mee peanpe ie ad ¥ Tek!) go 1.426) La variacién de Z. con € se muestra en Ia figura 11.8(b). Adviértase que, a partir de las ecuaciones (11-41a) y (1142), ZsLee = Z3 a1asy C. Linea acoplada (Z, = Z,) Poste es el caso mas deseable desde el punto de vista préctico. En él 1a ecunci6n (11.34) se reduce a (44a) Lani = 2 a, Figura 1.8. Impedancia, de una linea sin perdi Sortocireuito, (b) en cites © es decir, Vz = 0, se transmite la onda completa y no hay reflexion, La potencia te es totalmente absorbida por la carg: sion ests acoplada con Ia carga es posible una maxima transferencia de potencia. “iempio 12is | Ciera linea de transmision que opera a « = 10° rad/s tiene « = 8 dB/m, 6 = 1 | 2. = 60 + 740 Oy 2 m de largo. Si esta conectada a una fuente de 1070" V, Z, y termina én una carga de 20 ++ /50 0, halle 4) La impedancia de entrada, 6) La corriente en el extremo emisor. ©) La corriente a la mitad de la linea. Soluci6n: a) Pucsto que 1 Np = 8.686 dB, 5 oc = ggg 0-921 Np/m ya + jp = 0921 + flim ye = 2(0.921 + jl) = 1.84 + 72 De Ia aplicacion de la formula para tanh(x + jy) referida en el apéndice A.3 se obtiene tanh y€ = 1.033 — 0.03929 Zi + Zatanh ye © \ Zo Zi tanh ye s 20 ++ J50 + (60 + #40)(1.033 — 0.03929) 60 + 140) | p= fay + (20 + /80}CL- 03a = 70.09929) | Zom = 60.25 + 738.79 0 Fie 1b) La corriente en el extremo emisor es 1( De acuerdo con la ecuacién (11.28), Ye 10. Me) = Zo Z, ~ 6025 + 738.79 = 93.032-21.15" mA a0 ©) Para hallar la corriente en cualquier punto, se precisa de 1, = 1(@ = 0) = 93.032=21.15° mA Va = Zany = (71.66 £32.772)(0.09303/—21.15°) = 6.667/11.62°V A partir de ta ecuacién (11.27), Vg y Ver-Sin embargo, + ig, Ve = $Me + Zola) = £[6.667211.62" + (60 + #40)(0.09303/—21.15°)] = 6.687/12.08" ‘Se. $e — Zola) = 005182260" ‘A la mitad de fa linea, z = €2, yz = 0.921 + fl. Por tanto, la corriente en este punto es WE ote YE gr Ife 02) = Yen — ee (6ouTe!206)¢-01-__(O.0518eRH) IATA 60 + /40 “60 + 740 ‘Vale hacer notar que /1 est en radianes y equivale a /57.3°. Ast, 6.687720" 6-09 g-A72" 9, 95186200 O91 gists Iz = €2) = oa ee = 0,0369¢-782" — 0.001805" 6.673 — 34.456 mA 35.10/281° mA eee Vp = 1320" View Za 30 eH ees Sea Vine Si este aco ian te ee " GA Sens Baie Fal cl extreme emisor, ©) La constante de propagacion y. ron toda suerte de recursos (tablas, diagramas, graficas, ete.) para facilitar sus de disefio y anilisis Fue ast como surgieron medios graficos para reducir las ted hipulaciones implicadas por el cdlculo de las caracterfsticas de Iineas de transmis diagrama de Smith? es la técnica gréfica de uso mas comtin con ese propésito. S basicamente de una indicacion prafica de la impedancia de una lines de transmis forme se avanza a lo largo de ésta. Basta un poco de practica para dominar su ef 3 Inventado por Phillip HL. Smith en 1939. Véase PH. Smith, “Transmission line galculator"y trontes, vol, 12, 1939, pp. 39-31 y PH. Smith, “An improved transmission line esleulator” files, vol. 17, 1944, py 130-133, 318-325. Figura 11.10. Circulo de radio igual a Ia unidad Gnvel que te labora el diagrama de Smith. Examinaremos en primer lugar cémo se elabora el diagrama de Smith y después lo uti zaremos para calcular caracterfsticas de Iineas de transmision como T,,$¥ Zuye Aunque RO ¢s forzoso que sea asf, supondremos una Iinea de transmision sin pérdidas (2, — R,)- ‘El diagrama de Smith se traza dentro de un cireulo de radio igual a la unidad (IC| = como se muestra en la figura 11,10, Su elaboracidn se basa en la relacién enunciada en ia ‘feuacién (11.36) esto es, a1asy a IPlZe= 4, +7, (1.46) donde , y P, son las partes real e imaginaria del cocfitiente de reflexién I. ‘Para disponer de un solo diagrama de Smith aplicable @ cualquier linea de transmi- si6n —lo cual cs preferible a elaborar uno por cada linea con diferente impedancia ca- Pacteristica, como Z, ~ 60, 100 y 120 2—, se tsa un diagrama normalizado en el que todas las impedancias estén normalizadas respecto de la impedaneia caracteristica Z, de Ia Iinea particular en consiceracion. Fn el caso de la impedancia de la carga Z,, por ejem- plo. la tnpedancia normalizada 2, ests dada por — aun La sustitucién de la ecuacién (11.47) en las ecuaciones (11.45) y (11.46) resulta en od rer.4yr= a1.48a) Ser 1.486) Cuando P no va acompaflade por un subindice, alude al coeficiente de reflexion por voltaje en Ia carga (I, =T), La normalizaciOn ¢ igualacién de las componentes produce 1-r-r? -a-1yee ary a Pe FF @ Al reordenar los términos de Ia ecuacién (11.49) se obtiene [Booed Melia Estas dos ecusciones son similares a m+ Owe = Gere) radio = centro en (I, T+ En la tabla 11.3 se presentan los centros y radios de cfrculos r correspondientes a comunes de la resistencia normalizada r, y en la figura 11-11 los clreulos r basi ‘Tabla 11.3. Radios y centios de circulos reorrespondlientes valores metered soremiotg Ramd OT). coed (G0) ° A 0.9 ve aa oo} y 8 6233 2 ta Gay 3 te G0) ee é “ Figura 11.11. Circulos r correspondientes a r= 0, los datos de la tabla 11,3. La ecuacién (11.51) es a su vez un cfreulo x (efreulo de reac- fancia) con centro en (F171) = (1.2) «at.sée) ito = 2 1.540) En la tabla 11-4 aparecen los centros y radios de circulos x correspondientes a valores co- munes de x, y en la figura 11.12 el diagrama respectivo. Notese que mientras que r siem- pre es positiva, x puede ser positiva (en el caso de impedaneia inductiva) o negativa (en él de impedanéia capacitiva). ‘De la superposicion de los circulos ry x resulta el diagrama de Smith, como se mues- tra en la figtra 11.13, Bn él es posible localizar una impedancia normalizada 2 — 2+ J, por ejemplo, en el punto de interseccion del circulo 7 — 2 y el cireulo x — 1, el punto Py en la figura 11.13. De igual forma, z = 1 ~ 0.5 se localiza en Pz, donde se intersecan el Strode r= Ty 6] afreule x —0.5. “Aparte de los cfrculos r y x (mostrados en el diagrama de Smith), es posible trazar ctrenlos + 0 circulos de razon constante de onda estacionaria (los cuales nunca aparecen en el diagrama), centrados en el origen en tanto que s varia de 1 aoc. El valor de la razon, de onda estacionaria s se determina localizando el punto en el que un cfrculo s se cruza nabcrnclnepratt py matty (2) abe) hgh Figura 11.13, Tusteacin de los efveulos r,x y s en el disgrama de Smith. con el eje I, En Ia figura 11.13 se presentan ejemplos representativos de los efrculos # co- rrespondientes as — 1, 2,3 © cc, Puesto que ITly # se Telacionan de acuerdo con tn ceune cién (11.38), a los cfrculos s también se les conoce como efreulos IPl-en los que ITI varia linealmente de 0 a 1 conforme se avanza del centro a Ia periferia del diagrama, micn_ tras que s varfa de modo no lineal de 1 a oo. Conviene senalar lo siguiente acerea del diagrama de Smith: 4. En cl punto 7. del diagrama, r = 0,x = 0; es decir, Z, ~ 0 + /0,1o que indica que Poo representa un cortocircuito en la linea de transmisin. Fn el punto Pay? = OS ¥¥ = 00,0 Z,, = 00 + joo, lo que implica que P., corresponde a un cireuito abier- to en Ja linea. También en P.,,7 = Oy x — 0, de modo que abi se ublea otro corto- circuito en la linea. 2. Una revolucién completa (360°) en el diagrama representa una distancia de 4/2 en Ja linea. 1:1 desplazamiento en el diagrama en la direceion de las man 1oj equivale a desplazarse hacia el generador (o en direccion contraria ala carga), como lo indica Ia flecha G en las figuras 11-14 (a) y (b). Por consecuencia logica, el desplazamiento en la fale a desplazarse hacia la carga (o en direccién contraria al generador), como lo indi ca asu ver la flecha Jon la figura 11,14, De la figura 11-14(b) se desprende que, en la carga, no tiene sentido moverse hacia Ia carga (porque ya se esté en ella). ‘que lo mismo puede decirse respecto del extrem del penerador 3. En Ia figura 11,14(q) se detallan tres escalas en la periferia del diagrama. Inclui das para mayor utilidad, tienen sin embargo el mismo propésito, de modo que deberta ser suficiente con s6lo una de elias. Su fin es determinar en grados 0 lon- gitudes de onda la distancia desde la carga o el generador. Asi, el de las escalas ‘exterior e intermedia es determinar en longitudes de onda la distancia en la linea, desde el extremo del generudor y desde la carga, respectivamente, mientras que la escala interna es un transportador (en grados) para determinar 9, y Ia distancia des- de la carga 0 el generador. Puesto que una distancia de 4/2 en la linea corresponde a un desplazamiento de 360° en el diazrama, la distancia Aen la linea corresponde aun desplazamiento de 720" en el diagrama A 720° ass) De esta manera, es posible ignorar las escalas externas y usar Unicamente el trans- portador (Ia escala interna) para todos los edleulos de ®;-¥ distancia 4 Ving OcurTe en Ia ubjcaci6n de Zon, man 08 el diagrama [véase la couaciSn (11.394)}, © Sea, en la figura 11-14(a), en ef eje T positive 0 en OP.,. Va se localiza por SU parte en el punto con Zou, min 6M el dingrama o en la figura 11-14(a), en el cje F, hegative o en OP. Notese que Vina. ¥ Vins (© Zemin ¥ Zon.mm) e8t6n separados por 4/4 (6 180"), FI diagrama de Smith sirve Io mismo como diagrama de impedancia que de admitancia (¥ — 1/2). Como diagrama de admitancia (impedancia normaliza- day — Y/Y, ~ g + 7b), los cfreulos gy b corresponden a los cfreulos r yx res cei ¥.@ Generndor Tinea de trapamision ‘Caren cy Figura 11.14. (a) Diagrama de Smith en el que se ilustran las escalas en Ia periferia y los desplazamientos alrededor; (b) desplazamicntos correspondientes a lo largo de la linen de.tranamisién. Con base en estas importantes propicdades, el diagrama de Smith puede ra determinar, entre otras cosas, a) F = |T|Z0, ¥ 8: b) Zog: 0 Youn ¥ ©) 18s Ubi Veeos ¥ Vau slempre que se disponga de los valores de 2., Z,'y la longitud: Allguinos djemplos ilustrardn claramente como realizar todo esto y mucho mas, da del diagrama de Smith, un compas y una regla. Una linea de transmisiGn sin pérdidas de 30 m de largo con Z, = 50 01 aud pera a2 MTs Termina en tna carga do Z, © 60 + J40 1 Siu = O.6¢ en la linea, halle @) El cooficiente de reflexién T. Pirtaaacts Gavia tathctcntere ©) La impedancia de entrada, Sotuetén: Este problema se resolverd con y sin el dingrama de Smith Método 1 (sin el dingrama de Smith). ZZ, - 60+ 40 — 50 _ 10+ 40 Zev Z, ~ 6+ j40 + 50 ~ 110+ 740 0.3523 60° _ 4+ IF] _ 1+ 03523 TTF} ~ 10.353 ©) Bliesto gue u = of oA = ol, Wwe _ 282X100) _ 27 T-ee Recuérdese que At es Ia longitud elgetri Zi Zatan pe] Z. + jz, tn Be |_ 5060 + }40 + f50-tan 120°) - [50 + j(60 + 740) tan 120°] FSOG IRS IEVEY og oy pace G+4V3—16V8) = 2397 + f1350 Método 2 (con el dlagrama de Smith). ar bs 088 ee= a de la linea, aM 4) Se calcula la impedancia normalizada de la carga _ Ze _ 60+ 740 us aa 12 +08 En el diagrama de Smith que aparece en la figura 11.15, z, s¢ localiza en el punto P, don- de se cruzan los efreulos r= L2y x — 0.8, Para obtener fen z,,26 extionde OP hasta cru- far con cl efrculo r — 0, lo cual acurre en Q, y se mide OP y OQ. Puesto que OO Corresponde a [TF] — 1.en _ OP 320m IF = 99" dem = 0.516 Figura 11.15. vara el ejemplo 11.4 Cabe sefialar que OP = 3.2 em y OQ = 9.1 em proceden del diagrama de Smith g el autor: el que aparece en Ia figura 11,15 es una reduccion, pero manticne Ia raZOR El Angulo 6; se obtiene directamente del diagrama como el angulo entre O3 6, = angulo POS = 56" Ast | 1 = 0.3516 /56° | ») Para determinar Ja raz6n de onda etacionatia s, se traza un circulo con OF) dio y contro en O. Este es el eirculo de s constante 0 [T, Después se localiza el | qadl qus el tile p warm enam gat [Esto se demuestra facilmente estableciendo I’, = 0 en la ecuacién (11.492). El valor de ren este punto es 5: esto es, = =r (cuando r= 1) 21 ¢) Para obtener Zaq, primero se expresa € en términos de Ao en grados. te _ 0.6 (3 108) se So Cee a 30 €=30m= a= En raz6n de que A corresponde a un desplazamiento angular de 720° en el diagrama, la Iongitud de la linea corresponde a un desplazamiento angular de 240". Esto equivale & desplazarse 240° hacia el generador (o en direcciGn contraria a la carga, en el sentido de las del reloj) en el circulo s, del punto P al punto G. En G se obtiene 0.47 + 70.035 Por tanto, 2 Zckan = 500.47 + 0.035) = 23.5 + 1.75.0 Aunque ¢l diagrama de Smith proporciona resultados aproximados, para los fines de Ia ingenieria son sulicientemente cereanos a los exactos obtenidos con el método 1. Bae. ‘Una carga de 100 + j150 0 ests conectada a una Iinea sin pérdidas de 75 0. Halle: or bys ) La admitancia de la carga ¥;. 4) Zone 00 0.4 A desde la carga. ©) Las ubicaciones de Vinge ¥ Vin Tespecto de la carga si la Ifnea es de 0.64 de largo. A) Zoq, en el generador. Sotucién: @) Este problema puede resolverse con el diagrama de Si da de la carga cs 100 + 150 75 P se obtiene OP _ 6em 00 ~ 9.1m Angulo POS irl ane En consecuencia, P= 0.65040" - La impedaneia: 1.33 + 72 Esto equivale al punto P en el diagrama de Smith que se presenta en la figurs 0.659 Comprobacion: _ Zn ~ Z. _ 100 + j150 = 75 Ziv Z, 100+ fis0+ 75 a 0.659740" b) Se traza el circulo de s constante que pasa por P y se obtiene == 482 (Comprobacion: 1+ IFl _ 1+ 0.659 STS Irl 7 p= 0.659 = 4865 ¢) Para obtener ¥;, PO se prolonga a POP’ y se identifica el punto P’, donde el cfreulo Sec comstante se cruza con POP’. En P’ se obtiene yn = 0.228 — j0.35 La admitancia de la carga es Yi = Yays = Ay (0208 ~ j0.8) = 3.04 ~ ja.67 ms Comprobacion: 1 2 Ze wo tpi ~ 3.07 — 14.62 ms YL ) 0.40 corresponde a un desplazamiento angular de 0.4 < 720° = 288° sobre el cfreulo Gb s constante: Esto equivale a un desplazamiento de 288° desde P hacia el generador (en, la direcciGn de las manecillas del reloj) en el cireulo s, hasta llegar al punto R. En Ry 3 + 70.63 Por tanto, Zokem = 75 (0.3 + 40.63) 22.5 + j47.25.0. ‘Comprobacion: pe = 2% (0.4n) ~ 260° (0.4) 4 144° wine Zesecabae Za + jZ1.tan Be 75 (100 + /150 + 775 tan 144" “(7s + 7100 + 7130) tan 144°] 54.41 £65.25" Zone = 21.9 + 47.6.0. ©) 0.64 corresponde a un desplaizamiento angular de 0.6 x 720° = 432" 21 revoluicién + 72° Asf, se avanza 432°, o una revolucién més 72°, a lo largo del cfrculo s desde (extremo de la carga) hasta llegar al generador en el punto G. Observes q a G desde P se pasa una vez por el punto 7 (Ia posicién de Vaya) y dos ¥ to S a posicin de Ving). Desde la carga, es Fi Ter. Vinge 8 ubicw en 4 E120. Ving, 8€ ubica en 0.0555A + 3 = 0.5554 con tanto que €1 Unico Vig © ubica en 0.05SA + A/4 = 0.3055A. En G (extremo del generador), b Zoot 1.8 — f2.2 Zon = 75.8 ~ j22) = 135 — [165.0 Esto puede comprobarse mediante Ia eeuaci6n (11.34), donde pe = 27 @.6A) Como puede verse, el diagrama de Smith ahorra mucho tiempo y esfuerzo, ites 2 POSEN TE algunas aplicaciones de lineas de transmision Tas Iineas de transmisin se utilizan con diversos fines. Aqui nos referiremos asu uso en cl acoplamiento de cargas y la medicién de la impedancia A. Transformador de un cuarto de onda (acoplamiento) Cuando Z, # Z,, se dice que la carga esta desacoplada y que existe una onda reflejada en la linea, Para una maxima transferencia de enersia, sin embargo, ex descable que la a de transmision (Z, = Z,),a fin de anular la reflexion 1). El acoplamiento se consigue usando sceciones en corte de linens de recast ate da opts aie ete ones ad fae Tisai Sd Sut osama) 28 2m 20] Be asian wa] “Ze arse) priate eae Bk ee aus) Asf, mediante la incorporacin de una linea deA/4 al diagrama de Smith, se obtiene mitancia de entrada correspondiente a una impedancia dada de I carga. Asimismo, una carga desacoplada Z, puede acoplarse adecuadamente con una linea (con impedancia caracteristica Z,) insertando previamente en la carga una linea de transmi- sin de 4/4 de longitud (con impedancia caracteristica Z;), como se indica en Ia figura 11.17. Esa secei6n de 4/4 de la linea de transmision se Hama transformador de un cuarto de onda, ya que sirve para acoplar la impedancia como lo harfa un transformador ordi nario, Con base en la ecuacién (11.56), Z; se selecciona de tal manera que (Zim = Za) VEZ. 1.58) Figura 11.17. Acoplamiento de es transformador de 4/4 V Figura 11-18. Patrén de.on v A |” Sd'votaje de uns’ cay daa i t | Gy sin transtormador de | | Samformador dear. et meio o © a donde Z;, Z, ¥ Z_, son reales. Si, por ejemplo, se desea acoplar una carga de 120 Q linea de 75, el transformador de un cuarto de onda debe tener una impedancia cay tica de V((75)(120) = 950. Este transformador de un cuarto de onda de 95 acoplard uns cary Ue 75 © von una nea de 120 0. En la figura 11.18 (4) y (&) Ios patrones de onda estacionaria de voltaje sin y con cl transformador de A/4, mente. En esta figura puede observarse que pese a que entre el transformador persiste una onda estacionaria, a la izquierda de aquél la onda estacionaria ha d Go, por efecto diel acoplamiento, Sin embargo, la onda reflejada (o estacionaria), mina en la longitud de onda (0 frecuencia f) descada; en una longitud de onda ti diferente, habra reflexiGn, Asf, la principal desventaja del transformador de un ‘Onda es que se trata de un dispositive de banda angosta, sensible a la frecuencia, B. Sintonizador de secci6n de linea Gnica (acoplamiento) El mayor inconveniente del transformador de un cuarto de onda como dispo acoplamiento de Iineas desaparece en el sintonizador de seccion de linea nica, tontzador es una seccion abierta o-cn corto de una linea de transmisién de longi nectada en paralcio a In linea principal a cierta distancia € de la carga, como se its In figura 11.19, La impedancia caracterfstica de la seeciGn debe ser igual a la de ls principal, Aunque factible en teorfa, una secci6n en serie entrana dificultades de i Ra parts, una sensian an cincuite nbierto emite clerta energia a altas frecuencias. P to, cs preferible cinploar secciones paralelas derivadas en cortocircuito, Puesto que el propdsito es que Ziq, = Zo, es decir que Zeng — 1 0 Yeu ~ Len el [A de Ia linea, primero se traza el lugar geométtico y — 1 +'Jb (cfreulo'r = 1) ef rama de Smith, como se indica en la figura 11.20, Si se introduce en A una se hea en derivacidn de admitaneia y, — —Jb, entonces You = 1 +b + y,—1 + Jb — jb =1+j0 linea unica. | Figura 11.19. Acoplamiento con un sintoni 2, seecian conrocitente a =* Figura 11.20. Uso del diagram de Smith para determinar € y de un sintonizador Me seecion de inca nies en cortociseuito yen derivacisn. como es de desear. Puesto que b podria ser positiva o negativa, en la linea pueden ha- Harse dos posibles valores de € (=1/2). En A, y, — jb, € = € yen BY, — Jb, € = lm como se muestra en la figura 11.20. Bn vista de que In seccién de linea esta en corto Gi, — oo), su longitud d se determina hallando la distancia de P.. (en el cual =, = 0-70) ala admitancia requerida de la seccién y,,.En lo que se refiere a la seccion en A, d = dy'se obtiene como la distancia de P a A’, donde A” corresponde a y, — —jb, situadd en Ia'pe- riferia del diagrama, como se advierte en la figura. De igual manera, d — d,, se obtiene como la distancia de P.,.a B’ Gy, = jb)- ‘Se obticnen asfd — dy d= dg, correspondientes a A y B, respectivamente, como se observa en In figura 11.20. Reparese en que es invariable que d+ d, — A/2. Enere las dos posibles secciones en derivacion, Hormalmente se opla por acoplar la mas corta 0 la mas cereana a la carga. Cuando se opta por dos secciones en la Iinea en lugar de una sola seceién derivada, se efecttia un acoplamiento con doble seccion de linea, en el cual se tiene en cuenta el ajuste de Ia impedancia de la carga. C. Linea ranurada (medici6n de Ia impedancia) A altas frecuencias es muy dificil medir la corriente y el voltaje, ya que el tamafio de los, dispositivos de medicién aumenta excesivamente y cada circuito se convierte en una, Immea de transmision. La linea ranurada es un dispositivo simple para determinar la im- pedancia de una carga desconocida en altas frecuencias, hasta 1a repién de los gigahertz. Consiste en una seccién de Ifnea en cl aire (sin pérdidas) con una ranura en el conductor externo, como se advierte en la figura 11.21. Una sonda en la linea a lo largo del campo E Gig. 11.4) muestrea este campo, lo que permite medir la diferencia de potenci el propio sensor y su revestimiento externo. ‘La Iinea ranurada suele emplearse junto con el diagrama de Smith para determi- nar la raz6n de onda estacionaria + (Ia azn del vollaje maximo al voltaje minimo) ¥ Ja impedaneia de la carga Z,. El valor de s se obtiene directamente del detector cuando esta conectada la carga. Para hallar el de Z,, la carga se reemplaza por un cortocireul to y se identifica en la escala la posicion de los voltajes minimos (cuya determinacion es mds precisa que la de los maximos a causa de la nitidez del punto de cambio). Puesto que las impedancias se repiten cada semilongitud de onda, cualquier voltaje minimo Paods eugicy) eoain ports éo remorersts Gu in citga, Puscetinccieat® te datarasian ln © Figura 11.21. (a) Linca ranurada comun: (b) determinacién de la ubieacién ‘dela impedancia de la eargn Z, y de ine distancia del punto de referencia a la carga reemplazando el cortocircuito per ¢ identificanda Ia posicion de los voltajes minimos. La distancia € (la distancia fla carga) en términos de A permite ubicar la posicion de la carga de un cire Cl diagrama, como se muestra en la figura 11.22. La carga tambien podria I mediante €”,la distancia de Vi, al generador. Asi, para localizar € es posible ut oz, i ‘En sintesis, el procedimiento de empleo de la linea ranurada es el siguiente! 4. Conectada Ia carga, se obtiene s en el detector, valor con el que se traza 2. ‘Tras reemplazar la carga por un cortocircuito, se elige una posicién de volt nimo como punto de referencia de zy. 3. Nuevamente conectada la carga, se identifica la posicién de Vig ¥ se det ‘SL En el diagrama de Smith, se avanza hacia la carga una distancia € desde cin de Va, ¥ s halla Z, en ese punto. €= distancia hacta la carga Figura 11.22, Determinacion. ‘ncla hacia el poner ‘de la carga con Un indicador de onda estacionaria registra s'— 2 en una linea ranurads en el aire conee- tada a.una carga desconocida, en tanto que la eseala ubiea los minimos en 11 cm, 19 cm, Reemplazada la carga por un cortocircuito, los mfnimos se sittian en 16 em, 24 cm, SiZ, = 50 Qvealeule A, fy Zz- Solucién: Considérense los patrones de onda estacionaria que aparecen en Ia figura 11.23(a). De ell se deduce que En términos eléctricos, la carga puede situarse a 16 em o 24 em, Sila suponemos a 24 em, se encuentra a una distancia € de Vigg, donde, s €= 24-10 =sem= Sa — 03125 Figura 11.23. Determinacién de Z, mediante una linea ranurada: G@) patron de ondas, @) diagrama de Con" Smith para el ejemplo 11.6. ato corresponde a un desplazamiento angular de 0.3125 720° = 225° 4 2. Partiendo de la posicion de Vij, ¥ avanzando 225° hacia la carga (en d traria a la de las manecillas del reloj).se llega a la ubicaciGn de z,, como se ius figura 11.23(b). Ast, zp = 14 4 70.75 2 = Zot, = 50 LA + $0.75) = 70 + 375.2 Bjercicio 1.6 Pers Ri ‘Las siguientes medidas se tomaron usando la técnica de ta Imes ranuada ee eyes Goursig cu 2a umro cma... -tcon cortocireuit «= in2sem3isem.... SZ, — 50M, dewrine Z;. 9 Respucstar 9257S QN . i aagigiaag) | Une antena con impedancin 40 + 730 9 debe ser acoplada con una linea sin p Flemplo 14.7, | {90 0 mediante una seccion de linea en corto. Determine a) La admitancia requerida de la seccién de linea. &) La distancia entre la seccién y Ia antena. 4 6) La longitud de Ia secci6n. dl @) La razén de onda estacionaria en cada segmento del sisten Solweis: ou 04 + 703, Se localiza z,, en el diagrama de Smith, como se faz cl cireuio s de tal manera que y; se ubique en la posicin diamet a2, Asi 3, ~ L6— J1.2, Opcionalmente, y, puede encontrarse mediante > = 100 Zi. 7 40+ 730 Se determinan los puntos A y B donde el cfrculo s interseca con el efreulo ve 6 < a hye inod yon B.y, — 471.04, De este modo, la admitancia requerida de | ‘ae nea es ¥, = You = #/1.04,45 = = foams ‘ym Yor Toot zune it} cel ennen ~Sithl al een eels PN, Figura 11.24. Para el ejemplo 11.7. ) Con base en la figura 11.24, se determina la distancia entre 1a carga (antena en este caso) ¥, ¥ la secci6n de linea. En A, A _ (62° = =39") _ an% am = 0.36, Bn 8, (2 111.7. Transitorios en lineas de transmi ©) Se localizan los puntos A’ y B’ correspondientes a la admitanci “1.04 y j1.04, respectivamente. Se determina entonces la lon| cia de Pla A’ y BY): Be ay = B80 = 0.12224 272° dy ~ FER 0.37780 Notese que dy + dy = 0.54, como era de esperar d) De acuerdo con la figura 11.24, = 2.7. Esta es la razon de onda ests mento de In linen entre In seccion y la carga (fig. 11.18), mientras que a la Seccion s — 1, a causa del acoplamiento de la linea, y a lo largo de la que la seccisn est on cortocirculto, tancla desde ta Respuesta: €, = 0.093A, €, - Hasta aqui hemos supuesto que una Iinen de transmisién opera a una sola ff Embargo, en aplicaciones practicas como las redes de computo es posible enviar Schales de impulsos. De acuerdo con el andlisis de Fourier, un impulse pues ‘como una stiperposicion de ondas de muchas frecuencias. Ast, la difusion de impulsos cn In nea equivale a la transmision simultanea de ondas de diferent ‘Segdn cl andlisis de cireuitos, entre el eneendido de una Dateria o gene! pulsos conectado a una ifnea de transmision y la consecucion de valores est Corriente y cl voltaje en Ia ines transcurre cierto periodo de transicion, lami El comportamiento del transitorio inmediatamente después del clerre deli provocado por descargas causadas por rayos) sucle analizarse en el ambito de Id fon la ayuda de la transformada de Laplace. En afin de simplificacion, aqui t este problema en el mbito temporal Considérese la linea sin pérdidas de longitud € ¢ impect lustra en Ia figura 11.25(a). Supongamos que esta linea es de impulsos de voltaje V, con impedancia Interna Z, en 2 — Oy que termin ga resistiva pura Z,. En ¢l instante ¢ — 0 de cierre del interruptor, Ia corrient due sdlo “ve” Z, y Z,,de manera que Ia situacion inicial puede describirse com equivalente qué aparece en la figura 11-25(b). Con base en esta figura, la arranque en 2 — 0,7 — 0 esta dada por 1(0,07) @ © y el voltaje inicial es ¥@.0") = Ege %% aie ‘Tras cerrar el interruptor, las ondas I* = I, y V* = Va $e propagan hacia la carga ala ve- locidadt 1 —— (1.62) Vie Puesto que esta velocidad es finita, transcurre cierto tiempo hasta que las ondas, de di: reccion positiva, Hegan a la carga e interactéan con ella. La presencia de la carga no ejer- Ce ningtin efecto en las ondas antes del periodo de transicidn, dado por pee 11.63) ‘Las ondas llegan a la carga luego de ¢, segundos. El voltaje (o 1a corriente) en la carga es ‘Suma de los voltajes (© corrientes) ineidente y reflejado. Asi, Veet Vit Voe Vor TVe= G+ To ares) Ken) =P + 21 The~ TDS a1.65) donde I’, ¢5 el cotticiente de reflexion de Ia carga, dado en la ecuacisn (11.36); es decir, zp zy ma See 1.66) Las ondas reflejadas V- = T',V,¢ [> = —PyJ, vuelven al generador junto con las ondas V7. que ya se encuentran en la linea. Las ondas reflejadas llegati al generador en el ue Ys instante ¢ = 2f,, de manera que, V0, 24). = VE Yo = le + A +NDV, | Eiempto 11.0 oy © Figura 11.26. Diagrama de rebote de (a) una onda de voltaje yb) una onda de corrionte. VO.24) =A +P, +P DV, ng 10, 24,) PATef.) + = Pda 410,24) = — Ty + PP ey donde Tg es el coeficienté de reflexi6n del generador, dado por Zs* Zo Zee ze Te Las ondas reflejadas (én el extremo del generader) Vt = I'gh,V, eI = Poly pagan de nuevo hacia la carga, y el proceso contintia hasta que los resist absorben la energia del impulso. Rastrear los reflejos en un diagrama de rebote o reticular es més sencillo a guir las ondas de voltaje y corriente en su trayecto de un extremo a otro. Tal consta de una linea en zigzag que indica la posicin de la onda de voltaje (0 de. te) respecto del extreme del generador, como se ilustra en Ia figura 11.26, Para nar el valor del voltaje (© de la corriente) en cualquier momento, basta afladir asignados en el diagrama al momento respective. Con Feferencia a la Ifnea de transmision que aparece en Ia figura 11.27, ealcule 4a) Bl voltaje en los extremos de la carga y el generador en 0 < 1 < 6 js. ») La corriente en los extremos de la carga y el generador en 0 <1 < 6 ms. Figura 11,27, Para el ejemplo 11.8, Solucién: 4) Se calculan primero los coeficientes de reflexion por voltaje en los extremos del ge- nerador y la carga. da me 100 + 50 3. ry + Beads 2 = 0 3 "Zi ezZ. 2007 50 5 a <1 EI periodo de transicién t= £ = 102 — ays, El voltaje inicial en el extremo del generador es 50 v, = $02) -4v Remitidos a la carga estos 4 V, la punta del impulso Hega a la carga en ¢ = 1 = 1 ps. Luego de ser reflejada una porcidn del impulso, 4(3/5) ~ 2.4 V, ésta llega al generador en 1 — 31, ~ 2 ys En el generador se refleja 2.4(1/3) = 0.8 y asf sucesivamente. El proceso Completo se ilustra en el diagrama de rebote del voltaje que aparece en la figura 11.28, Figura 11.28. Diagrama de rebote del voltaje para el ejemplo 11.8. Con base en este diagrama de rebote es posible trazar V(0.2) ¥ VCC, nes de tiempo, lo que se muestra en la Figura 11.29. Bn ella se advierte qucya 106, los voltajon se aproximan a un valor asintético de i 200 = Map) av Esto era de esperar, dados los circuitos equivalentes en 1 = 0 y ¢='6 que se. en la figura 11.30 (véase el problema 11.46 para efectos de comprobacién). b) El coeficiente de reflexin por corriente en los extremes del penerador y Te = —13y —T, = ~3/5, respect Figura 11.29. Voltaje (fuera (a) en el extreme del ger (b) on cl extreme de la @ ven ws) 12. THANSTORGS IN NEE RETA «Hh SLE @ co Figura 11.30. Circuitos equivalentes a la Hnea de la figurs 11.27 en far Oy Wy r= sumbién 1(0, 0 € I(é, 0) se obtienen fécilmente del diagrama de rebote, esta vez referide a ln cortiente, el cual se muestra en la figura 11.31, mientras que en, la figura 11.32 se les diaprama como funciones de tiempo. Cabe hacer notar que I(€,9) = V(€, 0/Z,. Por tn weet tmara 11.32(b) pucde obtencrsc ya sea del diagrama de rebut de la cursicnte de To figura L131 0 feproduciendo a escala la figura 11.29(b) por un factor de 1/2, = 1/200. En letfiguras L1.30(B) y 11.32 se observa que las corrientes se aproximan # un valor asint6- tico de 12. - = 32 40m, i = 300° ie Figura 11.31. Dingrama de rebote de la corriente para el ejemplo 11.8, 100.) ma, Figura 11.32. Corriente ‘estala): (a) en el ext generador, (B) en cl e Earga, para el sjemplo 11, os Hus) Ejercicio 11.8 * Repita el ejemplo 11.8 si la linca de transmision esté en 14) Cortocireuito. << 5) Circuito abierto. > Respuestas: a) Véasc la figura 11.33. 6) Véase la figura 11.34. : veo dy vo. 13333 fe 7 Figura 11.33. Para el inciso a) del ejercicio 11.8. es Me, es Figura 11.34, Para el inciso b) del ejercicio 11.8, Una linea de transmision de 75 © y 60 m de longitud termina en una carga de 100 ©. Siun impulso rectangular de 5 jus de duracién y4'V de magnitud es emitide por el ge- nerador conectado ala Iinea, trace 1(0, 0) © 1(€,) con relacion a0 <¢-< 15 ys Adopte Z, = 25 Oyu Ole. Sotucién: En el ejemplo anterior el encendido de una baterfa genet una funcidn escalonada, un pulso de duracion o anchura infinita, En este ejemplo, el impulso es de una anchura fi- de 5 p18, Caleulemos primero los cocficientes de Feflexion por voltaje: 75 Baasv Ve Ve = 300 £ 60 u 01G X10) El tiempo que tarda V;, en su trayecto de un extremo a otro es 26, Guracion del impulso, de 5 is. Por tanto, habra empalme ‘El coeficiente de reflexion por corriente es S 2us 4 as, menor que Ia La corriante inieial 7. = ‘Sean /y rlos impulsos incidente y reflejado, respectivamente. En el extremo del generador: o2 donde Zz. [e: me 607? Ba ont Z.Ve, En conocimiento de es, ¥ Zq, 1a constante de fase y la vel que se propaga en la microcinta estan dadas por Re a. ~ 8.686 Se donde R, = gig es la resistencia peticular del conductor. La atenuacion das dicléctricas es (en dB/m) (eu = De, tan 273 * Den A donde A = u/fes ta longitud de onda de la Iinea y tand = o/we la tangente de Sustrato, La constante de atenuacidn lotal es la suma de la constante de at mica a, y la constante de atenuacién dieléctrica ay: es decir, En ocasiones «, es insignificante en comparaciSn con ,. Pese a su flexibilidad tacion, las Iineas de microcinta no son utiles para la transmisin en largas i sousaidle pu exoesiwalklecunce El sustrato de cierta Itnea de microcinta es de cuarzo (e, = 3.8). Si la razon de la anchu- fade la Iinen al grosor del sustrato es w/h = 4.5, determine a) La permitividad relativa efectiva del sustrato. }) La impedaneia caracteristica de la linea. ©) La longitud de onda de la Iinea a 10 GHz. Solucién: a) Puesto que w/h = 4.5, se trata de una cinta ancha. Con base en la ecuacién (11.70), 6) A partir de la ecuacién (11.71), 1200 °°" \/aaai (4.5 + 1.393 + 0.667 In (4.5 + 1.444)) =95760 Zi 3 x 108 1OV3.131 fen = 1.69 x 107 m = 16.9 mm Reepuestant a) 2.75, b) 84.030 y ©) 18.09 mm. A.10 GHz, una Iinea de microcinta tiene los parsmetros siguientes: A= 1mm w= 0.8 mm 66 10-+ eo, = 5.8% 107 Sim ‘Calcule la atenuacién debida a’ pérdidas de conducci6n y a pérdidas dieléctricas. Solucisns La razon wiht 8, Asi, de acuerdo con las ecuaciones (11.70) ¥ (11.71), _ 72,56 2\" oa 2+ 881+ BZ) 43 | ae es f vas” os * | e770 La resistencia pelicular del conductor es em efi, = ee lose ae sO Ba Vere ON. SBR I = 2.609 5 10-9 Set Mediante la ecuacién (11.75), ln constante de atenuscién de conduccién es R= 2.609 > 10-2 OB > 107? 3 67.7 = 4217 aim e% = 8.686 = Para hallar la constante de atenuaci6n dieléctrica se precisa de A. ue 3 x 108 f” FVeq 10x 10°43 = 1.447 < 107m x La aplicacién de la ecuaci6n (11.76) resulta en. 3.492 * 6.6 x 10 56 x 43 x 1.447 x 10 = 0.1706 arse f4g = 27.3 Calcul 1a atcnuncién debida a pérdidas Shmicas a 20 GEtz de una Cinta incgrada por un conductor de cobre de 2.5 mm de ancho sob Slumina, Adopte 30 0) come ln impedancia caractertstiea de ta linea. | sctuptcirar’0.904 auveafe dustin total cs ly nen ah Sec Resumen 1. Una linea de transmisién es comunmente descrita por sus parametros li (en O/m), L (en Him), G (en S/m) y C (en Fim). Las formulas para el Gy C de Ifneas coaxiales, de dos alambres y planas se proprocionaron en 1a 2. Los parémetros distribuidos se usan en un modelo de circuite equivalente para re- prosentar una longitud diferencial de la linea. Las ecuaciones de linea de transmni- Sion se obtienen aplicando las leyes de Kirchhoff y concediendo que Ia longitud, de la linea se aproxima a cero. Las ondas de voltaje y de corriente en Ia linea son VOa 8) 4 V56 + 608 (er — Bz) # Vee" cos (wi'+ Be) mi Wg P cos (ot — Az) — BE et cos (oo + Be) nea de dos ondas que se desplazan en direc Hat) Jo,que indica la existencia en ciones opuestas. 3. La impedancia caracteristica Z, (analoga a la impedanci das planas en un medio) de una linea esta dada por iL G+ joc 59 y la constante de propagaci6n y (por metro) por yet iB = VRF FOLNG + Ja) La longitud de onda y velocidad de onda son intrinseca 7 de las on- ae ea 4. El caso general es el de Ia Iinea de transmisiGn disipativa (G # 0 # R), el cual se examiné en primer término. En una linea sin pérdidas, R = 0 = G; en una linea Sin distorsion, R/. ~-G/C. En la transmision de energia son de descar Iineas sin pérdidas, y lineas sin distorsion en la comunicacién telefonica. El coeficiente de reflexin por voltaje en cl extremo de la carga se define como Vez s ZL - a 5. Te en tanto que la raz6n de onda estacionaria es LI! test de fasor a la corriente en forma de fasor es la impedancia en ese punto viendo hacia la carga, y seria la im- pedancia de entrada a Ia lines si ésta fuera de tal longitud. Fa una linea disipa pa ZZ, tanh ye Sp oe a donde ¢ c# Ia distancia de Ia carga al punto. En una linea sin pérdidas (« — 0), tanh yé = tan @€;en una linea en cortocireuito, Z, = 0; en una linea en circuito, bierta. 2 gecioctey eacaue dl tamaioorg lala airs ILA. {Cudles de tos enunciados a2, 13. a4, 7. El diagrama de Smith es un medio grafico de obtener caracterfsti transmisién como Ts y Z... Se le elabora dentro de un cireulo de rad unidad y se basa en la formula para T’, anteriormente referida. Re ry x cuenta con dos circulos explicitos (de resistencia y de reactancia). pifcito (de s constante), Permite determinar la ubieacion de un Seccién de linea y su longitud, asf como, en asociacién con una linen ¥ lar el valor de la impedancia de una carga desconocida, & Cuando en el extremo emisor de una linea se aplica sdbitamente un volt rriente directa, un impulso sigue en la linea un trayecto de uno a otro ext diagramas de rebote son utiles para analizar el comportamiento del ts 9. Las linens de (ransmision de microcinta se emplean en circuitos intepradg croondas. Fn el texto se presentaron formulas para crear Iinens de mic determinar sus perdidas, ¥ Eno son ciertos? a) Ry L son elementos en serie 1b) Gy Con elementos en derivacion. o e-t @) Lom ney RG =o ‘@) ‘Tanto R como G dependen de la conductividad de los conductores que componen lal /P) Solo R depende explicitamente de Ia frecuen 2) Fstos pardmetros no son globales, sino disteibuidos En una Iinea de transmision disipativa, Ia impedancia caracteristicn no depende de 2) La frecuencia de operaeim de In inen ©) La tongitud de ta tinea, ©) La carga en la que termina 1d) La conductividad de los conductores. ©) La conductividad del dieléctrico que separ nea. ©) Un intervalo de muy bajas frecuencias (R => wh, G => WC). 4d) Un intervalo de muy altas frecuencias (R wl, GC). {Cudl de los enunciados siguientes no es cierto en uns 4) Zoe ~ JZ cen wna linen en cortocireuito con € = NB. ) Zag = foo en tins thandl em ecetnnteetiney nent fA, nea sin pérdidas? 1s, 116. ©) Zens = Zp 00. una linea en circulto abierto con € = V2. @) Zyqe = Zp en una linen acopiada. ©) En una semilongitud de onda de una carga, Z.., = Z,,10 que en lo sucesivo se repite cada semilongitud de ond: Si una linea de transmision sin pérdidas es de 50 cm de longitud con L = 10 Fm, C= 40 pE/m y opera a 30 MIlz, su longitud eléctrica es de a) 20a 2) OZA ¢) 108° 4) 40 =. ‘@) Ninguno de los valores anteriores, ‘Haga coincidir las siguientes impedancias normalizadas con los puntos A, B, C, Dy E det igrama de Smith que se presenta en la figura 11-40. yore 1p worn Oath Zn oe ~» [Ze] wo [Ze] say caren eevptata r= 0) Una linea de transmision sin pérdidas de 500 m termina en una carga situada en Fen el dia grama de Smith que se presenta en la figura 11-41. 1 A~ 150 m, ,cuantos voltajes maximos a7 6 os a3 ‘e) Ninguno a Figura 11.40, Para la pregunta de repase 11.6. a9. oo Figura 11.41. Para la pregunta de Escriba cierto (C) o falso (F) frente a cada uno de los siguientes enunciados, 44) Todos los circulos ry x pasan por el punto (P,P) = (0). 4) Toda impedancia se repite cada 4/4 en el dingrama de Smith, ©) Un circulo # = 2 os igual a un efrculo [F] ~ 0.5 en el diagrame de Smith. 4) EI principio basico de cualquier método de acoplamiento es eliminar la centre Ia fuente y el dispositive de acoplamiento. ©) Una linea ranurada sélo permite determinar Z,- P En cualquier punto de una Iinea de transmisién, el cocficiente de reflexion, te cs el reciproce del coficiente de reflexion por voltaje en ese punto. Si,en una linea en el aire, méximos adyacentes se encuentran en 12.5 em y 37.5 cuencia de operacion ex de z a) 13 GHz 4 2) 600 MHz ©) 300 MHz Dos impulsos idénticos de 12 V de magnitud y 2 js de anchura inciden en ¢ = 0 en znca de transmisién sin perdidas de 400 m de iongitud terminada en tna carga. Sh Parador por 3 n= (como en cl caso de Ia figura 11.53) yu = 2% 10" m/s, gen qué In contribucion a V;, (6,1) del segundo impulso comienza a empalmarse con la del @) P= 05n8 2b) m2 ps r= 5us t= 55 ps ) = Ons i lestas: L116, dy €, 11.26, 6, L136, 11.4a, 6, 11.Se, 11.61) DB, B, ti) A, fit) E, iv) Cv) By vii) B, vill) A, 1174, 11a) C.b) Fre) Bed) Ce) Bf) B 11.96, 11.106. — Proeienas Gl 533 ALA. Una Iinea plana relleaa de aire con w = 30 cm, = 1.2 cm,1 = 3 mri tiene pineas conduc- toras con @, = 7% 10" Sim. Caleule R, L, Cy Ga 500 MHz. 412. Los conductores de cobre de un diodo, de 16 mm de largo y 0.3 mm de radio, estin separa- ios por una distancia de 2 mm, como se muestra en la figura 11.42. Halle la capacitancia en- tre ellos y Ia resistencia en corriente alterna a 10 MFIz. Z *IL3. En la seceién 11.3 se mencions que el circuito equivalents de Ia figura 11.5 no es el unico posible, Demuestre que las ecuaciones (11.4) y (11.6) manticnen valider en los cireuitos Rguivalentes tipo It y tipe T que sparecen on la figura 1143. 11.4. Gierta linea plana sin pérdidas de 78 0 no cumple un requerimiento basico. ,Qué fraccion Gee anchura de la cinta deberia aftadirae o climinarse para obtener una impedancia carac- teristien de 7502 16am Figura 11.42. Diodo para el problema 11.2. to Boe Hey aen, oo yah dee ts ven eas car vers azo © Figura 11.43. Para el problema 11.3: (a) cireuito equivalente tipo 11, (@) circulia equivalente tipo T- ms. a8, 19. a0. wa. aaa2, ‘Una linea telefonica posee los pardmetros siguientes: al R-400/m, G=400nSm, L = 02 mH, C= O5nPH 4) Si esta Iinca opera a 10 MrTz, calcule la impedancia earacteristica Z, y 2) {Despues de cuantos metros el voltaje se reduicira 30 dB en la lings? Una linea sin distorsién que opera a 120 MHz tiene R ~ 20 O/m, 2 — 0.3 63 pF/m. a) Determine 7. uy Z,. b) Qué distancia recorrera una onda de reducirse’a 20% de su magnitud inicial? c) ,Qué distancia recorrerd antes Cambio de fase de 45°? (Con referencia a una linea de transmisiGn sin pérdidas de dos alambres, des 4) La velocidad de fase u = 6 = Vie 120. og 4 2) Ls impedancia caracteristica 2, = 220 cosnmt LBL inciso a) es cierto con relacion a otras Kineas sin pérdidas? Las Iineas de par trenzado, semejantes a las de dos alambres, son de gran utili dustria teleronica. Considérese una Ifnea formada por dos alambres de cobre de didmetro con una distancia de 0.32 cm de centro # centro. Si los slambres estan por un material dielecirico con e = 3.Seq,halle £,Cy 25 ‘Una Iinea sin pérdidas tiene tna onda de voltaje V6.0 = Vo rontwe ~ Be) Halle 1a onda de corriente respective En una linea sin distorsién, la onda de voltaje est dada por V(E1) = 1204005 cos (10% + 26°) + 606-02 con (10% ~ 26) donde ¢' es la distancia desde la carga. Si Z, = 300 0, halle: 4) a, Ay 1,6) Z,6 Ht 14) Demuestre que un coeficiente de transmision puede definirse como men pers r, = 5) Malle 7, cuando la linea termina en: /) una carga cuyo valor es Z,. il) un eircuito to. ui) un contocireuito, v) Z, — 2, (ines acoplada. Los pardmetros distribuidos de una linea coaxial de 5.6 m de largo son K = 6. £ = 3.4 Em, G ~ 84 mS/m y C= 21.5 pF/m. Sila linea opera a2 MHz, caleule la daneia earacteristica y la duracion de Ia propagacién de exiremo a extrema. Figura 11.44, Parn el problema 11.16. 11.13. Una linea de transmision sin pérdidas que opera a 4.5 GHz tiene por parametos L = 2.4. Him YZ, ~ 85 0. Calcule la constante de fase By la velocidad de fase 1. 11.44. Un cable couxial de 50 0 alimenta 4 una antena de dipolo de 75 + /20 0. Halle Ty s. 11.45. Demuestre que una linea dé transmision disipativa de longitud € tiene una impedancia de entrada 2. © 7. tanh yé en cortocireuito y Z., ~ Z,,coth 7 en cireulto abjerto, Confirme Tn Seuacianes Git A y O13). 11.16. Halle Ja impedancia de entrada de Ia mea de transmisiGn coaxial en cortocireuito de la fi pura 1144n 2. = 65 + j38 02, y= 0.7 +725 im, €~ 0.8 m. Remitase a ta tinea de transmisiGn sin pérdidas que aparece en Ja figura 11.45. a) Halle Ty 'b) Determine Zen el generador. 11.18. Una Iinea sin pérdidas de un carte de onda de 100 0 termina en una carga de Z, = 210 0. Sil Voltaje en el extremo receptor ex de 80 V, {cual cs el voltaje en el extremo emisor? 12.19. Una iinea sin pérdial Seade Ia carga de 500 0 tiene Vi, — 10% V, Z, — $0, Halle 1a corriente en V8. 11.20. Una linea sin pérdidas de 60 0 est4 conectada a una fuente con Vy ~ 10/0" Vim ¥ Z, — 50 oy40 fy termina en una carga de 40 8. Si tiene 100 m de largo y 8 = 0.25 rad/m, calcule Zem¥ Ven 4) Blextremo emisor. b) Elextremo receptor. ‘¢) 4m desde la carga. 4) 3m desde Ia fuente. Figura 1145. Para el problema 11.17. Figura 11.46. Para problema 11.22. 11.21. Una linea de transmisi6n sin pérdidas con impedancia caracteristiea de 75 0 ‘carga sds 120. La longitud de la linea ex de 1,254, Sia linea est energizaday te de 160 V (rms) con impedancia interna de 50 (2, determine: a) la impeds ¥ b) 1a magnitud del voltaje en la carga. te 411.22, Tres lincas sin pérdlids estan conectadas como se muestra en la figura 11.40, Dy Considere Ia red de dos puertos que se muestra en Ia figura 11.47(a). La Variables de entrada y salida puede expresarse en forma matricial como. fe-lé 3 ee linea disipativa de la figura 11.47(b), demuestre que la matriz, oak ye Zygon ye 3 sonh ye cosh ve =a Una Iinea sin pérdidas de 50 © tiene 4.2 m de largo. A una frecuencia de of 300 Miz, In impedancia de entrada a la mitad de la lfnea es de SO ~ 760 0 Ffalle a tin de entrada en el generador y el cocficiente de reflexion por voitaje en In carga. un Oe. Sita impedancia de entrada de una lines en el aire de 60 41 que pers 8 20 Miz ¥ mde largo es de 90 + J150 0, caleule Z,.T'y ‘Una linea de transmision de 75 0 termina én una carga de 120 + 780 0. a) Hall b) Determine a qué distancia de la carga la impedaneia de entrada es puramente Una linea de transmision de 75 1 termina en una carga de impedancia Z,. Sila 5A/8 de largo, ealcule Z,,, cuando: a) Z,— j43 0,0) Z, = 25 ~ 765, 11.47. Para el prot 11.28. 11.29, 11.30. as. 11.33, n134. auas. Determine Ia impedancia de entrada normalizada cn AV desde la carga si: a) la impedaneia ‘cs 2 + j,b) su admitancia normalizada ex 0.2 ~ 0.3, 2) el coefieiente de ‘carga on 0.3 04 ‘Una Knea de transmisién tormina en una carga con admitancia ¥, = (0.6 + /0.8)/Z,. Halle Ia impedancia de entrada normalizada en 6 desde la carga. ‘Una lines de transmisin de 80 0. que opera a 12. MEiz termina en una carga Z;, A 22 mde Ia carga, In impedancia de entrada ex de 100 ~ 120 0. Siu — O.8e, 4) Caloule V4, Zens nts ¥ Zonta ) Halle Z,,y ia impedancia de entrada a 28 m de la carga. ©) gOusntor ZenimseY Zen min hay entre la carga y la impedancia de'entracda de 100 ~ 120.07 ‘Una antena conectada 2 una Iinen sin pérdidas de 150 0 produce una razén de onda esta- Glonaria de 2.6. Si ins mudicioney indicum que low voleaje maximow extan xeparades Por ‘120'em y que el dtimo maximo ocurre a 40 em de la antena, calcule 4) La frecuencia de operacién. ) La impedancia de la antena. ©) El coeficiente de reflexion. Suponga que se — e. Tin Fatt deceit, tetncionsvia obebivitad de tiie Tied sin’ pEtdides de 100 @ es'8, Stel primer voltaje maximo ocurre a 0.3A de la carga, calcule la impedancia de la earga ¥ el coe! Giente de reflexion por voltaje en la carga. ‘Una linea de 50 © termina en una carga de impedancia desconocida. La razén de onda es- tacionaria s — 2.4 en la linea y un voltaje maximo ocurre # A/S de la carga.) Determine In impedancia de la carga. b) -A qué distancia de ia carga se encuentra el primer voltaje mi- nimo? ‘Una linea sin pérdidas de 75 © termina en una impedancia de carga Z; desconocida. Si a una distancia de 0.2A de Ia carga el-voltaje es de V, = 2+/ Vy la corriente de 10 mA, Dos transformadores de a/d Sonektivin litid Was Ste tia thea de 500 a una carga de 75:01, como se llustea en la figura 11.48, 4) Determine la impedancia caracterfstica Z,; si Zz; ~ 30 0 y no hay onda reflejada a 1a iz~ quierda de A. 2) Si tos mejores resultados se obtienen cuando [ash - determine Z,, ¥ Zo2 para este caso. Dos antenas idénticas con impedancia de entrada de 74 0 som alimentadas por tres Kneas de transmision idénticas de un cuarto de onda sin pérdidas de 50, como se observa en la figura 11-49. Calcule la impedancia de entrada en el extremo de la fuente. ‘Sila linea descrita en el problema anterior se conceta a una fuente con un volta} impedancia interna de 80, caleule In potencia promedio que recibe cualduier ‘Considere las tres Imeas sin perdi que wpureon en Ia fut 11.50. 2, = 4) Zy, Sede 1a linea 1. 2B) Zig desde la linea 2. ©) Zam donde la tinea 3. ‘Una seceiGn de linea de transmision sin pérdidas se coloca en derivacion en lal como se indien en Ia figura 11.51. $i €, = A/4, €; — A/S y €, — 7X8, halle Yanns Ye puesto que Z, = 100 2, Z, = 200 + 150M. Repita los edlculos si la seceién en iablerta, Se desea acoplar una linea de 50 0 con una impedancia de carga de 60 — j50 0. Scceidn de Iitea de 50 01 que consiga exe acoplamiento. Halle la longitud de la tancia desde la carga. ‘Una seccién de linea de 0.124 de longitud se emplea para acoplar con una cas sin perdidas de 60 0 Sila seccidn se ubiea 0.34 de la carga, caleule 4) La impedancia de Ia carga Z;- 2) La longitud de tuna seccion de linea opesonal y su ubseacion respecto ae la ca ©) La raz6n de onda estacionaria entre la secci6n y la carga, Mediciones realizadas en una linea sin pérdidds indican que 3 = 42 con et maximo 9 \/4 de la carga. Determine a que distancia de la carga deberta ut ‘Gon de linea en cortocireuito y ealeule st longitud. (et 742 Figura 11.49. Para los problemas 11.36: 11.43. seas. 148. Figura 11.50, Para el problema 11.38. ‘Una Iinea sin pérdidas de 60 © que termina en una carga Z;, tiene una onda de voltaie co- ‘mo la que se muestra en la tigura 1152. Halle s,T'y 2, Las siguientes medidas procedentes de una linea ranurada corresponden a un sistema de 50 .Con carga: s = 3.2 y Vg adyacentes ocurren en 12 cm y 42 cm (Ia eifra mas ales 0 pre, senta del lado de la eargay: con cortocirculto: Viq, oeurre en 21 om. Halle la frecuencia de Operacion y la impedaneia de la carga. ‘Una Iinea ranurada cn el aire de 50 Ose aplica a Ix medicion de una impedancia de carga. Los minimos adyacentes se encuentran a I4.em y 22.5 cm de la carga cuando la carga des conocida est4 conectada ¥ Vaux = 0.95 V ¥ Vin = 0.45 V. Cuando la carga ex reemplazada Por un cortocirculto, los minimos ocurren n 3’3'em de la carga, Determine sf. Ty 2, Demuestre que en lo relative a un voltaje de corriente directa V; activado en ¢ = 0 (wéase 1a figura 11.30), los valores asintoticos (¢& €/u) de VUE. 16,45 son Vez | zpeean [Una linea sin pérdidas da 6) 0 std conectada a un ganerador de impuloos de 40 0. 1a Knee, ‘de 6m de largo y termina en una carga de 100 fi, Si'un impulse rectangular de Spe de dur ‘clon y 20 V de magnitud es emitide en la linea, halle V(O, 1) © 1(6,1) respecte de 0 =f = 10 05 ‘Adopte w= 3 > 10" mis El imtermuptor que aparece en la figura 11.53 se clerra en ¢ = 0. Trace el voltaje y la corriente en el lado derecho del interruptor respect de 0 =f ~ blu, Adopte 2, — 50 My tla 2 us. Suponga una lines de transmision sin perdidas. Figura 11.51. Para el problema 6—| 11538. a v1.50. asi. asa. ‘Figura 11.53. Para el problema 11.48. Figura 11.52. Para el prot 2,-0.0 Es ‘Con reterancia al sistema que aparece en I Agura 12.54, truce WEE, © HE.) orre spe Remitase @ Ia figura 11.55, donde Z, = 25 O, Z, ~ 50.0, Z, = 150.0, € = 150 enr= Ol impulzo que se muestra cn la figura 11,56 incide en Ia linea, 44) Trace los diagramas de rebote del voltaje y la corriente. i 2b) Determine V(0,#). V(E, DIO, 1) e Hea) respecte de 0 << 8 psi 1 ‘Una linea de microginta tiene 1 cm de grosor y 1.5 em de ancho. La cinta du cabre (or. = 1A 107 S/m), mientras que el sustrato es de un material dl oF S325 Gino ~ 0.002, Sila linen opera 825 Giz, halle: a) 2,9 ec) oY a Gia que recorrerd la onda en Ia lines antes de reduclrse 20 dB. of ‘Una linen de microcinta de 50 £1 registra un cambio de fase de 45° a 8 GHz. Sil sustrato es de f= 8 mm con ©, ~ 46, halle: a) la anchura de la cinta conduc itud de Ia Iinen {Uni sustraee'de eltmntin Co 06a) 6.2 nen db: Grown we cimpich Shsbi alates Giroulto de micracinta, Si el dischindor del eireuito pucde optar por un ancho de 5.0 mm para la linea, zeudl es el intervalo de la Impedancia caracterfstien de sta? Disehte una Iinon de microcinta de 75 sobre un sustrato de duroide (6, = 2:3) de grosor. Halle 1a anchura de la cinta conductora y Ia velocidad de fase. cas) Figura 11.56. Dos impulsos rectangulares para el problema 11.50, 12 Guias de ondas ‘Aquel que escriba un libro mejor, predique un sermon mejor © haga una ratonera m Jor que su vecing, vera llegar al mundo hasta su puerta. RALPH WALDO Eos 12.1. Introduccién Como se mencions en el capitulo anterior, una linea de transmisi6n se puede uss guiar energia clectromagnética de un punto (el generador) a otro (Is carga). Una ‘ondas hace lo mismo, pero posce algunas diferencias con una linea de transn especial de aquélla. En primer lugar. una linea de transmisién slo puede tol electromagnéticas transversales (ET), mientras que una guia de ondas puede toler chas posibles configuraciones de campos. En segundo, las lineas de transmision ll ‘eficientes a frecuencias de microondas (de aproximadamente 3-300 GHz), del efecto pelicular y Ins pérdidas dieléctricas, en tanto que las guias de onda en ese intervalo de frecuencias para obtener mayor ancho de banda y menor ate de senal. Por tiltimo, una Iinea de transmisién puede operar en una escala que ¥ el nivel de corriente directa (f= 0) hasta el de muy altas frecuencias, mientras q tay su tamano seria excesivo a frecuencias inferiores a las de microondas. Lae gulas de ondes méx comunce ron rectangularcs $ transversal podria ser de cualquier disefo uniforme. En la figura 12.1 aparecen ondas usuales.' El andlisis de I funciones de Bessel, tema que rebasa el alcance de este libra En consecuencia, ‘ocuparemos de gufas de ondas rectangulares. A partir del supuesto de gulas de pérdidas (o, = 0°, ¢ ~ 0), aplicaremos las ecuaciones de Maxwell con las con‘ In frontera adecuadas para obtener diferentes modos de propagacién de ondas y 10s 6 pos E'y H correspondientes, Respecto de otros tipos de gutas de ondas, vénse J. A. Seeger, Microwave Theory, Compon Devices, Prentice-Hall, Englewood Cliffs. 1986, pp. 128-133. © Analisis de purax de ondas © relacionados con este tema, por ejemplo, 5 ¥. Liao, Microwave Devices and Circuits. tee-Fall, Englewood Ctiffs, 1990, pp. 119-141. Ciesla Rectangular Considérese Ja gufa de ondas rectangular que aparece en Ia figura 12.2. La supondremos ‘ocupada por un material dieléctrico sin pérdidas (¢ ~ 0) ni fuente (p, = 0.5'™= 0) y do~ tada de paredes perfectamente conductoras (¢, ~ oc). Como se recordard, en las ecua- ciones (10.17) y'(10.19) se establecié que, en el caso de un medio sin pérdidas, las ‘ecuiaciones de Maxwell en forma de fasor se convierten en. WE, + 12E, — 0 a2ay WH, + 2H, = 0 a22) Wigura 12.2. Gufa de ondas rectangular con paredes perfectamente Conductoras rellena de un material sin pérdidas. donde k= wVue y se supone el factor de tiempo eM. Si se concede que E, Cen Eye Ea) ¥ Ay = Fan Hy He) las ecuaciones (12.1) y (12.2) constan en cada caso de tres ecuaeh Helmholtz. En otras palabras, para obtener los campos E y Hes preciso, ecuaciones esealares. Respecto del componente =, por ejemplo, In ecu. Puede resolverse por separacion de variables (solucion de producto), Cor tonees que acd Eqx,¥,2) = XC) YO) Z@) donde X(2), ¥@) y Z(z) son funciones de x, y y z, respectivamente. La sustit ecuacién (12.5) en la ecuacién (12.4) y la division entre XYZ dan como resi 24M, 7 Bei e aeee y Puesto que estas variables son independientes, cada término de la ecuacion ( ser constante, de manera que esta ccuacién pucde xpresarse como A IB Pe 7 donde — 12, —Ky ¥2 son cinstantes de separacién. Asf, Ia ecunci6n (12.6) se 5 siguiente manera: XT + xX =0 w+gy=0 Z+PZ—0 Continuando el mismo argumento del ejemplo 6.5, Ia solucién de la ecuak XG) =, cos kx +e sen kx YO) = e008 ky + cy sen ky Zz) = ese + ge Al sustituir la ecuaci6n (12.9) en Ia ecuacién (12.5) se obtiene FL, ¥, 2) = (€ COB KX #65 808 Kx) (C5 608 ky y F cg sen Ky) (ese + cye 74) Si suponemos, como siempre, que la onda se propaga a lo largo de Ia gufa de ondas en la Gireccion +2, la constante mulliplicativa cy — 0, ya que Ia onda debe sor finita on el inf ito fes deci, BG, 2 — 09) ~ 0} De abit que la ecuucién (12.10) ve reduzea Ef20.¥.2) = (Ay cos kx + Az sen kaa, cos ky + Assen kyo (12.11) donde Ay = e¢q Az = este ¥ asf sucesivamente. Siguiendo pasos similares, la solucién de Ia componente # dé la ecuacién (12.2) es HJ, ¥.2) = (By cos ex + Bz sen kx) (B, 608 Ky + Basen kyo" (12.12) En vez de despejar de 1a misma manera las demas componentes de campos Ean, Eye, Hyp y H,, de las ecuaciones (12.1) y (12.2), se emplean sencillamente las ecuaciones de Maxwell para determinarlas a partir de B., y H.., De Vv XE, = — jolt, VX, = jock, se obtiene a2a3a) 2.136) a2436) 2.134) (aza3e) a2a3p Ahora expresemos Eu, yr: Hay ¥ Hien términos de E,, y H,,,En el caso de Eun por ejemplo, se combinan las ecunciones (13.13) y (12.13) para obtener A (#En _ #2. ~ See a2a4y OF Jon az? ~ ox02 ay foieBg = De las ecuaciones (12.111) ¥ (12.12) se deduce claramente qué todas las componentes de campos Varian con z de acuerdo Con e~"+; es decir, to Bg VOR Bg wer i Por tanto, 2 wha one PE) EO? + one Ba 7 Se + he Jom Jom ox * ay Asi, si se concede que h? = 7 + wy = 9 + 1, Jonge Oy ay i Joy OH Hay _ fa, OH ee wax y Oe — Wax H, donde een eng De esta manera, es posible usar la ecuacisn (12.15) junto con las ecuaciones ( (012.12) para obtener Evy Eye Has ¥ Hye i ‘De las ecuaciones (12.11), (13.12) ¥ (12.15) se deduce que hay diferentes tipos. nes o configuraciones de campos, llamados modos. Exiaten cuatro categorias de a saber: 1. £,, = 0 = 1, (modo ET). Este es el modo electromagnético transversal etque tos campos Ey H son transversales a Ia direccion de propagaci fonds. Como se desprende de Ia ecuncidn (12.13), en esta cireunstancia Componentes de campos tienden a cero, de tal forma que Buy =O Hy: St ‘luye ast que una gula de ondas rectangular no puede tolerar el modo ET. 12.3. MoDOs MAGNENCOS THANSVERSALES (MTD mi 5:47) ‘Figura 12.3. Componentes de campos clectromagnéticos en una guta de ondas rectangular: (a) modo el, £, 0; (b) modo MT, 77, — 0. 2. Ry, = 0, H,, #0 (modes #1). Fin este ease, las componentes restantes (Fag ¥ Fy) dei campo eléctrico son transversales a la direccion de propagacion a,. Se dice entonces que tales campos se encuentran en modos eléctricos transversales (eT). ‘Vease la figura 12.3(a). 3. Ey, #0, Hy, ~ 0 (modos MT). En este caso, el campo Hes transversal a la direc in de propagacion de la onda, de lo que resultan los modos magnéticos transver- Sales (MTT). Vease la figura 12.3(D). 4, E., #0, H,, # 0 (modos 11). En este caso, ni el campo E ni el campo Hi son trans versales a‘ia direcci6n de propagacion de la onda, circunstancia a la que se le conace como modos hibridos (H). Cabe destacar la relacién entre k, en la ecuacién (12.3), y B, en la ecuacién (10.434). La constante de fase de la ecuacién (10.432) se dedujo con referencia al modo ET. En este modo f= 0, de manera que, con base en la ccuacin (12.16), 77 = Ky =a + JB = jk:es decir, p = k. En los demas modos. 8 * k. En Ins secciones siguientes examina- remos por separado los modos de propagacién MT y eT. .3. Modos magnéticos transversales (MT) Las componentes del campo magnético son transversales (o normales) en este caso a la direccion de propagacion de la onda. Esto implica fijar que /7, — 0 y determinar ,. Ey. ., Hy IT, mediante las ecuaciones (12.11) y (12.15) y las condiciones en la frontera. Despejaremos £, y después determinaremos a partir de él las demas componentes de ‘campos. En las paredes de la gufa de ondas, las Componentes tangenciales del campo E deben ser continuas: es decir, en (a2.17a) en 2.170) en a2a76) en a2a7a) Las ecuaciones (12.174) y (12-17e) implican que A, = 0 = A, en la e« manera que esta ecuacién se convierte en Eu donde £, = A,A,. Al aplicarse asimismo a la ecuacién (12.18), las ecu: (2.174) implican que Ey sen kx sen ky eo senka=0, senkb=0 Esto supone a su vez que kam mm, ma 12.3. kb = nm, No se cligen enteros negatives para m ¥ n en la ecuacién (12.202) por la 1 on el ejemplo 6.5. La sustitucion de Ia ecuacién (12.21) en la ecuacion (12.16 = eon (8) (Be ‘Las demas componentes de campos se obtienen de las ecu: niendo en cuenta que H., = 0. ASI, = CE) Boe (ME) CF) “fe (FF) Been (AEE) oe OH) 4. oe eo = (A 4 — “EE (GE) Baten (EF) ee a es (12.22) donde lo cual se obtiene de las ecuaciones (12.16) y (12.21). Conviene senalar ‘eouaciones (12.22) y (12.23) que cada conjunto de enteros m yn produce patron o modo de campos, llamado modo MT,,,,, en la gufa de ondas. El ent 12.3, Moos MAGNENCOS TRANSVERSALES (MT) 549) vale al nuimero de variaciones de medio ciclo en Ia direccién de x, mientras que el ente- ro 7 es el niimero de variaciones de medio ciclo en In direceién de y. Si, con relacion a ‘esas mismas ecuaciones, (7, 2) es (0,0), (0, 7) 0 (m, 0), todas las componentes de campos: tenderfan a cero. Asf, mim nin pueden ser iguales 4 cero, De esta forma, MT, €8 el MO do de menor orden entre todos los modos MT}. ‘Al sustituir la ecuacion (12.21) en la ecuacion (12.16) se obtiene la constante de pro- pagacién > V= eT 225) sdaai'h = wR: coo wif Sno (123) Rsonkedas ab foneral, 7 — a +30 eri ak Sais a Saito Rod Reeanraare nese GZ aca a Caso A (de corte): si aad =] yO 0 @=0=—8 El valor de que es la causa de esto se Hama frecuencia angular de corte w,: es decir, 2.26) Caso B (evanescente): si metus < rac B= En este caso no hay propagacién de onda, motivo por el cual a los modos respectivos de no propagacion’o atenuacién— se les llama evanescenes. Caso € (de propagacion): si o cual quiere decir que, con base en la ectuaci6n (12.25), 1a constants: a-Je—[]—[F] ste es el nico caso on el que hay propagacisn, ya que todas las compone pos posceran el factor e->* =e" ‘cada modo, carasterizado por un conjunto de enteros m y 1,1e corr frecuencia de corte f. De estar snaniers, la ule Ue gnidas opera come se obtiene de la ecuacion (12.26) como note pte E=l+cl NO donde u’ = Va = velocidad de fase de una onda plana uniforme en el me trico sin pérdidas (7 = 0, 44,2) que ocupa la gufa de ondas. La fongitud de Avesta dada por i Cabe referir con relaci6n a las ecuaciones (12.28) y (12.29) que MT}, ¢s. el moda} Ia menor frecuencia de corte (0 la mayor longitud de onda de corte). La con: fase de la ecuacién (12.27) puede expresarse en términos de f. como eaves -[4] a-ei- [AT «2230) donde B' = w/u’ = w/e = constante de fase de una onda plana uniforme en el medio dicléctrico. Vale seftalar que y para el modo evanescente puede expresarse en términos Sod nia manors "OT La velocidad de fase tp ¥ la longitud de onda on Ia guia catin.dadas respectivamente por (12.300) und 231 La impedancia intrinseen de onda del modo se obtiene de In ecunci6n (12.23) como = 1B) my el 2a donde 7’ = Vile = impedancia intrinseca de una onda plana uniforme en el medio. Reparese en In diferencia entre u’, 6" y m', por una parte, y t, 8 ¥ 7 por la otra. Las can~ tidades primas son caracteristicas de onda del medio dieléctrico no delimitado por la guia de ondas, como se explicé en el capitulo 10 (es decir, referentes al modo ET). Por ejem- plo, u” seria Ia velocidad de la onda si se eliminara Ia guia de ondas y el dieléctrico ocu- para todo el espacio. Las cantidades no primas son caracteristicas de onda del medio delimitado por Ia guia de ondas, ‘Como Ya se Mencions, los enteros m y 7 indican él niimero de variaciones de medio ciclo en la seccién transversal xy de la gufa. En la figura 12.4 se presenta, por ejemplo, la configuracién de campos en un momento fijo correspondiente al modo MT.y 12.4. Modos eléctricos transversales (eT) Vista lateral Campo _ Figura 12.4. Configuracion de campos correspondiente al modo MT3,. En los modos eT, el campo eléctrico es transversal (0 normal) a la direccion clon de Ia onda. Se fija Z, — Oy se determinan las demas componentes de Hi, H, y H, a partir de las ecuaciones (12.12) y (12.15) y las condiciones e tal como se hizo en los modos MT. Las condiciones en la frontera resultan que las componentes tangenciales del campo eldctrico deben ser continuas des de Ia gitia de ondas; es decir, us = en y=0 En en y=b E_=O0 en -x=0 E,-0 en x=a Con base en las ecuaciones (12.15) y (12.33), las condiciones en Ia frontera, Presarse como’ ots ay oH 12.4, MoOOS HLECTRICOS TRANSVERSALES (ET) MB 553 donde H, — B\By. Las demés componentes de campos se obtienen facilmente de las ecuacionss (12.35) y (12.15). en esta forma: st (SE) mom (MEE) en (Mem cz b= Sat (2) monn (EE) con (MEOH nae nif (G) Hoven (EE) (em anaes CB) meee (MEE) we (Bem za done Nek haiiplen cota ver m ym denoian el samero de variaciones de medio silo ea eee Se ta Buta En in Hgura 12.5 apasece por sjomplo, a con Ca eee ea ea arate in frecuisteln de corte Je Id longivud de onda de guracion de camp or 46) Tage pi ln'vclocidad de fase’, y In Wongitud de onda Ade tos See see ra a int de ioe modes MIT [veanne lak Seuaciones (12-28) « (12.319) Se oe ns eter (ns) pucde ver (0,1) © Ly 0), pero no (0-0 ¥ BO puede yaiae aorta Sniame ticenpo. porque sito fovzaria nis componcntes de cam aoe eee eeder meet, Esto implica que eT 2 ee, pueden tor et Fee a conan ad igs valores do ay blac dimensiones de I eta. Es commun modo mencr, dependiende elon pt in scuzclon (12.28), Aste iget ol modo Mmonor, PORQUE fog, — Hee < fan, = ME. Bate modo ne llama modo dominante deta gua de ondas a faa = NN Me Seipronmencion a NZI] \ 7; te Figura 12.8. Configuracion de campos correspondiente al modo ¢Tia y posee importancia préctica. La frecuencia de corte del modo eT 9 se obtien cin (12.28) como (1 = 1,1 = 0) 7 tom 2a en tanto que Ia longitud de onda de corte del modo eT,» se obtiene de la ecu: Dew =2a Notese que, de acuerdo con la ecuacién (12.28), Ia frecuencia de corte de whet + Bp? ab 1o cual es mayor que Ia frecuencia de corte de €1'yp, En consecuencia, MT), no. siderarse el modo dominante. 4 Adviértase asimismo que en la gufa no se propagara ninguna onda electrom: frecuencia F< fy (OA > ey) La impedancia intrinseca de los modos eT no es igual la de los modos M ecuacién (12.36) se deduce elaramente que (y — J) Cabe hacer notar respecto de las ecuaciones (12.32) y (12.39) que ner y Merson p te resistivas y varfan con la frecuencia, como se muestra en la figura 12.6 Rep bién en que er Meer = 1 En la tabla 12.1 aparecen importantes ecuaciones de los modos MT y eT para De las ecuaciones (12.22), (12.23), (12.35) y (12.36) se obtienen los pos de los modos MTT y eT: En el caso del modo dominante eTyy m= 1 ma que la ecuacidn (12.35) se convierte en. ram (Som H, En el Ambito temporal, H, ~ Re Ute) Hi, = H,cos (®) cos(wt — Bz) De igual manera, 9 partir de In councisn (12.26), 2, = %H4 11, sen (™=) coslwr — 2) 1, = SE tt, son (%*) sen(or — Bz) 4,0 Figura 12.7. VariaciGn de las com, de campos con ven el modo Fay, & i Figura 12.8, Lineas de campos del modo eT yg o propagacion =—— campo w ‘En la figura 12.7 se ilustra la variacion de los campos Ey H con x en un plano x—y —el plano cos(wt — 8.) = 1 para i, y el plano sen(wr ~ 82) ~ 1 para E, y I. por ejemplo— ch el modo eT,,. Las correspondientes Itneas de campos aparecen én la figura 12.8. ‘Una guia de ondas rectangular con dimensiones a = 2.5 em, b = 1. em operard por debajo de los 15:1 GHz. ;Cudntos modos eT y MT podra transmitir si se le rellena con un medio caracterizado por 7 = 0," ~ 4 tq,#2, ~ 1? Galcule la frecuencia de corte de los medos. Solucién: La frecuenci de corte esté dada por 2.5b0alb =25y Por tanto, -iV) 2+ oe ‘ | aay Te \ 3 10" zi ~ Fess Vie + 6250 Fog = 3 6S GH Lo que buscamos es f-,, ~< 15.1 GHz, Una manera sistemstica de hi en fijar mom incrementar el otro hasta que f.,, sea mayor que 15.1 GHz, cion (12.1.1) se desprende que de la fijacion de my el incremento den se pidamente una f,., mayor de 15.1 GHz. En el modo eTp: (ot = 0.0 = 1), fay = 32.5) = 7.5 GHz | modo €Ts( = 0,7 = 2), fog = (5) = 15 GHz modo €Toss fo, = 3(7-5) = 22.5 GHz Con relacion a f.. < 15.1 GHz, ast, el maximo = 2. Fijemos ahora 7 ¢ i . sea mayor de 15.1 GHz. mos m hasta que En el modo eTyo.(m = 1,1 = 0), fog = 3 GHz modo €Tap: fay = 6 GHz modo eTsy f. modo Tw fog = 12 GHz modo eT», fea = 15 GHz (igual que en el modo eT) mode 6Tain fog — 18 GH. 9GHz i de esta manera, con relaci6n a f._, < 15.1 GHz, el m maximo = 5. Habiendo do los valores maximos de m y mprobemos posibles combinaciones entre esos En eTy,, MT), (modos degenerados), fa, = 3.V725 = 8.078 GHz €T a, MT, foy = SVIOBS = 9.6 GHz. i Ty, MT 31, fay = 3VIS25 = 11.72 GHz eT ay MT si, fog = 3-V2225 = 14.14 GHz eT 12, MT 12, fe, = 3V26 = 15.3 GHz. ‘Serdn transmitidos los modos cuya frecuencia de corte sea menor que o igual: Griz; es decir, 11 modos eT y 4 modos MT (todos los modos aqui referidos, sal MT, eT ¥ eTy:)- La frecuencia de corte de estos 15 modos s¢ ilustra en el di lineal de fa figura 12.9. Frecuencias de corte de una gufa de ondas rectangular para el ejemplo 12-1 Escriba las expresiones instanténeas generales de campos de los modos MT y eT: Deduz- ca las de los modos eT, ¥ MTyz. Solucion: Las expresiones instanténeas de campos se obtienen de las formas de fasor mediante E = Re (eet) y Hi = Re (He™) ‘La aplicacién de estos valores a las ecuaciones (12.22) y (12.23) y el reemplazo de y y JB. resultan en los siguientes componentes de campos en los modos MT: |e (2) sn (G2) seer — 0 = BSE] on (AE) oe (292) star —20 si msde (EE) wee (EP) contee — 2) i, = $5 [FE] Boson (™E*) con (7G™) vemcar — He (27 e J ance 3 ) con (82) Seas ey sen (72) senior + zy 11 BCE) an (22) 292) te Ben Basen (22) etn (222) conta = 0) 12.4, Mo00s eLtermcos TmANSVERSALES (CT) ML Ser se, ~ ~$3(22) ss on (22) coe (252) een ~ ne) m= Ce) me (om (°F) nee H.-0 donde Bjercicio 12.2 ‘Una gufa de ondas rellena de aire y de 5 por 2.om tiene Eq, = 20 sen 40m sen SOmy 18" Vim. ais GHz, a) iCual ex el modo propagado? ») Halle B. 6) Determine B/F,. Respuestas: @) MTx,, 6) 241.3 rad/m y 6) 1.25 tan 40mx cot SOny. En una gufa de ondas rectangular con a Sem, b = 0.8em,o = 0,4 = wy 1 240m (22) eos (222) sen (100 — 2) atm Beene 2) El mero de oporsin. 3) te mecaeci de cone jee seen ones g ) La éonstante de propagacién y. ©) La impedancia intrinseca de onda 7. Solucién: a) De la expresién de /1, dada y las expresiones de campos del ejemplo anterior se des- prende claramente que m = 1,n = 3:es decir, que la gufa opera en MTy, 0 eT, Plijamos 1 modo MT,, (la eleccion del modo eT,, se analizaré en el ejercicio 12.3). | Por tanto, ae foo= SN TSR TOF * TORR IF = 221" (\Voaad + 1408) x 10% = 28.57 GH 2 B= Vue n/r - [2] = @ = 2af = 2 x 10" x 10%) 3x 10" 4) ¥ = J = ATI8Birad/m ee ° Bim con (2) So (292) Sono in| Propagacién de ondas en la guia El examen de In ecuacién (12.23) 6 (12.36) revela que todas las componentes de campos incluyen los términos seno 0 coseno de (rna/a)x © (nar/b)y por e 7, Puesto que sen = Ze" — e™) (a244a) cos 0 = 2 (el + em) azaapy una onda en una gufa de ondas puede descomponerse en una combinacién de ondas pla- nas reflejadas en las paredes de la gufa. En el caso del modo eT, por ejemplo, By ~~ aon (BE) erie = — SHB (ate — grin) @=ite (245) HR etme maipey — geet weg een El primer término de la ecuacién (12.45) representa a una gufa que se desplaza en la di- receidn 2 positiva en un angulo @=tan(Z) a2.46) con el eje z. EI segundo representa una onda que se desplaza en la direccién z positiva ‘en un angulo —0. Asi, este campo pede describirse como la suma de dos ondas ET pla- nas que se propagan a lo largo de trayectorias en zigzag entre Is paredes de la gufa en 3 — Oy x — a come se ilustra on la figura 12-10fa). La descomposicion del mode oT yp En dos ondas planas puede prolongarse a cualquier modo eT y MT. Cuando 1 y 7 dific ren de cero, Ia descomposicién produce cuatro ondas planas. La componente de ondas en la direccidn de = tiene una longitud de onda diferente que las ondas planas. Esta longitud de onda a lo largo del eje de la gua se llama longitud de onda de la guia de ondas y esté dada por (véase el problema 12:13) a azar) donde A’ = w'/f. ‘De las trayectorias en zigzag surgen tres tipos de velocidad: la velocidad del medio u", la velocidad de fase up y la velocidad de grupo u,. En la figura 12.10(b) se ilustra la rela- cion entre ellas. La velocidad del medio u’ — 1/V/ue €s como se explicé en las seeciones Figura 12.10. (@) del modo eTy en dos (@) Felncién entre Wey t © anteriores. La velocidad de fase u, es Ia velocidad a la cual los lugares g: fase constante se propagan por la'gufa y esta dada por la ecuacion (12.31) ale Esto indica que up = u’, ya que cos @ = 1. Siu’ = ¢, entonces u, es mayor at dad de la luz en ef vacio. :Esto contradice la teorfa de la relatividad de Ei cual un mensaje no puede desplazarse a mayor velocidad que la de la luz? no, porque la informacién (o energia) en una gula de ondas no sucle dest locidad de fase. La informacion viaja a Ia velocidad de grupo, Ia cual debe la velocidad de la luz. La velocidad de grupo u, es la velocidad con Ia que las ondas reflejadas repetidas se desplazan por la guia y esté dada por En vista de que el concepto de velocidad de grupo es muy complejo'y'rebasa los alcan- ces de este capitulo, baste decir que se trata en escncia de la velocidad de propagacion del envolvente de paquete de ondas de un grupo de frecuencias, Fs la velocidad de pro- pagacion de energia en la gufa, siempre menor que o igual a 1’. Las ecuaciones (12.48) ¥ (2.49) revelan que Ugly = > 42.50) relaci6n similar a la ecuacién (12.40). De ahf que la variacion de u, y 4, con Ia frecuencia Sea semejante a la que se mostro en la figura 12.6 respecto de nor'¥ moe ‘Una guia de ondas rectangular estandar rellena de aire con dimensiones a = 8.636 cm, b = 4.318 em es alimentada por un portador a4 GHz desde un cable coaxial. Determine ‘si por ella se propagaré el modo eT, De ser asf, caleule In velocidad de fase y la veloci- dad de grupo: Solucién: En el modo eT yo, f, = 4’/2a. Puesto que la gufa de ondas est rellena de aire, u’ = ¢ = 3 = 10%, Por tanto, ___3.x 108 Je. 25 ReS6 3 108 STG Como f= 4 GHz > f,, el modo eT,» sf se propagars por Ia gufa. _ 0 3 x10 Me" Va = Gap t= 73714 = 3:33 x 108 m/s u? __9 x 1018 = SEIS = 2.702 x 10" mis Para determinar el flujo de en Ia gufa de ondas, se halla primero el vector de Poynting promedio [a partir de Ia ecuacidn (10.68) | 2s En este caso, el vector de Poynting se encuentra a lo largo de la dir mera que Prom = FRO Bal Ente, 5 _ FuF + | 2a donde 7 = ner en los modos eT y 7 = nyrp en los modos MT. La potencia transmitida por la seceidn transversal de la gufa de ondas es Pa [Ppa af ph eat aca Syl ay ax La atenuacién en una gufa de ondas disipativa posee importancia pré aqui hemos supuesto guias de ondas sin pérdidas (o' = 0, ¢, ~ oo) en las cus }'= 1B. Pero cuando el medio dieléctrico es disipativo (o # 0) y las paredes de Son perfectamente conductoras (¢., # 00), ocurre una continua pérdida de p forme una onda se propaga por la gufa, De acuerdo con las ecuaciones (10,69) el flujo de poteneia en la guia es de la forma, Pyrom = Peet Para conservar esa potencia, el indice de decremento de Pom debe ser igual a de potencia promedio temporal P, por unidad de longitud: es deci AP prom az P= — 202 F prom a 2 BP rom En general, ema Oy donde «, y @, son constantes de atenuacién debidas a pérdidas Shmicas o de (e. 4) y a pérdidas dieléctrieas (e * 0), respectivamente. Para determinar a, recuérdese que en Ia ecuacion (12.1) se partio del suy un medio dieléctrico sin pérdidas (7 ~ 0). Con referencia a un dicléctrico dist yreciso incorporar el hecho de que o + 0. Todas nuestras ecuaciones anteri jendo validas en este caso, salvo que y ~ /f debe modificarse. Para hacerlo, se. 2a en la ecuacién (12.25) por la permitividad compleja de la ecuacién (10.40), nein CHS OY wan donde eae Sprung cr2sey La sustitucion de la ecuaciGn (12.58) en In eeuaci6n (12.57) y la elevacién al cuadrado de ambos miembros de I ecuaci6n resulta en 7 Bi + Byes = (7) + (BE) — otne + jouer ‘Al igualar las partes real ¢ imaginaria, oa — p= Cast (EY - ome (42.592) Denafla = oner oe 2-500) Si se supone que, a3 << £3. a3 — 63 — —B%,de la ecuacién (12.592) se obtiene a= ome CET CY ~ oven Jo cual ex lo mismo que # en la ecuaciGn (12.30). La sustitucién de la ecuacién (12.60) en Ia ecuacién (12.595) produce (42.60) ope concn at a2.61) ay ,-(£ G donde n' = Vale La determinacion de a, en los modos MT», ¥ €Ty, &s larga y tediosa. Iustraremos el procedimiento hallando a. en cl modo ¢T,. En este modo s6lo existen Ey, H, ¥ H.. La Sustitucién de la ecuacién (12.434) en la ecuaci6n (12.53) da como resultado yon a Pea Poet Jeno Jeno (42.62) pe eae La pérdida de potencia total por unidad de longitud en las paredes es Prom Pe lot Pe lyr + Pe leo + Pr lene 2.63) 20Px no + Pre leno) ya que en las paredes y = Oy y = box = Oy x = ase disipa el mismo Gia. En cuanto ala pared y = 0, Pelpew = $Re [nm f Wer + 17th) ax] =a = ~ Ba ES He [fF ms near tae + [f scot ae] donde &, es la parte real de Ia impedancia intrinseca 9, de la pared conducto se en In ecuncion (10.56), 4 1 Re-a os donde 5 es Ia profundidad pelicular. R, es la resistencia pelicular de Ia pared: siderarsele como la resistencia de 1 m por 5 por 1 m del material conductor. En« Ia pared x = 0, Prdy|uno= 3x, [" a3 ay Prlewo=4Re [ne f (er ( _ RHE tee ee dil OS ie ee aa) a r= na [o+$(1+ 22)] d Por tltimo, al sustituir las ecuaciones (12.62) y (12.67) én Ia ecuacién (12.55) se ob nun[o+$(1+82)]ay eS 5, modo eTyo se obtione. 12.7. CORRIENTE EN LA GLIA DE ONDAS Y EXCITACION BE MODGS mI 569. Siguicndo el mismo procedimiento, la constante de atenuacisn en los modos €T ig, (11 # 0) 68 -— eager 2Gre) ae wate Se 2 ee iad i vi -[4T (lay? m+ La constante de ateuacion total « se obtiene al sustituir las ecuaciones (12.61) y (12.69) ‘© (12-70) en Ia ecuacién (12.56). nte en la guia de ondas y excitaci6n de modos En lo que respecta a los modos tanto MT como eT, la densidad de corriente superficial Ken las paredes de la gufa de ondas puede hallarse mediante K=a x a2 donde a, ¢5 el vector unitario de salida normal a la pared y Hla intensidad de campo eva- Iuada en la pared, EI flujo de corriente en las paredes de la gufa para la propagacion en cl modo eT, puede hallarse mediante la ecuacién (12.71) en combinacién con las ecua- Clones (12.48) y (12.43). El remultada sc deseribe gréficamente on la figura 12-11. ‘La densidad de carga superficial p, en las paredes esta dada por Ps = 8,7 Dm a, cE, a272) donde E ¢s la intensidad de campo eléctrico evaluada en la pared de Ia gua, Fjemplo 12.5 (©) Modo MT), 2 Figura 12.12. Exeitacién de modos en una guin de ond: rectangular oi ‘Una guia de ondas sucle ser alimentada o excitada por una linea coaxial de ondas. En la mayor parte de los casos se utiliza una sonda (conductor cent Ifnea coaxial) para establecer la intensidad de campo del modo deseado y maxima transferencia de energia. La sonda se coloca de tal manera que se pi campos Ey H aproximadamente paralclos a las Ineas de los campos Ey Ht deseado, Para excitar el modo eT,y, por ejemplo, se parte del hecho de que, do con la ecuaci6n (12.43a), F, aleanza su maximo valor en x = a/2. Por tanto, se coloca en x — a/2 para excitar el modo ¢T yy, como se muestra en la figura Cuyas lincas de campos son similares a las de 1 figura 12.8. De igual forma, ps el modo MT;,,1a sonda se coloca a lo largo de Ia direccién de z,como se indi gura 12.12(6) Una gufa de ondas rectangular rellena de aire con dimensiones a = 4.em,b = 2, porta energia en el modo dominante a un indice de 2 mW. Si la frecuencia de. ¢s de 10 GHz, determine el maximo valor del campo eléctrico en la gufa. Soluctén: Cuando @ > 6, el modo dominante es eT jy. Las expresiones de campos correspont este modo (m ~ 1,1 = 0) se hallan en la ecuacion (12.36) 0 (12.43): ou a, Ey=0, Ey — —jE,sen (®) dies, t dora Eg uw 3x10 fom 3a" Wa toy 7 375 OBE 1 Ne ee ee - 108.7 vel] v!-Det ‘A partir de la ecuacién (12.53); la potencia promedio transmitida es re lee A ene Poon = [8 ad oS hab Por tanto, AnPpom _ 806.7) 2 > 10-9 _ ey ~ See XA 4067 Eq = 63.77 Vim. ‘Una guia de ondas con revestimiento de cobre (o", = 5.8 x 107 S/m) que opera a 4.8 GHz debe alimentar una antena con una potencia minima de 1.2 kW. Si la gufa esté ocupada por poliestirenc (a — 10-7 Sim, 2 = 2.55e,) y sus dimensiones son a — 4.2 cm, b = 2.6 Em, calcule Ia potencia disipada en una longitud de 60 em de la guia en el modo eTyo. Soluciém: Sea P, = potencia perdida o disipada P., = potencia transmitida a la antena P= potencia de entrada de la guia de manera que Py = Py + Ps, ‘Con base en la ecuacidn (12°54), = Pee En consecuencia, Pia Py Pay erm Py = Pe 1) Ahora es preciso determinar a a partir de De acuerdo con 1a ecuacién (12.61), Puesto que la tangente de pérdida 10°77 semana we a aa = 1.47 x 10-17 << 1 (medio dieléctrico sin pérdidas) fe _ am jaf Sma TONE W VE we SJ = > = 12879 x 10° mis Vue Ve, ui _ 1.879 x 108 -£-32 2.234 GH 1.334 x 10-15 Np/m En cuanto al modo eT», la ecuaci6n (12.68) resulta en, = geet donde _ [efe [eR aR RIOR ae IO ob oe 38 x 10" = 1.808 x 10-7 0 Por consiguiente, (on « 28/224) 2x 1aoe > 107(os + 29[22347) 2e x10 xsi — 22 4.218 = 10"? Np/m Obsérvese que ay << ez, 10 que indica que la pérdida debida a la conductividad finita de las paredes de la guia cs mas importante que in debida al medio dieléctrico. Ast, 4.218 = 10-3 Np/m y la potencia disipada es Pg = Py (OA — 1) = 1.2 x 10 er MA2TEHITO8 — 1) = 6.089 w Ejercicio 12.6 ‘Una guia de ondas de cobre (a7. = 1.1 x 107 mhos/m) de dimensiones a = 4.2 em, b= 1.5 em esta rellena de tefl6n (6, = 2.6.¢ = 10" mhos/m). La frecuencia de ope- racign es de 9 GHz. En relacién con cl modo eT yo, a) Caleule ay y ae 2) 1Cuél es la pérdida en decibeles en la gufa si ésta es de 40 em de largo? Respuestas: a) 1.206 x 10-9 Np/m, 1.744 = 10-* Np/m yb) 0.0606 dB. ‘Trace las Ifneas de campos del modo MT. Deduz las expresiones instantaneas de la densidad de corriente superficial de este modo. Sotucisr Del ejemplo 12.2 se obtienen los campos relativos forma: Bs = F(Z) Faces (St) sen (3) sont = 0) 2, (2) asen (2) con (32) senter = 60 +-G =) sen (2) conor ~ a2) $25 (2) Basen (22) cos (22) sentan — 82) = 55 (2) 2scoe (2) sen (52) seater — pe) H.=0 Respecto de las Ifneas del campo eléctrico, gue (DoW Respecto de las Iineas del campo magnético, Pant (22) ton { SF ae Ce) Repérese en que (B/E,)(Fh/H,) = ~ 1,10 que indica que as tineas de ose Peer tae Dt ep pe nen a $2 121S%oa ls que aparoton in ines fog campos ema a Stroma tape tsa oe hase on a Sada ay _ 4H, Kaa, XH = a, x He Hy) En x= 0,0, = 9,K = H,(0,y, 2,1) a,,e8t0 08, x = 9 (Z) mean (52) aontor — 0) we Enx =a,a, = —a,,K = —H,(a,y, 2,0) a -2 (2) Bj ton ® sen(wt — Bz) a, x Figura 12.13. Lineas de eampos de! modo para cl ejemplo 127. 12.8, ResONADORES DE CULMS DEONOAS i575) Eny = a. K = —7,(,0,2,0 a, $5 (F) ee sen (7) senor — pz) a, 9o (F) Basen (7) sentwr — 2) a, | Bjercicio 12.7 “Thace las ineas de campos corresp. Respuesta: Vease la figura 12.1/ nsidad de Bins dette staan (omaice) ntanes dood | lineas estén mas unidas (o mas separ oe Eny . Resonadores de gufas de ondas Los resonadores sirven principalmente para almacenar energfa. A altas frecuencias (de 100 M¥iz y superiores). los elementos de circuitos RLC son ineficientes como resonado- res, ya que jcnsiones de los circuitos resultan comparables con la longitud de onda de operaciGn, lo que produce una radiacion indeseable. A altas frecuencias, ast, los cireul~ tos resonantes RLC son reemplazados por ca Figura 12.14. Par el ejercicio 12.7, mado eT. uso comin en tubos de clistr6n, filtros de pasabanda y ondimetros. El horn das se compone en csencia de una fuente de potencia, una suta de ondas una cavidad (el horno propiamente dicho). Considérese la cavidad rectangular (6 caja conductora cerrada) que figura 12.15, Como puede verse, se trata simplemente de una gufa de anda acortada en ambos extremos. Es de esperar entonces la presencia de una oF tia, asf como de modos MT y eT de propagacion de ondas. Segdn la forma d de la cavidad, las ondas pueden propagarse en Ia direccion de x.) 0 z. Op direccin +z como “direccion de propagaciGn de ondas” pese a que, en circunstancias no hay propagacién, sino ondias estacionarias. Como se. seccidn 10.8 se explicé que una onda estacionari desplazan en direccion opuesta, A. Modo MT a z En este caso H, = Oy concedamos que B09, 2) = XC) YO) ZG) €s la solucién de producto de la ecuacidn (12.1). Siguiendo el mismo procs fen In seccién 12.2 ne obtiene X(O = €, 608 kex + 250m kx ¥Q) = c)e08 ky + cysen ky ZC) = es C08 kz + 6800 kez donde WORKER 4G +E = one Las condiciones en Ia frontera son: E,.=0 en E,-0 en E,=0,E,=0 en Figura 12.15. Cavidad rectangular. ‘Como se demostré en la seccién 12.3, las condiciones de las ecuaciones (12.7a y b) se satisfacen cuando ¢, = 0= b = ¢,esto implica que La = Ub > V/e, y de ahi que ol modo di Sen eTyy;. Nétese que cuando @ = b= ¢, la frecuencia resonante del modo mayor que la del modo eTjo,; Por tanto, eT jo, es el modo dominante. A modos con igual frecuencia resonante se les llama modos degenerados; un modo do} los demas septin In forma de excitacion de la cavidad ‘Una cavidad resonante prisctica tiene paredes de conductividad finita oy, puede pertler enargia slmncemaria. Fl faster ee calidad @ peamite determina’ oad 128 12.8. ResONADORES DE GUIAS'DEONGAS) mI) 579° Energia promedio temporal almacenada O = 20 Dérdida de energia por ciclo de oscilacin w w BEA Oe (2.89) donde T ~ 1/f ~ el periodo de oscilacién, P,, es la pérdida de potencia promedio tempo- ral en la cavidad y W es la energia total promedio temporal almacenada en los campos Cldcisteg x magnéden dantre, dela opvided. El @ de une cavided resonddora tucle b muy grande en comparacion con el de tn circuit resonante RLC. Siguiendo un proced miento similar al utikizado en la deduccidn de a, en In seccién 12.6, es posible demostrar que el factor de calidad del modo dominante eyo, estit dado por? (@ + Qabe erm = SBR + O) + acl +] 1290) donde 5 = Eye +e ——— es la profundidad pelicular de las paredes de la cavidad. fF sibeare En ol caso de una cavidud resonante de cobre (¢, = 5.8. 107 mhos/m) rellena de aire y con dimensiones @~ 5.cm,b—4emyc= 10cm, halle 4) Los cinco modos de menor orden. b) Es tactor de calidad det modo er" Sotucin: a) La frecuencia resontante esta dada por donde 3 Para efectos de comprobacién, véase 8. V. Marshall y GG. Skitek, Electromagnetic Concepts and “Applications, 3a. 6d., Prentice-Fiall, Englewood Cliffs, 1990, pp. 440-442. Por tanto, T+ [tee 15-V0.049? + 0.0625n? + 0.01 p? GHz Puesto que ¢ > a > bo Ve < Va < 1/5, 01 modo de menor orden es &T oy Baty 9 egg no oxinton, ya que m ~ {2.30 ¢ san = 12 losmodos My m OQ n— Od. yp haa frecuencia resonante del modo Cig e= Tom 7 ASV0.048 FO OO = 3.335 GHz El modo menor inmediatamente siguiente es €Tyy, (MTy,; no existe), con Tom = 1 VO F 00625 F OUT = 4.04 Giz El modo siguiente es Tyg: (MTiu2 nO existe), con Snug = ISVO04 + OF O04 = 4.243 GH El modo siguiente es MTyyo (Ty nO existe), con Tom = 1SVO04 + 0.0623 + 0 = 4.8 GHz Los dos modos siguientes son eT), ¥ MTy1; (modos degenerados), con Som = 150.04 + 0.0623 + 0.01 = 5.031 GHz EI modo siguiente es MT ya, con Sry = 150.04 + OF 0.09 = 5.408 GHz De menor a mayor, asf, los cinco modos de menor orden son Tio G35 GHz) Tou (4.04 GHz) Tee (249 Gls) MT io GsGH) eT OMT, (5.031 GHz) b) El factor de calidad de eT yo; ests dado por @ + 2) abe eg = & 2 QE +) + acl + YT (25 + 100) 200 = 107 ‘B[aCI25 + 1000) + 5025 + 100)] 1 _ Veh neee ois or VxG35 R10) te 107 SB XAT) or = 14358 * mene Las gufas de ondas son estructuras para encauzar ondas electromagneticas a altas fre- cuencias. En el andlisis de la propagacion de ondas electromagneéticas en una guta de ondas rectangular sin perdidas (¢. — 0°, & — 0) se aplican las ecuaciones de Maxwell. La resultante ecuscion diferencial parcial se resuelve con el método de separacion de Variables, De la aplicacin de las condiciones en la frontera a las paredes de la guia se Sbtienen Ins formulas basicas para 1a guia segun el modo de operacion. MT nu ¥ €Tyymy donde m y 11 SON enteros positives, son dos modos de propagacién (© pa- trons de campos). En los modos MT, m = 1,2,3,. y= 1,2,3,- yen los mo dos eT, m=0,1,2,..-¥n= 01,2... .,n—-m40. ‘Con cada modo de propagacién se asocian una constante de propagacion y una tre- cuencia de corte. La constante de propagacion y — a + JB depende no sdlo de los parametros constitutivos (©, #4, ¢) del medio, como en el caso de ondas planas en un. Espacio no delimitado, sino también de la dimensiones de la seccidn transversal (a, ) de la gufa, La frecuencia de corte es la frecuencia en la que y pasa de puramente real (atenuacién) a puramente imaginaria (propagacion). El modo dominante de opera- cidn es ol menor modo posible, aquel con la menor frecuencia de corte. Sia > b, el mo- do dominante es eT Las ecuaciones basicas para calcular Ia frecuencia de corte f,, la constante de fase 8 y fGrmulas para calcular las constantes de atenuacion debidas a un medio dieléctrico di sipativo y a paredes imperfectamente conductoras. La velocidad de grupo (0 velocidad del flujo de energfa) u, se relaciona con Ia veloci- dad de fase , de la propagacion de onda de acuerdo con lglg =u? donde u’ = 1/V/jze es Ia velocidad del medio, es decir, la velocidad de 1a onda en el medio dicléctrico no delimitado por la gufa. Aunque u, es mayor que w’,1, no excede de tp. El modo de operacién de una gufa de ondas est determinado por el método de exci- tacisn. Una eavidad resonante de una gufa de ondas sirve para almacenar energia a altas fre- cuencias. No es sino una guia de ondas acortada en ambos extremos, de ahf que su ana isis sea similar al de aquélla. La frecueneia resonante de los modos tanto eT como MT eesti ose) +(e) + [2] En los modos MT, m = 1,2,3, dos eT, m= 0,1,2,3,....2 = 0,3,2.3,.. yp el modo domininie (aquel con, menor frecuencia resonante) es eT), 8. El factor de calidad, el cual mide la pérdida de energfa en la cavidad, 42.41, En frecuencias de microondas las gufas de ondas son preferibles a las linens g para transportar energia electromagnética por Ins casas siguientes, excepio i) Lak peruidab én ine Mncas Go trinemnisiée’ ton! proKibiionmatra'fehnden 2) Las guias de ondas son de mayor ancho de banda y menor atenuaci6n de ©) Las lineas de transmision son de mayor tamafo. 4) Las lineas de transmisién slo toleran el modo FT, 32.2. Un modo evanescente ocurre cuando @) En una onda se presenta atenuacién, no propagacién. 2) La constante de propagacion es puramente imaginaria. ©) m= 0 =n,de manera que todas las componentes de campos tienden a cero, 4) La frecuencia de onda es igual a la frecuencia de corte. 12.3. EI modé dominante en las gufas de ondas rectangulares es a ery 2) MT, © Tie d) Te 12.4. _Elmodo MT puede existir en una guia de ondas rectangular @) Cierto. 1) also. 125. ;Cusles de las siguientes componentes de campos existen en el modo eTy? or ») zB, Ok oH, oH, 22.6. En una guia de ondas rectangular en ia que a = 26 y en la que la frecuencia de corte Wel mo: do cq, ex de 12 GHz, ln frecuencia de corte del modo My, es de 4) 3GHe ») 3V3 one ©) 12GHz a) 6V5 Guz ©) Ninguna de las anteriores. 32.7. Dentro de un tinal con seccién transversal de 4 por 7 m, un automévil!no recibirg una se- Ral de radio AM (f= 10 MHz, por cjemplo). 4) Cierto. 5) Falso. 42.8, Cuando el campo eléctrico alcanza su maximo valor, Is energia magnetiea de una cavidad se a) Sa maximo valor >) V2 de su maximo valor. 4 9 eee a we 1/2 de aw mbximo valor. ©) Cero 429, ,Cuti de los modor siguientes no existe en una cavided resonante rectangular? @ Tho B) eTon ©) MT no a) MT ya 42.10. ;Cusntos modos dominantes degenerados existen en una cavidad resonante rectangular en er ao bye oO3 as oc Respuestas: 1216, 122a, 123d, 12.4b, 12.5b, d, 12.6b, 12.7a, 12.8¢, 12.94, 12.10 12.4. a) Demuestre que una gufa de ondas rectangular no tolera los modos MTy ¥ MToy 1b) Explique Ia diferencia entre los modos eT ¥ MT yr: as. a2. 128. 32.10. ‘Si una gufa de ondas de 2 por 3 cm rellena de un material dieléctrico’éon. Griz cn el modo MT}, halle: a) Ia frecuencia de corte, b) la coustante de Gad de fase. ‘Una guia de ondas de 1 > 2 cm esté ocupada por agua desionizada con », cueneia de operaciones de 4.5 GHz, determine: a) todos los poribles modou Y sus frecuencias de corte, b) la impedancia intrinseca del modo Mayon, e) grupo del modo menor. Disefe una guia de ondas rectangular con proporcién dimensional de'S a 1. a banda k (18-26.5 Giz), Suponga que esta rellena de aire. Determine al por tdnel disefiada camo una gufa de ondas metilica rec se y-con dimensiones a= 8m y b — 16 m pasard a) una sonal de radio AM de En una gufa de ondas rectangular rellena de aire, la frecuencia de corte del 5 GH, miontras que la del modo eT, ex de 12 Gliz, Calcule 4) Las dimensiones de la gut 2b) La frecuencia de corte de los tres moos eT mayores. ©) La frecuencia de corte del modo eT, si Ia gufa estuviern ocupada por un ma digas cone, ~ 2.259, = 1 Una gufa de ondas rectangular huces rellena de aire tiene 150 m de largo y es un extremo con una placa de metal. Si en su entrada se Introduce un impulto en GH de frecuencia, Zeusmto tiempo tardara el impulso en Volver al siamo punt la gufa una frecuencia de corte de 6.5 GHz, Calcule las dimensiones de una gufa de ondas rectangular rellena de sire en la cuencia de corte de los modos MTy, y eT es de 12 GHz, Determine st a 8 Gi ‘sara o desvanecerd el mode dominante Las dimensiones de Ia secci6n transversal de una gufa de ondas rectangular rel son a= Gem yb = 3m. Puesto que soon (225) n (242) con00"r— 2) Vm ealcule In impedancia intrinseea del modo correspondiente y el flujo de potencia fen la gua, En una guia de ondas rectangular rellena de aire, un mode &T que opera EB, = 5 sen(@ns/a) cox(ary/b) sen(wt — 12¢) Vin Determine: a) el modo de operacion, b) ta frecuencia de corte, ¢) In imped: seca, a) My aan. aaa. aaa. waa, was. 1236. azar. san. En una guia de ondas rectangular rellena de aire con a = 2.286em y b ponente y del modo eT esta dada por .016 em, la com- E, — sen(2ersla) cos(Smy/b) sen(10m X 108% — Be) Vim Halle: a) e1 modo dle operacién, b) la constante de propagacisn 7, ¢) Ia impedancia intrin- seca 7. ‘Deduzca la formula splicable al modo MT;, para calcular Ia potencia promedio transmitida por la gute. a) Demuesire que en una gufa de ondas rectangular ET cael > Conran awn go ism rloatd s com 8 O eon ¥s par 8 Snoacaan, ghieo omer rn ‘Una guia de ondas rectangular de 1 3 em rellena de aire opera en cl modo eT; a una frecuencia 20% mas alta que la de corte. Determine: a) la frecuencia de operacién, b) Ia Velocidad de fase y de BrUpo. Un transmisor de microondas esti conectado con una antena a través de una guia de Ondas rellena de aire con seeciGn transversal de 2.5 > lem. Respecto de una transmisién ST Gris hatle ta razon de a) la velocidad de fase a Ia velocidad del medio y 6) la veloci- dad de prupe a la velocidad del medio. ‘Una guta de ondas rectangular esté rellena de polietileno (« = 2.25e,) y opera a 24 GHz. Sila frecuencia de corte de clerto modo eT es de 16 GHz, halle la velocidad de grupo y 1a impedancia intrinscea del modo. [La gufa de ondas rectangular cuya secciGn transversal se muestra en la figura 12.16 presen- Ee diteentinuidiad dicicetniea, Calcule ia razon de onda extaclonria st a gala upera a GUE en el modo dominante. Fl anstisis de gutas de ondas cireulares implica resolver la ecuacisn escalar de Helmholtz en Coordenadas cilindrieas; es decir, VE, + RE, = 0 Figura 12.46, Para el problema 12.17, a2a9, 1a (pA8s) «Ale he aE PEs aE, = 8 7) pt oe * az Suponieado ta solucion de producto Eqs tr 2) = RQ) WB) Z(2) demuestre que las ecuaciones separadas son: a Kze0 w+ Roo OR + PR’ + Ugo? — ha) R= 0 eae+e En el modo eTox, ,, ~ eee Mysentryibe7 By = 0 Halle Pom ¥ Pram , Si una gufa de ondas de cobre (e, = 5.8 < 107 S/m),1 > 2 cm y rellena de un Igetrico con © = 2.60.44 — stg. oy = 10-* Sim opera 8 9 GHz, evalite acy ay OMT ‘Una guia de ondas cuadrada de 4 cin por lado rellena de un dieléetrico con compleja e, ~ 16e,(1 ~ 10~*) ex excitada con el mode MT). Si opera a una free superior a ia de corte, calcule la atenuacion ay ZQué distaiicia recorterd la onda Antes de que su magnitud se reduizen 20%? lng paredes de la gufa de ondas cuadrada del problema anterior son de cobre: 107 Sim), halle a, ¥ la distancia que recorre la onda antes de atenuarse 30%. Una guta de ondas rectangular con a = 2b — 4.8 em esta rellena de teflén con y tangente de pérdida de 3 ~ 10-7. Suponga que sus parcdes estan recubie, (@, 4:1 10" Sim) y que por ella se propaga una onda elyy 44 GHz. Halles a) Una guia de ondas rectangular de cobre (er, = 1.37 > 10? S/m) con dimensiones @ yb ~ 1.5 em opera en el modo dominante a una frecuencia des GHz. Siesta tell f16n Gu; ~ 1,8, = 2.11, 0 ~ 0), determine: a) Ia frecuencia de corte dei modo dor Ia constante dé atenuacion debida a la pérdida en Ins paredes de la puta, Con referencia a una gufa de ondas cuadrada, demucstre que la stemuincién a £8 elmodo eT; cuande f= 2962/5 wz27. 1230. 21. {La constante de atenuscién de un modo MT esta dada por LA qué frecuencia aleanzardé a su méximo valor? Demuestre que en el modo eT’ z en una cavidad rectangular, ras BE (RE nan 8) an) aa #2*) Hi me CE mS) EE) a) en ¢l modo MT a z, Determine E, ‘Cuél es el modo dominante en uns cavidad resonante rectangular cuando wacbb Respecto de una cavidad rectangular rellena de aire con dimensiones a ~ 3em,b = 2 em,c seirSm, determine la frecuencia resonante de los modos siguientes: €Tiyy, €Tigx, MTy,0 ¥ MrT, Bnumere las frecucncias resonantes en orden aucendente. Si tna tavidad resonante rectangular con dimensiones a ~ 3 cm, b = 6 em y c = 9 em esta Stupada por polietiiens: (e = 2iSe,), halle ta feeetancia vexonante de lon cinco DIIMerOS modos de menor orden. ‘Una cavidad csibica rellena de aire opera a une frecuencia resonante de 2. GHz cuando se le Sheita en el modo eT, Determine sus dimensiones. Respecto de tna cavidad cabica de cobre (@r, = 1.37 107 Sim) rellena de aire y de 3.2 cm por lado calcule: a) la frecuencia resonante del modo €Tygy,d) cl factor de calidad de ese todo. Disefe una cavidad ecibica rellena de aire cuya frecuencia resonante dominiante sea de 3 GHz. ‘Una eavidad enbica rellena de aire de 10 em por Indo tiene E = 200 sen 30mrx sen 3079 cos 6 X 10% 73 Antenas Los diez mandamientos del éxito “Trabaja con ahinco: el trabajo intenso es la mejor inversién. * Estudia con esmero: el conacimiento permite trabajar mds inteligente y efi ‘Toma iniciativas: los caminos trillados se convierten en tums, ‘Ama tu trabajo: después derivarss placer de dominarlo. . Sé exigente: los métodos desalinados dan resultados desalifiados, ‘Ten espiritu de conquista: ast podras combatir y vencer toca dificultad, CCultiva tu personalidad: esta es a tn individu le que el perfume a ta fly Ayuda a los demas: la verdadera prucha de la grandeza en los neocon ‘porunidades. ). Sé democrético: si no respetas a tux companerns, Jamas serse un lide de Haz siempre tu mejor esfuerzo: quien ha hecho su mejor esfuerzo le ha todo. Hacer menos es hacer nada, ‘Cranes Mi 13.1. Introduccion Hasta este momento no nos hemos preguntado aun cémo se producen ondas el néticas. Como se recordard, los campos electromagnéticos son producto de. tricas. Si In fuente varia en el Gempo, las ondan electromagnéticas se prop: radiaciGn. La radiacién puede percibirse como el proceso de transmision de e trica. La radiacién o emisidn de ondas en el espacio se cumple eficientemente de estructuras conductoras o dieléctricas Hamadas antenas. ra puede emitir ondas electromagnéticas, pero no todas son mecanismos de ‘Una antena también puede concebirse como un transductor para él acopl Ja linea de transmisin 0 guia de ondas (vias de encauzamiento de la onda por! el medio circundante o viceversa. Fn la figura 13.1 se ilustra esta funcion. Las ai indispensables para una radiacién eficiente y el acoplamiento de impedaneias fin de minimizar la reflexion. Se sitven del Voltaje ¥ Ia corriente de la linea de sidn (0 de los campos electromagnéticos de Ia gufa de ondas) para emitir una - tromagnética en direccién al medio. Pueden usarse para transmitir o recibir electromagnética. r Medio cireundante Figura 13.1. Antena como dispositive de acoplamiente entre Ta'cstructura de guia y el medio cireundante. En Ia figura 13.2 aparecen antenas de uso comin. La antena de dipolo de la figura 13.2(a) consta de dos alambres rectos tendidos a lo largo del mismo eje. La antena de cua: dro de la figura 13.2(b) se compone a su vez de Una o mds Vueltas de alambre. La antena hielicoidal de la figura 13.2(e) consta de un alambre en forma de hélice sostenido en un plano conectado a tierra. A todas estas antenas se les conoce como antenas de alambre; Se'usan en automéviles, edificios, aviones, barcos, etc. La antena de bocina de la figura 15.20), cjemplo de antena de abertura, es una seeciGn piramidal de una gufa de ondas que sirve de transicién entre la guia y el medio circundante. Dada la facilidad para instalarla Sl ras, resulta Util en varias aplicaciones, como en aviones. En el reflector de disco para- Bolico de la figura 13.2(e) se aprovecha el hecho de que las ondas electromagnéticas son Feflejadas por una lémina conductor, Cuando se Ie emplea como antena transmisora, en Cl punto focal se coloca una antena de alimentacion, va sea de dipolo o de bocina. La fadiacion que procede de la fuente se refleja en cl disco (a la manera de un espejo), de lo que resulta unhaz de rayos paralelos. Este ultimo tipo de antenas se utilizan en las co- thunieaciones, como radares ¥ en la astronomia, Til fenomeno de la radiacion es complejo, de abs que su anélisis se haya pospuesto a este capitulo, No se intentara una amplia exposiciGn de la teorfa de antenas; limitaremos Stiestro estudio a los lipos basicos: dipolo hertciano, dipolo de media onda, monopole de Un cuarto de onda y antena de cuadro pequefia. Los campos de radiacion de cada tipo se determinaran siguiendo estos pasos: 1, Selig at irtown de coordanmier mfeoande y a deenxina i pete atcngne™ seers 2 SRST iar gem = ant = 0 A 3 Sedetermina Ba panirde Vx = 228 0 sin perdidas Go ~ 0) 4, Se daicula cl Campo temoto y se determina la potencia radiada promedio temporal mediante = nH X a, suponiendo un medio w= [mms donde Ppee = Eo, > HP) Conviene advertir que, en este capitulo, Prag Cquivale @ 1a Pyro de 1a ecuacién (10.70). el | — 7 IN| co (©) Refector de dios parsblico Figura 13.2, Antenas comunes. '. Dipolo hertciano Por dipolo hertciano se entiende un elemento de corriente infinitesimal 7 dl. Aun elemento de corriente no existe en la realidad, es esencial para calcular por integra el campo de ana antena practica, Considérese el dipolo hertciano que aparece en la figura 13.3. Supongamos qu ubica en el origen de un sistema de coordenadas y que porta una corriente wn (constante a todo Io largo del dipolo) 1 = J, cos wt. De acuerdo con la ccuacion (9.54), potencial magnético vectorial retardado debido al dipolo en el punto del campo P dado por amlnjar A amr Figura 13.3. Dipole hértciano portador Getcorriente J = 7, 008 wt. donde [JJ es la corriente retardada dada por = recone (¢—£) = fyc08 (wr ~ pr UI) = t.c0sa (6-2) = s,c08 (wr — pr) ee = Re [Teor donde (= iu = 2niAy w= 1/-Viae «Se dive que In corriente en el punto P es rerardada st cmusa de un rotardo de propagecion r/o Tetardo de fase fr de O a P. Al sustituir la Eatncion (13.2) en Ia Seuacion (13.1) ex posible exprotar Aven forma de fasor como pall elm Ay = ee a3.) La transformacién de este vector de coordenadas cartesianas en esféricas produce A, = Ar Aon Aes) donde An = Aq e080, Ay = —Aysen®, Ay =O a3.) Sin embargo, B, = nH, ~ V < A, asf,el campo Hse obtiene como Hew aa.say Hyp =O Hy 13.56) El campo Ese halla mediante V > H = ¢ 3/00 V > 1, ntat Ey = BS 3.64) Be, — Usd 3.66) Eu =0 s (43.6e) donde eee Jz FI detenido examen de las ecuaciones de campos (13.5) y (13.6) revela {a de términos que varian entre I/, Lir'y Ur El termnino Lr ce el campo ci ‘que correspond al campo de un dipole elgetrice [vease la ecuacton Gast domina a los dems en una region muy cercana al dipole hertclane. Ei ta campo inductive, predecible a partir de Ia ley de Biot-Savart [vease la eon Este término s619 es importante en un campo préximo, es decir a distanciag clemento de corriente. El termino i/r es el campo lejano 0 remot 6 cape paarto que cs el Unleo término que permanece an in Zona remotes So Heel Inuy algjado del clemento de corriente. Aqui nos ocupareinos primordial po lejano 0 zona de radiacion (Br = 1 0247 = A), donde low torminos em i Bog aprtoaran on Proge dalivaesttoas dp Ack ont co perme Injen Toft TEBE noe, Eu = otal Hy HH, ~ Ha = By = By, = 0 Cabe sefalar respecto de Ia eeuacién (13.74) que los términos de radiacién de se hallan en la misma fase temporal y son ortogonales, al igual que los campo ‘onda plana uniforme. Asimismo, que los campos de la zonas proxima y lejanae dicionados a ser las desigualdades fr << 1 y Br => 1, respectivamente. De mai pecitica, Ia frontera entre las zonas proxima y remota (o lejana) esta definida por de r, dado por a eat donde d es la mayor dimension de Ia antena, ‘La densidad de potencia promedio temporal se obtiene de esta forma: Prom = Ee (es 2H) = Be Ee Hw) ? 2 Ma? a, La sustitucin de la ecuacién (13.7) en la ecuacién (13.9) produce a su ver Ia pot radiada promedio temporal. Pet f nth fC aaee RnB deem ee [ sen? 0 do sen? 0 r? sen 0.0 db ‘aan? No obstante, [eemoae = [0 ~ eo acme y B? = 42/42. De ahf que la ecuacién (13.10) se convierta en Pau Bz [ at] caste Sil vacio ox el medio de propagacicn, 9 = 120% y Pea = 400 (2) 2 as. Esta potencia equivale a la potencia disipada por la corriente J = I, cos wt en una resis- tencia ficticia yagi es decir, Prag Fins Bena Poa a Rn 43.12) donde ig, ¢s €l valor de rafz media cuadratica [rms, root mean square] de I. De las ecua- ciones (13.11) y (13.12) se obtiene a3.3a) on ~ 802 [EF a 43.138) La resistencia R,q9, llamada resistencia de radiacién, es una propiedad caracteristica de la antena de dipold hhertciano. De las ecuaciones (13-12) y (13.13) se deduce la necesidad de antenas con gran resistencia de radiacién para emitir grandes montos de potencia al es- pacio. Si, por ejemplo, di! = 1/20, Raq ~ 2 O, bajo valor que indica una capacidad de emi- Sion de montos de poteneia relativamente reducidos. Cabe hacer notar que 1a Rag de la ecuacién (13.136) se refiere a un dipolo herteiano en el vacfo. En cl caso de un dipolo en, un medio distinto sin pérdidas, se sustituye 7 = Vile en la ecuaciGn (13.114) ¥ Ryya Se determina mediante la eeuacién (13.134). ‘Adviertase que se ha supuesto al dipolo hertciano como infinitesimalmente peque- fo (# di = 1 o dl = A/10). Asi, su resistencia de radiacion es muy reducida, de manera quan aa in: guiieiion an Gita anaphaciaanacenn tiene Gectiteendtiaiiin. sent. Maid ae supuesto una corriente uniforme en el dipolo, lo que implica que la carri tremos no es igual a cero, algo pricticamente imposible a causa de que el dante no es conductor. Sin embargo, nuestro anslisis demostrard ser una Valida y til de una antena con df = A/10, Una antena mas practica (y tal ve portanie de todas) es el dipolo de media onda, tema de la siguiente seceign. 13.3. Antena de dipolo de media onda Una Ifnea de transmisi6n (una Ifnea de dos alambres, por ejemplo). El campo di dipolo pusde obtenerse Tacilmente si se considera que consiste on Una cat Jos hertcianos. EI potencial magnético vectorial en P debido a una longitud di di (= dz) del dipolo portador de una corriente de fasor L, = 1, cos Bz &s Mo 608 BE dz 5. yy a arr’ Diecncign Figura 13.4. Dipoto de media wa REE ’ eaipeo —| © 13.3. ANTENA 0€ OWSiS BF MEIN GHER a 5BR Notese que para obtener Ia ecuacién (13.14) hemos supuesto una distribuci6n sinusoidal de corriente, puesto que la corriente debe tender # cero en 10s extremos del dipolo: aun que también seria posible una distribucion triangular de carriente (véase l problema 13.4), los resultados serfan menos exactos. La distribucion real de corriente en la antena no se conoce con precisidn; se determina resolviendo las ecuaciones de Maxwell suje~ tas a las condiciones en Ia frontera en la antena, procedimiento matematico complejo. Sin embargo, el supuesto de Ia corriente sinusoidal aproxima la distribueion obtenida mediante la resolucién del problema con valor en la frontera y es de uso frecuente en la Teorfa de antenas. ‘Sir 5% €, como se explicé en la seecién 4.9, dedicada al dipolo eléctrico (fig. 4.21), zed oY 2 cos Asf, puede sustituirse 1’ — r en el denominador de Ia ecuaci6n (13.14), donde es necesa- ta ia magnitud de la distancia. En cuanto al término de fase en el numerador de la mis- ma ecuacion, la diferencia entre fir y ir’ es significative, de manera que r’ se reemplaza por r — 2 cos 8, no por r. En otras palabras, el término del coseno se mantiene en él ex. Ponente y se ignora en ¢i denominador, pues el primero implica la constante de fase y el Segundo no. De este modo, Ay Bes [os con pe de = sas) Lhe, 4-10 ( gst2008® os = Bheeem [een con ete Con base en las tablas de integrates del apéndice AS, (acon be + bon be) [egononac’ Selecta Baan bn) ‘Al aplicar esta expresin a la ecuacién (13.15) s0 obtiene ise“ eieon (pcos 0 cos Bz + B Sen 62) |** —~ aar —# cos? 6 + B® Me oe Puesto que 6 = 27/A 0 BAIA = m/2 y —co¥? 0 + 1 = sen” 01a ecuacin (13.16) se con- A = pele (eterna + py — e-komenno — By} 3.7) serio Saltas od diate PES EG ce tee" cos (Ecos) eee 43.18) As Del empleo de la ecuacion (13.4) junto con el hecho de que B, — “Jwe,, los campos magnético y eléctrico en la zona Iejana ( LA yup) ae obtionee de seta forme: 7 (tame (Sone) ‘SP sen? O ro La potencia radiada promedio temporal puede determinarse de la manera we Zerece Ba atte a i Adviértase de nuevo que los términos de radiacién de Hy, y Ey, 60 encus i ma faze temporal y son ortogonales. | De Ia apiencian de las ceuactones (13.9) y (13.19), 1a densidad temporal se oblicne de este modo: ‘ 4 | onan = Fn Wal? ae a | se (Fou) a | eg Tee Ey ‘08 | eg ee 4 i donde se ha sustituido 7 = 1207 suponiendo el vacfo como medio de prop: Ja naturaleza del integrando de la ecuacin (13.21), pe Gen,. pea sone ie Esto podria ilustrarse facilmente con un diagrama elemental de Ia variacion do con 0. Por tanto, ia [7 cos? G. = @) ao i Pra Al cambiar variables, u = cos @, y el empleo de Ia fraccién parcial feduce Ia ecuacién G322) 0 Pras = 603 [ ao 323) [pea (Ee) El reemplazo de 1 + u por v en él primer integrand y de 1 — w por v en el segundo re- sulta en, ® sz oa ‘sen? Low. Pac = 3008 (972 ae ssf EHS ay ress ji puesto que cos w +. La integracién de Ja ecuacién mar tah ot Sar: (13.25) término por término y Ia evaluacién en el limite conducen a a[@m? _ @m* , @m* amt, Poa = 1518 Sey ~ Seay + Stety ~ 8 ] 43.26) 305672 La resistencia de ra facilmente de las ecua 6n Rag de Ia antena de dipole de media onda se obtiene nes (13.13) y (13.26), astt , Bem -73n 43.27) 4. Antena monopolar de un cuarto de onda Obsérvese el significative incremento de Ia resistencia de radiaci6n det dipolo. Sere ESS tiparaciGn con la del dipolo hertciano. En consecuencia, aquél puede Gkpacio mayores montos de potencia que éste. 7 ‘Le impedancia de entrada total Z-,, de la antena es la impedancia registrady terminales de la antena y esta daca por ’ Zoem = Rens + 5X donde Ruy — Rag en el caso de una antena sin pérdidas. La deducciGn del valor de Is {NGS ZZ implicaria un procedimiento muy complicado que rebasa el alcance d Tenet Baile Mlber que X,n, — 42.5 ©, de modo que Ziq = 73 + j42.5 2 cuando (ea Gel dipole es €— A/S"'La reactancia inductiva cae répidamente a cero al r Hgesamente cea longitud. Cuando € — 0.485 A, el dipolo es resonante, con Xau = 0 Herttica, asi, un dipolo 2/2 se disefia de tal forma que Xeq Se acerque a cero y Z- Fr cute valor de la resistencia de radiacion de la antena de dipolo A/2 explica la abl cabi coaxial extandar de 75 €, De igual manera, tal valor ¢s facil de acoplay Tee ae transmision. Junto con la propiedad de resonancia, estos factores son tn > extendido uso de la antena de dipolo. ‘La antena monopolar de un cuarto de onda consta bésicamente en ta mitad de una. sean Rbe de media onda situada en un plano conductor a tierra, como se ilustra Fae diel Scuntena cx perpendicular al plano, habitualmente supuesto como i Heute ononte conductor La alimenta un cable coaxial conectado a su base. Seem tS S con Ia teorfa de las imagenes expuesta en la seccién 6.6, es posible plazbr cl plane infinite perfeclamente conductor a tierra por Ia imagen del monopole, Pea een a monopolo A/a con su imagen en Ia repidn sobre el plano a tierra es igual ae eee do's un dipole V2. Ast, la ecuacién (1319) también es aplicable al eae ee ee taunts, la integracion ‘de 1a ecuacion (13.21) slo cubre la superficie Fo Ee aa ee i ence oe a= n/a), pucato-que ok monapola i ; aaeterea to a traves de esa supelficic. Esto quiere decir que s6lo irradia Ia mitad Paice el dipolo con igual corrionte. En el caso, asf, de una antena monopolar M Prag ~ 18.28 13 y 2P ma B Raa = Figura 13.5. Antena monopolar. 3 Rna = 3650 43.30) Por la misma razon, la impedancia de entrada total de un monopolo A/4 es Zoom = 36.5 4 P2125 O Antena de cuadro pequefio La antena de cuadro posee importancia practica. Se le usa como antena indicadora de direccién (0 cuadro de exploracién) en I deteccion por radiacion y como antena de te- levision para freevencias ultraaitas, El érmino pequeno implica que las dimensiones del cuadro (como p,) son mucho menores que A. ‘Considerese la pequena espira (o cuadro) filamentosa circular de radio p, portadora de una corriente uniforme /, cos «wf que se muestra en la figura 13.6. Esta espira podria ge ca fang ea ape oon ose saree oo Rene 6 ee ee oe ele ses ca aq faa ass donde [1] = 1, cos (wt — ir’) ‘obtiene A en forma de fasor: Re [Tel ~ Pr} Al gustituir [7] en la ecuacién (13.31) se antes 332) La evaluaciGn de esta integral supondria un largo procedimiento. Es posible demostrar que, en el caso de un cuadro pequefio (p, <= A), 7” puede ser reemplazada por ren el de- nominador de la ecuacién (13.32) y A, s6lo posee la componente $, dado por = GS + jprje“™ sen 3.33) Figura 13.6. Antena de cusdro pequefio. ira con N. javek,, de la donde $ = mp? = 4rea de la espira. En el caso de una es ‘A partir del hecho de que B, — #H, — V< A,y Vx H, se obtienen los campos eléctrico y magnético en esta forma: Bas IHL sono [2 Ht = ES coo [= Bs od sta = IES ono [1B + Lu = Bay = Hye = Al comparar las ecuaciones (13.5) y (13.6) con la ecuacién (13.34) es posible haturaleza dual del campo debido al dipolo eléctrico de la figura 13.3 y al di tico de In figura 13.6 (véase también la tabla 8.2). En el campo lejano (o remoto) manece el término (de radiacién) 1/r de la ecuacisn (13.34). Asi, en el campo mes de campo de la ccuacién (13.35) se han obtenido con referencia a una espira circular pe también cs posible emplearlas para cuadros pequefos con una vuelta (S~ a"), (S = Na?) 0 de cualquier otra configuracién en tanto sus dimensiones sean ret (d = 2/10, donde d ex Ia mayor dimension del cuadro). A manera de ejercicio, el podria demostrar que, mediante las ecuaciones (13.134) y (13.35), la resistencia de_ cidn de una antena de cuadro pequefio es Regg = 220-0 13.5, ANTENA DE CUADEO PEQUERO fl GOT! En un punto en @ = 4/2, a2 km de una antena en aire, xe precisa dé und intensidad de: campo magnético de 5 jA/m. Sin considerar las pérdidas ohmicas, ,cudnta potencia debe transmitir la antena si se trata de 4a) Un dipolo herteiano de 4/25 de longitud? ) Un dipolo de media onda? c) Un monopolo de un cuarto de onda? 4) Una antena de cuadro con 10 vueltas y radio p, = 4/207 Sotucién: 4) En un dipoto herteiano, Ler.) — fodtsen 0 i amr Qe A Qe donde di = 025 6 Bat = 2%. A. = 2%. por tanto, ae) aa (2 x 10°) ~ 10% 5x 10-¢ _ 407700.5)? 2 = SGsyF 2) Bn un dipolo 4/2, 20m mA Pyug = W212 Ryyg = U2@0m}? X 10-473) 144 mW ©) En un monopole 1/4, 200 mA como en el inciso b). S /212Rigg = U2(20m)? % 10-%(36.56) = 72 mw’ 4) En una antena de euadro, Uts,| = 72S sone En el caso de una vuelta, $ = 1p,2, En el de N vueltas, $ — Norp,2. Asi we 1o-t = Blot ea]? Sere 10" 2x10 LA BO ge aye — 2E seag 6 Jx10-* = 22 x10 = 40.53 mA 320 w4s* 6 y2[ 22)" Fat eg [&] be Oe ay = n20 9 x 100[4]' = 23.0 1 12 Rea — 4. (40.5392 = 10-8 Pont = 411 Rew ~ 800.535 92) = 158 mw Ejercicio 13.1, £ {Un dipole herteiana de longitu M100 se ubica on ol oxigen y 2 aliment cor ome econ BoC eceb eer meses eee a) r= 05,0 ~ 30° 1b) r= 200A, 0 = 60° Respucstar: «) 0.2119 sen (10% ~ 20.5") up mA/m y b) 0.2871 sen (0% ae : f Una intensidad de campo eléctrico de 10 :V/m se mediré en un punto de observacion 6 '#/2, 4 500 km de una antena de dipolo (resonante) de media onda que opera en sire & 50 MFIz. 4) {Cudles la longitud del dipolo? ) Calcule la corriente con la que debe ser alimentada la antena. ©) Halle la potencia promedio radiada por la antené @) Sia la antena se conecta una linea de transmision con Z, = 75 , determine la raz6n, de onda estacionaria. Sotucion: © _ 310° a) Ta longitud de onda A= $= 35 toe ~ om. De este modo, Ia longitud del medio dipolo es € b) A partir de la ecuacién (13.19), a $-3m iss ($e) ee ‘2ar sen @ 1£o.| 2mr sen @ na.cos (Feos a) _ 40 10° 2m (500 > 10") (1) 1200 1) = 83.33 ma, 2 ku BO Pot = BP, Regs = 63.339? 10-6 73 int = $F Rang = 3 (83.39) = 252.5 mw Zu * p= Ze Zo (z, — Zen este caso) 1734 jas = 75 2-2 + 12.8 TBH HAS TIS Tas + HDS 22.55 702.09" GES = 02703 18:87 _ A+ Ir) _ 1+ 02763 1— Ir} 102763 1.763 ts el sjemplo 152 sta antena de dipolo és reemplarada eae 5 Respuentan o) 1.5 mn 0) 84.93 mA 6) 1268 mW y a) 13.6. Caracteristicas de las antenas mena RINNE Sk I) OF GORENG OES FERIAB ARES Ginenosss) cioaily ats oct, se ¢ el Habiendo considerado los tipos elementales de antenas, examinemos ahora portantes caracterfsticas de una antena como radiador de energia clectromags tas caracterfsticas son: a) patron de antena, b) intensidad de radiacién, ¢) directiva y d) ganancia de potencia. A. Patrones de antena El diagrama de la amplitud de un componente especificada del campo Fes un ps campo 0 patron de voltaje; el del cuadrado de la amplitud de E, un patron depo Para evitar el trazado del diagrama tridimensional del patron de antena, se (sem separado el |E,| normalizado contra 0 con @ constante (patrén del plano Eo pat tical) y el |E,| normalizado contra ¢ con @ = 7/2 (patron del plano Ht 0 patron ho tal). La normalizacion de |E,| $e realiza respecto del valor maximo del JE}, de mod el valor maximo del [E,| normalizado es la unidad. En el caso del dipolo hertciano, por ejemplo, el |E,| normalizado se obtiene d ecuaci6n (13.7) como. KO) = \son a “ lo cual es independiente de . De Ia ecuaci6n (13.37) se obtiene el patrén del plano J diagrama polar de (0) en el que 0 varia de 0" a 180°,como se muestra en la figura L Notese que el diagrama es simétrico en torno al cje z (0 ~ 0). En cuanto al pal plano H,se fija @= 7/2 para que (@) — 1,10 cual equivale a un circulo de radio 1, Se ilustra en la figura 13.7(b). De la combinacién de los diagramas en las figuras ¥ (b) resulta el patron de campos tridimensional de la figura 13.7(c), en forma de d EI patron de potencia de la antena es un diagrama de la potcneia promedio t '® pron! — Pyroge CON FelaciGn a una distancia fija r. Esta vez se trazan por separadd cotta @ con $ constante ¥ Pam contra cond constante. Con referencia al dipold Rertciano, el patron de potencia normalizado se ob f4cilmente de la ecuacién (13.37) © (13.9), en esta forma: $20) = sen 0 to cual se representa graficamente en Ja figura 13.8. Obsérvese que en las figuras. y 13.8(b) aparecen cireulos, ya que f(@) es independiente de $, y que cl valor de OP él @ © © Figura 13.7. Patrones de campo del dipolo hertciano: (a) patron del plano Pnormalizade 0 vertical (f = constante — 0); db) patron del plano A normalizade {horizontal (6 = w/2); (e) patron teidimensional je potar we 2 ‘e @ cy Figur 13.8. Patron de potencia del dipolo hertciano: (a) ¢ = constante = 0; (@) 0 ~ constante = 7/2, figura 13.8(a) es 1a potencia relativa promedio respecto de ese @ particular, F 2 ]@ ~ 45°), ast, la potencia promedio equivale # In mitad de In potencia dio (a cual beurre en 0 = 77/2), B. Intensidad de radiacién La intensidad de radiacién de una antena se define como UO.P) =F Porm Con base en Ia ccuacién (13.39), 1a potencia radiada total promedio puede expres Pan $2, = 0.9) xen 0ao to sen odode as fo, = EL yeonan donde dO = sen 0 dé dds es el dngule sdlido diferencial, on estercorradianes (51). B que Ia intensidad de radiacion U(@, }) se mida en watts por estereorradian, valor promedio de U(6, 6) es Ia potencia radiada tot Toro = it C. Ganan: directiva Aparte de los patrones de antena anteriormente descritos, a menudo nos interesam tidades mensurables como la ganancia y la directividad para determinar las caract cas de radiacion de una antena. 7 directiva puede considerarse como la capacidad de una antena para potencia radiada en una Ia intensidad de radiacién en una direccién dada (8, ) a In intensidad de radiacis medio; es decir, Mediante Ia sustitucién de la ecuacién (13.39) en la ecuacién (13.42); Pom Pureds expre- sarse en términos de ganancia directiva como, Gu Prom @ gett Prd a3.43y La ganancia directiva G,(0, 6) depende del patron de antena. En el caso del dipolo hert- ciano (lo mismo que del dipolo 4/2 y el monopolo 4/4), en la figura 13.8 se advierte que ‘Poaom 8 Maxima en 8 = w/2y minima (de cero) en @ = Oo 7. Asf, el dipolo hertciano irra- dia potencia en una direcciGn transversal a su longitud. Respecto de una antena isorrdp!- a (aquella que irradia por igual en todas direceiones), G — 1. Sin embargo, esta antena no es real sino ideal. Tis D = Get = Gy max (3.440) Ae Unse p-* a3.4ap) D = 1en.una antena isotrépica; éste es el menor valor que D puede adoptar. En cuanto al dipolo hertciano, G00) =15sen?0, Dm 15 3.45) zn cuamto al aipolo 4/2, Gi0,0) = TAF)» dD = 1.64 3.46) donde 7 = 1207, Rug = 73. 2y 3.47 D. Ganancia de potencia Nuestra definicién de la ganancia directiva en la ccuacién (13.42) no tiene en cuenta Ia ‘piedide dhenina de polantie F, db Ik anthha, Py oo Gobo & eque.el wcedmnor dal que eatd hecha Ia antena es de conductividad finita. Como se ilustra en 1a fi potencia de entrada total a la antena, 8 donde J... es 1a corriente en las Pan = Pe + Pra ne eee 5 Maul? Re + Ra) terminales de entrada y R; la resistencia de ‘mica dela antena. En otras palabras, Pj., es Ia potencia aceptada por la ant minales durante el proceso de diferencia entre ambas es Pa la radiaciGn, y Pjaq 1a potencia radiada por ‘La ganancia de potencia G,(0, 6) de la antena se define como potencia disipada dentro de la antena, ar UO.) eto, 4) — AEE \La raz6n de la ganancia de potencia en cualquier direccién especificada’a la g reccional en esa direccion es la La Introduccion de la ecuacion eficiencia de radiacion 7, de la antena; esto es, Ge _ Peas © Ge Pen (413.48) resulta en ‘ = Pres Fea” Rea + RG En muchas antenas, n, se acerca a 100%, de manera que G,'~ G,. Directividad uelen expresarse en decibeles (4B). Asf, D (AB) = 10 10849 D G (AB) = 1010810 G Cabe mencionar en este punto que los patrones de radiacién de una antena, habitualmente en la region del campo lejano, concebida por lo general como exist ae inn = 2 Figuen 13.9. Relacin entre Pays Pe 13.6. Camacrenisnicas OF LAS ANTENAS lB 609, ydea payor dimensién de la antena. Por ejemplo, d = € en la antena de dipolo elée- rico y "3p, en la de cuadro pequeno. Demuestre que la ganancia directiva del diplolo hertciano es G0. 6) = 1.5 sen? 0 ¥ que Ia del dipolo de media onda es cost(Z cos 0 GAO.) = 1.64 st(Gone) ‘Soluctons De senerdo con Ia ecuacisn (13.42), 640, 6) - LO Jr@a a) Respecto del dipolo hertciano, Aw sen? (0) GLO.) = Tae eS a CS) [fr eaas 7 Steno como se solicit6. 2) Respecto del dipolo de media onda, 4m =Gr °) G0, 6) = sen Con base en Ja ecuacion (13.26), Ia integral del denominador da como resultado 21 (2.2188). Por tanto, 4 cos*(% cos 0) gent Bar LIBBY co*(Z cose) sen" G0.) = 1.64 como se solicit6. Kjerciéio 13.3 Caloule la dirsctividad del @) Monopolo herteiano. 4) Monopole de un cuarto de onda. a Respuestas: a) 3 yb) 3.28. Determine la intensidad de campo eléctrico a 10 km de una antena con gat Semple 13-4 | vade 5 dB y que irradia una potencia total de 20 kW. Sotucis: 5 = G, (AB) = 10 tog Gy 0.5 = logy Gy—> Gy = 10° = 3.162 A partir de la ecuaci6n (13.43), uP Prom = So Sin embargo, lz Peron = +5 Ast, __ 120m(3.162)(20 10°) | 2m (10 10°}? | = 0.1948 vim Ejercicio 13.4 i Cierta antena con tna eficien a) La de entrada es de 0.4 W. | ) La potencia radiada es de 0.3 W. Respuestas: a) 16.53 y b) 20.94. Z 13.6, CARACTERISTICAS DE LAS ANTENAS mE GTP ee ae | Determine su directiviaaa. : Solucién: directividad se define como ‘s u, oEke ey - D = Lex sue. | A partir de la U dada, : : mn ag [YAO Pal, Ey deft [Lawn e tte ena ad ad Ap [[semoao [seme ae “aE Fa ~cos20) do [™ (cos! 4) acon #) Sea Oe ee 3 13.7. Arreglos de antenas En muchas aplicaciones practicas (como una estacién radiodifusora de disefiar antonas con mayor potencia radiada en ciertas direcciones. qi equivale a demandar que cl patron de radiacion se coneentre en la Tal propésito es diffcil de lograr con un solo elemento de antena. Un permite obtener mayor directividad que la que puede ofrecer unas Es conveniente y practico, aunque no indispensable, que los elementos idénticos. Examinaremos primero el caso simple de un arreglo de dos el prolongar después nuestros resultados al caso general, mas complicade, de un WN elementos. Considérese una antena compuesta por dos dipolos hertcianos situados en ¢ lo largo del eje z pero orientados en paralelo al eje x, como se muestra en la fi Supongamos que el dipolo en (0, 0, d/2) porta corriente T,,— Ina y que el ©, 0, —d/2) porta corriente 1, = 1,0, donde « es la diferencia de fase entre corrientes. Al variar el espaciamiento d y la diferencia de fase a, puede lo los campos procedentes del arreglo interfieran constructivamente (se sumen) direceiones de interés ¢ interfieran destructivamente (se cancelen) en otras d El campo eléctrico total en el punto F es la suma vectorial de los campos debi ‘elementos individuales. Si P se ubica en Ia zona del campo lejano, el campo. total en P se obtiene de la ecuacin (13.7a) de este modo: E,-E,, +E, = BLA heel OCS ee Bl (oo, Pca tom] Notese que sen 8, presente en la ecuacion (13.7a), ha sido reemplazado en este cos @, ya que el elemento ilustrado en la figura 13.3 sigue la direecién de z, mientras Jos de la figura 13.10 siguen Id dé x. Puesto que P se encuentra lejos del arreglo, 0; ~ @ = 0; ¥ %,— a) — a), En la amplitud puede fijarse r, — r ~ ra, pero en la fase se emplea, a nor Loose 13.540) oo nor+ Se0s8 3.546) En consecuencia, la ecuaci6n (13.53) se convierte en = LBL ay 9 gr hi giant + -favon O26, 3.55) ao : IB a cog geen con Pa cue patacion Go ath eenice con pio ooegnaa eae yeaceeeans enna Bea aac cepe eg ca an ee eee eee (ps con0 + 0) Jy ee as aase En general, asf, el campo lejano debido a un arreglo de dos elementos esté dado por E (total) ~ (& debido al elemento en el origen) x (factor de arreglo) (13.57) De la ecuacién (13.55) se desprende asimismo que |cos 6] es el patron de radiacion debido aun solo elemento, mientras que el factor de arreglo normalizado, cod 1/2(Bd cos 0+ a]. es cl patron de radiacion del arreglo si los elementos fueran isotropicos: Tales patrones pu~ den considerarse respectivamente como un “‘patrén unitario” y un “patron de grupo”. Asi, el “patron resultante” es el producto del patron unitario y el patron de grupo; es decir, Patron resultante — Patra unitario X Patron de grupo a3.s8) Esto se conoce como multiplicacion de patrones. El patrén de un arreglo puede diagra- marse por multiplicacion de patrones casi camo si se hiciera por inspeceiGn, de modo que aquélla es un instrumento muy util para el diseno de un arreglo. Cabe senalar que, a di- ferencia del patrén unitario, el patron de grupo es independiente del tipo de elementos que componen el arreglo, siempre que no ocurran cambios en el espaciamiento d,la di- ferencia de fase a y la orientacion de los elementos. Prolonguemos ahora los resultados del arreglo de dos elementos al caso general de un arreglo de V elementos, el cual se muestra en la figura 13.11, Supongamos que el arreglo 8 Jineal en cuanto que los elementos estan igualmente espaciados en una linea recta y se tienden a lo largo del eje z. Supongamos asimismo que el arreglo es uniforme, de manera que cada élemento es alimentado con corriente de igual magnitud, aunque de cambio de fase a progresivo, es decir Ty, = 1,20, Izy = Lele. Ls, = FoZ2a. y ast sucesivamente. temesn: Secatinnetans-te-sncaueteeninen taptenes aalk Aaeesbied~ tne, henainadiie. ait: eaeaiesiies tameaeinc nile atin Figura. 13.11. Arteglo lineal snifor facilmente a partir de la ecuaci6n(13.57) una vex conocido el factor de renela al arregio lineal uniforme, el factor de arregio es 1a suma de las ea todos los elementos. Ast, : FA S14 8 Pete ee 4 et be donde Y= Bd cose +a Tn ta ccuacién (13.60), 8 — 2n/A, mientras que dy « on el espaciamientery to de fase entre los elementos, respectivamente. Adviértase que el micinbra Ia ccuacion (13.59) es una serie geométrica de la forma, lteter te tat Asi, Ia ecuacién (13.59) se convierte en Io cual puede, expresarse como ey a1 isin Sem ON WI2) = Sen (W2) an El factor de fase ef ~ 9¥2 Ho estarfa present si el Atreglo estuviera centr: del origen. Ignorando este término carente de importancia, Fam Notese que esta ecuncién se reduce a la ecuacién (13.56) cuando N = 2, como es de es perar. Reparese de igual forma en lo siguiente: 1. FA posee el valor maximo de N; asf, el FA normall entre N. El maximo principal ocurre cu: ado se obtiene dividiendo FA sda = Oreste es, O=pdcosa+e 0 cose= —% 3.65) 2. FA tiene nulos (0 ceros) cuando FA — 0; es decir, Sete 266) compe eco can aaubes aol ea easton Etec te crensrmreet eguejune’carecctoa:nocmaat al cfs a ee ee aa ech Oe cere ean dooce dat ees iateandy 0x6 [5 egemere ae == [ Fstas observaciones son stiles para la representacién gréfica de FA. En la figura 13.12 se presentan los diagramas de FA correspondientes aN = 2,3 y 4, (Con referencia al arreglo directivo de antenas de dos elementos que se presents en Ia fi- ura 13.10, trace el patron de campo normalizade cuando las corrientes son alimentadas: @) En la misma fase (a = 0),d = 0/2 b). 907 fuera de fase (@ = #/2),d = A/4 El campo normalizado del arreglo se obtiene de las ecuaciones (13.55) a (13.57) como, 110) = |>0s0con [3 (ad coro + «|| 2m a @) Sia =0,d = 02, Bd — 2A — we, Por tanto, f@) - loos al Jcos & cos 0) + 4 a Patron resultante | =" patrénunitario > —_patrén de grupo El diagrama del patrén unitario es sencillo, pues, como se advierte en la figura 13.13(a), se . me Figura 13-12. Factor de are a lineal uniforme. ° = at Nar ° a ae me @)v=3 . jones reduce a una version invertida del patron de la figura 13.7(a), relative al dipolo Para trazar un patron de grupo, antes es preciso determinar sus nulos ¥ Saunte ateeinnioe (es cere cos (Zeos@) = 0+ Zeose = 6 = 0", 180° En cuanto a los maximos, cos (Home) aa-rersem.a —" 13.7. Anmectos OF ANTENAS mH 617" Figura 13.13. Para el inciso @) del ojemplo 13.6; patrones de campo Enel plano que contiene los ejes de los elementos, Th path de grape ne uet on ta figice13A12(5). 80 ps igre polar obtah pact srecode fou (Zeus) | cond = 0736 0°.A57,. - 46073 nla nconporacon de 10s nulos y miximos en @ = 0°, 180° y @ — 90°, respectivamente. La multiplicacion de Ia figura 13.13(4) por Ia figura 13.13(D) da como resultado el patron de la figura 13.13(c). Gabe destacar que los patrones de campo de Ia figura 13,13 se ubican en el plano que con- fiene los ejes de los elementos, Notese que: 1. en el plano yz, el cual es normal a los ejes Ue tos elementos, el patron unitario (= 1) es un efreulo (véase la figura 13.7(b)), mientras que cl patron de grupo permanece como en la figura 13.13(b); por consiguiente, en este aso el patron resultante es igual al de grupo, 2. En el plano xy, @ = 7/2, de tal forma que El patron unitario tiende a cero mientras que el patrén de grupo (— 1) es un cfreulo. b) Sia=mnia,d— Alay pd —~ 22 A so) - leos 01 con Z (eos 8+ 1) 4 = 4 patron resultante patron unitarlo patron de grupo {El patron unitario se mantiene como en Ia figura 13.13(a). Bn cuanto al patron de grupo, el nulo ocurre cuando cos (1 + €08 0) = 0-5 (1 + cos 0) = ae oes os et yn can cas ee = Sleaze sen] = osm eant a's onnyae 0-4 6 = 0°, 180" Elemplo 13.7 eb @ ) oo Figura 13.14. Para el inciso 4) del ejemplo 13.6; patrones de campo en el 1s contione Tos sjes de los elementos. by n sen Z(1 + cos@) = 0 cos@= -1 0 0 Cada patron de campo Se obtiene variando @ = 0°, 5°, 10°, 15%,. . 180%, 0 = 180" corresponde al maximo Valor de FA, mientras que 6 = O° correspa ‘Aci, on Ia figura 13.14 #6 presentan los patrons uniturio, de grupo y fesul ho que contiene ios ejes' de los elementos. Obsérvese on los patrones de arreglo transversal (a ~ 0) de Ia figura 13.13 es bidireccional, en tanto que cl al gitudinal (a — a) de la figura 13.14 es unidireccional. Ejercicio 13.6 Repita el ejemplo 13.6 con relacion a los casos siguientes: @) @ =, d = 02, b) a= =n, d = NA. Respuesta: Véase Ia figura 13.15. Considere un arreglo de tres elementos con raz6n de corriente 1:2: 1, coma se i figura 13.16(a). Trace el patron de grupo en el plano que contiene los ejes de los el Solmeién: Para efectos analiticos, dividamos el elemento intermedio de la figura 13.16(a). de corriente 2170", en dos elementos, portador cada uno de ellos de corrient Esto resulta en cuatro elementos en lugar de tres, como se muestra en Ia figura I Si se consideran los elementos 1 y 2 como un grupo y los clementos 3 y 4 com Brupo, se obtiene el arreglo de dos elementos de la figura 13.10(c). Cada grupo & vez.un arreplo le dos elementos con d= A/2, a= 0, de manera que el patron de § del arreglo de dos elementos (0 el patron unitario del arreglo de tres clementos) e8 © Figura 13.15. Para el ejercicio 13.6. Figura 13.16. Para el ejemplo 13.7: (@) arreglo de tres sera wont ramus de Corricnte 4e2r45 @) ¥ we ona uo fea 0 ST Gy anregion Squiventes de tee somone ©, : ©, : 4 lar al que se presents en la figura 13.13(b). Ambos grupos forman por su p glo de dos elementos semejante al descrito en el inciso @) del ejemplo 13.6.4 @ ~ 0, de modo que también en este caso el patron de grupo es isual al a 13.13(b). En esta situaci6n, el patron de esta filtima figura es representative t trén unitario como del de grupo. Fl patron de grupo resultante se muestra 13.17(c). Este patron no es el patrén resultante, sino el-patron de grupo del cicmentos. Fl patron de grupo resultante del arreglo seria la figura 13.17(c), Por el patrén de campo del tipo de elementos. 7 ‘Un método opcional para determinar el patron de grupo resultante del tres elementos ilustrado en la figura 13.16 consiste en seguir pasos similares adoptaron para obtener la ecuacin (13.59). El factor de arreglo normali tron de grupo) es entonces Casa om tom amo) era, = fom (Zooe)| fou (Zomal ‘ i + wait pains pag de resultante de grupo Los diagramas de estos patrones son idénticos a los de la figura 13.17. Si dos arreplos de tres elementos, como el arreglo ilustrado en la figura 13.16(a), espaciamiento de 2/2, de ello resulta un arregio de cuatro elementos con razén de 1:3:3:1 como el que aparece en Ia figura 13.18. Dos arreglos de cuatro elementos Figura 13.17. Para el ejemplo 13: Rota y See eh obtencion del patron de propo. Fesultante del arreglo de tres elementos de la figura 13.16(@). Patron wnitario Patron de grupo Patrgn de grupo a So Teauftade ‘ 13.8, Area HFCTIVA YA ECUACION OE FAIS OI. 629 espaciamiento de A/2 resultarfan a su vez en un arreglo de cinco elementos con razén de corriente 1:4:6:4:1. La continuacion de este proceso deriva en un arreglo de N elementos, con un espaciamiento de 1/2 y longitud (N ~ 1)A/2, euya razon de carriente son los coc, Heientes binomiales. Un arreglo de esta clase recibe el nombre de arreglo binomial lineal, 10 aria sept 140 Thauen 13-18. Arregio, de cuatro na $e vn na TSB pare th Gfercicio 15.7 Bjercicio 13.7 i ib 4) Trace el patron de grupo resultante del arreglo de cuatro clementos con razén de cortiente 1:3:3:1 que aparece on Ia figura 1318, 6) Deduzea una expresion para el patrén de grupo de un arreglo binomial lineal de WN elementos. Suponga que éstos se ubican a lo largo del eje 2, estan orientados en Paralelo al eje xy tienen un espaciamicnto d y un corriemiento de fase entre ellos a Biecnaie os Veceauy ania a aortas ee de efectiva y la ecuacién de Friis ‘Cuando la onda electromagnética de entrada es normal a Ia superficie entera de una an- tena receptora, la potencia recibida es J ma 8 = en 5 sen En Ia mayorfa de los casos, sin embargo, la onda electromagnética de entrada no es nor- mani’s!Ieuperticie cuteredd Id austenn, fe cual Vuslve ilecewaries ls idem del dren efectiva de una antena receptors, El concepto de frea efectiva o abertura efectiva (seccién transversal receptora de una antena) es de uso comin en el anal Figura 13.19. Para el inciso a) del ejersicio 13.7. Esto-es, se eee Con referencia a la ecuacién (13.68), cabe hacer notar que ¢l area efectiv de la capacidad de la antena para exiraer energia de una onda electromage ‘Deduzcamos la formula para caleular el area efectiva del dipolo herte dad de antena receptors, El cireuito de Thevenin equivalente a la ant aparece en la figura 13.20, donde V,, ¢s el voltaje en cireuito abierto induel minales de la antena, Zq — Kya +) Xqq 1a impedancia de la antena y Z,. Ia impedancia externa dela carga, Ia cual podria ser la impedaneia de entr de transmision que alimenta a la antena, Para una maxima transferenci Z,, = Zim YX, = Xam. La potencia promedio temporal transmitida Ga ex cntonces 2p Ival 7 P= 3 [2] Bm edVeal?, Rens Enlel caso del dipolo herteiano; Riad = 802°C)? ¥ Vi. Fut, donde £ es la efectiva de campo paralela al eje del dipolo. En consectiencia, Ia ecuaciGn (13.6 Vierte en ex P= a0? La potencia promedio temporal en. la:antena es BB Porom — 35, ~ 2400 La inserci6n de las ecuaciones (13.70) y (13.71) ena eenacian (13 68) resulta, 3a? oe as pk ar Figura 13.20. Circuito de Thévenin equ a una antena receptora. ; 13.8. Amita ErECTIVA V 1A FCUACION DERI MY 623) donde Ds 1.5 es la'direetividad del dipole hertciano: Aunque la eetiacion (13.72) 4e de Five con relacion al dipolo hertciano, es aplicable a cualquier antena si D se reemplaza por G,(0, #). Fn general, ast, 43.73) Supongamos ahora dos antefias en el vacfo separadas por una distancia r, como se muestra en la figura 13.21, La antena transmisora tiene area efectiva A,, y ganancia di rectiva G,,y transmite una potencia total P, (~ P,,.)- La antena reesptora tiene area efec- tiva “1. y Banancia directiva Gg, y recibe una potencia total P,. En la antena transmisora, sant _ 407 prom agate = , Poon ™ Ges Cat 13.74) Al aplicar las ecuaciones (13.68) y (13.73) se obtiene la potencia recibida promedio tem- poral, en esta forma: ~ Pr Perom Aer = 2 Gar Prom 43.75) La sustituci6n de Ia ecuacién (13.74) en la ecuacin (13:75) resulta en De meeste te, : es i ee ape motaps pon Ja paris mene 2.0 inh Se on Soo Be ee cairiee eer seas Samoans ge Anoeeee ee FCS Figure 1421. Antcnas transmisora y receptors en'et vacio. (efanipOTATB | Halle cl Area cfectiva maxima de un dipolo 1/2 de alambre que opera a: ta potencia recibe el dipolo de una onda incidente plana con intensidad, a Solucién: a a= Sexe. 4) 3. 10° ADF 730% 108 — 10" G0.) ~ 7200) = POE 30) = 1.04740) GAP, b) mie = 1.64 ante — S104) asl zs 2n A, 13.05 = 71.62 nW Una antena transmisora y otra receptora separadas 200 A entre sf registran una g directiva de 25 y 18 dB, respectivamente. Si la potencia recibida debe ser de 5 mW, Je la potencia minima transmitida, 3 Solucién: Puesto que Gy (dB) = 25 dB = 10 loge Gun Gy = 105 = 316.23 De igual manera, Gu (dB) =18ab 0 = 1018 = 63.1 13.9, BCUACICRES CRRA MB (62S Al aplicar Ia ecuacién de Friis se obtiene ren eala[elm os amr a Terex \-3 [ 47 =< 200 A? a sx 10s [S22 SIE = 1583w Ecuaciones del radar Los radares son dispositivos electromagnéticos tiles en la deteccién y localizacién de ‘objetos. El término radar es el acronimo de la expresion radio detection and ranging Cdeteccion y ubieacién por radio"). En un sistema de radar comin como el que apare- ce en la figura 13.22(a) se transmiten impulsos de energia electromagnética a tun objeto distante. Una misma antena cumple las funciones de transmisién y recepciGn, de modo que cl intervalo temporal entre los impulsos transmitido y reflejado permite determinar Ta distancia en la que se encuentra el objetivo. Sir es la distancia entre el radar y el obje~ tivo y c Ia velocidad de la luz, cl tiempo transcurrido entre los impulsos transmitido y re- cibido es 2v/e, De Ia medicion de ese periodo se deduce 7 Figiaaiet gyaienes Marae | ceecne oeet ee pe ep eS ena ates eeu La capacidad de un objetivo para dispersar (o reflejar) energfa se seccién transversal de dispersion «(0 seccion transversal de radar), Ia Unidades de area y puede medirse experimentalmente. = lan arr? donde &, es la densidad de potencia incidente en el objetivo Ty ®, la densidad de cia dispersada en el transceptor O, como se muestra en la figura 13.22(b).. ‘Alpartir de la ecuacion (13.43), In densidad de potencia incidente 2, en vo Tes Gu PR Bie gare La potencia recibida en el transceptor O es PAL ®, P, w= Fe Debe senalarse que 9, y ®, denotan la densidad de potencia promedio wattsim?, mientras que Pjqq ¥ P, se refieren a Ia potencia promedio temporal watts, Puesto que Gy, = Gy = Gy Any = A, y (13.79) en Ia ecuacion (13.77) resulta en. lo 300 “abla 13.1. Designaciones de UHE '300-1.000 Mi cL 11000-2000 Miz e ‘$000:8000 Ms x ‘8000-12500 Matix © ‘ie265 Gn De acuerdo con Ia ecuacién (13.73), A, = MG,/4z. Por tanto, Ia ecuacién de transmision del radar en el vacio, basica para medir la secci6n transversal de dispersion de un objetivo. Al despejar r en la ecuacin (13.81) se obtiene M2 Gio Pas as (43.82) la ecuacién de distancia del radar. Dada la potencia minima detectable del receptor, esta ‘ecuacion determina la distancia o el aleance méximo de un radar, aunque también per~ mite obtener informacién de ulilidad en ingenierfa sobre los efectos de los diversos Ppardmetros en el rendimiento de un sistema de radar. El radar considerado hasta aqui es del tipo monostdtico, en raz6n del predominio de este tino en las aplicaciones practicas. En un radar bisidiico, el transimisor y el receptor estan separados. Si las antenas transmisora y receptora se hallan a distancias r, y 7, del objetivo y Gy, * Gy, en el caso del radar bistatico la ecuacién (13.81) se convierte en ‘Las frecuencias de transmision por radar van de los 25 a los 70000 MHz. En la tabla 13.1 se clasifican tales frecuencias y Se indica la denominacién con que las conocen los in~ genieros de radares. P, ‘Un radar de banda S que transmite a 3 GHz irradia 200 kW. Determine la densidad de potencia de la sefial a distancias de 100 y 400 millas nauticas si el drea efectiva de la Bntena del radar es de 9 m?, Considerando un objetivo de 20 m? 300 millas nauticas, Calcule la potencia de la sefial reflejada en el radar. Solucion: La milla néutica es una unidad comin en las comunicaciones por rds 1 milla néutica (mn) = 1852 m <_ 310° a Seeger an 4m Gu =F Ag = Bia 9 = 36000 En el caso de r= 100 mn = 1.852 10° m = GaPrus _ 36007 X 200 X10" amr “ae (L852) X 10 = 5.248 mW/mn? e En el caso de r = 400 mn (1.852% 10°) m ous Mediante Ia ecuncién (13.806), donde r = 300 mn = 5.556 = 108m 2 x 20 x 36007 > 200 > 10 es [ae 5.5367]? x 10° Mediante la ecuacién (13.81) se obtendria el mismo resultado. ame e En este capitulo se expusieron las ideas y definiciones basicas de Ia teoria de lantenas. Los tipos basicos descritos fueron el dipolo hertciano (0 de longitud diferencial), dipalo de media onda, monopolo de un cuarte de onda y antena de cuadre pequeno. Si se conove Ia distribucion de corriente de una antena, en teorfa es posible hallar el potencial magnético vectorial retardado A,con el que a su vez pueden determinarse los campos clectromagnéticos retardados Hy E mediante A H=vx4, B=nHxa 7 n Los campos de la zona lejana se obtienen manteniendo tnicamente los térmi nos Vr, Fl andlisis det dipolo hertciano es fundamental para el de otras antenas. La re- ducida resistencia de radiacién de este dipolo limita su utilidad practica, El dipolo de media onda es de longitud igual a A/2. De uso mds practico y fre- cuente que el dipolo hertciano, su impedancia de entrada es de 73 + j42.5 0. Elmonopolo de un cuarto de onda es Ia mitad de un dipolo de media onda sobre un plano conductor. [Los patrones de radiacién de uso mas comtin son los de intensidad de campo, in- rensidad de potencia ¢ intensidad de radiacion. El patron de campo es el diagra- ma de |E,| osu forma normalizada /(@). El patron de potencia es el diagrama de Perom 0 SU forma normalizada f2(@)- L&'ganancia directiva es la razon de U(@, ¢b) 2 su valor promedio. La directividad es el maximo valor de la ganancia directiva, Un arregio de antenas es un grupo de elementos de radiaci6n dispuestos para producir caracteristicas particulares de radiacion. Su patron de radiacion se ob- tiene multiplicando el patrén unilario (debido a un elemento del grupo) por el patron de grupo, el diagrama del factor de arreglo normalizado. En el caso de un arregio lineal uniforme de N elementos, sen (NY/2) ‘sen (W/2) FA= donde y = fad cos @ +e, 6 = 2m/A, d — espaciamiento entre los elementos y @ = corrimiento de fase entre los elementos. La formula de transmision de Friis caracteriza el acoplamiento entre dos antenas en términos de su ganancia directiva, distancia de separacion y frecuencia de operacion. En un radar bistatico (con anten: cia recibida esta dada por transmisora y receptora separadas), la poten- Tora En un radar monostético, 7, = 72 = ry Ga = Gar 124. Una antena ubicada en clarta cludad'es fuente de andas de radio. 4 stax en llegar a una poblacion a 12000 km de exa ciudad? @) 368 2) 204s ©) 20ms a) 40. ms @) Ningsmo de loa tempos anteriores 413.2, ,Cusl es el término de radiacidn en In ecuacién (13:34)? 4) Eltérmino wr D) Bl término U2 6) Eltermino 1/5 4) Todos tos anteriores 433. Un muy pequefo alambre delgado de longitud 4/100 tiene una resistencia de @ 09 ») 008.0 2792 a) 790.2 4134. Una antena monopolar de un cuarto de onda que opera en aire a uns. debe tener una longitud total de a) >A ») 300m ©) 150m a) 73m EA 435. Siuna antena de cuadro pequeno de una vuelta tiene una resistencia de 1 Goudntas vueltas se necesitan para producir una resistencia de radiacién de @) 150 ») 125 ©) 50 4) 28 os 13.6, 137. 13.9. Una antena receptora se sits mera es de 500 cm*y la densidad de potencia en la localidad receptora es de 2 mW/m?, a Po- tencia reeibida total ex de: A una distancia de & km de una antena diferencial, la intensidad de campo es de 12 4V/m. ‘La intensidad de campo en una localidad a 20 km de la antena es de @) 75 aim 2) 30 nVim ©) 48 uVim a) 1.92 wVim ‘Siuna antena thene Une es de OWir, Up = 4:5 Wist ¥ 7, = 95%, su potencia de entrada @ 222W » aw ©) 35.35 w a) 5952W ‘Una antena receptora ubicada en un aeropuerto tiene tnia dimension méxima de 3m y ope- ra'a 100 MHz. Un avin que, en direcei6n al aeropuerto, se halla a 1/2 km de la antena so ‘ncuentra en la region del eampo lejano de sta @) Cierto. 2) also. 100 m de Ia antena transmitora. Si el 4rea efectiva de la pri- @) 10nw 2) 100 aw: o1ew ® 10nw ©) 100 nw 13.10. Sea Rel alcance maximo de un radar monostatico. Si, dada una secciGn transversal del ra- dar de 5 m?, en K/2 se encuentra un objetivo, jcudl deberfa ser Ia xeccin transversal de Sste on 32/2 para resultar en una intonsidad de sefal igual a la del radar? 4) 0.0617 m? b) 0.885 m? ©) 15 me a) 45 me () 405 mi? Respuestas: 13.1d,13.2a, 13.3, 13.4d, 13.5¢, 13.60, 13.7d, 13.84, 13.9¢, 13.106. 33.1. El potencial magnético vectorial en el punto P(r; 0,6) debido a una antena 132. Un dipole hertciano en el vacto situado en el origen tiene d€ = 20 em es = 10) 133. Una fuente de 2.A que opera a 300 MHz alimenta a un dipolo hertciano de 5 134, 138, 13.6. da en el origen ext dado por 50cm a, = 2 donde 7 — 23 + 92 + 23, Halle E(r, 8,6, y M(r,8, 6,1) en el campo lejnno, Halle [Z,| en el punto distante (100, 0,0). tud situado en el orizen. Halle , y Hi, en (10, 30", 90°), 2) Ep ver de la distribucion contante de enrriante sip 1 ot apatap ce 282bapoage ona crcl wieguter eo loam 4 (A tet on gun 1323. Demme ge : nacme [sf Jo que equivale a la cuarta parte de In ecuncién (13.13). Ast, Rug depende de idm de corriente, 8) Calcule Ia longitud del dipole resultante de una resistencia de radiaciGn de 0.5. Una antena puede disenarse como un dipolo eléetrico de 5m de largo a 3 MHz, sistencia de radiacidn suponiendo una corriente uniforme en su longitud. Si un dipolo de media onda es alimentado por una Iinea de transmision de 50 Q, coeficiente de reflexion ¥ la razon de onda estacionai ‘Una antena de radio de automévil de 1 m de largo opera en Ia frecuencia de MHz: ,Cuanta corriente se necesita para transmitir 4 W de potencia? Figura 1323. Pequena antena de dipoto con distribucién. triangular de corriente: para el problema 13.4. was. 139, waaa0. waa. 4) Demuestre que las expresiones de campo lejano relativas a un dipélo'delgado de longi tnd € portador de cortiente sinusoidal /, cos Sz son He amr . wos (Scns) cont (Pista: Remitase a In figura 13.4 y a ta ecdiaci6n (13.14) +b) En una hoja de coordenadas polares, trace f(0) del inciso a) respecte de € = A,3A/2y 2A. Con referencia al problema 13.4, 4) Determine E, y Hen el campo Iejano, b) Calcule Ia directividad del dipolo. ‘Una antena situada on Ia superficie de un terreno plano transmite una potencia promedio de 200 kW. Suponiendo que la totalidad de la potencia es emitida de manera uniforme Sobre la superficie de tin hemisferio con Ia antens en el centro, ealeule @) el vector de Poynting promedio temporal 50 km &) el campo eléctrico maximo en ese sitio. Una antena de cuadro de 100 vueltas y 20 em de radio que opera a 10 MHz en el aire de- bbe rendir una intensidad de campo de 50 mV/m a una distancia de 3m del cuadro. De- termine 4a) La corriente con la que debe ser alimentada ta anteha. 1b) La potencia promedio radiada por la antena, ‘Trace los patrones de los campos F y H normalizados para 42) Un dipolo de media onda. b) Un monopole de un euarto de onda. A partir del resultado del problema 13.8, trace los patrones de campo verticales de antenas inonopolares de longitudes € — 34/2, A, SA/S, Cabe indicar que el monopolo de SAS es de uso muy comdn. El campo de zona lejana de una antena en el vacfo esta dado por wp, = 588228 ema, vin donde = «Vjigéy. Determine Ia potencia radiada. El campo eléctrico producido por una antena en el campo lejano es g, = 0 cm cosa cos ba, ‘Trace el patrén vertical de In antena, Su diagrams debe incluirla mayor eantidad posible de — 413.16. Con referencia a un dipolo hertciano, demuestre que la densidad de poteneia poral se relaciona con la potencia de radiacion de ncucrda con 13.17. En el campo lejano, una antena produce = 2sen 8.608 Igo pees, eet a wim, O<0<7,0< . Calcule la potencia maxima recibida por la antena. Adopte 1-64 como disecti dipolo. La antena de un satétite sincrénico en Srbita transmite una potencia de 320 W: Si te ganancia de 40 dB a 15 GHz, calcule la polencia recibida por otra antena con Una gs de 32.dB a una distancia do 34567 km. [La ganancia directiva de una antena es de 34 dB. Si emite 7.5 kW a una distancia de halle 1a densidad de potencia promedio temporal a exa distancia Dos antenas idénticas en una eémara rorda estn separadas 12 m y orientadas en funciG tuna maxima ganancia direecional. A una frecuencia de 5 GHz, la potencia recibida por 5.30 dB inferior a 1a transmitida por Ia otra. Calcule la ganancia de ambas en a {Cust os ta maxima potencia que puede recibirse a una distancia de 1.5 km en el vacto Un circuito a 1.5 GHz compuesto por una sntena transmisora con ganancia de 25 diy t lntena receptora con ganancia de'30 dB? La potenela transmitida ex de 200 W. 13.39. Un radar de impulsos de\banda Lcon una antena transmisora y receptora comin y gan cia directiva de 3500 opera a 1500 MHz'y transmite 200 KW. Si el objeto se encuentra kkm del radar y su seccion transversal de dispersin ox de 8m, halle 2) La magnitud de Ia intensidad de campo eléctrico ineidente del objeto: 1b) La magnitud de la intensidad de eampo eléctrico dispersada en el radar, ©) El monto de potencia absorbido por el objeto. 4) La potencia que la antena absorbe de la onda dispersa. 3.40. Una antena transmisora con une frecuencia portadora de 600 MI Iz produce 80 W de tencia. Halle la potencia recibida por otra antena eh el vacfo a una distancia de T kim. pPonga que la ganancia de potencia de ambas antenas es igual a la unidad. 13.41, Un radar monostético que opera a 6 GHz rastrea un objetivo de 0.8 m? 4 una distancia 250 m. Si la gananeia es de 40 dB, calcule Ia polencia minima transmitida para producit potencia de retorno de 2 pW. PrRomLonns. 5S SS 13.42. Las antenas del sistema de radar bistatico que aparece en Ia figura 13.26 se encuentran fnras de tierra y catan scparadas 4 km entre af micntrar que el abjetivo, de 2-4 mine haifa a una altura de 3 kim El sistoma opera a5 Glir. Con Gy de 36 dB ¥ Gy de 20.00, ‘determine la poteneia radisda minima necosaria para obtener tna potemcia de Fetorne Se soe to Ww 14 Temas actuales EI futuro tiene varios nombres: para los débiles es lo imposible, para los creyentes je desconocido y para los osados y sagaces lo ideal. 14.1, Introduccion Aparte de la propagacién de ondas, Iineas de transmisién, gufas de ondas y ante} ten otras reas de aplicacisn del electromagnetismo. Entre ellas estan las terferencia y compatibilidad clectromagnéticas, la fibra Gptica, las comuni satelitales, el bioelectromagnetismo, la maquinaria eléctrica, la meteorologia por circuitos aprendidos en cursos anteriores y de conceptos de clectromagnetisme a dos en los capitulos precedentes. 14.2. Microondas Hinata ta facha axisten tres madios para ol transports de miles de canales a largas diet cias: a) enlaces de microondas, b) cables coaxiales y ) fibra Sptica, tecnologia rel mente nueva de la que trataremos mas adelante. Para efectos comparativos, Ia seflal de una estacién de radio de AM es de alreded 1 MHz, mientras que la de una estacion de FM es de aproximadamente 100 MHz. El su comportamiento sea mas semejante al de los rayos de luz que al de las ondas de Ordinarias. Probablemente el lector ya conoce aparatos de microondas como el hori microondas, el cual opera a 2.4 GHz; la television por via satélite, que opera a alre de4 GHz, y el radar de vigilancia policiaca, el cual funciona a aproximadamente 22. G Entre las caracteristicas a las que las microondas deben su atractivo para Las cOm nicaciones estan su amplio ancho de banda disponible (la capacidad para la transmi de informacién) y sus propiedades direccionales de onda corta, Puesto que el ancho de banda disponible limita el monto de informacién que es posible transmitir, el especie de las microondas brinda mas canales de comunicacion que las bandas de fadio 9 tele. vision. Las comunicaciones por microondas se han extendido a causa de la demanda, siempre creciente, de asignacién de canales. ‘Un sistema de microondas' esta normalmente integrado por un transmisor (el cual incluye un oscilador de microondas, gufas de ondas y una antena transmisora) y un subsis. tema receptor (el que a su vez incluye una antena receptora, una linea de transmision © guia de ondas, amplificadores de microondas y un receptor). Una red de microondas sucle Consistir en la interconexién de varios componentes y dispositivos de microondas. Existen diversos componentes de microondas, con sus Variantes: Los mas comiunes son: + Cables coaxiales, lineas de transmisién para la interconexién de los demas compo- = Resonadores, cavidades para cl almaccnamiente dle uuslas clecteounpnsticas, = Secciones de gufas de ondas, las cuales pueden ser rectas, curvas 0 en espiral. = Antenas, para la eficiente transmisin o recepeién de ondas clectromagnéticas. + Terminadores, los que, disefiados para absorber la potencia de entrada, actian como primeros puertos. * Atenuadores, los que, diseflados para absorber parte de la energfa electromagné- tica que pasa por ellos, reducen cl nivel de potencia de la sefial de microondas. * Acopladores direccionales, formados por dos gufas de ondas y un mecanismo para el acoplamiento de sefiales entre ellas. + Aisladores, para que Ia energfa fluya en una sola direccisn, + Circuladores, diseniados para permitir Ia alimentacién o extraccién de energia en varios puntos de entrada/salida. = Filtros, que suprimen senales indeseables, separan sefiales de diferentes frecuen- cias o hacen ambas cosas. Eluso de las microondas se ha ampliado sustancialmente. Son ejemplo de ello las tele- comunicaciones, radioastronomia, topografia, radares, metcorologia, television de UF, enlaces terrestres de microondas, aparatos transistorizados, ealefaccion, medicina y sist mas de identificaci6n. A continuacién se describiran s6lo tres de estos casos. 1. Telecomunicaciones. La transmisi6n de informaci6n analégica 0 digital de un pun- to a otro es la principal aplicacion de las frecuencias de microondas. Estas se propagan a lo largo de una linea recta como un rayo de luz y no sufren deformaciones en In ionosfera, como las sefiales de menor frecuencia, 1O cual hace posible la existencia de satélites de co municacion. Un satélite de comunicacién es, en esencia, una estacion de relevo de microon- das para el enlace de dos 0 mas transmisores y receptores terrestres Recibe sefiales a una frecuencia, las repite o amplifica y las transmite a otra frecuencia. En la figura 14.1 se pre- sentan dos modos usuales de operacién de la comunicacin por savélite, En la figura 14.1(a) Para un estudio completo de las microondas, véase D. M. Pozar, Microwave Engineering, 2a. ed. John Wiley, Nueva York, 1998. el satélite proporciona un enlace punto a punto, mientras que en Ja figura 14.1(b) estable- ce multiples enlaces entre un transmisor lerrestre y Varios receptores también terrestres, 2. Sistemas de radar. Los sistemas de radar fueron el motive mas importante del desarrollo de la tecnologia de microondas, dada la obtencién de mayor resolucién en ins- trumentos de radar a mas altas frecuencias. Sélo la region de microondas del espectro, podia brindar la resolticién requerida con antenas de tamafo razonable. La capacidad de Ia microondas para concentrar nitidamente una onda radiada explica su utilidad en esta aplicacion. Ademss de servir para detectar aviones, gular misiles supersGnicos, observar y tastrear patrones climaticos y controlar el trafico aéreo en aeropuertos, también se usan Yadares en alarmas contra robo, mecanismos para la apertura de puertas de cocheras ¥ detectores de velocidad de In policta. 3. Calefuceién. La energia de las microondas es més facil de dirigir, controlar y con- centrar que Ia de ondas electromagnéticas de menor frecuencia. Asimismo, a frecuencias de microondas ocurren varias resonancias at6micas y moleculares, lo que ha generado areas de aplicacion cn ciencias basicas, deteccién remota ¥ métodos de calefaccion. Las propic- dades ealefactoras de la energia de las microondas son utiles en numerosas aplicaciones in- Gustriales y comerciales, entre las que destaca el horno de microondas, representada en la figura 14.2. Al oscilar el magnetron, de las cavidades resonantes sc extrae energia de mi- croondas La reflexién de las paredes estacionarias y el movimiento del ventilador inducen Ia distribucion de esa energia, lo que acelera y uniforma el proceso de coccin. Ademas de Utilizarse en In cocina, las propiedades calefactoras de las microondas tambien se emplean fn la diatermia fisica y en la deshidratacion de papas fritas, papel, telas, ctcétera, Un circuito de microondas consta de componentes como fuentes, Ifneas de transmi- sion, guias de ondas, atenuadores, resonadores, circuladores y filtros. Una manera de ana- lizar tal cireuito consiste en relacionar las variables de entrada y salida de cada componente. Aunque para relacionar esas variables pueden usarse diversos conjuntos de parametros, en el andlisis de circuitos de microondas suelen emplearse los parametros S, dada In imprecision a altas frecuencias del voltaje y la corriente. Los pardmetros de dis- persion o pardmetros § se definen cn términos de variables de ondas, de mas facil medi Cidn a frecuencias de microondas que el Voltaje y Ia corriente. Ventitador ‘Guia de ondas {| ZaISy La 3 Figura 14.2. Horo de microondas Fuente: N. Schlager (ed.), How Products are Made, Gale Research Inc., Detroit, MI, 1994, p. 289. Considérese la red de dos puertos que se presenta en la figura 14,3, les se relacionan con los pardmctros de dispersion de acuerdo con byw Suey + Sata By = Saya, + Szstta (2]-[ S][e) bal lsu Sea) Las donde a, y a; representan las ondas incidentes en los puertos 1 y 2, respect micniras que b, y b, representan las ondas reflejadas, como se indica en la fi cuanto a la matriz 8, los términos no diagonales representan cocficientes de de onda de voltaje, en tanto que los términos dingonales representan cocficien flexion. Si la red es reciproca, tendra las mismas caracteristicas de transmision auier direccisn; es decir, 0, en forma matricial, Sa = Sa Sila red es simeétrica, entonces n ebettg Sata le Tag es ee donde Zp = Ze BLS Za ee Ons a Sa Sa oP, A Figura 14.3. Red de dos puertos. De igual manera, cl cocficiente de reflexién de salida (con V, = 0) puede expresarse en términos de la impedancia del generador Z, y los pardmetrog S como. aay donde cas) Los siguientes pardmetros § corresponden a un transistor de microondas que opera a 2.5 GHz: Sy) = U.8D/—3U", Siz = U.U//90", Say = LOBZ120", Sy, = U.8DZ—AU". Determine el coeficiente de reflexién de entrada cuando Z; = Z, = 75.0. Soluctén: A partir de la ecuacisn (14.7), Zp ~iey Carmen Te Por tanto, la aplicacion de la ecuacion (14.6) resulta en. Ty = Sy = 0.852=30° Hjercicio 14.1 En un acoplador hfbrido, la razén de onda de voltajé estacionaria de los puertos de entrada y salida esta dada respectivamente por ° 2 a 1+ Sul oe erst LAS T= 1sil Caleule 5) y 5, de! acuerdo con 1a matriz de dispersién siguiente! = [ed 8] s-[foe om Respuesta: 2.333, 1.5. 14.3. Interferen: y compatibilidad electromagnéticas Todo dispositive clectrénico es fuente de campos electromagnéticos rad emisiones radiadas. Estas emisiones suclen ser un subproducto accidental d dispositive. El ambiente clectromagnético es resultado de mecanismos como estaci de radio y television, radares e instrumentos de navegaciGn, los cuales: Glectromagnética al operar. Cualquier dispositive electronico es susceptib faflucncia se percibe en todas partes. Entre sus efectos estan los “fantasms Gion de imagenes de television, la interferencia de sistemas de radio de policiales y de transitorios-de lineas de energia con computadoras personales Gscilacion de un circuito de radio receptor o transmisor. La CE? se consigue cuando dispositivos electrénicos coexisten en armonta y funciona de acuerdo con el propdsito para el que se le fabric6, en presencia ( de los demas La IF es el problema —aparicion de voltajes 0 corrientes inde: influyen en el rendimiento de un aparato—y la CE la solucion. El objetivo de & rantizar la afinidad de sistemas o subsistemas, lo que se obtiene con la aplicaci hicas de disefio probadas. ‘La CE es un area en expansion a causa de Ia creciente densidad de circuitos cos en los modernos sistemas de computacion, comunicacién, control, ete. Pero, [htorecar # ingenteror elactrione y am eomputaciSn, también incumpbe a los ing mecanica automotriz. La progresiva aplicaciGn de sistemas electronicos en autom para clevar la eficiencia en el uso de combustible, reducir la emisiGn de conta fen garantizar la seguridad y brindar asistencia al conductor ha vuelto urgente la lidad en la operacion normal, A continuacién se expondran las fuentes y caracte! la TE, para después examinar las técnicas con las que se le controla. A. Fuentes y caracteristicas de la IE Clasifiquemos primeramente las causas de la TE, lo que facilitard la determinacion d dios de control. Como ya se mencion6, cualquier dispositive electrénico puede ser fi Wunque tal no sea la intencidn del disefiador. La IE puede ser causada den Para un estudio completo de las microondas, véase D. M. Pozar, Microwave Engineering Sam Weinie Pranee Youn, 1058. Enlace de relevo ‘$< Susccptorce de interierencia Figura 14.4, Casos comunes de problemas intersistemas de IE. Fuerte: JIN. Violette et al, Electromagnetic Compalibilly Handbook, Van Nostrand Reinhold, Nueva York, 1987, p. 4. fuera de un sistema, y representar por tanto un problema intrasistema o intersistemas. En la figura 14.4 se ustran problemas intersistemas de Il. For lo comun se lama emisor a 1a fuente y susceptor al dispositive afectado. Fn las tablas 14.1 y 14.2 se refieren a su vez cau- sas habituales de problemas intra c intersistemas. Unos y otros pueden controlarse general- mente scl ingenicro creador del sistema sigue ciertas p. ‘de diseflo. En el caso de problemas intrasistema, por ejemplo, es posible apli jas de conexion a Herta e interconexidn, blindaje de circuitos y dispositivos y filtracién. Tabla 14.1. Causas de IE Intaststema anor Susceptores| Fucntes de potencia Relevadores, ‘Tramamisores de radar Roceptores de radar “Tranamisores méviles de radio _—-Receptores indviles de eadio Slatemas de encendido de autos __Receptores de radio de autos SEL UST “CLIN) ssuoestunUIODD|a1 op TeuOIeUIDIUT UOTEA vt Tod sepe> Jun Soles op Sepeatiop o1per ap souo}oelusue[sos $e] S>[PUO!EUIIU SOPE aiqvive souo!sotnsoa so] © seio{ns Upiss sersuoNsoy5 ap SOUOCUREE SET "AI BL aHrotMudis wun a[duins viousnsa1y ey sepesndun soured sey onus sopesosou JO. SOTUSUE|OI SO] “SLEIUN[OA UOEIUDUNE|Sa! E[ "TCHNSAPUY USIP! DerINdosuos oJons vIs> “=> v] v sepuUTLEvoUD SouO!on JouOdW eid se JO} ¥f a1qos Uo!soIpstin{ ouoH eantesodo wouose eunguty anb Osan . ‘sepuipes & sepronpuoo souorsius sn1u9 jo19y1p ¥| ensnT Os “PT VIMBY vl UAT "udgiooI So] Nb SODIUONS9I> SOANISOT 2}sIut svsne> tapand anb soy ‘oveisde un 10d soprpadsop sostna9}9 sodures Bs souO|sjuIo su] “pox Uso B FOpMIIOUaD UPIGUIN) soasodsIp sOz10 UOD UID ord topand apuop “eyB19u9 op uHUIOD pas L] US SePHNPONUT Hos “(o|dUEIO!S ap a1uatt109 ap UuIO} ap 2[4¥9 19) swoHpIau! SeHIO}99KwN JOd sepeHtod -onb IpUoD SoUOTSTUID UOS “So[qesoPUT (soqUIE O SooHSUBLU ‘SODLagIO Sod. 9 (Goquite o seius11109 ‘Saley[On) SepInPUOD SUOISILD UOS OoIspq USSU AS S0] 10d sepeiouad) sofeiytiie 6 sopesnyeu 198 uopand a] 9p Soman SOT tei 9p o1pes 9p soiordooost —eppiod exper 9p sovostuumreat oe Seioperndtuion no ha fijado limites a las emisiones radiadas y conducidas de dispositivos clectrénicos co- mo maquinas de escribir, calculadoras, televisores, impresoras, modems y computadoras personales. La comercializacion en Fstados Unidos tle un aparato eleetronico supone, Ta previa verificacidn de que sus emisiones radiadas y conducidas no excedan los limites im- [puestos por la PCC. Asi, en el disefio de los dispositives electronicos que actuallmente se fa- rican cn ese pafs es pricticamente forzoso incorporar los principios de la CE, pues de 10 contrario es muy poco probable que tales aparatos resulten aprobados en esa inspeccién. B. Técnicas de control de la IE [Los tres medios de uso mis frecuente en el proceso de disefo para controlar o suprimir la TE: son la conexidn a tierra, el blindaje y la filtraci6n. Aunque cada una de estas tec nicas interviene de distinta manera en el disefo de un sistema, Ia adecuada conexién a tlorra puedo minimicar la nececidad de blindaje y GltraciGn,y el blindaje apropiade lane cesidad de filtracion. Las abordaremos por ese motivo en el orden citado, CONEXION A TIERRA La conexién a tierra ex el establecimiento de una trayectoria de conduccisn eléctrica en- tre dos puntos para la conexidn de los elementos eléctricos y electronicos de un sistema, ‘ya sea entre sf 0 con un punto de referencia, al que puede llamarse terra. Un plano a tie fra ideal serfa un cuerpo con cero potencial y cero impedancia que sirva como referencia A todas las sefales de un sistema de circuitos asociados y al cual sea posible transferir cualquier corriente indeseable a fin de eliminar sus efectos. Fl propésito de la tierra flotante es el aislamiento eléctrico de circuitos © equipo res- pecto de un plano a tierra comin, Sin embargo, esta técnica de conexién a tierra puede entrafiar riespos. La conexi6n a tierra en un solo punto minimiza los efectos de corrientes a tierra en grandes instalaciones, mientras que I de multiples puntos minimiza la longitud de los conductores a tierra. Fl plano a tierra podria ser un alambre a todo lo largo del sis- toma 0 un cuerpo conductor de tamaifio considerable ‘La interconexion es el establecimiento de una Trayectoria de baja tmpedancia entre dos superficies metalicas. La conexidn a tierra es un concepto de circuitos, y la intercone- Xion st aplicacion fisiea. Fi propésito de ésta es volver homogénea una estructura en 1o Que se refiere al flujo de corriontes eléctricas, para evitar el desarrollo de potenciales entre las partes metélicas, los que pueden resultar en IE. La interconexion brinda proteccién contra choques eléctricos, trayectorias de retorno de corriente en circuitos eléctricos ¥ conexiones con planos a tierra para antenas y minimiza de igual forma la diferencia de po- fencial entre dispositivos. Permite la conduccin de grandes montos de corriente de fuga, Hay dos tipos de interconexién: directa e indirecta, La interconexién directa supone el contacto de metal con metal entre los elementos conectados, mientras que la indirecta Jo efecttia mediante puentes conductores. ‘La resistencia en corriente directa Ry de una interconexiGn suelé indicar la calidad de ésta. Tal resistencia esta dada por ee sao) donde € es la longitud de la interconexi6n, o su conductividad y S el drea de transversal. Al incrementarse la frecuencia aumenta también Ia ConexiGn, a causa del efecto pelicular. Ast, la resistencia en corriente alt dada por € wow Re donde w es la anchura de la interconexisn y 6 la profundidad pelicular. La efectividad de interconexion pucde expresarse como la diferencia (en los voltajes inducidos en un equipo con interconexion y sin ella. BLINDAIE El propésito del blindaje es confinar la energ{a radiada a una region esp. que entre en una region especifiea, Los blindajes pueden adoptar la forma de compartimientos o de cables y conectores, El blindaje puede ser solido, no s6lido (pantallas, por ejemplo) yp estila en cables. En todos los casos se caracteriza por su efectividad de bi ‘cual se define como densidad de potencia incident densidad de potencia transmit ER = 1010830 donde la densidad de potencia incidente es Ia densidad de potencia en mmediciGn antes de la instalacion de un blindaje y la densidad de potencia fegistrada cn el mismo punto después de la instalacién. En términos de inte Campo, la efectividad de blindaje también puede definirse como la razén del ¢ io £, impregnado al campo incidente E,. Asi, EB esté dada por EB = 20106 2 En el caso de campos magnéticos, EB ~ 2010p9 22 Por ejemplo, una ldmina de aluminio (material con & = 3.5 < 107 Sim, = for de 0,01 mm de grosor tiene a 100 MFz una EB de 100 dB. Asi, un gabinete de para computadora, de mucho mayor grosor, se considera un blindaje muy efic protege los circuitos internos contra campos externos tanto como impide cion de tales circuitos al exterior. En estas condiciones, la emision radiada de. ma de computacién es producida por aberturas en el gabinete como grictas, de unidades de disco, ete., y por cables externos como los de toma de corriente dient. FI“rRACION Un filtro eléctrico es una red de resistores, inductores y condensadores constantes, agru- pados 6 distribuidos, con escasa oposicion a clertas frecuencias al El paso de otras, lo que reduce sustancialmente la interferencia conducida. ‘La caracteristica peculiar de un filtro es la pérdida de insercién (PT) como funcién de a frecuencia. La PI se define como. Hae sas donde V; es el voltaje de salida de una fuente de sefales con filtro en el circuito y V3 el Voltaje de salida de Ia fuente de sefales sin el filtro. Los filtros de paso angoste son de Uso comtin para efectos de CE. Su pérdica de inserei6n esta dada por PL = 10 logy (1 + F*) dB aaa6) donde ‘mfRC, _ para filtro capacitive FUAfLIR, para filtro inductive a4a7) y fes la frecuencia. .4. Fibra Gptica A mediados de la década de 1970 se admitié que Ia tecnologta de cobre serfa inadecua- Ga para las futuras redes de comunicacion. En vista de cllo. Ia industria de las telecomu- Ricaciones invirué intensivamente en la investigacin que derive en la fibra Optica, atractiva opcién a Iineas de transmisién aldmbricas como las de cables de par trenzado y coaxial. La fibra optica” iene las siguientes ventajas sobre el cobre: + Ancho de banda. Posee muy alta capacidad para portar informaci6n. Permite la fransmision de bits en serie, lo que reduce considerablemente el tamafio, costo ¥ complejidad de los equipos. + Arenuacién. Esta es tan escasa que Ia fibra Sptica puede cubrir largas distancias sin necesidad de repetidores. = Suscepribilidad a perturbaciones. Ni emite ni es vulnerable a la interferencia elec- tromagnética, Su inmunidad a ésta se debe a que carece de partes metalicas, lo que Ia exime de corrientes de conduccion. 3 Bxiston excelentes libros sobre la Sibra éptica. Véase, por ejemplo, S.-W. Meardon, The Elements of Fiber Optics, Regenta/Prentice Hall, Englewood Ciifts, NJ, 1993, = Seguridad. Dificulta la intereepcién dolosa, porque al realizar derivacio krumpe casi infaltablemente la comunicacion, a + Costo. Su costo se ha reducido drasticamente en los dltimos afos, ¥ segutn diendo. Lo mismo ha ocurrido con el de componentes asociados Come f Fes y receptores Spticos. Estas impresionantes ventajas sobre medios eléctricos explican Ia reciente de Ia fibra Sptica como medio de transmisiSn. En sistemas de comunicaciGn, tacin, redes de televisin por cable, automatizacion industrial y sistemas d y foro. El nucleo se compone de uno o mas hilos delgados de vidrio o plastica, El Iniento es la capa de cristal 0 pldstico que rodea al nuicleo, el cual puede ser cle indie Ionado © gradual. En el primer caso, el {ndice de refraccion del miicleo es pero sufre un cambio abrupto en la interfaz nucleo-revestimiento, mientras qi Segundo varia con Ia distancia radial desde el centro de la fibra. El forro cine Fecubierta o un haz de ellas Es de plastico u otros materiales y protege contra M doblez, etestera, Al introducirse en el nucleo, un rayo de luz se reflojard internamente al int medio mas denso y si el angulo de incidencia es mayor de cierto valor critico. flejado asf hacia el medio original y el proceso se repetira en el recorrido de la h nucleo. Este tipo de propagacion es multimodal, con referencia a la variedad de de reflexién, como se ilustra en la figura 14.7.’Esto provoca que Ia sefial se d desacompasadamente y limita el indice de recepeidn precisa de los datos. La red Gael anguis omies. SSeortida por el forro Figura 14.6. Fibra optica. 144. Fiona Orne, my 651 Figura 14.7, Modos de transmisién por fibra optica. Fuente: W. Stallings, res, 48. ed., Mace Local and Metropolitan Area Networ lan, Nueva York, 1993, p. 85. del radio del miiclee produce en cambio un modo de propagaci6n uniforme qq Ia distorsion. ‘Un sistema de fibra Optica es similar a un sistema de transmisin com se observa en la figura 14.8, esta integrado por un transmisor, un medio di gital y las convierte en sefiales opticas, que irradia modulando la salida de una luz (sualmente un diodo emisor de luz [light-emitting diode, LED] 0 rayo laser ceptor, donde un fotodiodo la reconvierte en eléctrica. Fl rendimiento de un enlace de fibra Gptica depende de las caracterist tura numérica (AN), atenuaciGn y dispersion de la fibra. Atenuacion y di sionan la senal a su paso por aquell: ¥iste es el pardmetro mas importante de una fibra 6ptica. Fl valor de AN esta d do por el indice de refraccion del nucleo y el revestimiento. Por definicion, Tefraccién nde un medio es __ velocidad de Ia luz en el vacte Velocidad de la luz en el medio Vieni Puesto que 4, = Hg en Ia mayorfa de los casos précticos, [st = Ve Jo que revela que el indice de refraccién es en esencia la rafz. cuadrada de la coi relaci6n a materiales comunes,” — Len aire, — 1.33 en agua y= 1.5 en vidrio. ‘Cuando un rayo de luz se propaga del medio i al medio 2, debe satisfacerse Snell, my sen 0, = nz sen 0s fgets “Gssagtee’ | Ceble we Se cree comers] LT emopticns Miedioge hesiaenaine cE Pow Figura 14.8, Sistema comin de fibra éptica. 14a Fiona ornce 653 donde #, es el dngulo incidente en el medio 1 y 0; el 4ngulo de transmisién en el medio 2: La reflexion total ocurre cuando @ — 90°, lo que resulta en = 8, aa21) donde 0, es el dngulo critico para la reflexién interna total. La ecuaci6n (14.21) sélo es Valida si'm, > n,,ya que el valor de sen 0, debe ser menor que o igual a 1. ‘Otra manera de analizar la capacidad de conduccin de luz de unia fibra consiste en medir el dngulo de aceprancia 0,, cl maximo dngulo en el cual los rayos de lux que entrant en la fibra seran atrapados por | nucleo de sta. Sabernos que el dngulo maximo ocurre cuando 0, 8 el dngulo critico, lo que satisface Ia condicién de reflexion interna total. Asi, Fespecto de una fibra de indice escalonado, AN = sen 0, = m sen 6. = Vint = 7 4.22) donde 1, ¢s el indice de refraccin del ndcleo y 7; el del revestimiento, como se muestra fen la figura 14.9, En virtud de que el nucleo suele ser de silice, n, = 1.48. Los valores mas. frecuentes de AN fluctian de 0.19 0.25. Cuanto mayor sea este valor, mayor sera la ‘energia Sptica que Ia fibra puede tomar de la fuente. ‘Cuando una fibra puede tolerar numerosos modos se Nama flbra multimodal de in- dice escalonado. Bl volumen de modos V esté dado por vet Vaal a2) donde d es el didmetro del nucleo de la fibra y Ia longitud de onda de Ia fuente 6ptica. ‘Con base en Ia ecuacién (14.23), el némero N de modos que se propagan por una fibra de indice escalonado puede estimarse como wate (4.24) _ ses ATENUACION Como se explics en el capitulo 10, la atenuacion es 6ptica en este caso. La atenuacién de la potencia en una fibra Optica (© per bra) esta regida por ap SP = ap donde a es la atenuacién y ? la potencia 6ptica. En la ecuacién (14.25) se que Ia onda se propaga a lo largo de z. Al resolver la ecuncion (14.25), la en la entrada de la fibra y la potencia P(€) de la luz después de € se relucionan do con PO) = Poye* La atenuacién @ se expresa habitualmente en dif/km y la longitud € de la fibra, este caso, la ecuacién (14.26) se convierte en : e+ 10 gs, 2 Asif, Ia potencia de Ia luz se reduce a decibeles por kilémetro al propagarse por La ceuncion (14.27) puede expresarse como P(e) = PC) + 10-00 Cuando € = 100 km, PO) _ f10-' para cable coaxial PO ~ lio para fibra a Jo que indica que la pérdida de potoncia on cl cable coaxial es mucho mayor que ‘ocurre en Ia fibra, z diseminacién de impulsos de luz al propagarse por la fibra, lo que pro’ se empalmen con periodos Sila dispersion excede dle cierto Iimite, puede contundie ceptor ya que los impulsos representan ceros y los periodos unos. Las fibras de mo forme resienten mucho menores efectos de dispersion que las multimodales. (41) | Sic! diametro e indice de refraccién del nucleo de una fibra de indice escalon: ‘de 80 jum y 1.62, respectivamente, y su abertura numérica es de 0.21, calcule: a) el Jo de aceptancia, b) €l indice de refraccin que la fibra puede propagar con una de onda de 0.8 jzm, c) el numero de modos que puede propagar con esa misma I pasty aepeliendy 144 Fawn Orns M655 Solucién: @) Puesto que sen 0, = AN = 0.21, entonces 0, = sen“1 0.21 = 12.12° b) De AN = V/ni = ni, se obtiene mma = Vil = AN = Vi62? = 027 = 1.606 o na nd AN v = tt Vad = TAN (80 X 10-9) X 0.21 _ fo ony 08 x 10-* SAY: Por tanto, Impulos de luz 20 propagan por un cable de fibra con atenuacisn de 0.25 dB/km. Deter mine Ia distancia a traves de Ia cual la potencia de los impulsos se reduce 40%. Solucion: EI hecho de que la potencia se reduzca 40% significa que Pe) PO) 04 =06 En consecuencia, PO) 10 ¢ = Crowe Fey 10. 1 = 925806 =8.874km a4 142, 443, Las microondas son ondas electromagnéticas de muy corta longitud de one propagan a lo largo de una Ifnea recta, a la manera de los rayos luminoso tanto, pueden ser facilmente concentradas por antenas en una direccion, en radares, conduceién, navegacion y calefaccién. ‘La compatibilidad electromagnética (CE) es la capacidad de dispositivos el cosy electronicos para operar on su medio electromagnético sin sufrir degradaciones inaceptables como resultado de IE. ‘La interferencia electromagnética (IE) es la falta de CE. Puede suprimirs diante conexion a terra, blindaje y filtracion. Una fibra Gptica es una estructura dieléctrica de guia de ondas que cuencias Spticas y consta de una region nuclear y una region de revest Las ventajas de la fibra dptica sobre el alambre de cobre son: 1. gran banda, 2. baja atenuacion, 3. inmunidad a TE, 4. seguridad y 5. bajo costo. Las microondas poseen larga longitud de onda. a) Clerto. 2) Falso. La longitud de onda en el vacfo de una sefial de microondas a 3. GHz de fr 2) Imm 2) 10 mm, 2) 1oem aim 4Cual de las siguientes no es fuente de TE? 2) Fibra éptica. 5) Computadora personal. ©) Radar policial a) Avion. ¢) Lémpara fuorescente. 144, 147. 149. La fibra optica es = ‘@) Una linea de transmisisn, 1) Una guia de ondas. ©) Ambas cosas. [A diferencia del cable coaxial y de par trenzado, la fibra Sptica es inmune a a) Teansmisi6n a alta frecuencia. b) Atenuacion de senal. ¢) Pérdida de potenci 4) Interferencia electromapnética. 2 locidad ni costo, el Gnico reparo es Ia interferencia de una extacion de radio cercana. ZCudl de los medios siguientes serfa el max apropiado para implementar la red? @) Microondas b) Cable coaxial ©) Fibra optica. ) Radio. Las aplicaciones de fibra dptica incluyen 4) Cable submarine, ‘b) “Telecomunteaci6n de larga distancia. €) ‘Transmision de datos a alta velocidad. 4) ‘Tnstrumentacion médica. ¢), Todas las anteriores LLos rayos de luz estin confinados dentro de una fibra éptica simple por medio de 1a) ReflexiGn interna cotal en la cara externa del revestimiento, 1b) ReflexiGn interna total en la interfaz nicleo-revestimiento, ©) Reflexién en el forro de la fibra. @) Refraccion. €) Defraccién. Si el indice de refracciGn del ndcleo de una fibra Gptica es de 1.45 y el de su revestimiento ‘de 1.42, la abertura numérica de la fibra es de a) 012 ») 0.18 ©) 020 4) 038 414.10. Un cable de fibra Sptica de 20 km de largo tiene una potencia de si pérdida de la bra es de 0.48 di/km, jeual es su potencia de entrada? a) 52 ew ») 19 aw ; ) 7uW. a) 2 Bw ae Respuestas: 14.1b, 14.26, 14.34, 14.4b, 14.5d, 14.6, 14.7e, 14.8b, 14.96, 14.104, iplicaciones de microondas diferentes de Ins ref Problemae J 141. Explique brevemente algun 142. Un ail conjunto de parametros, conocidos como pardmetros de transfe) seiretacionan nnn Ine ands ineidante y reflojarin Aa aenerde san 4 (e)-[ 2] [2] 61 (7a Tal Las 4) Exprese tos parémetros T en términos de los parametros S. ») Halle T cuando “i 02 0.4" * s=[o4 03] : 4143. Los pardmetros $ de una red de dos puertos son: Sy) = 033 ~ 70.16, Spy = Sy = 0.56, Sz = 0.44 — 0.62 7 Halle los coeficiente de reflexion de entrada y salida cuando ZZ, ™ 50 Qy Z,= 144. ;Por qué componentes regulares agrupados de circuitos como resistores, inductores Gensadores no pueden ser utilizados a frecucneias de microondas? 4145. En ef vacio, una seal de microondas tiene una frecuencia de'8.4 GHz. Caleule la “147. La pérdida de insercién de un ciretite con filtro puede calculavée en términes de sua trot A,B, Cy Dal terminar en Z, ¥ Z,,como se muestza en la figura 14.10. Demuestre qu AZ, + B+ CZ,Z,+ DZ, aos area 14.8. Si cl alto y ancho de In secciGn transversal rectangular de tina varilla dé plata son de OF 1.2 em, respectivamente, halle: PI = 20108 @) Laresistencia en corriente directa por 1 ea del conduetie: 1) La resistencia en corriente alterna por 1 km del conductor 6 MHz. 149. La velocidad de Ia Iuz en un medio dado se mide como 2.1 * 10" m/s. Halle el indice de oe Figura 14.10. Para et problema 147. 14.10. saz, nasa. 4437. Qué utilidad ofrece Ia fibra éptica en el aislamiento contra la TE? > r El ndcleo de una fibra de vidrio tiene un didmetro de 50 jum y un indice de refraccién de 11.62, mientras due el indice de refraccion del revesutniento es Ue 1-604 Sten esa fibra se em plea luz con una longitud de onda de 1300 mn, halle: @) La abertura numérica. 2b) EL éngulo de acepracisn, ) El ndmero de modos de transmision: ‘Una fibra Spica con radio de 2:5 yum e indice de refraccidn de 1-45 tiene sire como revesti- micnto, Si es iluminada por tn rayo de 13 jam de luz, determine: av 2”) AN 6) Una estimacién de cusntos modos puede propaga. Una bra Sptica con atenuacion de 0.4 dB/m tiene 5 km de lareo.m, = 1.53.1, — 1.43 y dia metro de 30 yim. Halle: 4) El Angulo méximo'en éheual rayos de luz entraran en Ia fibra y serdn atrapados 1) Bl poreentaje de la potoncia de entrada recibida. Un diodo de rayo ldser es! capa de acoplar 10 mW en una fibra con atenuacion de 0.5 dByum, Si la fibra es de 850 m de largo, calcule la potencia recibida en su extremo final. La atenuacion y del capitulo 10 esta expresada en Np/m, mientras que la atenuacién a, de este capitulo 10 est en dB/km. ;Cudl es la relacion entra ambas? ‘Un sistema de ondas luminosas se sirve de un enlace de fibra de 30 km con una pérdida de 0.4 dBicm, Stel sistema precisa de al menos 0.2 mW en el receptor, ealcule Ia potencia mi hhima que debe ser transmitida por la bea 4) Explique las ventajas derivadas del uso de cable de bra Sptica Ss, naheux neste ditemedietiniionnaiondl 15 Métodos numéricos La formula de la ignorancia es conformarse con fos Julclos y conocimientos p 5.1. Introduccién En los capitulos anteriores se estudiaron variag técnicas para resolver problem tromagnetismo y obtener soluciones en forma cerrada. Una solueidn en forma e ‘cas se obtuvieron suponiendo situaciones ideales. Al deducir la formula para eale Pacitancia de un capacitor (© condensador) de placas paralelas, por ejemplo, d Supuesto que el efecto de borde era insignificante y que Ia distancia de separ Jas placas era muy reducida en comparacién con el ancho y lirgo-de éstas, De blemas con limites coincidentes con superficies coordenadas. Las soluciones. tienen la ventaja inherente de ser exactas. Asimismo, es facil observar su comp. to cuando los parametros de los problemas varfan. No obstante, s6l0 los proble! configuracién simple toleran soluciones analiticas de las veloces computndoras digitales. Las tres técnicas numéricas simples de emp commtin en electromagnetismo son: 1, el metodo de momentos, 2. cl método de las rencias finitas y 3.e1 método del elemento finito. La mayor parte de los problems tromagnéticos implican ecuaciones diferenciales parciales © ecuaciones integrale Jo general, las ecuaciones diferenciales parciales se resuelven con el método de las di cias finitas o del clemento finito, mientras que las ecuaciones integrales se resuelven el método de momentos. Aundue los métodos numéricos ofrecen soluciones apro das, éstas son suficientemente precisas para los fines de la inges Ia impresion de que las téenicas analiticas son obsoletas a causa de los métodos 1 ricos: mas bien se complementan. Como se observara mas adelante, todos los mé numéricos suponen una simplificaci6n analitica que facilita su aplicacin. I codigo de Matlab elaborado para implementar en computadora los conceptos dos en este capitulo ha side simplificado y se explica por sf solo. Su notacién coincide del texto en 1a mayor medida posible, y se define cuando es necesario, La codification que 42 propone aquf no es Ia Gnica factible; un programa de computacion puede escribirse de varias maneras, Por tanto, el usuario puede modificarla de acuerdo con sus objetives. 2. Trazado de campos En la secciGn 4.9 usamos lineas de campos y superficies equipotenciales para representar un campo electrostatico. Sin embargo, las ?epresentaciones gréficas de la figura 4.21, re- lativa a campos clectrostaticos, y las figuras 7.8(b) y 7-16, relativas a su vez a campos map- netostaticos, son simples ¢ insustanciales y de cardcter cualitative. Distribuciones de carga mas complicadas hacen necesarias ilustraciones més precisas. En esta seccion $0 Presentaré una téenicu numérica que puede convertirke en uh programa de computaciGn Ritcractive. Bata (Scniea petera pusttos de datos para ineas de campos eléctricus y linens ‘equipotenciales para configuraciones arbitrarias de fuentes de puntos. ‘Lineas de campos eléciricos y lineas equipotenciales referentes a fuentes de puntos coplanares pueden trazarse con programas simples. En el caso de N cargas puntuales localizadas en los vectores de posici6n ¥),F2,.- » Ryn 1a intensidad de campo eléctrico Ey €l potencial V en el vector de posicin ¥ estan dados respectivamente por Sy Ou (ev) E- Fi amet eP asap ¥ A eh as2) Silas cargas se cncuentran cn el mismo plano (= ~ constante), las ecuaciones (15.1) y G5.2) se convierten en S Oni(x ~ xa)a. + Oy — yeas] BO 2 ame @ = a FO Tt . ve Se asa) i tel — AY +O IT Para trazar las Imeas del campo eléctrico se siguen estos pasos: 1. Se selecciona un punto de partida en la Ifnea del ampo. 2. Se calculan £, y £, en ese punto mediante la ecuacién (15.3). 3. Se avanza una corta distancia a lo largo dé la Iinea del campo hacia un nuevo pun- to en el plano. Como se observa en Ia figura 15.1, un movimiento 4€ a lo largo de Ja Iinea del campo corresponde @ movimientos 4x y Ay alo largo de las direccio- nes x y y, respectivamente. De esa figura se desprende que Ax _E, Ee ae ~~ (e+ BR Figura 15.1., Corto desplazamiento en una linea de campo. 2 ae- 5, seal 7 (222 Bey Se TEE h egy De igual forma, Se avanza asta lo largo de la Iinea del campo desde el punto inicial (x,y) a un punto x’ — x + Ax yy + Ay 4, Se rearesa a los pasos 2 y 3 para repetir los eslculos. Se siguen generando n puntos hasta completar una linea dentro del intervalo de coordensdas d completar la linea, se vuelve al paso 1 y se elige otro punto de partida. Tén cuenta que, al existir un numero infinito de ineas de campo, es muy probable cualquier punto de partida se ubique sobre una de ellas. Los puntos generado: » pueden trazarse a mano 0 con un graficador, como se ilustra en la figura 15.2. Para trazar las Itneas equipotenciales se siguen estos pasos: 1. Se selecciona un punto de parti. 2. Se calcula el campo eléctrico (Z,, F,) en ese punto mediante Ia ecuacién (15.3), Figura 15.2. Puntos generados en Iineas del es (Continuas) y lineas equipotenciales (punteadas). plo 15.1 3. Se avanza una corta distancia a lo largo de la Iinea perpendicular a la Itnea del Campo F en exe punto, Recuerdese a este respecto que si una Iinea tiene pendien- fem, una linea perpendicular debe tener pendiente —l/m. Puesta que una linea, del Campo £ y una linea equipotencial que se cruzan en un punto dado son re Procamente ortogonales en ese punto, ~ae~z, ann Te gy em) Aye AEs Bee 15.8) tae + BST So Asi, se avanza a lo largo de Ia linea equipotencial desde el punto inicial (x,y) a un huevo punto («+ x,y +b Ay). Para comprobar éste, se calcula el potencial en los puntos nuevo ¢ inieial mediante Ia ecuacion (15.4); ambos valores deben ser igua~ Tes, porque los puntos se sitdan en la misma linea equipotencial, 4, Se represa a los pasos 2 y 3 para repetir los cdlculos, Se siguen generando nuevos, puntos hasta completar una linea dentro del intervalo de x yy dado. Iras comple~ {ar la Iinea, se vuelve al paso 1 y'se clige otro punto de partida. Los puntos gene- Fados se unen » mano o con un graficador, como se muestra en Ia figura 15.2, La linea del campo magnético debida a varias distribuciones de corriente puede trazarse, Gon base en la ley de Biot-Savart, siguicndo el mismo razonamiento. Es posible desarrollar programas para determinar la linea del campo magnético debida a una corriente lineal, Eneespira de corriente, un par de Helmholtz y un solenoide. También es posible escribir programas para trazar lineas de campo eléctrico ¥ magnético dentro de una guia de ondas Fectangular o el patron de radiacién de potencia producido por un arreglo lineal de antenas verticales de dipolo eléctrico de media ond: scriba un programa para trazar las Ineas de campo eléctrico y equipotenciales debidas a: 2) Dos cargas puntuales @ y —4@ ubicadas en (x, ») = (—1,0) y (1, 0), respectivamente. b) Cuatro cargus puntuales O, ~Q, Oy —Q ubicadas en (x,y) = (<1, —1),, 1), 1) ¥ C_1. 1). respectivamente. Adopte Q/4 me = Ly A€ ~ 0.1. Considere él intervalo *Sexy=3 Solucién: El programa que aparcce en la figura 15.3 se labors con base en los pasos descritos en a secelon 13-2. Los comentarios insertados permiten que el programa se explique por sf Solo, Para generar cl diagrama que se muestra en la figura 15.4(a), cargue el programa plotit cn el directorio Matlab. En el indicador de comandos de Matiab, teclee plotit (1 —4}[—1 0; 1 0} 4, 1,041, 0.01, 8,2, 5) nuimeros cuyo significado es provisto por el programa. Mas adelante se ofrecen explica- Ciones adicionales sobre éste. ‘Puesto que las Iineas dei campo lineal £ emanan de cargas positivas y terminan en cargas nepativas, parece razonable generar los puntos de partida (,,,) de esas ineas Gn pequchos circulos centrados en las ubieaciones de carga (eg) CHt0 e5, x, = xg + eos s.ta) yim ve trsene as.1.18) function piotit (charge Che piot is to bewithin che range —Sec,r5 ckmpiela, ekea, DLE, DLV, inction plotit. (chaxgew, location, ckEPLeld, ckilg, DLE, DLV, MLE, toa vector dontaining Location = a matcix were each Llomgpieid = Fiag wet col plore S ckka ~ Ping sat to 1 plote the equipetential lines pur or BEY = the increment along Ea V lines "MLE = No. of PrField lines per charse prs i> Plots very PEG point (1-0, if Pré = 5 then plot every Sth pine? Wipote that constant '9/4*Bietlirh 1s sac equai’ co 1.0 4m a charge location ge apie bheraan at cule ror conventence, the etarcing points (xs, xs) exe xeatelly Siaeriaited about charge Locations Xo ~ location (1,2) NO > length (charge): Le, (oxmrieray peta = a=pir(Z-2) / (we) + XE=xQ(W) + 0; 1pe0R (THETA) + Yervats) + Oli-een(@HREA) + LE (-moa@y, prs) & Ri pina 4 ncremant ana new poine G&¥) Figura 15.3. Programa de computacién para el ejemplo 15.1. my omy FOL) Cee YONI ORM) fe. cr <=.00005) leart (CEQ (9))72 + CHE - OLD? AE Ceabe ue) = 5) | (abecemy == 59) Fiqura 15.3. (Continuactén,) for x =2:m0 XS = XQUR) + FACTOR" COS (ANGLED + Le ( abe(xs) = 3 | abe(ve) 2-5) pier (aya) for 7 = ame Bx = BE + OUI) (RV-XOCIII/ RTS) s 8 (3,¥) nur INCREMENT TW ‘Tite OPPOSITE DIRBCTION ie (apecnw) > 5 | apace) > 5) Figura 18.3. (Continuacion.) plot Ow.) 2 Figuea 18.3. (Continuacion). donde res el radio del efrculo (por ejemplo, r = 0.1.0.0.05) y @ un Angulo prescrito ele- gido para cada linea del campo £. Los puntos de partida de las Iineas equipotenciales pueden generarse de diferentes formas: a lo largo de los ejes xy y, a lo largo de la linea PSY asi sucesivamente. No obstante, para que el programa sea lo mas general posible, los puntos de partida deben depender de las ubicaciones de carga, como en el caso de las Iineas del campo E, Podrian seleccionarse por medio de la ecuacién (15.1.1), pero con @ fija (45°, por ejemplo) y r variable (0.5, 1.0, 2.0, .-.. por ejemplo). El valor de la longitud incremental A€ es crucial para conseguit diagramas exactos. Cuanto menor sea el valor de AC, los diagramas seran mas precisos, pero tambien se generaran mas puntos, lo cual puede causar problemas de almacenamiento en la memo- fia, Por ejemplo, una linea podria constar de mas de 1000 puntos generados. En vista en este caso del gran numero de puntos por trazar, serfa conveniente almacenarlos en un archive de datos y seguir una rutina de graficos para trazarlos. ‘En el programa que aparece en Ia figura 15.5 se han insertado diferentes comproba- ciones para las Iineas tanto del campo F como equipotenciales: a) Comprobaci6n de un punto singular (:E = 07). 4) Comprobacién de si el punto generado esta demasiado cerca de una ubicacion de carga: 6) Comprobacién de si el punto se encuentra dentro del intervalo dado de —5 < x, poe d) Comprobacin de si las curvas dé las Iineas (equipotenciales) vuelven al punto de partida. FI trazo de los puntos generados con relacion a las dos y cuatro cargas puntuales se muestra en las figuras 15.4(a) y 15.4(b), respectivamente, Figura 15.4, Para el ejemplo 15.1: diagramas de lineas datcampo £'y lincas equipotenciales debidas a (a) dos ‘cargas puntuales y db) cuatro eargas puntuales (an cum Srupois pidimensiona, Ejercicio 15.1 Eseriba un programa completo para trazar las: Jineas de campo el ‘quipotenciales debidas a cargas puntuales coplanares. Ejecute el = 3:es decir, tres cargas puntuales —Q. + Oy — Gaerne onl oe Metceameme adopte Giaree 4, Ae > 61.6 O01, pe Figura 15.5. Para el cjercicio 15.1, 3. Método de las diferencias finitas EI método de las diferencias finitas! (MDF) es una técnica numérica simple de utilidad para resolver problemas como los que se resolvieron analiticamente en el capitulo 6. Un Problema esta inequivocamente definide por tres cosas: 4. Una ecuaci6n diferencial parcial, como la ecuacién de Laplace o Ia de Poisson, 2. Una region de solucion. 3. Condiciones en la frontera, iniciales 0 ambas. Por ejemplo, una solucién de diferencias finitas de a ecuaciGn de Poisson o Laplace se obtiene en tres pasos: L division de la region de solucion en una cuadricula de nodor 2. aproximacién de Ia ecuaciOn diferencial y las condiciones en la frontera por medio de un Conjunto de ecuaciones algebraicas lineales (Ilamadas ecuaciones en diferencia) en Puntos de Ia cuadricula dentro de Ia region de solucion, y 3. resolucién de este conjunto de ecuaciones algebraicas. { Para una amplia exposicion del método de Ins diferencias finitas, véase G, D. Smith, Numerical Solution of Partial Differential Equations: Finute Difference Methods, 28. ed Clarendon, OMord, 1978, Paso 1. Apliquemos el método de las diferencias finitas a la determinacién del p. cial eléctrico en Ia region que se muestra en la figura 15.6(a). La region de solucion xe dividido cn mallas rectangulares con puntos de cuadricula © nodos. Un nodo en la tro tera de la region en el que el potencial esta especiticado se lama nodo fijo (fijo por problema), en tanto que los puntos dentro de la regiGn se Haman puntos libres (libres El sentido de que el potencial en ellos se desconoce). ‘Paso 2, Nuestro objetivo es obtener Ia aproximacién de diferencias finitas de ecuacién de Poisson y utilizarla para determinar el potencial en todos los puntos Recuerdese que Ia ecuacin de Poisson esta cada por vv = ass ot ete ewe sep desl nnn! Som Sco em 75.) pe foarplioadpoc ps B= fy do manera pos eee te A partir de la definicion de Ia derivada de V(x, y) en el punto (x. ¥.) OV) _ VG + Ax ye) — VGxe — Ax ye) ox |e. Figura 15.6, Patron de solucién de diferencias finitas: (a) division de Ia solucion tootapanciarediennediicliedes titcennlilanic damian tiasilimnaiientadien aa 94 19 donde Ax es un incremento suficientemente reducido a lo large dex IEn cuanto a la se gunda derivada, la derivada de Ia primera derivada V", av. ove _ Vilas + Ax/2, yo) Be lag BE Vite + Ax, yo) ven v, saa xy? Las ecuaciones (15.10) y (15.11) son las aproximaciones dle diferencias finitas de la pri mera y segunda derivadas parciales de V respecto de x, evaluadas en x — x,- La aproxi macidn de la scuacion (15.10) se arocia con un error an el orden de Av, mientras que Ia de la ccuacion (15.11) se asocia con un error en el orden de (Ax)*, De igual manera, PV) 9 Von Ya + AY) — 2V Cen Yo) + V Cor Yo — AY) av Tye (ry Vises > Vay + Ve an ‘AI sustituir Ias ccuaciones (15.11) y (15.12) en Ia ecuacién (15.95) y conceder que Ax = Ay = ise obtiene. (as.12) Miser ene + Vang avy = Oe | Oe Gee ee nie eee as.13) er, oie asa) La aproximacién de diferencias finitas de esta ecuacion se obtiene de Ia ecuacién (15.13) al fijar py — O;es decir, Vv Ferg + Vins + Viner + Vis (as.1s) sta ecuaciGn es en esencia una aproximacién de diferencias finitas de cinco nodos det jpotencial en el punto central'de una malla cuadrada. En la figura 15.6(>) se presenta una molécula de cinco nodos de diferencias finitas, deducida de la figura 15.6(a). Apl esta molécula, asf, Ia ecuacién (15.15) se convierte en. i HEM + Vat V+ WD ‘sta ecuacién exhibe claramente la propiedad de valor promedio de Ia ecuacion de place. En otras palabras, la ecuacién de Laplace puede interpretarse como un medio. ferencial de enunciar que el potencial en tn punto especifico equivale al promedi del potencial en los puntos vecinos. Paso 3. Para aplicar la ecuacién (15.16) fo (15.13)] a un problema dado suele se se uno de estos dos métodos: A. Método de iteracién Para comenzar, se asigna al potencial en los nodos libres un valor inicial de cero 0 cual- Auier otro razonable valor estimado. Manteniendo inalterado en tode momento el poten: cial en los nodos fijos, se aplica Ia ecuncion (15.16) a cada nodo libre hasta ealeular potencial en todos ellos. Los valores del potencial obtenidos al final de esta primera ite- Facién no son exactos, sino meramente aproximativos. Para dotarlos de mayor precisiGn, se repite el ediculo en cada nodo libre a partir de los valores anteriores para determinar huevos. La modificaci6n iterativa o reiterada del valor del potencial en cada nodo libre prosigue hasta aleanzar un grado de precision prescrito @ hasta que el valor anterior y él huevo en cada nodo son satisfactoriamente cercanos. B. Método de Ia matriz en banda La aplicacion de la ecuacin (15.16) a todos los nodos libres resulta en un conjunto de ccuaciones simulténeas de la forma (41 = 1) as.a7 donde [4] es una matriz escasa (es decir, con muchos términos cero),[V] se compone del _ potencial desconocido en los nodos libres y [B] es otra matriz cn columnas formada por El potencial conocido en los nodos fijos. La matriz [A]es tambien una matriz en banda, ya que sus términos diferentes de cero se agrupan en torno a la diagonal principal en razon de que el potencial en cada nodo solo se Ve afectado por los nodos vecinos mils Proximos ‘La matriz escasa en banda se inverte facilmente para determinar [V] Asi, de la matriz[V] se obtiene el potencial en los nodos libres de esta forma: (y= tar (a ass) El método de las diferencias finitas puede aplicarse a problemas con variacién en el tiempo. Considérese, por ejemplo, la ecuaci6n de onda unidimensional (10.1), as.19) donde w es la velocidad de la onda y @ la componente de campo E © H de la onda clec~ tromagnética. Las aproximaciones de diferencias de las derivadas en (r,./,) 0 en el hodo, de orden (i, /) que se muestran en la figura 15.7 son FO) Pinny — 2D. + Mors ae ste: fat > 2520) eo) A204 Po ah an ae ‘Al insertar Ins ectinciones (15.20) y (15:21) en In ecuncign (15:20) y despejar @,,, se ob- Pay = M1 + Porgy + 20 ~ 2) Oy — Oyo (5.22) onde ws peel (as2a) Es posible demostrar que para que la soluci6n referida en la ecuacién (15.22) sea esta~ bie. a = 1, Para iniciar el algoritmo de diferencias finitas de la ccuacién (15.22) se em- plean las condiciones iniciales. Suponemos que en f= 0, 28/8 = 0 y utilizamos la Aproximacién (central) de diferencias (véase la pregunta de repaso 15.2) para obtener Po a> G1 Zar =o B= @, «as.2a) [La sustitucion de la ecuacion (15.24) en la ecuacién (15.22) y la adopeion de j = 0 (¢ = 0) preducen Diy AD 1,9 + HL, FAA — 2) Dy ~ His 15.7. Patron de soluci6n de diferencias ‘ecuncion de onda. ample 15.2 By FlalD 50 + Prvrja) +21 — a) 6) ass) Con Ia eeuacion (15.25) como formula “de partida”, por medio de Ia ecuscién (15.22) puede obtenerse directamente el valor de Pen cualquier punto de Ia cumdriewla. Convieng Sefialar que los dos métodos descritos para resolver Ia ecuacién (15-16) no se apliean a la ecuacisn (15.22), porque ésta puede usarse directamente con la ecuacién (15.25) como, formula de partida. En otras palabras, en este caso no se tiene un conjunto de ccuaciones, simultaneas; la ecuacién (15.22) es una formula explicita. El concepto del MDF puede prolongarse a la ccuaciéii de Poisson o de Laplace y a las ecuaciones de onda en otros sistemas de coordenadas. La precision de este metoda depende de la calidad de la cuadricula y del tiempo que se invierta en la depuracion de los valores del potencial. Fl tiempo eh In cnimpitndora puede raducirse 2 inaremenuane, In exactitud ¢ fadice de convergencia si las estimaciones de los valores iniciales son taco. nables, se obtiene provecho de la simetria (de ser posible), se (rabaja con una malla tan Pequenia como se pueday se utilizan moléculas de diferencias finitas mas complejas (ig, 15.41). Una limitacion de este método es la necesidad de recurrir a una interpolacion de. algiin tipo para determinar soluciones en puntos no situados en la cuadricula, Una forma, obvia de-evitar este inconveniente consiste en utilizar una cuadricula mas fina, lo que sin ‘embargo implica gran nimero de calculos y la ocupacién de un vaste espacio de memo- Fiaen la computadora. a Resuelva el problema unidimensional con valor en la frontera —@" ~ x7,0 =x =1 su jeto a DO) = 0 = D1), Aplique el método de las diferencias finitas. Solucton: Se obtiene primero la aproximacién de diferencias finitas de la ecuacién diferenei "= 33, 1a ecuacién de Poisson en una dimension. Después se divide el dominio entero 0 =< y= 1 an Nsagmentos iguales de lonpitud » (— LUN), some ne mucstra en te, figura 15.8(a), de tal forma que haya (N+ 1) nodos. PO) _ Pxy +h) ~ 24a) + xa — h) ae lene a oe Figura 15.8. Para el ejemplo 15:2, © gn M28 + Os nag = S29 On Ast 20, = — 3h Or Pa B 2 2) By =F (Py + Os + ape) Del empleo de este sistema de diferencias finitas se obtiene una solucion aproximada de diversos valores de NF codigo de Matlab correspondiente se presenta en la figura 15.9, [El numero de itcraciones N7depende del gracio de cnactitudl que se slesee. Lat aan puublenies unidimensional como éste quiza sea suficiente con N/ = 50;en problemas bi o tridimen- sionales se precisara de valores mayores (tabla 15.1). Cabe sefialar que los valores de en puntos extremos (nodos fijos) se mantienen fijos. En la figura 15.10 se muestran las Soluciones relativas a N= 4/10. ‘Esta solucién puede compararse con la soluci6n exacta, la cual se obtiene de la si- guiente manera, Puesto que d'dx? = —x?, una doble integracion resulta en. x sHtarte 2 pery danni» ph (2m) phiewenr. = (2.0-%.73) /12-07 (linet)! phi phies) phd (3mea) +phs (Linea) se. -20N823 /2; iaeiteal ais iil iain die iii Bete LAD ed Get dls gina’ ca Bp Ee & ° Figura 15.10.’ Pate el ejdmplo'152; dingrama de @ (2). La curva Gontinus corresponde a N= 10 la curva punteada aN — 4. donde A y B son constantes de integracién. Con fundamento en las condiciones. frontera, 0) =0->B=0 1 1 Bra 0 aat En consecuencia, la solucion exacta es @ = x(1 — x°)/12, la cual se calculo en la 15.9 y result estar muy cerea del caso N = 10. a1) =0+0= Determine el potencial en los nodes libres del sistema potencial que aparece en In ra 15.11 aplicando el método de las diferencias finitas. Solucién: Bate problema vo resolver mediante, loe métodos de iteracién y.de spatciz on bands Figura 15.11. Para el cjemplo 15.3. ov |__Js _| -— a Método 1 (método de iteraciém). Se establecen primero como equivalentes a cero los valores iniciales del potencial en los nodos libres. Después se aplica la ecuacién (15.16) a cada nodo libre utilizando los potenciales circundantes mas recientes cada vez que se calcula el potencial en un nodo. En cuanto a la primera iteracion: Vy ~ 14 +20 +0 +0) —5 Vz = V4(S + 0 +0 + 0) = 1.25 Va = WA(S +20 +0 +0) = 6.25 Vi. = 14.25 + 6.25 +0 +0) = 1.875 y asf sucesivamente. Para evitar confusiones, cada vez que se calcula un nuevo valor en Un nodo libre, se tacha el valor anterior, como se muestra en la figura 15.12.'Tras caleular Vi Se inicia la segunda iteracion en el nodo 1: Vy = 1/4 + 20 + 1.25 + 6.25) = 6.875 Vy = 1/4(6.875 + 0 + 0 + 1.875) = 2.187 y asf sucesivamente. Después de cinco iteraciones se obtienen los valores no tachados de la figura 15.12. Luego de 10 iteraciones (Ias cuales no se muestran en esa figura) se obtiene, Vi = 1004, Vz, = 4.956, y= 15.22, V4 = 9.786 V,= 2105, Vg= 18.97, V,= 15.06, Vy 11.26 Método 2 (método de matriz en banda). Este método manifiesta la estructura escasa del problema. Se aplica la ecuacion (15.16) a cada nodo libre y los términos conocidos {potencial preserito en los nodos fijos) se agrupan en el lado derecho, mientras que los tér- minos desconocidos (potenciales en nodos libres) se ubican en el Indo izquierdo del sis- tema resultante de ccaciones simulténeas, el cual se expresara en forma matricial como ay = 1 En cuanto al nodo 1, —4¥, + Va+ Va = -20— f Meropos Numencos & o as Figura 15.12, Para el ejemplo 15.3; los valores sin tachar son Ja solucion, de las cinco iteraciones, Vit 4¥, + Vy En cuanto al node 3, Vin 4Vy + Va + Vs = -20 En cuanto al nodo 4, Vast Vy 4V, + Vg = En cuanto al nodo 5, V5~ 4V5 + Ve = —20~ 30 En cuanto al nodo 6, Vat Vs— 4V5 + V, = —30 En cunnto al nodo 7, Ve 4V, + Va = —30—0 | En cuanto al nodo 8, V, — 4¥y=-0-0-30 | " Notese que se cuenta con cinco términos en cada nodo, ent raz6n de que se esté usando ONSSS ideuin de cinco nodos. La siguiente es la expresin en forma matricial de las ocho ccuaciones obtenidas: [Fs Rs vs 20 1 Ge vs ° Je va | | 20 eae a Bl tans oan as0 vi|7| 50 oa ONS ve) | =30 o o o oO va 30 oO 0 oO oO | Vs ba) Lavy =18) donde [4] es 1a matriz escasa en banda, (V]la matriz en columna integrada por los valores des: sonocidios del potencial en los nodos libres y [#] la matriz.en columna formada por el poten- cere los nodos fijos. La naturaleza de “banda” de [A] se indica con un marco punteado, Ropareee cn que Ia matriz [A] pudo haberse obtenido directamente de la figura 15.11 sin tener gue apligar la ecuacién (15.16) @ cada nodo libre. En tal caso, simplemente se sulableced los términos de la diagonal (0 autotérminos) como Ay = m4 y se fija Ay = 1 ene os taran visculadus & Ay 0 ai no cotan directamenta vineulados. Por jem- eee 2 20. poraue los nodes 2 y 3 no estén vinculados, en tanto.que Aas > Awa = 1+ eieraue los nodos 4y 6 estan vinculados. De manera similar, la matriz [B] se obtiene di- eerie conte dela figura 15-11 estableciendo que #,es igual a menos la suma de los va~ sees yStencial en los podos fijos vineulados con el nodo J. Por ejemplo, Bs = — 20 + Wy porque si node 5 esta vinculado con dos nodos fijos con potencial de 20 V y 30 V. Si ei ngde tno esta vineulade con ningtin nodo fijo, B, = 0. “Al invertir la matriz [4] con Matlab se obtiene IM =f4r a) Vi = 10004, Vz; 4958, ¥y= 15.22, Va = 9.788 Vy = 21.05, Vy = 18.97, V¥, = 15.06, Vy = 11.26 resultado aceptable en comparacién con el obtenide mediainte el método de iteracién. igure 15.13. Para el ejercicio 15.3. Iiptgigia| | Odtenga Ia solucion de 1a ecuacion de Laplace respecto det tanque de\longitud infin cuya seccidn transversal rectangular se ilustra en Ia figura 15.14, Sea V, = 10-V, Vz = 100 Vs=40Vy Va= OV. : aa _ | Bste problema se resolverd siguiendo ef método de iferacion: Ea este caso, |} de sohucion Gene una frontera regular. Se puede escribir facilmente uh programa para, terminar el potencial en los puntos de la cuadrfcula dentro del tanque. Se divide | en mailas cuadradas. St se decide utilizar una cuadricula de 15 < 10, el numero de p »| tos de cuadricula a lo largo de x es 15 +1 = 16 y el mimero de puntos.a lo largo dey 10 +1 = 11. El tamafio de la malla # = 1.5/15 = 0.1 m. Pa cuadricula de 15 > 10 x Figura 15.14. Para el ejemplo 15.4, ee : 6.20) Figura 15.18. Para cl ejempto 15.4; 0.1) 5.10) cuadrfoula de 15 5 10, 0.5) 20) = mucctra on la figura 15.15. Los puntos de la cuadrfoula se han numerado (é,/) @ partir del extreme inferior iquierdo del tanque. Teniendo en cuenta la aplicacion de la ecuacion (G5.15) y del método de iteraci6n, el programa de computacién que se refiere en la figu- fa 15.16 se elaboré para determinar €l potencial en los nodos libres. En la tabla 15.1 se presentan los valores del potencial en los puntos (x, y) = (0.5, 0.5), (08, 0.8), (1.0, 0.5) ¥ (8, 0.2), correspondientes a (i,j) = (3, 5), (8, 8), (10, 5) ¥ (B, 2), respectivamente, luego. de 50,100 y 200 iteraciones. Se presentan asimismo ios valores exactos [véase el inciso ©) del problema 6.18), obtenidos con el método de separscion de variables y un pro- grama similar al contenido en la figura 6.11. Valga insistir en que el grado de precision depende del tamano de mallia . Siempre os Conveniente que ft sea lo mas pequefio posi- ble. Adviertase también que los valores del potencial en los nodos fijos se mantuvieron, constantes durante la realizacién de todos los caleulos. aa 13.0) Phot (LAPLACE's EQUATION) SHOWN IN FIG. 15-14 © SG Tas° powmweran ar ORED POINT (4,5) OR Gey) WITH Figura 15.16. Programa de computacién para el ejemplo 15.4 vale = vay ae for go2eny-a : = vase vas 3a Pasay = o.se(wa + vars Vin) = O.seiwa + v2) vGuay) = 0.s*(s + vay? Vincsny) = 0.52 + v3)e Wonow Bin vc4,3) USING EO. (25.15) APTER ni ZTERATIONS for de2eny-2 " vided) = 0-286 (wCbeaedd) * videgead + wedged ena ran = “ oa tetere). v(3.9)~ V2.8), ¥65.3)) {aime deny) weed) I ‘ | diary off core Figura 15:16. (Continuacion). | ~ as | Tabla 15.1, Solucion del ejemplo 15.4 (por el metodo iy de Keracién) en puntos selecton. Goondanedat 2 eo 50100 200_ Valor exacto wo mae 2249 22s a 4 M7. Mase aso Gass nak fim 41s 432) aaa 2098 2097 2089 15.4. MEOOO OF MOMENTOS: mH 68a a Figura 15.17. Para el ojercicio 15.4. . 15.4. Método de momentos Como el método de las diferencias finitas, el método de momentos(MM)? tiene Ia venta- ja de la sencillez conceptual. En tanto que el primero se utiliza para resolver ecuaciones diferenciales, con cl segundo se resuelven ecuaciones integrales. ‘Supongamos que se desea aplicar el método de momentos a la ecuacién de Poisson (45.94). Es posible demostrar que una soluci6n integral a esa ecuacion es pudv v~ | arer (as26) Como se recordara, en el capitulo 4 se explicé que una ecuncin como 1a (15.26) puede deducirse de la ley de Coulomb. Recuérdese también que, dada la distribucion de carga, p(x, ¥.2), €8 posible hailar el potencial VG y, 2), el eampo eléctrico E(x, y, 2) y la carga total @. Pero'si se conoce el potencial Vy la distribucion de carga se dexconoce, ze6mo determinar 9. a partir de la ecuacién (15.26)? Convirtiendo a ésta en ecuacién integral Su forma general es vee) = [fxn a0 ae pean ey ee ee ea ope ae ee areas ine ana come 3 Harrington fue cl primero en usar el término método de momentos en Ia bibliografia occidental. Para mayores detalles sobre este metodo, veae KR. F- Harrington, eld Compumation by Moment Methods, Krieger, Malabar, FL. 1968. taco detcdémir ie a ednremsitcen comm pars reeciversmonciones itogill ete se meintise US ah Sele compromicrs sejortonn un Semple Cian et site Stor Segue cneh fess de als SY Nong LE. > 9) spun Se umsetn an ta Rgice 28.18 Scpdgane ape nolo mation won feel Ge Bindi propésit on determinar ia Sentilad Ge carga pra lange de Soe slambet nS Sears sitnctote dtamucoma tistics Sctarnameas oh purtisioic eaetaal ads carmpor saccindan Ea ecuncion Ca.26) e seduos on alguien pants al sins soo Soares * prdt ven [get casas) Tract ae Ia euuacién (15:28) ne apes pinion de obeervaciin dn cualquier part de Hinsbpach tn panto WG yc liseode ponte de econianions 45.29) 1 [* px) dy arrest, lye — vt Recuérdese que la integracion es en esencia Ia determinacién del érea que se encuentra de- ‘bajo de una curva, Si Ay es reducida, la integracion de f(y) sobre 0 = y= L esté dada por [10 4y +700 ay + Fon ay + + + Fow) Ay : zs 45.30) ~ Zyros donde el intervalo 1 se ha dividido en N unidades de longitud Ay. Dividiendo el alambre cn. N segmentos de igual longitud A como se muestra ‘en la figura 15-19, la ecuacion (Gi5.29) se convierte en Bebe ig ys sy ee anys Lye =x Lye — al Le = yal aso1) donde A = L/N = Ay. Bl supuesto de la ecuacién (15.31) es que la densidad de carga des- conocida p, on el segmento de orden k es constante. Ast, en la ecuacidn (1531) se tienen, Figura 15.18. Alambre conductor delgado bo bs. Figura 15.19. Division del alambre en N segmentos, constantes desconocidas p,, px... py: En virtud de que la ecuacién (15.31) debe soste- erse en todos los puntos del alaibre, se obtienen N ecuaciones similares seleccionando: N puntos de acoplamiento en yy, Ya, --=sYur---Jw en el alambre. Ast se obtiene pa pa pwd heres mit a ER os op EO naa Ta val * Ty = yal * Ty: — yal Ga we ao wd. gee 7 eye (5.326) Ta = al * Tos = val Ts — yal aa me pu Sacer et Pa Pig 15.320) reVe = Toy = yal” Ty = 3al * Dy = nT o La idea de acoplar el miembro izquierdo de la ecuacién (15.29) con el derecho en los puntos de acoplamiento es semejante al concept de obtencién de momentos de Ia me~ Eanica, lo que explica el nombre del método que se estd exponiendo. Obsérvese en la fi- gura 15.19 que los puntos de acoplamiento y,, yz. -. » ¥y Se sitian en el centro de cada Scgmento, La ccuacion (15.32) puede expresarse en forma matricial como (2) = (Allo as.33) donde [B] = 470,V_| ” as.34) Ay AB + Aw oe te | ta-| ¢ a a j be te ee Arn = as Po : be (ol =| 2 : as. pw En a ecuacion (15.33), [p] es Ia matriz cuyos elementos se desconocen, Puede det arse (p] mediante is eciacidn (15.33) aplicande la regla de Cramer, Ia inversisn matt ‘To la técnica de eliminacin gaussiana, Por inversion matricial, f= arta) as37), donde [AT es la inversa de la matriz [A] Se deben tomar precauciones al evaluar los el mentos (0 autotérminos) de la diagonal de 1a matriz [A] de 1a ecuacién (15.32) 0 (15. Puesto que el slambre es conductor, en st superficie es de suponer una densidad de car 2a superficial ps. De ahi que en el centro de cada segmento, . 1 ae fe psa dd dy nb vicentroy 4, ’ Vicentro) = 224252 1m (2) (as.38) rey | oa) donde py, = 20 aps. Ass, los autotérminos (m= n) son Am = 210 (4) as.39) La ecuaci6n (15.33) se convierte entonces en. a) yee aS 2in(# ae a 1 Ta= al Pe 1 |=4meve|o] soy ’ PN 1 ee ae (8) yw = onl Low = yal @ ‘Si se emplea la ecuaci6n (15.37) con la (15.40) y se concede que Vj = 1 V.L = 1m, @ = 1mm y N = 10 (A = LIN), puede claborarse un programa de Matlab como el que se presenta en la figura 15.20, el cual se explica por sf solo. En él se invierte la matriz [4] y Se traza p, contra y. El diagrama resultante se muestra cn la figura 15.21. Fste programa determina asimismo la carga total en el alambre mediante o- fou asi) SGruncién quayranda expremireacen forma ifisarsta comes e- saa «as.42) Con Jos parametros seleccionados, el valor de la carga total fue de Q = 8.536 pC. Si se desea, el campo eléctrico en cualquier punto puede calcularse mediante pr dt a ee (15.43) Ia cual puede expresarse como MAR n-3 23 «as.44y donde R = IR| y Rar = @~xpat Om ya, + ~ ea, r= (x,y, 2) es el vector de posicién del punto de observacion y m= (rn Yas £4) el del pun- to fuente. ‘Para obtener Ia distribucion de carga de la figura 15.21 se adopts N = 10. Cabe supo- ner que un valor menor habria producido un resultado menos exacto, y un valor mayor Un resultado mas exacto, Sin embargo, un valor excesivo de N puede dar origen al pro- blema de tener que calcular la inversion de la matriz cuadrada (4) Asf, la capacidad de los recursos de esleulo que se tengan al alcance limita Ia exactitud dei experimento nemo mE Merooos wunecos BuO 1g Tm LINE CHARGE DENOTTY, RHO = ZNV(A}"B for 3a1eN eG 3) AG S)-DELTA/abe (8) “309 + DU. 3)=2-0+ta@ (DBEEA/AAD » = Anvtahs eum = gum + RHO(Z) (uo.01 Broe(e, RHO) : Raber (em), taper ene t (pc/m) ‘> ‘Figura 15.20. Programa de Matlab para calcular la distribuci6n de carga aval ciuaiien tet een a 15.4, MerOo OE MENENIE! m 68 Or 0203 08 05 06 07 08 09 4 ein) Figura 15.21. Diagrama de p, contra y. Emplee el método de momentos para hallar la capacitancia del capacitor de placas parale- Jas que aparece en la figura 15.22. Adopte a= 1m,b = 1m,d=1mye, = 1.0. Soluctén: Sea Ia diferencia de potencial entre las placas V, = 2 V, de manera que la placa superior P, se mantiene en +1 V y la placa inferior P, ch —1 V. Se desea determinar la densidad de carga superficial ps en las placas con objeto de hallar la carga total en cada una como o- J rsas Figura 15.22. Capacitor de placas paralelas para el ejemplo 13.5. 590, Meropos Numerics: Una vez conocida Q, la capacitancia puede caleularse como 2@_e2 3 c-2-2 Ya 2 Para determinar ps mediante el método de momentos, se divide P, en n sul AS, AS,,... AS» ¥ P, en n subsecciones: AS, . 1, AS, + 3, ---; ASzq. El potencial V; ceniro de una subseccion representativa AS; && i vi [pete SA ams |, Are ~ 7 tres J, Ry ~ Eom L Se ha supuesto una distribucién de carga uniforme en cada subsecci6n. La dltima cién puede expresarse como donde En consecuencia, Van Sy Ay m3 15.4. Meto00 Ge MoMENTOS! GBT Jo cual produce un conjunto de 2n ecuaciones simulténeas con 2n densidades de carga desconocidas p,. En forma matricial, Aa fe OT aes 2 An An arm fa] ofa fo =a Bans Aana 1+ anand Lean} 1-2 (Allp] = (2) Por tanto, el = (arm) donde [Bes la matriz en columna que define los potenciales y [4] la matriz cuadrada que contiene a los elementos A,. Para determinar A,, considérense las dos subsecelones # y J de la figura 15.23, las cuales podrfan encontrarse én diferentes placas o en una misma, ten mee LS donde Ry = [ay — 298 + 0, — yt + ZIT? Si, simplificando, suponemos que las subsecciones son cuadradas, wa AC =H oe pte demovtrar que | Oe amar, ' th Figura 15.25, Subsecciones ¢y / para el ejemplo 15.5. y au ae ae ego” Au = 26191 + V3) = SE 8814) I programa de Matlab incluido en la figura 15.24 se elabor6 con estas formulas, n= 9,€ — 2651 pF;con n= 16, C= 27.27 pF, y conn = 25, C — 27.74 pF. & BESTANGE.D, AND MAINTAINED AT 2 VOLT AND 1 VOU DrvipeD, - Rem) = pes (ma - 0.59) Figura 15.24, Programa de Matlab para el ejemplo 15.5. 15.4. MeroDO-DE MOMENTOS MM 693. aceon) = 5) Rom mare (CCL) =-K699)92 = (ECE) -KIS))IMB + (ZEEE) ABD AGEs) = DENBY (4. *DS*BOER) + eum = Sum + RMO(Z): OQ = sume cotn2y) Se ane to) 790; ter Figura 15.24. (Continuacién) 3.5. Método del elemento finito . Figura 15.25. Alambres conductores paralelos para el ejercicio 15.5. para el ejercicio 13'5 xm wn oa Sas os on El método del elemento finito (MEF) tiene st origen en el campo del andlisis estructural. No fue aplicado a problemas de electromagnetismo hasta 1968. Como el de las diferen= cius finitas, ef método del elemento finito es util para resolver ecuaciones-diferenciales. ‘Tal como se sefald en Ia secci6n 15.3, el método de las diferencias finitas representa 1a region de solucion con una red de puntos de cuadrfcula, de modo que su aplicacion se dificulta en problemas con fronteras de forma irregular. Estos problemas pueden mane~ El andlisis del elemento finito de un problema implica basicamente cuatro pasos: 4) discretizacion de la regiGn de soluci6n en un ntimero finito de subregiones o elementos, 5) deduccion de las ecuaciones que rigen a un elemento representative, ¢) reunion de todos los elementos en la region de solucién, d) resoluctén del sistema de ecuaciones obtenido. Az Discretizacién de los elementos finitos La region de solucion se divide en cierto nimero de elementos fintios, como se ilustrt ‘en Ia figura 15.26, donde la region se ha subdividido en cuatro elementos no empalmados entre sf (dos de ellos triangulares y dos cuadrildteros) y siete nodos, Se busca entonces 3 Véase PP Silvester y R. L. Ferrari, Finite Elements for Electrical Engineers, Cambridge University Press, Cambridge, 1983. Figura’ 15.26. Subdivision det clemento finito representative: i iNanerouenode «de un dominio irregular, D Nemero de elemento una aproximacién del potencial V, dentro de un elemento e y después se interrelacionan las distribuciones de potencial en los diversos clementos de tal forma que el potencial sea continuo a uno y otro lados de la frontera entre los elementos. La solucion aproximada de la region entera es ven = Sues cua donde N es cl mimero de elementos triangulares en los que se ha dividido la region de Sohucion. ‘La forma més comtin de aproximacion de V, dentro de un elemento es la aproxima- cién polinomial: Vice») = ab be bey (5.46) en el caso devun elemento triangular y Vice y) = a+ bx + ey + dry as.a7) en cl de un clemento cuadriltero. F potencial V, es en general de no cero dentro del ele- mento e, pero de cero fuera de e. Fs dificil aproximar la frontera de la regién de solucion con elementos cuadrildteros, ttiles en problemas con fronteras suficientemente regulares. En vista de ello, en el analisis de esta seccion usaremos elementos triangulares. NOtese que nuestro supuesto de variacion lineal del potencial dentro del elemento triangular, incluido en Ia ecuacin (15.46), equivale a suponer que el campo eléctrico es uniforme dentro del clemento: es decir, E, = -vv. Wa, + ca) as.48) B. Ectaciones que Figen’a los elementos Considérese el elemento triangular representativo que aparece en la figura 15.27. El po- tencial V.;, Va V.q en los nodos 1,2 ¥ 3, respectivamente, se obtiene mediante la ecua- Son 3.88), aac: Va tm n|fe : []-[; = >| [*] 5.49) Vea 1 x3 ysd Le. Los coeficientes a, b y ¢ se determinan a partir de la ecuacisn (15.49) como « Lox om] P Vea fé]-[: = 3] [2] @ss0) el AIRY La sustitucin de esta expresién en la ecuacién (15.46) resulta en a [Gays — ase), Gap Says) \ Gaps — 2901 Var ve=0 x vig | Or— Gs Gr) || va Gy Ge SG. x0 ILve Vie Seine oss donde ey Sh leas — 29) + Gs — x + Gs — DY] as.522) oe ee | cee ee y A es el rea del elemento 6; esto os, Be eo Baim [Li ad ¥4| Aa” ys es = ey) + Gay — ae) + Gas — 02), A= 12 (G2 = D089) — Gy — 4)O2 — 9) a5.s3) El valor de A es positive si los nodos se numeran en direcei6n opuesta a la de las mane- cillas del reioj (comenzando por cualquiera de ellos), como lo indica la flecha de Ia figu- ra 15.27. Cabe hacer notar que de la ecuacién (15.51) resulta el potencial en cualquier punto (x,y) dentro del elemento siempre que se conozca el potencial en los vertices, Esto conirasta con el andlisis de diferencias finitas, en el que sélo ke conace el potencial con los puntos de la cuadricula. Repérese asimismo en que a, son funciones de interpolacion lineal. Se les llama funciones de forma del elemento y poseen las propiedades siguientes: adm = {i tof «as.saay Figura 15.27. Elemento triangular represemtativ la numeracidn local de nodos 1-2-2 debe seguir la direceign contraria a la de las manevillas del rela}, ‘come lo indica la flechs Bees = 1 aas.saey En la figura 15.28 se ilustran, por ejemplo, ls funciones de forma ay ¥ a. La energfa por unidad dé Tongitud asociada con el slemento ¢ esta dada por Ia ecua- cion (4.96); 68 decir, 2 fattest 2 w, = 4 feteras = 3 felvv.ras as.s5) donde se ha supuesto una regién de solucion bidimensional sin carga (ps = 0). De acuer- do con la ecuacién (15.51), sin embargo, wv, = Siva ve asso) La sustitucién de la ecuacién (15.56) en Ia ccuacién (15.55) da como resultado 1 w. S Rovl f var vapas]y, oss eee ee eee oes cp = J Vay Va, ds as.58) deunelementotlangular @ 98 Meropos NuMEmicos Ia ecuacién (15.57) puede expresarse en forma matricial como We = $e (Va)? [CO] [Ve] as: donde el exponente T denota Ia trasposicion de In matriz, Ves = [¥] (15.602) Ves. y oe ce cg to -|cR ce ce (15.608) af ae oh. = La matriz [C0] es Ia mairiz de cocfictentes de los elementos. Ei elemento matricial CO, de la matriz de cocficientes puede considerarse como el acoplador entre los nodos 7 y j; su valor se obtiene de las ecuaciones (15.52) y (15.58). Por ejemplo, on = [ va,-vards ch ) = giglos — Gs — 0) # Gs — ener = an) fas (as.61ay = Fy lOr — ys — 9) + Ga = ea — 2) De igual manera, C8 = gig (02 — 95)? + Gea — 22)*] as.616) 2 1 ft = gy (02 901 — 92) + Ga — a) — 2] as.616) c= Bi loa — + Ge — a7) asia) 8 = Bi [Oa = 901 — 2) + Gr — Ga = 1 cas.1e) = Zo lo1 = 92)? He -— 9) casey Asimismo, i cach CH=CH ca-ce as61g) No obstante; nuestros célculos se facilitarfan si definimos Pom G27. Pa = Os — Ke Pa = 01 9D (15.62) A= —. 22 &-W. A-w&ow Dados P,y Q, (¢ = 1,2,3 son los ntimeros locales de nodos), cada término de la matriz de coeficientes de los elementos se determina de esta mancr 1 oP AP) + 20))| (45.626) 3 Leno. P22) | (5.626) donde Notese que P, + Py + Py = 0 = O, + O, + Qyy, por tanto, by p= 0- > cH. Esto puede servirnos para comprobar nuestros célculos. C. Reunién de tédos los elementos ‘Habiendo considerado un elemento representativo, el paso siguiente es reunir todos los elementos en la region de solucidn. La energia asociada con Ia reunion de todos los ele- mentos en Ia malla es w= Sw. ~ devel iv) «a5.63) donde vs Ve hte ase) Yi y mes el ntimero de nodos, N el mimero de elementos y [C]la matriz de coeficientes global 0 general, en la que se conjuntan las matrices de coeficientes de los elementos particulares. ‘Ahora el principal problema es obtener [C]a partir de [CO] El proceso de agrupacion de las matrices de coeficientes de los elementos particul res en la matriz de coeficientes global se comprendera mejor con un ejemplo. Considére- se Ia mala de elementos finitos integrada por tres clementos finitos que se presenta en. Ia figura 15.29. Obsérvese Ia numeracion de los nodos. La numeracin 1,2, 3, 4y 5 es la numeracién global, mientras que la nuimeracion ij-k es Ia numeracion local, correspon- Gionte a la nameracién 1-2°3 del elemento de la figura 15.27. Con referencia al clatmeine©. Figura 15.29. Reunién de tres elementos: -/-« Comresponde a la numeracion local 1-2-3 det 3 de Ia figura 15.29, por ejemplo, la numeraciGn global 3-5-4 corresponde a la numeracion, Jocal 1-253 del elemento de la figura 15.27. Adviértase que la numeracién local debe se fiir una secuencia de direccion contraria a la de las manecillas del reloj a partir de cual- Biicr node del elemento. En cusnto al elemento 3, por ejemplo, podria elegirse 4-3-5 9 SES cn lugar de 3-5-4 en correspondencia con 1-2-3 del elemento de la figura 15.27. Ast, fa mumeracion de la figura 15.29 no es Unica, y cualquier numeracién que se emplee Uerivard siempre en la misma (C} Si se adopta la numeracién de Ia figura 15.29, es de SGponer que la matriz de coeficientes global ser de la forma Cu Ca Cx Cu Cs Cu Gm Cas Cos. Cs Cu Ca Ca Cu Css Ca Ca Cas Cae Cos Fu Cn Ca Cu Css tel as.65) matriz de 5 x 5 puesto que estén implicados cinco nodos (n = 5). También esta ver Cy es, UE Nloplador entre los nodos / yj. C, se obtiene con base en que la distribucién de po- fencial debe ser continua a una y offo lado de la frontera entre los elementos. La con- {Hbucion« la posicion fj en [C] procede de todos los elementos que contienen nodos i Ji para hallar C,,, por ejemplo, se observa en la figura 15.29 que el nodo global 1 pertene- Udatioe clementos 1 y 2 y #3 el nodo local 1 en ambos; por tanto, Cy = CR + CR s.662) En cuanto a Cy, el nodo global 2 s6lo pertenece al elemento 1 y es igual al nodo local 5; por tanto, a= C® 15.660) En cuanto a Cy, el nodo global 4 equivale a los nodos locales 2, 3 y 3 de los elementos 1, 2y 3, respectivamente: asi, Cu = CN + H+ CB a5.66¢) En cuanto a C,y/el Vinculo global 14 equivale #168 vinculos locales 12 y 13 de los clemen- tos 1'y 2, respectivamente; en consecuencia, Cu = CR + CR «as.66a) Puesto qué no hay acoplamiento (o Vineulacién directa) entre los nedos 2 y 3, Cat Ga = 0 as.66e) x liendo este procedimicnto mediante la inspeccién de la figura 15.29, se obtienen to- dos los términos de la matriz de cocficientes global, en esta forma: RR cy ca Qieg 0 ce cee 2) o {c] =| cH Oo -ce+ ce CR + CR ‘ce (5.67) +P Qi Oe ay cyree rey cy ° Oe a a Obsérvese que en nodos compartidos se empalman matrices de coeficientes y que la ma- triz de coeficientes global [C] contiene 27 términos (nueve por cada elemento). Vale des tacar asimismo las propiedades siguientes de la matriz [C} 4. Es simétrica (C, = C,), al igual que la matriz de coeficientes de los elementos. ‘no existe acoplador entre los nodos # yj, es evidente que, con relaci6n a gran ndmero de elementos, [C]se convierte en una matriz escass nda. 3. Es singular. Aunque esto no es del todo obyio, puede demostrarse mediante la matriz de coeficientes de los clementos de la ecuacién (15.608), D. Resolucién de las ecuaciones resultantes Sabemos por el célculo de variaciones que la ecuacién de Laplace (o la de Poisson) se Satisface Cuando la energia total en la region de solucion es minima. Fs preciso entonces: que las derivadas parciales de W respecto de cada valor nodal del potencial sean de coro: es decir, ow aw ew Oa sale av, aw ave m:0,)" -oheas tp as.68) O en la malla de elementos finitos de la figura 15.29, por ejemplo, Ia ccuacién (15.65) se sustituye en la ecuacidn (15.63) y se obliene Ia derivada parcial de W respecto de V,. Asi se obtiene aw oe a QC + Vala + VaCis + Vili + Vas + V2Ca + Vala + Vilas + Ven, O= Vien + Van + Vig + Vie + Vis (5.69) En general, 2W/0V,= O-conduce a o~ Sven as donde 7 es el ntimero de nodos en Ia malla. Al expresar la ecuacién (15.70) para todos Jos nodos k = 1,2, -...,8e obtiene un Conjunto de ecuaciones simultaneas a partir de las cuales es posible hailar la solucion de [(VJ" = [V, V2... ». Val. Esto puede hacerse de dos maneras, similares i Ins empleadas para resolver ecuaciones de diferencias finitas obten} das de la ecuacion de Laplace (0 de Poisson). MEroDO OF FTERACION. Este método es semejante al que se. utiliz6 en el MDF. Supongamos que el node 1 de la figurw 13.29 cy un node Line 21 poteusial cn exe nude puede ulteuciee de he Scunelon (5.69) como fee Wand: Rveu asm as.72) Esto se aplica iterativamente a todos los nodos libres de la malla den nodos. Puesto que C,, ~ Osi el nodo k no esta directamente conectado con el nodo #, slo los nodos direc. tamente vinculados con el nodo & contribuyen a V, en la ecuacién (15.72). ‘ASf, Si se conoce el potencial en los nodos vinctlados con el nodo K, es posible deter- minar V, mediante la ecuaci6n (15.72). El proceso de iteracién comienza asignando al potencial en los nodos libres un valor de cero o el valor del poteneial promedio. Viste = "12 (Vines + Vines) as.73) donde Ving ¥ Vmax S07 los valores minimo y maximo del potencial prescrito en los nodos: jos. El potencial en los nodos libres se calcula mediante Ia ecuacion (15.72) a partir de tales valores iniciales. Habiendo calculado el nuevo valor de todos los nodos libres al fi- nal de la primera iteracién, ese valor se convierte en el valor inicial de Ia segunda itera- Cidn. El procedimiento se fepite hasta que el cambio entre iteraciones subsecuentes se vuelve insipnificante. Meron DE LA MATIZ EN BANDA Sise_numeran primero todos los nodos (Gs.63) puede expresarse como libres y después los nodos fijos, la ecuacién wetaw, vale S[%] astg ont 15.5. Méro00 on. aeneaTO mE mm 703: donde los subindices f'y p se reficren a nodos con potencial libre y fijo (o prescrito), res- pectivamente. Puesto que V, es constante (consta de valores fijos conocidos), s6lo se di- ferencia respecto de V, de modo que la aplicacién de In ccuscién (15.68) a la ecuacién (5.74) produce CM p+ Cy¥p = 0 [eqlt¥A = “1E_1¥ 1 a5.75) Esta ecuacién puede expresarse como [Aly] = 1) a5.76a) [m= ] (5.766) donde [¥] = [VA [4] = [Cpl y [8] = [Cp] [Vp]. Puesto que, en general, [A] no es singu- lar, el potencial en los nodos libres puede hallarse mediante la ecuacién (15.75). [V] puede despejarse en la ecuacién (15.76a) con Ia técnica de climinacin gaussiana 0 en Ia ecuaci6n (15.766) mediante la inversion matricial si la matriz por invertir no es muy grande. Notese que de la ecuacién (15.55) cn adelante nuestra solucisn se ha restringido aun problema bidimensional que implica a la ecuacién de Laplace, VV ~ 0. Sin embargo, los conceptos basicos desarrollados en esta seccién pueden prolongarse al andlisis de cle- mentos finitos de problemas que impliquen Ia ecuacion de Poisson (V?V — —p,/e, VA — 3) 0 le councion de onda (Wy ya — 0). El sutilinis de elementos finltos envrana et uso de gran cantidad de memoria de una computadora para almacenar los elementos de Ia matriz, asf como la dedicacién de mucho tiempo a esa tarea. No obstante, se dispone ya de varios algoritmos que aligeran en cierta medida este inconveniente. El MEF ticne varias ventajas sobre el MDF y el MM. Primero, 8 apto para regiones de solucién complejas. Segundo, su generalidad permite claborar un programa multiusos ara la resolucion de una extensa gama de problemas. Un solo programa puede servir para Fesolver distintos problemas (descritos por las mismas ecuaciones diferenciales parciales) con diferentes regiones de solucién y condiciones en la frontera; asi, lo unico que varia son los datos de entrada. Sin embargo, el MEF no esta libre de contratiempos. Es mas dificil de comprender y programar que el MDF y el MM. Impone asimismo el a veces tedioso proceso de preparacion de los datos de entrada, Considere la malla de dos elementos que aparece en Ia figura 15.30(a). Con base en el método del elemento finito, determine el potencial dentro de la malla. . Figura 15.30. Para cl ejemplo 15.6: (a) malla de'dos elementos, (b) numeracion local y bau 20 plobal de los elemento. TP @a. te Gata e121 227, Solucts: Las matrices de coeficientes de los elementos pueden calcularse mediante Ia ecuacion (5.62). En cuanto al elemento 1, compuesto por nodos 1-2-4 correspondientes a la nus meracién local 1-2-3, como se indica en la figura 15.30(b), PH-13, P09, P= 04 Q)=-02, Q,=-04, 2, = 06 A= 12 @54 + 0.16) = 0.35 La sustitucién de estos valores en la ecuaci6n (15.626) da como resultado 1.236 —0.7786 —0.4571 [c%) ~| 07786 0.6929 0.0857 0.4571 0.0857 0.3714 ase En cuanto al elemento 2, de igual manera, integrado por nodos 2-3-4 correspondientes Ia numeracion local 1-2-3, como se indica en la figura 15.30(b), Py=-06, FP, =13, Py = -0.7 Q=-09, G,=02, Q,=07 A = 12 (0.91 + 0.14) = 0.525 Por tanto, ® bs « 05371 0.4571 0.1 | fe™] =| 04571 08238-03667 05.62) =o1 -0.3667 0.4667 La aplicacion de la ecuacién (15.75) resulta en. Cn Cul[Ve]__ [Ca Ca] [vs lez exlll--le el] assay Esto puede expresarse en forma mas conveniente como 210-0. TPs aio 0) 0 cy 0 Ca}, | fea —ce [Pvp oo 10 [wl pro 1 tlys (eens Bc, 0 cad Lvs. -Cu -Ce. rise (15.6.4) Cx = CY + CH = 0.6929 + 0.5571 = 1.25 Ca = Cay = C® + CH 0.0857 — 0.1 ~ —0.0143 CY + CR = 03714 + 0.4667 = 0.8381 Ga = OR = 0.780 Cay = C® = —0.4571; Cy = CY = 0.4571 Cua = CB = —0.3067 Notese que en la matriz de coeficientes de los elementos seguimos la numeracién local, y.cn Ia matriz de coeficientes global la numeracion global Ast la matri¢ cuadrads [C) se Sbtiene como i o oo t>. jeyafo° 125 0 =o01a3-}— yore a tae. 0. “Oey 45.65) 0 -00143 0 ose yy la mateia [B]}dal mlombro derecho de Ia ecuacién (15.6.4a) como, a ° tay= | 4372 ass6) S007 Hacer: inom tf SER SED RRA ° 3.708 10.0 4.438 [Iv] = [ert] Asf, Vj =10, Vz = 3.708, Vg = 10 ¥ Vi = 4.438.,Una ver conocidos los valores del poten- cial en los nodos, mediante la ecuacidn (15.51) puede determinarse el potencial en cual quier punto dentro de la mala. Ejercicio 15.6 Calcule ta matriz dé cocticientes global de la malla de dos elementos que aparece: : 13.91 Suanaore) ctnods lente Sana “feed PBs econ Wine Ch ha igure 15 51fe), by ctrnode 2 exta vinculade sto 4 bom numeracion local como ne indica ona figura 13.3100). | i cor PF 9.9964 005, 0246s 08 oon a7 8% Om Respuesta: @) |, .0.2464° —0.75) 1.5964 -0.6 fe" ioe be 48 ° 2=097777 00 —1.056 ce Omio3 098 o.a3K6 098 208-008 : -Oasae 108177 Nodot: @.1)__ Node 3: @.2.4) Figura 15.31, Para el ejercicio 15.6. jo Neo 2: (G2). Node 4213, 16)> °F ? ry ry [Bemple 15.7 Escriba un programa para resolver la ecuacién de Laplace mediante el método del clemen- to finito. Aplique el programa al problema bidimensional que se presenta en In figara 15.32(a). Solucién: La-region de soluci6n se divide en 25 elementos triangulares de tres nodos, de Io que re- sulta un ndmero total de 21 nodos, como se indica en la figura 1532(b). Este es un paso necesario para disponer de datos de entrada que definan la geometria del problema. Con fundamento en lo expuesto en la seccion 15.5, en In figura 15.33 se presenta un programma general de Matlab para la resolucion de problemas que impliquen la ecuacion de Laplace x Figurn 15.32. Para el ejemplo 15.7: o | } & viz) = pormerzaL ar Nope 7 : MATRICES P(X) AND Q (1) ARE DEFINED IN EQ. (15.62—) SPEND LOCAL COORDINATES XU(3), YE(S) FOR ELEM T om RECz, (20339) Pa) = m2) - ee, ec) = ua) = EC) 15.33, Programa de computacién para el ejemplo 15.7. Figu 9) DETERNEIE COmFRICrENT MATRIX FOR ELEMENT T Cea(B+R! + 020'1/ (4. 0*AREA) + for gaa:3 BGR) = vAEGO: (2,3) am B(x) se Cre = Meropo ort FLEMENTS MTo _709 % aHiRD Srep - SOLVE THe syenm™ OF mQUATIONS ere: © tuimpys Figura 15.33. (Continuacton.) con elementos triangulares de tres nodos. La elaboracion de este programa implies basi- camente los cuatro pasos indicados en la figura, los que se detallan a continuacién, Paso 1. Se introducen los datos que definen el problema, Este es el Gnico paso que depen- de de Ia geometria del problema. Mediante un archivo de datos, se introduce el ntimero de Glementos, el nmero de nodos, el nimero de nodos fijos, los valores prescritos del potencial fen los nodos fijos, las coordenadas x yy de todos los nodos y una lista de identificaciGn de los nodos pertenecientes a cada elemento en el orden de la numeraci6n local 1-2-3. En el caso del problema de la figura 15.32, en las tablas 15.3, 15.4 y 15.5 se presentan los tres conjuntos de datos de coordenadas, relacin elementos-nodos ¥ poteneial prescrito en los nodos fijos, respectivamente. Sola mala se elementos finite de “ Nodo xy Modo xy 3 0s oo iS as of 6 %o oo i 62 os & 92 03 15 00 os Hlememo nim. 1-2? = _ Elemento nd 1 2 3 3 Ag al ” 2 a: ie 3 ee 18 a. 12-6 é a 9 8 » Bou oY é So fo a is aa ° s ‘6 ou 2 ie 48 10 7 8 DB B % ‘Tabla 15.5. Potencial preserita en-nodos flo. " < 2 ome Potencial " Fotencia! : | Node nim: prescrito Node nd. preserito = ¢ 2 oo 20 1009, | 2 oo 2 S00, 4 60 io ‘30, | 0 16 oo ° 309, 2 80 ‘Tabla 15.6. Datos de entrada para el prozrama de elementos =~ in | gb a Ping Bana = oro 0.0" 8.8)1 } Paso 2. Este paso entrafia la determinaciGn de la matriz de coeficientes de los elementos [CW] de cada elemento y de la matriz de coeficientes global [C]. Se aplica el procedimicn. fo que se explicé en el ejemplo anterior. La ecuacién (15.6.4) puede expresarse en forma, (2, ][%] tam — (1 En esta etapa se calculan tanto Ia matriz “global” [C] como In matriz. (4 aso 5. Se invierte la matniz global obtemida en el paso anterior. Los valores del poten- cial en todos los nodos se obtienen mediante multiplicacién de matrices, como se indica en la ecuacin (15.76b). En lugar de invertir la matriz. global es posible despejar cl poten cial en los nodos con la técnica de eliminacién gaussiana. Paso 4. Se extrac el resultado de los ealculos. ‘Los datos de entrada y salida se presentan en las tablas 15.6 y 15.7, respectivamente. “abla 15.7, Datos de salida de! 1 000-000 0.000 3 0m 020 “o.000 140400 tm 13 bso © 40100000 16 B00 8.800.000, 21.00 ton "$0.00, Resumen Figura 15.34, Para el ejercicio 15.7. Rjercicio 15.7 wr a aay | elemento finito. Divida Ia region Repils el ejemplo 15.3 aplicando el métod ‘de solucion en elementos triangulares, como se muestra 15.34. Compare ia solucién con la obtental ofa eae a fas diferen- as finitas € Respuesta: Véase el ejemplo 15.3. 1. Lineas de campos eléctricos y lineas equipotenciales debidas a fuentes de puntos coplanares pueden trazarse siguiendo la técnica numérica que se presents en es te capitulo. El concepto basico puede prolongarse al trazado de Iineas de campo magnético. 2. Un problema clectromagnético en forma de ecuacién diferencial parcial puede re- solverse mediante el método de las diferencias finitas. La ecuacion de diferencias finitas que aproxima la ecuacin diferencial se aplica en puntos de cuadricula es paciados de modo ordenado sobre la regin de solucién entera. La cantidad de Campos en los puntos libres se determina empleando el métode mds apropiado. 3. Un problema électromagnético en forma de ecuacién integral se resuelve facil” mente mediante el método de momentos. La cantidad desconocida dentro del signa. de integral se determina igualando ambos miembros de la ccuacién integral en un niimero finito de puntos en el dominio de la cantidad, 4. Mientras que el método de las diferencias finitas se restringe a problemas con regio nes de solucion de forma regular, el método del elemento finite es apto para proble- mas de geometria compleja. Este método implica dividir la region de solucion en. elementos finitos, deducir las ecuaciones para un elemento representativo, reunir todos los elementos en Ia regién y resolver el sistema de ecuaciones resultante. En este capitulo se expusicron ejemplos ilustrativos de la aplicacién de cada método a problemas practices y, cuando fue necesario, se proporcionaron programas de compu tacion para la resolucion de tales problemas. 15.1. En el punto (12,0) de un campo eléctrico debido a cargas puntuales coplanares, se tiene E03 a, — 0.1'm, Vim. Un despiazamionto difercncial de 0.05 m sobre una lines equipo tencial en ese punto conducira al punto 4a) (1.04,2.03,0) 2) (0.96, 1.97.0) ©) G.04,197.0) 4) (0.98,2.03.0) 152. ;Cuél de Ins siguientes no es una aproximacién correcta de diferencias finitas de dVide en, Soein = Ane Vie +) = Vera) a # vi Vor =m) sieetenren wy Vets h) = Vers = 1) Viz +h) = Vee = bh ay Veet MV o Viet Wa) = Visa ba) 153. El elemento wiangular que aparece en la figura 15.35 se encuentra en el vacfo. El valor aprox @ 10 B) 75 o3v a ov 154, Para efectos de anslisis de diferencias finitas, una placa rectangular de 10 por 20 em se divi de en ocho subregiones mediante lineas paralelas a los extremos de la placa separadas 5 em. ‘entre st. {Cudntos nodes libres hay si los extremos estan conectados a alguna fuente? a) 15 By 2 29 ao 23 ea ev Figura 15:35. Para las preguntas de repaso 15.3 y 15.10. Oi eo 45.5. En la ecuacién de diferencias V, = Vy 1+ Vas 1 608 Ve = Vs = 1y comenzando con los ‘Va despues'ae la tereera teraction es 1a) Es densa (es decir, contiene muchos términos diferentes de cero). b) Bsté.en banda. ©) Es cuadrada y simétrica. 4) Depende de Ia geometria del problema dado. toes que 4) La soluci6n en uno de ellos resulta on una matriz excasa. ) La solucion en uno de ellos es conocida en todos los puntos del dominio, ©) Uno de ellos se aplica a Ia resolucign de ecuaciones diferenciales parciales 4) Uno de ellos se limita a problemas sin variacién en el tiempo. 15.8. Sila placade la pregunta de repaso 15.4 se discretiza para el andlisis del elemento finito de manera que se tenga el mismo numero de puntos de cuadricula, ;cuantes elementos trian Bulares resuttan? a) 32 ») 16 12 ao 159. _,Cudl de los enunciados siguientes sobre funciones de forma no es cierto? @) Son de naturaleza interpolatoria. 5) Deben ser continuns en todo el elemento. ¢) Su suma es idéntica @ ta unidad en cualquier punto dentro del elemento. 4) La fanci6n de forma asociada con un node dado tiende a cero en cualquier otra nado. @) La funeidn de forma ascciada con un nodo equivale a cere en'see nodo. 15.10. El érea del elemento de Ia figura 15.35 es ® » ° a orn Respuestas: 15.14, 15.26415.3a, 15.4e, 15.5¢, 15.66, 15.7a, 15.86, 15.9¢, 15.10d. 15. Con base yu tee. on ol progsams deectito on oli jernplo 15-1 0 ca on oSiligo oquiyslente alg borade por usted, tract las lneas de campo eléctrico y Mncas equipotenctales de los casos si guientes: 4) Tres cargas puntuales de —1,2y 1 C ubicadas en (—1,0), (0,2) y (1,0), respectivaments, 5) Cinco cargas puntuales idénticas de 1 C situadas en (—1,~1);(—1)1),, 1.) ¥ ©. 0), respect o=x=1 Sujets a ¥(0) = 0. ¥(1) = 10. ute el método (iterative) de las diferencias finitas para hallar (0.23), Adopte A'0,25 y realice cinco iteraciones 153. ab Qhimendedestericain inhi, 29 Se eee — 3 a) se i av y OY eax = 0.5. ‘V__ 1007 1505620134 25261-30452 2) Los datos de Ia tabla anterior se han obtenide de V ~ 10 senh x. Compare ol resultado, que obtuve en el inciso a) con los valores exactos, [Ca formula del inciso a) es una formula de diferencias hacia delante, la del inciso 6) una f6rmula de diferencins hacia atras ¥ las de los incisos d) y e) formulas de diferencias centrales, Promenas mt 717 Figura 15.36. Cundricula de diferencias finitas en ‘coordenadas cilindrieas: para el problema 153, ASA. Demusstre que la ccuacién de diferencias futitan parm lw ecuncion de Laplace en coordena. das cilindricas, V— Vip, 2). 68 4 Views) 2 [Vewse +0) -+ Vinee m+ (14) v's dwar (3 A)vatne donde h = Ar = Ap. 45.5. Con base en la representacién de diferencias finitas en coordenadas cilindricas (p, b) en tun punto de cuadricula que se muestra en In figura 15.30, concediendo que pm Spy =n Ad de manera que Vip, Pip = VOndp,. ndd) = Vi. demucstre que eels | (1) vee (a4 ea, alae ava sve] 1 i Zetia Jon coats ins do langue condor Gundrado ve mandeoe on -10,0,30 FBT Baraca a poate cones dol tnaaaee 415.7. Siga el MDF para calcular el potencial en los nodos 1 y 2 del sistema de potencial que apa- rece en el figura 13.37. 30v Figura 15.37. Para el problema 15.7. 415.10. asa. sow? — yew Fiyura 15.38. Para el problema 15.9. “OD: Senin =m ye 6; donde A os el tama do ‘Repita el problema 15.7 si ps 1a malta. ‘Considere ol sistema de potencial que se presenta en la figura 15.38. a) Asigne alos nodos libres un valor de cero y caleule el potencial en ellos en cinco iteraciones. b) Resuelva este mismo problema con el métede de ia matriz en banda y compare el nuevo resultado con el que obtuvo en el incino 4). _Aplique la técnica de matriz en banda para establecer un sistema de ecuaciones simulténeas de diferencias para cada tino de lox problemas representados en la figura 15.39. Obtenga las ‘matrices [Aly [#1 4) 2Oué modificaciones harfa a las matrices [4] y [] del ejemplo 15.3 si la regién de solu- Sion taviera una densidad de carga py? +) Exeriba un programa para despejar el potencial en los puntos de cuadrfeula que se ius tran en Ia figura 15.0 suponiondo una densidad de carga py — xy — 1) nClm= Use el método iterative de Ins diferencias finitas y adopte ®, — 1.0. ele asv @ © alice aid, Micon th creciitinaits tine. asa. 15.16. Figura 15.40, Para el problema 15.11. ‘La eouacion de onda bidimensional est dada por Lee _ oe, ee ; fot ~ an tae Concediendo que P/q., denota la aproximiscion de ditercincias finitas de Pan tf). de inucstre que el slstoms de diferencias fnitas para Ia ceuacion de onda es Phh A 20h — PLEA (Phornt Chynt.n— 2 Phan) + (hart + Ohnr = 2h.) donde h = Ax = Az y a = (ean). Escriba un programa en el que se emplee el sistema de diferencias finitas para resolver la ‘ccuncion de onda unidimensional av av Seu S¥ oszs to dads tas condiciones en Ia, frontera V(O, 1) = 0, V1, 4) = 0, ¢ > 0 y Ia condicisn inicial ‘2Via0 (x, 0) = 0. VGr, 0) = sen mx,0 21 _ Ge + Jy) | Gen — J) _ 882 + re Gat iy) Gin Im) +d Bateww tye Ma = VET iy = Vr? = Vr 02 Bs (ek iy)" = em = 2" Ln (m = entero) 2" = (e+ fy) = AM eh =P" Join + ImkIn (k = 0.1.2... wt — 1) In (ro) = In + Ine” = In + 10+ Rk (k= entero) A.3. Funciones hiperbélicas e us sen x = SS, cosh x = senh x tanh x = SS8R%, con x = ot esch x = sech x = 4 sen jr = jsenh x, cos jx = cosh x senh jx = sen x, cosh jx = cos x senh (x + y) = senh x cosh y + cosh x senh » cosh (x + y) = cosh x cosh y = senh x senh y senh (x jy) = senh x cos yj cosh x sen y cosh (x = jy) = cosh x cos y = j senh x sen y tanh (x + fy) = eae ‘cosh 2x + cos 2y */ cosh 2x + cos2y cosh? x — senh? x = 1 sech?x + tanh?x = 1 sen (x jy) = sen x cosh y + j cos x senh y cos (x * jy) = cos x cosh » = jsen.x sen y we Armiorce A 4. Identidades logaritmicas log xy = logx + log y log = logx — lon y log x" = n log x logig ¥ = log x (logaritmos comunes) Jog, x = In x (logaritmos naturales) Si fel ain G by a 5. Identidades exponenciales . i ot esieet Sees donde ¢ = 2.7182 werner fen em Inew=x .6. Aproximaciones de can‘ jades pequefias Stix <1, GQorm1sne enute ma+onx send Ex 0 iim 8% 1 cose = 1 tanx =x A.7. Derivadas SiU = UG), V = VG) ya = constante, Bor ave @ sor me mn ~ coor feu = eau Beanv = sooty sean u = coon or a aw ieosnu = senny $2 a ay ww fiuanu = set 3. Integrales indefinidas $7=U0),V = VEO ¥ 4 = conan, [uammetc Jeav=uv—[vav megracion por partes) mote +, met cae fGews = a forae e+ 8 amoaes [eanaere [orarnleve [retax-Ger-yte J eer ax = Flee - 20x +2) + [mxav-xmx—x+e [sen axax = —Leos ax + [cosaxax=Lsenax +e ftenax a? Linecdaxrncrt tmcosan + 6 1 [escarai't Di (eecad Hanan) +c sentax dx = 5 cost ax dx = 5 4 282 sen arte = 4, (oon ax — ax conan) +6 weonar dem few ax gaspen as) 3G teen ba dn = yoy (aon be — boob) + 6 acon bx dns a obo pet G sen(a~ b)= _ sen a x fica Seeiabde = we wee ab) — arb) sctlatega tnt: => SSG — Es ote be oe 2a =>) 2a + BY sen (a b)x | sen(a + bx en pte tee (feceree onan 1 pothenaz Lennart 1 osname teense iii ces iabcin aged ax ew de ere eax wat 4 ~ fing +e) +0 att Zac ee Io (ee e>a oe |i es dmtttec, 20 [rte eritve J eer EEA | Bee Ve rece Sa@tae- sete Sete - vere? om (M2E# 45) [ata Septem Geter a)re Ao. - Integrales definidas (Cosnmennneae (eomecainees= {Sp TIT : ee [Conn mecon mein (am OTe men [P son murson nade = [" senmerenneas = {2 men A) — (A+ AYA BD 25.72 a) 7-681 b) —2a, — 5a, ©) 137.43" d) 11.022 ©) 17.309 4) Comprobacién 1b) cos 6, cos 6, + sen ®, Sen 82, cos 0, cos 0: — © |sen a) 103 6) —2.175a, + L631a, — 4.8938, ©) —0.175a, + 0.6318, — 1.893m, sen 0, sen 0, Capitulo 2 27. 29. 2a. 243. 247. 2a9. a) P(O5, 0.866, 2) » °) 1.061, 2.121) @) T(.464,2,0) @) pz cos d — pt sen d cos + pz send 3) PQ Sen? 8 sen? & + cos 8) o) aptgton, + tad, (conto + S2222)a, + ono (cone —4)ae 2) Eom + ems ren a, » (G+ et Se - n e TET VaT Tae Comprobacion ae a + ym, by ea.3 1b) r(sen? 0 cos sb + r cos? @ sen a, +r sen 0 cos 0 (cos ¢ — rcos 0 sen d) my, 3 4) rsen 0 [sen @ cos 0 (rsen 0 + cos) a, + send (r cos! @— sen 0 cos b) a, +3 cos ag) Sa, — 21.218, » VIF (om + Pos @) Una linea infinita paralela al eje z %) Punto 2, —1, 10) ©) Un cirenio de radio'r'sef 0 = 5, es decir, la interseccién de un cono y una esfera ‘@) Una Kinea infinita paralela al eje z ) Una linea semiinfinita paralela al plano x-y P) Un semicireulo de radio 5 en el plano x-y @) 3,9, + 7a, ) 143,26 ©) ~ 8.789 gaet ex). 44720; 4 080440, + 2.2360, 107 m/s 10.23. 0.1203 mm, 0.126 2 1025, 294% 10-8 m 10.27. q) 131.6 0 5) 0.1184 cos? (2m 10% — 6x)m, Wim? ©) 03535 W 10.29. a) 2.828 x 10% rads, 9-225 + 0.3183 cos (10% — &x)a, Am sen (at — 2z)a, A/m 10.31. 1b) =10 cos (oot + 2), Vim, 26. 10.33. 26.038 10-° Fim 10.35. a) 0.5 < 10% rad/m. a2 ©) ~26.53 cos (0.5 = 10% + z)a, mA/m 4) 1.0618, Wim? 0.97. (4) 6.283 m, 3 > 107 rad/o,7.32 cos (wt — c)ay Vion 4) =0.0265 cos (wt — z)a, A/m ©) 0.268, 0.732 @) Ey — 10 cos (wr ~ 2)a, ~ 2.68 cos (we + 2), Vien, HE, = 7.32 0s (wt — za, Vim, Piprom = 0-231, Wim? Prpcom = O23, Win 10.39. Vedse'ia figura C2 9 cos (wr + x)a, mam 10.41. Comprobacion, H, — {/2° [k, sen (kex) sen (Kye + Ke costkec)€08 (Kya) 10.43. ) 36.87° 2) 79.58, + 106.18, mWia* ©) (15188, + 2.0248.) sen (wt + dy — 32) Vim, (1.877A, — 5.968,) sen (wr 9.538 ~ 32) Vim 10.48. g) 15% 10" rad/s 2) (~8a, + 6a, — Sa,) sen (15 x 10% + 3x + 4y) Vind Anovoicee 751 Fizugs C2. Para el problema 10.39; curva n correspond as "778," ~ 0, 1,2, Capitulo 11 11-1. 0.0104 O/m, 50.26 nH/m, 221 pF/m, 0 Sim 113. Comprobacion WLS.) 13.34 /=36.24° , 2.148 < 107 m/s 5) 1.606 mm 798.3 rad/m, 3.542 % 107 m/s Comprobacion Sa) OA112, 2397 2) 34.63 7-40.65" 0 1119. 0.2740" A 121 @) 46.87 0 3) 48.30V. 11.23. Comprobacién 41125. 10.2 + f13.80, 0.7222/154" 6.2 Bs 075 0 70.7 41.29. 0135 + 70.24 131. g) 125 MHz b) 2+ jo ©) 044/120" 11.33. 4) 35+ j34.0 by O75 2 411.35. a) 24.5.0 b) 55.33 0, 67.74. 10.25 W 20 + f15 mS, —j10 mS, ~6.408 + /5.189 mS, 20 + /15 mS, /10 ms, 2.461 + j5.691 mS a) 3424 p14 &) 0.38, 0.4730, ©) 265 11.43. 4,0.6 /—90", 27.6 — 52.8.0 2A, 1.764 GHz, 0.357 /=44.5°, 70 — 740.0 411.47. Véase la figura C3 41.49. Véase la figura C4 151. a) 77.77 0.18 %) 0.223 dB/m, 4.974 dB/m ©) 3.848 m 11.53. 9.1122 = Z, <21.03.0 veo.9 yay Figura €.3. Para el problema 11.47. 10.9 18) q va.o oo ssiore 75.026 ° 7 + $ t $ Fie a 10.9m0 + 33 —o 00.17 ° 7 + $ + $ oe Figura C4, Para el problema 11.49. Capitulo 12 Comprobacion 12.3. a) Véase Ia tabla C1 2) ners = 573.83 O. mers = 3.058 0 ©) 3.096 x 10? m/s 4) No ») St 430 ns 375.1 9,0.8347 W a) eT &) j400.7/m. ©) 9853.0 @) Comprobacién 'b) 4.06 x 10° m/s, 2.023 cm, 5.669 < 10* m/s, 2.834 cm 1 SS Antwoice © 1245. a) 110% b) 0.8381 12.47. 4.917 12.19. “ 12.21, 0.04637 Np/m, 4.811 m 1225. a) 2163 © 10 Nplm 3 Mats = 10-9 Np 1225. Gomprobacién 1227. Comprobaciom: — fx (2) (2%) sy sen (™#) cos ("22") con (22) 1229. @) eT 0) Mino ©) eT yo 12.31. Véase la tabla C2 Modo f, (He) Sto 3335 io 38 12.33. a) 6.629 GHz b) 6.387 12.35. 2.5 (sen 30mx cos 30mya, + cos 30mx sen 30mya,) sen 6 10° re) Capitulo 13 isa, 13.5. 137. 139. 13.11. 43.43. 13.15, 13.17. 13.19. Avevorce © om 755 SAP en (oot — ir) (—sen dag + €08 6 cos dra,) Vim 228 sen (et — Br) (sen dag + cos 6 cos bay) A/m. 94,25 mV/m, j0.25 mA/m 1.974 9 2RAT A ple! sen 0 Big, = PE ee Hg, = Ew 1s 2) 0.9071 wA b) 25 nw ‘Vease la figura C.5 ‘Vease Ia figura C6 8 sen 0 cos 8 @) L5sene 6) 15 Figura €.5. Para el problema 133. 13.21. 13.28, 13.27. 1331, 13.33, 13.37. . Figura C6. Para el problema 13.15. 1.50? sen? 0 ae a) 3.0840 99.97% a) 15 cen? 0,5 by Gen? 0 con 6 ©) 66.05 cos? 0 sen? 2, 66.05 “inbitade a ). 18a on.9 cos (1 pd cos 0 )ae Véase la figura C7 Vénse la figura C8 0.2686 es B 2snw Dias pw 19 aB ° Figura ©.7. Para el problema 13.27. Cay 13.39, 13.41. Capitulo 14 14, 143, 145, 147, 149, 14.41, 14.13, 14.15. 447. 2) 1.708 Vim. b) 11.36 wV/m ©) 30.95 mw. @ 1.91 pw 77.52 Explicacion 0.33 ~J0.15, 0.5571 ~ 0.626 3571 Comprobacion 1.428 a) 0.2271 ) 13.13" ©) 376 @) 29.23° &) 63.1% eto = BORG, Explicacion Figura €.8. Para el problema 13.29. 58 mw Artnorce C Japitulo 15 - Véase Ia figura C.9 7, 1-56 Figura €.9. Para el problema 15.1, prem 15.11, a) La matriz [A] permanece igual, pero a cada término de la matriz [B] se debe afladir —fipsie b) Vz = 4.276, Vy = 9.577, V, = 11.126 Vi = "2013'V, = 2.919,'V, — 6.069 V, = —3.424) Vv, = —0.109,V, = 2.909 El refultado numérico coincide por completo con la solucién exacta; por ejemplo, respecto de r— 0, V(O, 0) ~ 0, V0.1, 0) = 0.3090, V(0.2, 0) = 0.5878, (0.3, 0) V(0.4, 0) ='0.9511, V0.5, 0) —'1.0, V(0.6, 0) — 0.9511, ete. 12.77 p¥/m (numérico), 12.12 pF/m (exacto) IS.17. Véase la tabla C3 0.809, @ erodes) Cwph 10 Baas, 20 Bos 17% 1132 15.19. a) Exacto: C = 80.26 pF/m, Z, = 41.56 O; en cuanto a Ia solucién numérica, véase Ia tabla C4 2.) 20° Bis6e S197 b) En cuanto a los resultados numéricos, véase la tabla C5 N_ Ciphim) 2,0) 191075330903 15.21. Comprobacion 1523. @) En (1.5, 0.5) a lo largo de 12 y (0.9286, 0.9286) alo largo de 13 %) 56.67 V 0.8788 —0.208 ° —0.6708 08 us2e 0-12 0.1248 ° ni 1.408 —0.208 ~0.6708 0.1248 0.208 1.0036, 27. 18 V,20V 29. Véase la tabla C.6 For iene Soo ‘Comprobacion Apéndice D a, alr wranceatan Cantidad tankiades) simboto sr ee See ae pes Permitivided del vacto (Fim) a fase x10 1 Pomeabidad del veto (Em) = sea wae 107 Inmpodanclaintrfeacca del veto (D2) = wee 1200 Yotocda ott ene vac ov) 2900 10 ax Car det cocton (©) =1.0020 > 10-* H16 x10 Moss et eee kx) me 2.1068 3 10" sax 10% Mase et proten (hs) | ress x 10-7 67% 10-7 Neanicaeat = 10749 > 10-= er x 10-8 onsite de otrmann 6) a 1.38087 10-> 138% 10-8 Nimo de Avogndio (emo) ™ 022s x10 oxo Consinnte de Fanck =) e a2 10™ e210 ‘Acslracin por gravedad (oie?) . on os onsite universal de gravtecin « 6.50 > 10-" 6.65 9 20- one) Blectrdin-vole (1) ‘Roencia Prete Simboio 10% xa io" he io Gi ior Megs esto prose aooperzodnm Maydscule Mindsewla Nombre ‘Maydseula — Mindscula Namere a a Bota z é r y Gamma o é a a Dele in * = = Epsilon e 2 z é Zeta = os Sigma ° 3 Then y : Ipstton A a Comba * * Pat te 2 Mer a = mesa Derivadas vectoriales Coordenadas cartesianas (x.y, ) AW Aa, + Aya + Aa, a, av, ay Yt on™ aA, , 2A, +a te ey oz A A, [ts Ae a [RS — Vay ~ oe 1 * Vaz ev ev ay ae * ay * at veya Coordenadas cilindricas (o, d. 2) A= Apa, + Agng + Am ae ee a op pag" * az 13 1AAy | 2A. vi an=d es wa) + 3° + a, pas 1|'3 VRAD lip oF Ap be (ase - Sst ]= +L Pp ab a 1a (,av) 1a 12 (av) 4 Lee, oe VV pap \P ap) * page * ast Coordenadas estéricas (r, 0, 6) Am Ap, + Aga + Aung veiw 1 VV = or * 7 36 ™ * Teen 8 a n oy 1 ae ? Faen@ 20 (409°) + Teen Gog ray (rs0n 8) my 2 2 a6 a0 A, ray (rsen 0) Ay Faewe [do (oven Elie ean — 1a (av te aw wv (ASL) + aetaa ao (eno indice analitico Aisiadores, 161,162. Véase tambien Sean ‘anillo cargado, 118 Se radincion longitudinal, 15 toinductancia, 336 Indale electrostético, 186 ‘Se'radincion, 592 ‘Slectroneaico, 103,592 Itrotacional, 87 remoto (0 ljano), 592 compote) as [Stmantoas on el tiempo, 389 Slasfieacion de, 86-88 ccepacttancia, 224-230 SSenpetibllited ctoctromngnditin fo), Componentes de microondas, 639 ‘valores de, 737 ‘Sonrervacton del flujo magnético, ‘de propaacisn, 410,479 ate 1m fwoice anaimeo ieeerico, 142 ‘Svldna, 606 ‘raenct, 6-73 ‘finioign, 60, ‘© Laplace, 84, 199-202, 288, 671, ‘¢ Poisson, 199-202, 291, 670, 671, "283, 368-380, 418, 438, 451, lividad de Blindaje, 63 fividad de union, 638 trowtation, 103 ‘pin 40, 339-384 oF de arrosio,613 Se de calidad $78, 579 Vaare fuerrn electromatrie Sfinicion, 650 Ioterost, 329 ‘Sxponcnciales, 730 Togaeitemieas 730, telecast 727-728 ‘So radincion, 603-7 incerferencia elestromagneticn (lo), Lamina de sorriente infiniea, 275 inpleciaogees ‘de Ampere, 262,273,290 de Biot Sewart, 262-306, 290, 307 splegciones dein 126-120, de In corriente de Kirchhoff, 10, ‘Siaiparion, 80 fom erento abort, 489 gait Portia 80 “de Majo elders, 144 songicad ‘Soda ein in de ondan magnetizacién, 324,329 ‘masnitud, 5 ‘équina copindara xerogréicn, 204 material homoxeneo, 173 ‘material notropico, 175 ‘materiales no magnéticos, 327 Imatriz de cocficientes de ion imatris global de cocticiente, 699 ‘de Imagenes, 240 ‘Se momentos, e3-di7 Be 32 BRR. BR microondas, 638 moder 546.348 ‘sidetrice transversal (e8), 547, ‘tnd mera om), modo degenerados, 576 tmoléeula de cinco nodon, 672 ‘momento magnetice dipolar, 317 monopole, 148 ‘mullplieacion de patrones, 613 odo jo, 670 Bucleo Co kernel), 683 hnamero de onda, 412 ‘de piano tniforme, 425 Sittcmametcntanevoral Com), Favtmetion de dapersin, 64h Se Fadincion, 604 perdida de insereién, 639 stores de 739 ‘ompleja, 422 ‘Sel acto, toa Polaron, 7icia as otencialos magnsticos, 284-287 orenciates retardndon 389) producto, Nectar, 13-43 profundidad de penetracién. Véue radar) 625,041 Fadlecion reflojada, 514 Fistn do outa oxteciomaria (roe), 444 eR accab 16 ait sb. “ae radincion, $93 Tesonador de guins de ondas, 575 ZSparacion de variables,212, 221 inronizador de roceiom de linen diniea, Iwoice anaumco 767 Sierici cavipotencint 44 mesnetice, 326 ‘mperatare’ Curie 328” Se In civergencia, 72,125, 8 Papmting 30,457 siompe de einen, 181,229 torque 316 {eal fala, 153 transtormasior de cuarto de onda, 505 ‘rayectoria amperiana,274 werlabten complajan, 728, 720 ‘de distancia, 8 ‘rattiptiencion de, 11 ‘35 te, 563 volumen diferencial, 54, 55,57, 89 prea i i

You might also like